Vous êtes sur la page 1sur 184

Álgebra Comutativa

um tour ao redor dos anéis comutativos


FT
Eduardo Tengan
(ICMC-USP)
)
28
7:
(1
RA
0
01
,2
29
D
ct
,O
ET
FT
)
28
7:
(1
RA
0
01
,2
29
D
ct
,O
ET

Copyright c 2010 E. Tengan


Permission is granted to make and distribute verbatim copies of this document provided the copyright
notice and this permission notice are preserved on all copies.
“To get a book from these texts, only scissors and glue were needed.”

J.-P. Serre, in response to receiving the 1995 Steele


Prize for his book “Cours d’Arithmétique”
FT
)
28
7:
(1
RA
0
01
,2
29
D
ct
,O
ET
D
ET
,O
ct
29
RA
,2
01
0
(1
FT
7:
28
)
Chapter 1

Prefácio

Quando terminei de escrever meu livro anterior, estava tão esgotado que prometi a mim mesmo: meu
próximo livro será intitulado “A Tabela dos Primos Pares (versão resumida)”.
A Teoria de Anéis possui diversas aplicações nas mais diversas partes da Matemática, tais como
Combinatória, Geometria Algébrica, Teoria dos Números, Análise, e até mesmo fora da Matemática,
como na Culinária (anéis de cebola), no Transporte (anel rodoviário). No cinema, anéis têm obtido
grande destaque, em pelı́culas como “O Senhor dos Anéis”, “Matrix” e “Corpo Fechado”.
Os pré-requisitos para este livro são poucos, Bourbarki Lang

1 Devo ler este livro?

2 Terminologia Frequente e Notações


Utilizamos a já consagrada notação N, Z, Q, R, C para denotar os conjuntos dos números naturais
(incluindo o zero), inteiros, racionais, reais e complexos. Denotamos ideais por letras góticas. Além
disso, ao longo de todo o livro utilizaremos a seguinte terminologia:
1. Claramente: Não estou a fim de escrever todos os passos intermediários.
2. Lembre: Não deverı́a ter que dizer isto, mas. . .
3. Sem Perda de Generalidade: Farei apenas um caso e deixarei você adivinhar o resto.
4. Verifique: Esta é a parte chata da prova, então você pode fazê-la na privacidade do seu lar, quando
ninguém estiver olhando.
5. Não consegui verificar todos os detalhes, então vou quebrar a prova em pedaços
Esboço de Prova:
que não pude provar.
6. Dica: A mais difı́cil dentre as muitas maneiras de se resolver um problema.
FT
7. Analogamente: Pelo menos uma linha da prova acima é igual à prova deste caso.
8. não me lembro do enunciado (na verdade, nem tenho certeza se provei isto
Por um teorema anterior:
)
ou não), mas se o enunciado estiver correto, o resto da prova segue.
28

9. Prova omitida: Acredite, é verdade.


7:
(1
RA
0
01
,2
29
D
ct
,O
ET
D
ET
,O
ct
29
RA
,2
01
0
(1
FT
7:
28
)
Chapter 2
anel
anel zero
morfismo! de
módulo
morfismo! de
grupo de unid
anel produto

Vamos ser amigos dos anéis! ideal

Em contraponto ao restante deste livro, este capı́tulo inicial tem um caráter, digamos, mais exploratório:
veremos anéis comutativos em seu formato “bruto”, ainda não lapidados por uma abordagem teórica
e sistemática, a ser adotada a posteriori. Cabe aqui bem lembrar que Álgebra Comutativa não é uma
área isolada do resto da Matemática; muito pelo contrário, é uma disciplina que bebe de diversas fontes,
como a Análise, a Teoria dos Números, a Geometria e a Topologia, entre outras. Conhecer esta interação
é importante, não só para compreender como a Álgebra Comutativa se posiciona dentro do Cosmos
matemático, mas também para entender a motivação dos teoremas, métodos e exemplos que formam o
tronco desta bela disciplina.
Muito bem, mas o que de fato é feito neste capı́tulo? Afinal de contas, queremos menos palavras
e mais ação! Começamos com uma breve revisão das definições e conceitos básicos que serão utilizados
ao longo de todo o livro. Logo em seguida, introduzimos os grandes protagonistas no estudo dos anéis
comutativos: os ideais primos. Veremos o papel que eles assumem em diversos exemplos concretos. Por
fim, encerramos este prelúdio definindo a topologia de Zariski do espectro primo de um anel, sinalizando
um dos temas recorrentes deste manuscrito: que anéis comutativos são, sobretudo, objetos geométricos
por natureza.

1 Notação e Convenções
Esta seção é uma “coleção de pré-requisitos”, definições e conceitos assumidos como conhecidos e que
serão frequentemente utilizados em todo o livro. Sugerimos que o leitor não perca muito tempo nesta
seção, fazendo apenas uma leitura rápida para se familiarizar com as notações empregadas.
Começamos com a noção de anel: como você já sabe, um anel nada mais é do que um conjunto
onde podemos somar, subtrair e multiplicar; neste livro, convencionamos que o termo não adornado anel
significará sempre anel comutativo com 1. Note que o menor anel do universo, o anel zero A = 0 (com
um único elemento 0 = 1) é um legı́timo anel e não está banido por esta convenção. Um morfismo
de anéis φ: A → B é um mapa que preserva soma e produto, i.e., φ(a1 + a2 ) = φ(a1 ) + φ(a2 ) e
FT
φ(a1 · a2 ) = φ(a1 ) · φ(a2 ) para todo a1 , a2 ∈ A, e que (ainda por decreto) satisfaz φ(1) = 1. Um
A-módulo M é, moralmente falando, um “espaço vetorial sobre A” em que 1 · m = m para todo
m ∈ M . Um morfimo de A-módulos ψ: M → N é uma “transformação A-linear” entre M e N :
)
28

ψ(a1 · m1 + a2 · m2 ) = a1 · ψ(m1 ) + a2 · ψ(m2 ) para todo a1 , a2 ∈ A e m1 , m2 ∈ M .


Recorde que uma unidade u ∈ A é um elemento que possui inverso multiplicativo u−1 ∈ A. O
7:

conjunto de todas as unidades de A, juntamente com a operação multiplicação, forma um grupo abeliano
(1
RA

A× , o grupo de unidades de A. Por exemplo, Z× = {±1} e C[t]× = C× = C \ {0}.


Q
0

Dada uma coleção de anéis Aλ , λ ∈ Λ, definimos o anel produto λ∈Λ Aλ como o anel dado pelo
01

produto cartesiano dos Aλ , com a soma e multiplicação efetuadas coordenada a coordenada. O elemento
,2

neutro deste anel é a tupla constante com todas as entradas iguais a 0 e a identidade é a tupla constante
com todas as entradas iguais a 1.
29

Lembre ainda que um ideal a de um anel A é um A-submódulo de A, ou seja, um subconjunto


D

a ⊂ A fechado por combinações A-lineares: x, y ∈ a e a, b ∈ A ⇒ ax + by ∈ A. Ideais generalizam a


ct

noção de conjunto de múltiplos de um elemento. Dada uma famı́lia arbitrária {bλ }λ∈Λ de elementos de
,O

A, o conjunto de todas as combinações A-lineares (finitas) de elementos nesta famı́lia


ET


a1 · bλ1 + · · · + ar · bλr r ∈ N, ai ∈ A, λi ∈ Λ

é um ideal de A, o ideal gerado por {bλ }λ∈Λ . Note que este é o “menor” ideal de A que contém o
conjunto {bλ }λ∈Λ . O ideal gerado por a1 , . . . , an ∈ A será denotado por uma das duas seguintes formas:

(a1 , . . . , an ) = A · a1 + · · · + A · an
8 Vamos ser amigos dos anéis!

Ideais da forma (a), isto é, gerados por um único elemento, são chamados de ideais principais. ideal! principal
ideal! próprio
Ideais podem ser multiplicados e somados: dados dois ideais a e b, a · b é o ideal
P gerado por todos morfismo! quociente
os produtosSa · b com a ∈ a e b ∈ b. Dada uma famı́lia de ideais aλ , denotamos por λ aλ o ideal gerado
pela união λ aλ . Em particular, para ideais finitamente gerados, temos
(a1 , . . . , am ) · (b1 , . . . , bn ) = (a1 b1 , a1 b2 , . . . , ai bj , . . . , am bn )
(a1 , . . . , am ) + (b1 , . . . , bn ) = (a1 , . . . , am , b1 , . . . , bn )
Um ideal a de A é dito próprio se a 6= A, isto é, se a é um subconjunto próprio de A. Note que a é
próprio se, e só se, 1 ∈
/ a ou, mais geralmente, se, e só se, A× ∩ a = ∅ (da sabedoria popular: “a melhor
maneira de se livrar de um ideal próprio é dar uma unidade a ele”). De fato, se A× ∩ a = ∅ então 1 6∈ a e
portanto a 6= A. Reciprocamente, se a é próprio mas existe u ∈ A× tal que u ∈ a então a = au−1 · u ∈ a
para todo a ∈ A, o que é absurdo. Observe que todo anel, com exceção do anel 0, possui ideais próprios
(o ideal nulo, por exemplo).
Ideais possuem um importante papel não só em nossas vidas mas também nas vidas dos anéis, sendo
ingredientes essenciais na promoção da igualdade: dado um ideal a ⊂ A, o anel quociente A/a é o anel
obtido “igualando-se” elementos que diferem por um elemento em a; formalmente, os elementos de A/a
são as classes laterais do ideal a, que serão denotadas por uma das seguintes três maneiras:
a + a = a mod a = a ∈ A/a (a ∈ A)
(sendo a última notação a utilizada se o ideal a está claro pelo contexto). Escrevemos ainda
a≡b (mod a) ⇐⇒ a − b ∈ a ⇐⇒ a = b em A/a
de modo que as propriedades usuais de congruências se verificam:

  a + c ≡ b + d (mod a)
a ≡ b (mod a)
⇒ a − c ≡ b − d (mod a)
c ≡ d (mod a) 
ac ≡ bd (mod a)
Por exemplo, para provar a última propriedade, note que se a − b ∈ a e c − d ∈ a então ac − bd =
c · (a − b) + b · (c − d) ∈ a. Estas propriedades nada mais expressam do que a compatibilidade das
operações do anel com a relação de equivalência dada pelo quociente. Isto mostra que as operações em
A/a
def def
a±b = a±b e a·b = a·b (a, b ∈ A)
estão de fato bem definidas, isto é, independem da escolha dos representantes de classe a, b.
O anel quociente vem equipado de fábrica com um morfismo quociente ou morfismo projeção,
claramente sobrejetor:
FT
q: A ։ A/a
a 7→ a
)
28

Ainda no que tange a quocientes, temos os seguintes resultados muito importantes, ainda que de demon-
strações singelas. O primeiro é o princı́pio “zero vai em zero”: para mostrar que um morfismo de um anel
7:

quociente A/a para um outro anel B está bem definido, basta verificar que 0 7→ 0. O segundo fornece
(1
RA

condições suficientes sob as quais este morfismo é um isomorfismo. O terceiro identifica os ideais do anel
quociente A/a com os ideais de A contendo a.
0
01

Teorema 1.1 (Propriedade Universal do Quociente) Sejam A e B anéis e seja a um ideal de


,2

A. Dar um morfismo φ: A/a → B é o mesmo que dar um morfismo φ: A → B tal que φ(a) = 0.
Explicitamente, se φ: A → B satisfaz φ(a) = 0 então existe um único morfismo φ: A/a → B tal que
29

φ(a) = φ(a) para todo a ∈ A, ou seja, tal que o seguinte diagrama comuta:
D
ct

φ -
A B
,O
-

ET
φ
∃!

q
?
?
A/a
Aqui q: A ։ A/a denota o morfismo quociente.
9

Teorema 1.2 (Isomorfismo) Seja φ: A → B um morfismo de anéis. Então o kernel de φ módulo quoci
nilpotente
reduzido
def
ker φ = {a ∈ A | φ(a) = 0} divisor de zer
elementos ass
corpo de fraç
é um ideal de A e φ induz (pelo teorema anterior) um morfismo φ: A/ ker φ ֒→ B que é injetor e que elemento! irre
domı́nio de fa
portanto estabelece um isomorfismo entre A/ ker φ e a imagem de φ. domı́nio de id

Teorema 1.3 (Correspondência de Ideais) Seja A um anel e a um ideal. O mapa quociente q: A ։


A/a estabelece uma bijeção entre
 
ideais b de A tais que b ⊃ a ↔ ideais de A/a
b 7→ q(b)

Se N é um A-submódulo de M , podemos definir o A-módulo quociente M/N de maneira análoga,


como o conjunto das classes laterais de N . Mutatis mutandis, os três teoremas anteriores valem também
no contexto de módulos.
Lembre que um elemento a de um anel A é dito nilpotente se existe um número natural n tal que
an = 0. Um anel A 6= 0 é chamado de reduzido se seu único elemento nilpotente é o 0. Um elemento
a 6= 0 é um divisor de zero se existe b 6= 0 tal que a · b = 0. Recorde que um anel A 6= 0 sem divisores
de zero é chamado de domı́nio: mais explicitamente, um anel A é um domı́nio se A 6= 0 e, para todo
a, b ∈ A, temos a · b = 0 ⇐⇒ a = 0 ou b = 0. Num domı́nio, vale a “lei do cancelamento”: se c 6= 0
então a · c = b · c ⇒ a = b (já que a · c = b · c ⇐⇒ (a − b) · c = 0 ⇐⇒ a − b = 0). Para elementos a, d
em um domı́nio A, escrevemos ainda

d|a (lê-se “d divide a” ou “a é múltiplo de d”)


⇐⇒ a = b · d para algum b ∈ A
⇐⇒ (d) ⊃ (a) (como diz o velho ditado, “no mundo ideal, conter é dividir”)

Dois elementos a, b de um domı́nio A são ditos associados se eles diferem de uma unidade (multiplica-
tivamente falando), isto é, a = b · u para alguma unidade u ∈ A× . Ideais são “insensı́veis a associados”
no sentido que
(a) = (b) ⇐⇒ a e b são associados
De fato, temos que (a) = (b) é equivalente a a | b e b | a, ou seja, à existência de elementos u, v ∈ A
tais que a = b · u e b = a · v. Se a e b são associados, digamos a = b · u com u ∈ A× , temos também
b = a · v para v = u−1 ; reciprocamente, se a = b · u e b = a · v temos a = a · vu, ou seja, a = b = 0 ou
FT
vu = 1 ⇒ u, v ∈ A× e em ambos os casos a e b são associados.
Para um domı́nio A, denotaremos ainda por
)
na o
28


Frac A = a, b ∈ A, b 6= 0
b
7:

a c
(1

o seu corpo de frações: aqui, duas frações e são identificadas se, e só se, ad = bc; e as operações
RA

b d
são definidas do modo usual:
0
01

a c a·d+b·c a c a·c
+ = e · =
b d b·d b d b·d
,2
29

Seja A é um domı́nio. Um elemento π ∈ A \ (A× ∪ {0}) é dito irredutı́vel se ele só possui fatorações
triviais: π = a · b ⇒ a ∈ A× ou b ∈ A× . Um domı́nio A é chamado de domı́nio de fatoração única
D
ct

(DFU) se todo elemento a 6= 0 de A pode ser fatorado de maneira essencialmente única como produto
,O

de irredutı́veis, ou seja,
1. (Existência da fatoração) a pode ser escrito como a = π1 π2 . . . πm com πi irredutı́veis;
ET

2. (Unicidade da fatoração) Se a também se escreve como a = ρ1 ρ2 . . . ρn com ρi irredutı́veis então


m = n e existe uma permutação σ: {1, 2, . . . , m} → {1, 2, . . . , m} tal que πi é associado a ρσ(i) ,
i = 1, 2, . . . , m = n.
Um domı́nio em que todo ideal é principal é chamado de domı́nio de ideais principais (DIP). Por
exemplo, Z e C[t] são DIPs. Temos ainda os seguintes importantes resultados (ver apêndice):
10 Vamos ser amigos dos anéis!

Teorema 1.4 Todo DIP é um DFU. álgebra


complexo
sequência exata
Teorema 1.5 Se A é um DFU então A[x] também é um DFU. produto direto
soma direta
Uma A-álgebra é por definição um morfismo de anéis φ: A → B. Muitas vezes, φ (dito morfismo
base) é claro pelo contexto e por isso nos referimos ao próprio anel B como sendo uma A-álgebra. Por
exemplo, o anel de polinômios A[x1 , . . . , xn ] é uma A-álgebra via a inclusão A ֒→ A[x1 , . . . , xn ]; além
disso, qualquer anel A é uma Z-álgebra pelo morfismo natural Z → A (que leva 1 ∈ Z em 1 ∈ A). Note
que φ não é necessariamente injetivo, mas se a ∈ A e b ∈ B denotamos φ(a) · b simplesmente por a · b,
por abuso de linguagem. Finalmente, um morfismo f : B → C de A-álgebras é um morfismo de anéis
compatı́vel com os morfismos bases φ: A → B e ψ: A → C, isto é, tal que o diagrama
f -
B C
6

-
ψ
φ

A
comuta (f ◦ φ = ψ). Utilizando o abuso de linguagem acima, um morfismo de anéis f : B → C é um
morfismo de A-álgebras se, e somente se, f é A-linear: f (ab) = af (b) para todo a ∈ A e b ∈ B.
Relações lineares entre módulos são geralmente expressas através de sequências exatas. Uma
sequência de morfismos de A-módulos
··· - Mi+1 - Mi
fi+1
- Mi−1
fi
- Mi−2
fi−1
- ···
fi−2

é um complexo se fi−1 ◦ fi = 0 ⇐⇒ im fi ⊂ ker fi−1 para todo i. Um complexo é uma sequência


exata se im fi = ker fi−1 para todo i. Em particular,
0 - M - N
f
- P
g
- 0
é uma sequência exata se, e só se, f é injetora, g é sobrejetora e ker g = im f , de modo que g induz um
isomorfismo N/f (M ) ∼ = P . Neste caso dizemos que a sequência acima é uma sequência exata curta.
Uma maneira de interpretar uma sequência exata curta é imaginar o módulo do meio como “composto”
pelos módulos das pontas. Por exemplo, se os módulos acima são k-módulos onde k é um corpo (i.e., M ,
N e P são k-espaços vetoriais) então dimk N = dimk M + dimk P .
Note que toda sequência exata (M• , f• ) pode ser quebrada em sequências exatas curtas
0 - im fi+1 - Mi - im fi - 0
de modo que o estudo de sequências exatas gerais pode ser reduzido ao estudo das sequências exatas
curtas.
FT
Dados dois A-módulos M e N , o conjunto HomA (M, N ) de todos os morfismos φ: M → N de A-
módulos também é um A-módulo (com a soma e o produto por escalares em A induzidos pelas operações
em N ). Além disso, dada uma famı́lia de A-módulos Mi , i ∈ I, podemos construir dois novos A-módulos:
)
28

o produto direto Y
7:

Mi
(1

i∈I
RA

que, como conjunto, é igual ao produto cartesiano dos Mi , sendo a soma e o produto por escalares
0

realizada componente a componente; e a soma direta


01

M
Mi
,2

i∈I
29

Lcom mi 6= 0
que é o submódulo do produto direto cujos elementos são as tuplas (mi )i∈I quase nulas, i.e.,
apenas para um número finito de ı́ndices i. Um módulo que é isomorfo a uma soma direta i∈I Mi onde
D
ct

cada Mi ∼ = A é chamado de módulo livre sobre A. Por exemplo, espaços vetoriais sobre um corpo k
,O

são k-módulos livres.


Observe que temos os isomorfismos canônicos
ET

Y Y  Y M 
HomA (T, Mi ) = HomA T, Mi e HomA (Mi , T ) = HomA Mi , T
i∈I i∈I i∈I i∈I
Q
onde o primeiro isomorfismo leva (φi )i∈I no morfismo φ: T → i∈I Mi cuja i-ésima coordenada é φi ,
L
enquanto que o segundo isomorfismo leva (ψi )i∈I no morfismo ψ: i∈I Mi → T dado por ψ (mi )i∈I =
P
i∈I ψi (mi ), que faz sentido uma vez mi = 0 para quase todo i ∈ I.
11

2 Ideais Primos e Maximais ideal primo


ideal maxima
espectro
Nesta seção, iniciamos o estudo dos anéis propriamente ditos. Para realmente entendermos um anel,
precisamos sobretudo conhecer seus ideais. Dois tipos de ideais desempenham um papel primordial: os
ideais primos e os maximais.
Definição 2.1 Seja A um anel.
1. Um ideal p de A é dito primo se satisfaz uma das (e portanto todas!) seguintes condições
equivalentes:
i. A/p é um domı́nio;
ii. p é um ideal próprio e a · b ∈ p ⇐⇒ a ∈ p ou b ∈ p para todo a, b ∈ A;
iii. p é um ideal próprio e a · b ⊂ p ⇐⇒ a ⊂ p ou b ⊂ p para quaisquer ideais a, b de A.
2. Um ideal m ⊂ A é dito maximal se satisfaz uma das (e portanto todas!) seguintes condições
equivalentes:
i. A/m é um corpo;
ii. m é maximal no conjunto parcialmente ordenado (por inclusão) dos ideais próprios de A,
isto é, se a é um ideal próprio de A e a ⊃ m então a = m.
Observe que como todo corpo é domı́nio, todo ideal maximal é primo.
Intuitivamente, p é primo se p 6= (1) (afinal de contas, ao contrário da crença popular, 1 não é
um número primo!) e “p divide o produto a · b se, e só se, p divide a ou p divide b”. A equivalência
(i) ⇐⇒ (ii) na definição de ideal primo é apenas uma paráfrase em termos de quociente: A/p é domı́nio
se, e só se, primeiro, A/p 6= 0, ou seja, p 6= A e, segundo, a · b = 0 ⇐⇒ a = 0 ou b = 0 para todo
a, b ∈ A ou, em outras palavras, a · b ∈ p ⇐⇒ a ∈ p ou b ∈ p. Para ver que (iii) ⇒ (ii), basta tomar
a = (a) e b = (b). Finalmente, para mostrar que (ii) ⇒ (iii), suponha por absurdo que p ⊃ ab mas p 6⊃ a
e p 6⊃ b. Neste caso, existem elementos a ∈ a \ p e b ∈ b \ p, logo por (ii) temos a · b 6∈ p. Mas a · b ∈ a · b,
o que contradiz a hipótese p ⊃ ab.
A equivalência (i) ⇐⇒ (ii) na definição de ideal maximal decorre do fato de que a correspondência
de ideais no teorema 1.3 preserva a relação de inclusão, de modo que m ⊂ A é maximal dentre os ideais
próprios de A ordenados por inclusão se, e só se, (0) é o único ideal próprio de A/m. Mas um anel B
possui um único ideal próprio (necessariamente o ideal nulo) se, e só se, B é um corpo. De fato, se B
é um corpo e b 6= (0) é um ideal de B, então b possui um elemento b 6= 0, que é uma unidade, logo
1 = b · b−1 ∈ b ⇒ b = B. Reciprocamente, se (0) é o único ideal próprio de um anel B, então dado b 6= 0
temos (b) = B e portanto 1 ∈ (b), logo existe c ∈ B tal que bc = 1, isto é, b ∈ B × , mostrando que todo
elemento não nulo é uma unidade e que, portanto, B é corpo.
Dado um morfismo de anéis φ: A → B, se q é um ideal primo de B, então a sua pré-imagem
FT
φ−1 (q) é um ideal primo de A. De fato, note primeiramente que φ−1 (q) é próprio, pois de outra forma
1 ∈ φ−1 (q) ⇐⇒ 1 = φ(1) ∈ q, contrariando o fato de q ser próprio. Por outro lado, temos que
)
28

a · b ∈ φ−1 (q) ⇐⇒ φ(a) · φ(b) = φ(a · b) ∈ q ⇐⇒ φ(a) ∈ q ou φ(b) ∈ q ⇐⇒ a ∈ φ−1 (q) ou b ∈ φ−1 (q)
7:
(1
RA

Definição 2.2 Dado um anel A, chamamos de espectro primo ou simplesmente espectro de A o


conjunto de todos os ideais primos de A, denotado por Spec A. Se φ: A → B é um morfismo de anéis,
0

denotamos por
01

Spec(φ): Spec B → Spec A


,2

q 7→ φ−1 (q)
29

o morfismo entre espectros induzido por φ.


D
ct

Exemplo 2.3 (0) ∈ Spec A se, e só se, A é um domı́nio.


,O

Exemplo 2.4 Se A é um DFU e π ∈ A é irredutı́vel, então (π) é um ideal primo. De fato, (π) é próprio
ET

pois π ∈
/ A× e, pela fatoração única em irredutı́veis, temos que ab ∈ (π) ⇐⇒ π | ab ⇐⇒ π | a ou
π | b ⇐⇒ a ∈ (π) ou b ∈ (π).
Exemplo 2.5 Seja A um DIP (por exemplo, A = Z ou A = C[t]). Então
 
Spec A = (0) ∪ (π) | π é irredutı́vel
12 Vamos ser amigos dos anéis!

De fato, já sabemos pelo exemplo anterior que se π é irredutı́vel então (π) é primo. Reciprocamente, se
um ideal (principal) não nulo (π) é primo então π 6= 0 é irredutı́vel: π ∈/ A× pois caso contrário (π) = A
e se π = a · b então como a · b ∈ (π) e (π) é primo temos a ∈ (π), digamos, logo existe c ∈ A tal que
a = πc ⇐⇒ a = abc ⇐⇒ 1 = bc (note que a 6= 0 pois π 6= 0 e A é domı́nio) e portanto b ∈ A× , isto é,
a fatoração de π é trivial.
Em particular, temos  
Spec Z = (0) ∪ (p) | p é primo
 
Spec C[t] = (0) ∪ (t − a) | a ∈ C

Exemplo 2.6 Se A é um DIP, todo ideal primo não nulo de A é maximal. De fato, para mostrar que
A/(π) é um corpo se π é irredutı́vel, tome a ∈ A tal que a 6= 0, ou seja, tal que π ∤ a. Como A é DIP,
temos que (π, a) = (d) para algum d ∈ A. Mas então d | π, logo, como π é irredutı́vel, d é associado ou
a 1 ou a π, sendo que o último caso não ocorre pois d | a mas π ∤ a. Resumindo, (π, a) = (1) e assim
existem r, s ∈ A tais que
r · π + s · a = 1 ⇒ s · a = 1 em A/(π)

Ou seja, a é uma unidade em A/(π), como querı́amos mostrar.


Exemplo 2.7 Se a é um ideal qualquer de um anel A e q: A ։ A/a é o mapa quociente então pelo
teorema da correspondência de ideais (teorema 1.3) temos que Spec(q): Spec A/a ֒→ Spec A é injetor,
com imagem dada pelos ideais primos em Spec A que contêm a. Por exemplo, temos

Z 
Spec = (p) p primo, p | n
(n)
C[t] 
Spec  = (t − a) a ∈ C, f (a) = 0
f (t)

(Lembre que “conter é dividir” e t − a | f (t) ⇐⇒ f (a) = 0)


O próximo teorema mostra que ideais maximais (e portanto primos) existem em abundância.
Teorema 2.8 Seja A 6= 0 um anel não zero. Então A possui um ideal maximal. Consequentemente:
1. Spec A = ∅ ⇐⇒ A = 0
2. se a ⊂ A é um ideal, então a é próprio se, e só se, a ⊂ m para algum ideal maximal m de A.

Prova Seja A 6= 0 e seja P o conjunto de todos os ideais próprios de A, parcialmente ordenados por
inclusão. Devemos mostrar que P possui um elemento maximal, o que seguirá do lema de Zorn. Note
FT
que P 6= ∅ pois (0) ∈ P (aqui utilizamos A 6= 0). Temos agora que mostrar que qualquer cadeia C em P
é limitada superiormente. Considere
)
[
28

u= c
7:

c∈C
(1
RA

Se mostrarmos que u é um ideal próprio de A, claramente u será um limitante superior de C em P e o


resultado segue. Primeiramente, u é ideal pois se x, y ∈ u existem ideais g, h ∈ C tais que x ∈ g e y ∈ h
0

e podemos supor que g ⊂ h (lembre-se de que C é uma cadeia). Assim, ax + by ∈ h ⊂ u para quaisquer
01

a, b ∈ A. Além disso, u é próprio, pois caso contrário 1 ∈ u e assim 1 ∈ c para algum c ∈ C, contradizendo
,2

o fato de que c ∈ P é próprio.


Agora, para mostrar o item 1, note que se A 6= 0 então Spec A 6= ∅ pois A possui pelo menos um
29

ideal maximal, que é primo; por outro lado, se A = 0, não há ideais próprios e, em particular, não há
D
ct

primos, logo Spec A = ∅. Para obter o item 2, se a é próprio então A/a 6= 0 e portanto A/a possui um
,O

ideal maximal, que corresponde a um ideal maximal m de A com a ⊂ m pelo teorema da correspondência
de ideais (teorema 1.3). A recı́proca é imediata.
ET

Encerramos esta seção com uma generalização do famoso Teorema Chinês dos Restos, que afirma
que dados inteiros m1 , m2 , . . . , mn , dois a dois coprimos (i.e., gcd(mi , mj ) = 1 se i 6= j), então existe um
isomorfismo de anéis
Z ∼
- Z × Z × ···× Z
(m1 m2 . . . mn ) (m1 ) (m2 ) (mn )
13

Por exemplo, para m1 = 3 e m2 = 5, temos a seguinte tabela, cujas linhas são indexadas por elementos teorema chinê
ideais! coprim
de Z/(3) e as colunas, por elementos de Z/(5) e cuja entrada na posição (a, b) ∈ Z/(3) × Z/(5) é o
elemento de Z/(15) correspondente:

0 1 2 3 4
0 0 6 12 3 9
1 10 1 7 13 4
2 5 11 2 8 14

Teorema 2.9 (Chinês dos Restos) Seja A um anel e sejam a1 , . . . , an ideais dois a dois coprimos,
isto é, ai + aj = (1) para i 6= j (esta condição é por exemplo satisfeita se os ai são todos ideais maximais
distintos). Então
1. a1 ∩ · · · ∩ an = a1 · . . . · an
2. Temos um isomorfismo
A A - A × ···× A

=
a1 · . . . · an a1 ∩ · · · ∩ an a1 an
dado pelo mapa natural

a mod a1 ∩ · · · ∩ an 7→ (a mod a1 , . . . , a mod an )

Prova Observe que, em geral, a1 ∩ · · · ∩ an ⊃ a1 · . . . · an . Para mostrar a inclusão oposta, procedemos


por indução em n, o caso n = 1 sendo trivial. Para n = 2, existem ai ∈ ai tais que 1 = a1 + a2 pois a1 e
a2 são coprimos. Assim, se c ∈ a1 ∩ a2 temos c = ca1 + ca2 ∈ a1 · a2 , como desejado. Agora seja n > 2.
Basta mostrar que a1 ∩ · · · ∩ an−1 e an são coprimos, pois neste caso por hipótese de indução (para n − 1
e 2 ideais) concluiremos que (a1 ∩ · · · ∩ an−1 ) ∩ an = (a1 · . . . · an−1 ) ∩ an = (a1 · . . . · an−1 ) · an . Como ai
e an são coprimos, existem ai ∈ ai e bi ∈ an tais que ai + bi = 1 para i = 1, . . . , n − 1. Assim

1 = (a1 + b1 ) · · · (an−1 + bn−1 ) ∈ a1 . . . an−1 + an ⊂ (a1 ∩ · · · ∩ an−1 ) + an

o que mostra que (a1 ∩ · · · ∩ an−1 ) + an = (1), encerrando a prova do item 1.


Para o item 2, temos que o mapa natural

A → A/a1 × · · · × A/an
FT
a 7→ (a mod a1 , . . . , a mod an )
)
tem kernel a1 ∩· · · an , logo induz um morfismo injetor A/(a1 ∩· · · an ) ֒→ A/a1 × · · ·× A/an . Para mostrar
28

que este morfismo é sobrejetor, observe que por “A-linearidade” basta encontrar, para cada i = 1, . . . , n,
7:

pré-imagens para os vetores da forma (0, . . . , 0, 1, 0, . . . , 0) (com 1 na i-ésima entrada), ou seja, elementos
ei ∈ A tais que
(1
RA


ei ≡ 1 (mod ai )
0

ei ≡ 0 (mod aj ) para j 6= i
01

De fato, neste caso teremos que a = b1 e1 + · · · + bn en será uma pré-imagem para um vetor arbitrário
,2

(b1 , . . . , bn ). Por simetria, podemos assumir i = n. Pela demonstração do item anterior temos que
29

a1 ∩ · · · ∩ an−1 e an são coprimos, logo existem elementos en ∈ a1 ∩ · · · ∩ an−1 e b ∈ an com en + b = 1.


Temos que en satisfaz as condições pedidas.
D
ct
,O

Exemplo 2.10 Seja k um corpo e considere o anel k[x], que é um DIP e logo um DFU. Seja f (x) ∈ k[x]
um polinômio não nulo e seja f (x) = c · p1 (x)e1 · · · pn (x)en , c ∈ k × a fatoração de f (x) em potências
ET

de polinômios mônicos irredutı́veis distintos pi (x). Note que se i 6= j então (pi (x)ei ) + (pj (x)ej ) =
(pi (x)ei , pj (x)ej ) = (1), pois k[x] é um DIP e (pi (x)ei , pj (x)ej ) é gerado por um elemento que divide
pi (x)ei e pj (x)ej , logo associado a 1. Pelo Teorema Chinês dos Restos temos

k[x] k[x] k[x]


= × ···×
(f (x)) (p1 (x)e1 ) (pn (x)en )
14 Vamos ser amigos dos anéis!

Observação 2.11 Tome cuidado: embora x e y não possuam fatores comuns no DFU k[x, y] (para k anel de séries formais
um corpo), os ideais (x) e (y) não são coprimos pois (x) + (y) = (x, y) 6= k[x, y]. E, de fato, k[x, y]/(xy)
não é isomorfo a k[x, y]/(x) × k[x, y]/(y)! Uma maneira de ver isto é a seguinte: o anel produto possui
um idempotente não trivial e = (1, 0), isto é, um elemento e diferente do elemento neutro e da identidade
satisfazendo e2 = e. Por outro lado, todos os idempotentes de k[x, y]/(xy) são triviais: se f ∈ k[x, y] é
2
tal que f = f em k[x, y]/(xy), então xy | f (f − 1) e pela fatoração única em k[x, y] temos que, primeiro,
x | f ou x | f − 1 e, segundo, y | f ou y | f − 1. Testando as 4 possibilidades, obtemos apenas f = 0 ou
f = 1. Por exemplo, se x | f e y | f , temos xy | f (novamente pela fatoração única) e assim f = 0. Por
outro lado, se x | f e y | f − 1 então f = x · g e f − 1 = y · h para algum g, h ∈ k[x, y], logo x · g − y · h = 1,
o que é imposı́vel (o lado esquerdo se anula para x = y = 0 mas o direito não).

3 Anéis que aparecem na Natureza


Álgebra Comutativa não é uma disciplina isolada; muitos de seus métodos e exemplos foram inspirados
em partes diversas da Matemática, como a Análise, a Teoria dos Números, a Geometria e a Topologia.
Vejamos alguns casos concretos.

3.1 Séries Formais


Começamos com um exemplo de um anel modelado em séries de potências estudadas em Análise, mas
com a vantagem de não termos de nos preocupar com as irritantes questões de convergência. Seja A um
anel. O anel de séries formais A[[t]] com coeficientes em A é o anel cujos elementos são “polinômios
infinitos” na variável t, i.e., expressões da forma

a 0 + a 1 t + a 2 t2 + · · · , ai ∈ A

sendo a soma e a multiplicação definidos da maneira usual, como no anel de polinômios:

(a0 + a1 t + a2 t2 + · · ·) + (b0 + b1 t + b2 t2 + · · ·) = (a0 + b0 ) + (a1 + b1 )t + (a2 + b2 )t2 + · · ·


(a0 + a1 t + a2 t2 + · · ·) · (b0 + b1 t + b2 t2 + · · ·) = a0 b0 + (a0 b1 + a1 b0 )t + (a0 b2 + a1 b1 + a2 b0 )t2 + · · ·

O anel de séries formais em várias variáveis A[[x1 , . . . , xn ]] é definido indutivamente por A[[x1 , . . . , xn ]] =
A[[x1 , . . . , xn−1 ]][[xn ]].
Por exemplo, em Z[[t]] temos

(1 − t) · (1 + t + t2 + t3 + · · ·) = 1

de modo que 1 − t e 1 + t + t2 + t3 + · · · são unidades em Z[[t]] (intuitivamente, a soma da “PG infinita”


FT
1
1 + t + t2 + · · · é 1−t ). Em geral temos
Lemma 3.1.1 Seja A um anel. Então
)
28

A[[t]]× = {a0 + a1 t + a2 t2 + · · · ∈ A[[t]] | a0 ∈ A× }


7:
(1
RA

Prova Um elemento a0 + a1 t + a2 t2 + · · · ∈ A[[t]] é uma unidade se, e só se,


0
01

(a0 + a1 t + a2 t2 + · · ·)(b0 + b1 t + b2 t2 + · · ·) = 1
,2

tem solução nas “variáveis” bi ’s, ou seja, se, e só se, o seguinte “sistema triangular” possui solução:
29
D

a0 b 0 = 1
ct

a1 b 0 + a0 b 1 = 0
,O

a2 b 0 + a1 b 1 + a0 b 2 = 0
ET

..
.

Assim, se a0 + a1 t + a2 t2 + · · · é unidade então a0 b0 = 1 ⇒ a0 ∈ A× . Reciprocamente, se a0 ∈ A× ,


podemos recursivamente definir b0 = a−1 −1
0 e bn = −a0 (an b0 + an−1 b1 + · · · + a1 bn−1 ) para n ≥ 1, que é
2
solução do sistema acima, logo a0 + a1 t + a2 t + · · · ∈ A[[t]]× .
15

Exemplo 3.1.2 Um caso interessante ocorre quando o anel de coeficientes é um corpo k. Neste caso, anel local
como k[[t]]/(t) ∼
= k, temos que (t) é um ideal maximal; como o complementar de (t) é exatamente k[[t]]× , inteiro algébr

temos que todo ideal próprio está contido em (t), que é portanto o único ideal maximal de k[[t]]. Um anel
com um único ideal maximal é chamado de anel local.
Vamos mostrar agora que o espectro de k[[t]] consiste apenas de dois elementos:

Spec k[[t]] = {(0), (t)}

De fato, como k[[t]] é um domı́nio, temos que (0) é um ideal primo. Se p 6= (0) é um ideal primo e f ∈ p
é um elemento não nulo, “descascando” a maior potência de t que divide f podemos escrever f = tn · u
com u ∈ k[[t]]× e n ≥ 1 (pois p ⊂ (t)). Agora f = tn · u ∈ p ⇒ t ∈ p ⇒ (t) ⊂ p e como (t) é maximal
devemos ter p = (t).

Séries formais possuem inúmeras aplicações. Por exemplo, elas podem ser utilizadas para “resolver”
recursões, como mostra o seguinte

Exemplo 3.1.3 (Sequência de Fibonacci) A sequência de Fibonacci Fn é a sequência definida re-


cursivamente por
F0 = 0, F1 = 1 e Fn = Fn−1 + Fn−2 para n ≥ 2

Assim, seus primeiros termos são

F0 F1 F2 F3 F4 F5 F6 F7 F8 ···
0 1 1 2 3 5 8 13 21 ···

Para encontrar uma fórmula explı́cita para Fn , considere o elemento S = F0 +F1 t+F2 t2 +F3 t3 +· · · ∈ R[[t]].
Temos
S = F0 + F1 t + F2 t2 + F3 t3 + F4 t3 + · · ·
S·t= F0 t + F1 t2 + F2 t3 + F3 t3 + · · ·
S · t2 = F0 t2 + F1 t3 + F2 t3 + · · ·

e como Fn = Fn−1 + Fn−2 para n ≥ 2 temos portanto

t
(1 − t − t2 ) · S = F0 + (F1 − F0 )t ⇐⇒ S =
1 − t − t2
FT
√ √
Agora sejam α = 1+2 5 e β = 1−2 5 , de modo que 1 − t − t2 = (1 − αt)(1 − βt). Utilizando “frações
1
parciais” e a fórmula da “soma da progressão geométrica” 1 + αt + αt2 + · · · = 1−αt temos portanto
)
28

t 1  1 1  X αn − β n n
7:

S= 2
= · − = ·t
1−t−t α−β 1 − αt 1 − βt α−β
(1
RA

n≥0
0
01

αn −β n
Assim, comparando coeficientes, temos que Fn = α−β para todo n ≥ 0.
,2

3.2 Inteiros Algébricos


29

Uma grande fonte de anéis comutativos é a Teoria Algébrica dos Números. O conceito central aqui é o
D
ct

de inteiro algébrico:
,O

Definição 3.2.1 Dizemos que θ ∈ C é um inteiro algébrico se θ é raiz de um polinômio mônico


ET

p(x) ∈ Z[x] (lembrando: um polinômio mônico é aquele cujo coeficiente lı́der é igual a 1).
√ √
Por exemplo, os números α = 1+2 5 e β = 1−2 5 do exemplo anterior são inteiros algébricos pois
são raı́zes do polinômio mônico com coeficientes inteiros x2 − x − 1 = 0. Inteiros algébricos generalizam
o conceito de inteiro; uma das motivações para esta definição é o seguinte lema, que caracteriza os
elementos de Z como sendo exatamente os inteiros algébricos que moram dentro de Q:
16 Vamos ser amigos dos anéis!

Lemma 3.2.2 Seja θ ∈ Q uma raiz de um polinômio mônico com coeficientes inteiros

f (x) = xn + cn−1 xn−1 + · · · + c0 , ci ∈ Z

Então θ ∈ Z.

Prova Escreva θ = a/b com a, b ∈ Z primos entre si. Substituindo x = θ em f (x) e limpando os
denominadores obtemos
an + cn−1 an−1 b + cn−2 an−2 b2 + · · · + c0 bn = 0

Como b divide todos os termos a partir do segundo, temos que b divide an também. Mas como a e b são
primos entre si temos que a única possibilidade para que isto ocorra é b = ±1, logo θ = ±a ∈ Z.

Se θ é um inteiro algébrico, raiz de um polinômio mônico f (x) = xn + cn−1 xn−1 + · · · + c0 ∈ Z[x]


de grau n, temos que o conjunto

def
Z[θ] = {a0 + a1 θ + · · · + an−1 θn−1 | ai ∈ Z}

é um subanel de C. De fato, este conjunto é fechado por soma e também por produto, já que aplicando
várias vezes a relação

f (θ) = 0 ⇐⇒ θn = −cn−1 θn−1 − · · · − c0 ⇒ θn+i = −cn−1 θn+i−1 − · · · − c0 θi

para i ≥ 0 podemos escrever qualquer potência em θ de grau maior ou igual a n em termos de potências
de grau menor do que n. Note a importância do fato de f (x) ser mônico, o que dispensa a necessidade
de dividir a relação acima pelo coeficiente lı́der de f (x). Conclusão: Z[θ] é um anel que é finitamente
gerado sobre Z como Z-módulo, algo que não se vê todo dia! Anéis como Z[θ] têm diversas aplicações
em Teoria dos Números, vejamos uma:

Exemplo 3.2.3 Seja Fn o n-ésimo número de Fibonacci. Vamos mostrar que

m | n ⇒ Fm | Fn

Suponha que m | n, digamos n = mk com k ∈ Z. Utilizando a fórmula explı́cita de Fn encontrada no


exemplo anterior, obtemos
FT
Fn Fkm αkm − β km
= = m = (αm )k−1 + (αm )k−2 (β m ) + (αm )k−3 (β m )2 + · · · + (β m )k−1 (∗)
Fm Fm α − βm
)
28

def
Agora observe que tanto α como β = 1 − α são elementos do anel Z[α] = {a + bα | a, b ∈ Z}. Este anel
7:

tem uma propriedade muito interessante: todo elemento de Z[α] é um inteiro algébrico: como α+ β = 1 e
(1
RA

αβ = −1, multiplicando pelo “conjugado” temos que a + bα é raiz do polinômio mônico com coeficientes
0

inteiros
01

 
x − (a + bα) · x − (a + bβ) = x2 − (2a + b)x + (a2 + ab − b2 )
,2

Desta forma, de (∗) temos que FFm n


é um elemento de Z[α] e, portanto, é um inteiro algébrico. Mas um
29

número racional que é um inteiro algébrico deve ser inteiro, ou seja, FFm
n
∈ Z ⇐⇒ Fm | Fn , o que encerra
D
ct

a prova.
,O

Utilizando o fato que Z é um domı́nio de ideais principais, é fácil verificar que


ET

Spec Z = {(0)} ∪ {(p) | p é um número primo}

e que, com exceção de (0), todo ideal primo é maximal. Para encontrar o espectro de anéis mais
complicados como Z[α] acima, podemos utilizar o mapa f : Spec Z[α] → Spec Z associado à inclusão
Z ֒→ Z[α], como mostra o seguinte
17

Exemplo 3.2.4 Seja Z[α] e f : Spec Z[α] → Spec Z como acima. Vamos descrever Spec Z[α], mostrando
em particular que, como em Z, todos os ideais primos com exceção de (0) são maximais. Dado q ∈
Spec Z[α], observe inicialmente que f (q) = q ∩ Z, de modo que temos dois casos a considerar:
• f (q) = (0) ⇐⇒ q ∩ Z = (0). Neste caso, provaremos que q = (0). Para isto, supomos que q 6= (0) e
vamos mostrar que q ∩ Z 6= (0). Seja a + bα ∈ q um elemento não nulo. Multiplicando pelo “conjugado”
a + bβ, obtemos como no exemplo anterior que (a + bα)(a + bβ) = a2 + ab − b2 ∈ q ∩ Z, que é um elemento
não nulo já que a + bα 6= 0 e a + bβ 6= 0.
• f (q) = (p) ⇐⇒ q ∩ Z = (p) para algum número primo p. Neste caso, como q ⊃ (p), pela corre-
spondência de ideais, temos que determinar Spec Z[α]/(p). Como Z[α] ∼ = Z[x]/(x2 − x − 1), temos que

Z[α]/(p) = Fp [x]/(x − x − 1) e assim temos alguns subcasos, de acordo com a fatoração de x2 − x − 1 em
2

Fp [x]. Se x2 − x − 1 é irredutı́vel em Fp [x], temos Fp [x]/(x2 − x − 1) é um corpo e assim (p) é maximal


em Z[α]. Por outro lado, se x2 − x − 1 = (x − ā)(x − b̄) é redutı́vel em Fp [x] com fatores distintos, então
pelo Teorema Chinês dos Restos temos que

Fp [x] ∼ Fp [x] Fp ∼
= × = Fp × Fp
(x2 − x − 1) (x − ā) (x − b̄)

é o produto de dois corpos, de modo que seus ideais primos são os maximais (0) × Fp e Fp × (0), que
correspondem aos ideais maximais (x̄ − ā) e (x̄ − b̄) em Fp [x]/(x2 − x − 1), ou seja, aos ideais maximais
(α − a, p) e (α − b, p) em Z[α], onde a, b ∈ Z são levantamentos de ā, b̄ ∈ Fp . Finalmente, se x2 − x − 1 tem
raı́zes múltiplas em Fp [t], o que ocorre quando (x2 − x − 1, 2x − 1) = (1) em Fp [t] (critério da derivada),
isto é, quando p = 5 (verifique!), então F5 [x]/(x2 − x − 1) ∼
= F5 [x]/(x − 3̄)2 . Se (p(x)) é um ideal primo
de F5 [x] contendo (x − 3̄) , então p(x) | (x − 3̄) , de modo que devemos ter (p(x)) = (x − 3̄). Assim,
2 2

F5 [x]/(x2 − x − 1) tem um único ideal primo (x̄ − 3̄), que é maximal e corresponde ao ideal maximal
(α − 3, 5) de Z[α].
Resumimos os casos acima no seguinte diagrama esquemático, onde os ideais maximais são representados
por pontos e os ideais (0) pelas linhas cheias (a escolha desta representação, que deve ficar mais clara
posteriormente, foi feita em analogia com os “anéis geométricos” correspondentes das seções seguintes):

(11, α − 4)

(2) (3) (7)


(α − 3, 5) Spec Z[α]

f (11, α + 3)
FT
Spec Z
)
(5) (2) (3) (7) (11)
28
7:

f : Spec Z[α] → Spec Z


(1
RA
0

Observação 3.2.5 Temos que x2 − x − 1 é redutı́vel em Fp [t] se, e só se, seu discriminante ∆ =
01

2
1 − 4 · (−1) = 5 for um quadrado perfeito em Fp . Utilizando a lei de reciprocidade quadrática (ver
,2

por exemplo J.-P. Serre,  Arithmetic”, capı́tulo I), temos que isto ocorre para p 6= 2, 5
 “A course in
29

exatamente quando p5 = 1 ⇐⇒ 5p = 1 ⇐⇒ p ≡ ±1 (mod 5).


D
ct

3.3 Anel de funções contı́nuas e anel de funções holomorfas


,O

Análise e Topologia são duas outras grandes fontes de anéis comutativos. Dado um espaço topológico
ET

X, podemos considerar o seu anel de funções contı́nuas reais

def
C(X) = {f : X → R | f é contı́nua }

onde a soma e o produto são os usuais de funções. Note que C(X)× consiste nas funções contı́nuas
f : X → R que não se anulam em nenhum ponto de X (o inverso de f é 1/f , que é contı́nua). Em geral,
18 Vamos ser amigos dos anéis!

C(X) não é um domı́nio (por exemplo, se X = R podemos encontrar funções não identicamente nulas f
e g que se anulam em (−∞, 1) e (0, ∞), respectivamente, de modo que f g é identicamente nula).
Muitas propriedades geométricas de X refletem-se em propriedades algébricas de C(X). Por exemplo,
cada ponto P ∈ X define um ideal mP de C, o ideal das funções que se anulam em P :

def
mP = {f ∈ C | f (P ) = 0}

Este ideal é um ideal maximal de C(X), já que o mapa “avaliação em P ”

C(X) → R
f 7→ f (P )

é sobrejetor (tome funções constantes por exemplo) e tem kernel mP , de modo que C(X)/mP ∼ = R, que
é um corpo, mostrando que mP é maximal.
Se Y é outro espaço topológico e φ: X → Y é uma função contı́nua, temos um morfismo induzido
de anéis na direção oposta, dado por composição com φ:

φ# : C(Y ) → C(X)
g 7→ g ◦ φ

Note que o morfismo de espectros associado Spec(φ# ): Spec C(X) → Spec C(Y ) leva mP em (φ# )−1 mP =
mφ(P ) .
Adicionalmente, se supusermos que X compacto, podemos mostrar que todo ideal maximal de C(X)
é da forma mP para algum P ∈ X. De fato, basta mostrar que para todo ideal próprio a de C(X) existe
um ponto P ∈ X tal que a ⊂ mP . Suponha, por absurdo, que tal não aconteça, isto é, que para cada
ponto P ∈ X podemos encontrar uma função aP ∈ a tal que aP (P ) 6= 0. Seja UP uma vizinhança aberta
de P onde aP não se anula (aqui usamos a continuidade de aP ). Como X é compacto, existe um número
finito de pontos P1 , . . . , Pn tais que a união dos UPi cobrem X. A função a2P1 + · · · + a2Pn ∈ a e não se
anula em nenhum ponto de X, o que é um absurdo, pois a2P1 + · · · + a2Pn é então uma unidade mas a é
próprio.
Desta forma, para X compacto e Hausdorff temos uma bijeção

{pontos P de X} ↔ {ideais maximais mP de C(X)}

relacionando a álgebra de C(X) com a geometria de X! Já mostramos no parágrafo anterior que P 7→ mP
FT
é sobrejetor; por outro lado, a injetividade segue do lema de Urysohn (X é normal), que garante a
existência de uma função contı́nua f : X → R tal que f (P ) = 0 mas f (Q) 6= 0 para dois pontos P 6= Q
)
dados, de modo que P 6= Q ⇒ mP 6= mQ . Veremos mais tarde que os ideais mP não esgotam todos os
28

ideais primos de C(X).


7:

Além disso, como vimos antes, se φ: X → Y é um mapa contı́nuo com Y também compacto e
Hausdorff, temos que o mapa de espectros associado Spec(φ# ): Spec C(X) → Spec C(Y ) se restringe aos
(1
RA

subconjuntos dos ideais maximais de X e Y , levando mP 7→ mφ(P ) . Assim, a bijeção acima permite
0

recuperar o mapa φ a partir do morfismo de anéis φ# !


01

Temos também uma versão “analı́tica” do exposto acima: se U é um subconjunto aberto de C,


,2

definimos os anéis
H(U ) = {f : U → C | f é uma função holomorfa}
29

M(U ) = {f : U → C ∪ {∞} | f é uma função meromorfa}


D
ct

Assim, H(U ) é um subanel de M(U ). Estes anéis são muito mais bem comportados do que o anel de
,O

funções contı́nuas reais; por exemplo, se U é conexo, então H(U ) é um domı́nio e M(U ) é um corpo. De
fato, se f, g ∈ H(U ) são tais que f (z)g(z) = 0 para todo z ∈ U mas f (z0 ) 6= 0 para algum z0 ∈ U então
ET

por continuidade f (z) 6= 0 para todo z em alguma vizinhança aberta V de z0 e assim g(z) = 0 para
todo z ∈ V . Mas zeros de funções analı́ticas não nulas em abertos conexos são isolados, logo g deve ser
identicamente 0 em U . De maneira análoga, prova-se que M(U ) é um corpo. Utilizando-se o teorema de
fatoração de Weierstraß (veja por exemplo J. B. Conway, “Functions of One Complex Variable”, VII.§5),
pode-se inclusive mostrar que M(U ) é o corpo de frações de H(U ).
19

3.4 Conjuntos Algébricos Afins conjuntos alg

Dados polinômios f1 , . . . , fd ∈ C[x1 , . . . , xn ], podemos considerar o conjunto de seus zeros comuns


def
V = V (f1 , . . . , fn ) = {(a1 , . . . , an ) ∈ Cn | fi (a1 , . . . , an ) = 0 para todo i = 1, . . . , d}
Subconjuntos de Cn da forma acima são chamados de conjuntos algébricos. Note que podemos
também tomar o ideal I = (f1 , . . . , fn ) e considerar
def
V = V (I) = {(a1 , . . . , an ) ∈ Cn | f (a1 , . . . , an ) = 0 para todo f ∈ I},
o que fornece uma definição equivalente para conjuntos algébricos em termos de ideais de C[x1 , . . . , xn ]
(veremos mais tarde que todo ideal deste anel é finitamente gerado pelo teorema da base de Hilbert).
Exemplos familiares de conjuntos algébricos incluem o espaço todo Cn (definido pelo polinômio nulo),
qualquer ponto (a1 , . . . , an ) ∈ Cn (definido pelos polinômios x1 − a1 , . . . , xn − an ), retas e cı́rculos no
“plano complexo” C2 (como os dados pelas equações x = y e x2 + y 2 = 1), entre muitos outros.
Note que se p, q ∈ C[x1 , . . . , xn ] são tais que p ≡ q (mod (f1 , . . . , fd )) então p(a1 , . . . , an ) =
q(a1 , . . . , an ) para todos os pontos (a1 , . . . , an ) ∈ V . Assim, os elementos do quociente
def
A = A(f1 , . . . , fd ) = C[x1 , . . . , xn ]/(f1 , . . . , fd )
podem ser vistos como funções polinomiais de V em C. Por outro lado, como na seção anterior, um
ponto P = (a1 , . . . , an ) ∈ V define um ideal maximal
def
mP = {a ∈ A | a(P ) = 0}
das funções polinomiais em V que se anulam em P . Aqui, por outro lado, podemos dar uma descrição
muito mais explı́cita de mP , como sendo o ideal gerado pelas funções xi − ai ∈ A:
mP = (x1 − a1 , . . . , xn − an )
De fato, por um lado é claro que xi − ai ∈ mP de modo que mP ⊃ (x1 − a1 , . . . , xn − an ). Para mostrar
a inclusão oposta, seja f ∈ mP , representado por um polinômio em C[x1 , . . . , xn ] que ainda denotamos
por f . Temos que
f (x1 , . . . , xn ) ≡ f (a1 , . . . , an ) (mod x1 − a1 , . . . , xn − an )
e como f (a1 , . . . , an ) = 0 por hipótese, temos que f ∈ (x1 − a1 , . . . , xn − an ), o que encerra a prova.
Dados dois pontos distintos de V , que diferem em suas primeiras coordenadas a1 e a′1 digamos,
temos que x1 − a1 ∈ A estará no ideal do primeiro ponto mas não na do segundo. Assim, a associação
V → {ideais maximais de A}
(a1 , . . . , an ) 7→ (x1 − a1 , . . . , xn − an )
é injetiva. Veremos mais tarde (pelo Nullstellensatz Hilberts) que esta associação é na verdade uma
bijeção, e assim como na seção anterior novamente testemunhamos uma estreita relação entre a álgebra
FT
do anel A e a geometria do conjunto V !
Lemma 3.4.1
)
28

1. O conjunto
1 n o

∈ C(x) λ ∈ C
7:

x−λ
(1
RA

é linearmente independente sobre C. Em particular, C(x) possui dimensão incontável sobre C.


2. Se V é um espaço vetorial de dimensão contável sobre C e T : V → V é um operador linear,
0
01

então existe λ ∈ C tal que T − λ não é bijetor em V .


P
,2

ai
Prova Se temos uma relação de dependência linear 1≤i≤n x−λi = 0, multiplicando por x − λ1 e
substituindo x = λ1 concluı́mos que a1 = 0; repetindo este procedimento, obtemos ai = 0 para todo i.
29

Para provar (2), suponha por absurdo que T − λ é bijetor para todo λ ∈ C. Tome qualquer vetor v ∈ V
D
ct

não nulo e considere o conjunto


{(T − λ)−1 v ∈ V | λ ∈ C}
,O

Como este conjunto é incontável e a dimensão de V é contável, existe uma relação de dependência linear
ET

X
ai (T − λi )−1 v = 0
1≤i≤n
P
Seja f (x) = 1≤i≤n ai (x − λi ) ∈ C(x), que é não nulo pelo item (1). Escrevendo f (x) = p(x)/q(x)
−1

com p(x), q(x) ∈ C[x] e fatorando estes polinômios em termos lineares, temos que p(T ) e q(T ) são
transformações lineares bijetoras em V , logo p(T )q(T )−1 v = 0 implica v = 0, uma contradição.
20 Vamos ser amigos dos anéis!

Teorema 3.4.2 (Nullstellensatz) Todo ideal maximal de C[x1 , . . . , xn ] é da forma (x1 − a1 , . . . , xn −


an ) com ai ∈ C.
Prova Seja m um ideal maximal de C[x1 , . . . , xn ]. Temos que V = C[x1 , . . . , xn ]/m é um corpo que tem
dimensão contável sobre C (pois isto já vale antes do quociente). Assim, para o operador linear em V
dada pela multiplicação por xi existe ai ∈ C tal que a multiplicação por xi − ai não é bijetora em V , o
que só é possı́vel se xi − ai = 0 já que V é um corpo. Logo m ⊃ (x1 − a1 , . . . , xn − an ) e devemos ter a
igualdade já que ambos são ideais maximais.

Mais ainda: dados dois conjuntos algébricos V ⊂ Cn e W ⊂ Cm , definidos por ideais I ⊂


C[x1 , . . . , xn ] e J ⊂ C[y1 , . . . , ym ], digamos, podemos considerar os mapas polinomiais φ: V → W entre V
e W , isto é, mapas que são da forma φ(a) = (p1 (a), . . . , pm (a)), a ∈ V , onde p1 , . . . , pm ∈ C[x1 , . . . , xn ]
são polinômios fixos tais que

g(y1 , . . . , ym ) ∈ J ⇒ g p1 (x1 , . . . , xn ), . . . , pm (x1 , . . . , xn ) ∈ I (∗)

Note que a última condição (∗) garante que (p1 (a), . . . , pm (a)) ∈ W para todo a ∈ V . Esta condição só
precisa ser verificada num conjunto de geradores de J, que é finito (pelo teorema da base de Hilbert, a
ser provado mais tarde). Como no caso dos anéis de funções contı́nuas, associado a φ temos um morfismo
de anéis dado pela composição com φ:

φ# : A(J) = C[y1 , . . . , yn ]/J → A(I) = C[x1 , . . . , xn ]/I


g 7→ g ◦ φ

que neste caso possui uma descrição muito mais explı́cita: φ# é o morfismo de C-álgebras dado por y i 7→
pi (x1 , . . . , xn ). Novamente a condição (∗) garante que este morfismo está bem definido. O morfismo de
espectros correspondente Spec(φ# ): Spec A(I) → A(J) se restringe aos subconjuntos de ideais maximais
de A(I) e A(J) e leva mP 7→ mφ(P ) , de modo que pela bijeção entre pontos e ideais maximais podemos
recuperar o mapa φ a partir do morfismo de C-álgebras φ# !
Exemplo 3.4.3 Como C[t] é um domı́nio de ideais principais, temos que Spec C[t] é formado por (0) e
por ideais principais gerados por polinômios irredutı́veis em C[t], ou seja,

Spec C[t] = {(0)} ∪ {(t − a) | a ∈ C}

Note que obtemos desta forma uma bijeção a ↔ (t − a) entre pontos a ∈ C da “reta complexa”e ideais
maximais (t − a) ⊂ C[t] do anel de funções polinomiais nesta reta. Como acima, (t − a) é o conjunto das
FT
funções polinomiais que se anulam no ponto a. Representando os ideais maximais por pontos e o ideal
(0) pela linha cheia, temos o seguinte diagrama esquemático:
)
28

(t + 2) (t) (t − 1)
← (0)
7:

−2 0 1
(1
RA

Spec C[t]
0
01
,2

Exemplo 3.4.4 Vamos determinar Spec C[x, y]/(y 2 − x3 + x). Para isto considere o morfismo de C-
álgebras φ: C[x] → C[x, y]/(y 2 − x3 + x) dado por φ(x) = x. Note que este morfismo é injetor (nenhum
29

polinômio na variável x apenas pode ser múltiplo de y 2 − x3 + x), de modo que podemos√ pensar em
D

C[x] como subanel de C[x, y]/(y 2 − x3 + x) e que este último é obtido “adicionando-se x3 − x a C[x]”.
ct

Observe ainda que como y 2 − x3 + x é um polinômio irredutı́vel no domı́nio de fatoração única C[x, y],
,O

temos que (y 2 − x3 + x) ⊂ C[x, y] é um ideal primo e C[x, y]/(y 2 − x3 + x) é um domı́nio. Utilizando a


relação y2 = x3 − x, todo elemento de C[x, y]/(y 2 − x3 + x) pode ser representado unicamente como um
ET

polinômio p(x) + q(x)y de grau no máximo 1 em y.


Agora seja f : Spec C[x, y]/(y 2 − x3 + x) → Spec C[x] o morfismo associado a φ. Como no exemplo 3.2.4,
temos dois casos considerar:
• f (q) = (0) ⇐⇒ q ∩ C[x] = (0). Como no exemplo 3.2.4, vamos mostrar  que q = (0). Se q 6= (0)
e a(x) + b(x)y ∈ q é um elemento não nulo, temos que a(x) + b(x)y · a(x) − b(x)y = a(x)2 −
21

b(x)2 (x3 − x) ∈ q ∩ C[x] é um elemento não nulo, pois é o produto de dois elementos não nulos no elementos hom
domı́nio C[x, y]/(y 2 − x3 + x).
• f (q) = (x − a) ⇐⇒ q ∩ C[x] = (x − a) para a ∈ C. Como no exemplo 3.2.4, devemos calcular
Spec C[x, y]/(y 2 − x3 + x, x − a). Seja b ∈ C tal que b2 = a3 − a, de modo que C[x, y]/(y 2 − x3 +
x, x − a) ∼ = C[y]/(y 2 − a3 + a) = C[y]/(y 2 − b2 ). Se b 6= 0, temos pelo Teorema Chinês dos Restos que
C[y]/(y − b2 ) ∼
2
= C[y]/(y − b) × C[y]/(y + b) ∼= C × C, que possui dois ideais primos (0) × C e C × (0),
correspondendo aos ideais primos (e maximais) (y − b, x − a) e (y + b, x − a) do anel original. Se b = 0
(isto é, se a3 − a = 0 ⇐⇒ a = 0 ou a = ±1) então só há um primo (y) em C[y]/(y 2 ), que corresponde
ao ideal primo (e maximal) (y, x − a).
Resumindo: Spec C[x, y]/(y 2 −x3 +x) consiste no ideal (0) e nos ideais maximais (x−a, y−b), exatamente
um para cada ponto (a, b) ∈ C2 da curva complexa y 2 = x3 − x. Novamente representado cada ideal
maximal pelo ponto correspondente da curva e o ideal (0) por toda a curva, temos que f corresponde
à “projeção no eixo x” como mostra o seguinte diagrama esquemático (compare com o diagrama no
exemplo 3.2.4):

(a, b) (x − a, y − b)
f
Spec C[x]
(x − a)

Spec C[x, y]/(y 2 − x3 + x)

3.5 Anéis graduados


Ao contrário da crença popular, anéis graduados não são aqueles com um diploma. A origem do nome
anel graduado seria mais clara se ele fosse rebatizado “anel grau-duado”, mas até os matemáticos estão
sob a égide das regras ortográficas. . .
Definição 3.5.1 Um anel Z-graduado (ou simplesmente anel graduado é um anel A cujo grupo
aditivo admite uma decomposição M
FT
A= Ad
d∈Z
)
como soma direta de subgrupos abelianos Ad tal que
28


ai ∈ Ai
7:

⇒ ai aj ∈ Ai+j
aj ∈ Aj
(1
RA

para todo i, j ∈ Z. Os elementos de ad ∈ Ad ⊂ A são chamados de elementos homogêneos de grau


0
01

d. Um morfismo de anéis φ: A → B entre dois anéis graduados é um morfismo de anéis graduados


se φ respeita a graduação: φ(Ad ) ⊂ Bd para todo d ∈ Z.
,2

Um A-módulo Z-graduado M é um A-módulo cujo grupo aditivo é uma soma direta de subgrupos
29

abelianos M
M= Md
D
ct

d∈Z
,O

de modo que 
ai ∈ Ai
ET

⇒ ai mj ∈ Mi+j
mj ∈ M j
para todo i, j ∈ Z. Como no caso de anéis, os elementos em Md são chamados de homogêneos de grau d
e um morfismo de módulos graduados é um morfismo de módulos que respeita a graduação.
Se Ad = 0 para d < 0 diremos que A é um anel N-graduado. Analogamente definimos módulos
N-graduados.
22 Vamos ser amigos dos anéis!

O exemplo canônico de anel graduado é o anel de polinômios k[x1 , . . . , xn ] sobre um corpo k, que ideal homogêneo
série de Hilbert
admite a graduação M
k[x1 , . . . , xn ] = k[x1 , . . . , xn ]d
d∈Z

onde k[x1 , . . . , xn ]d = 0 se d < 0 e k[x1 , . . . , xn ]d é o k-espaço vetorial de dimensão n+d−1
d gerado pelos
monômios xe11 xe22 . . . xenn de grau d = e1 + · · · + en para d ≥ 0.
Um ideal a de A que é graduado como A-módulo, ou seja, admite a decomposição
M
a= (a ∩ Ad )
d∈Z

é chamado de ideal homogêneo. Se a é homogêneo, o anel quociente A/a também é graduado de


maneira natural pois podemos escrever
A M Ad
=
a a ∩ Ad
d∈Z

L
Lema 3.5.2 (Ideais Homogêneos) Seja A = Ad um anel graduado e a um ideal de A. As seguintes
condições são equivalentes:
1. a é um ideal homogêneo;
P
2. a é tal que um elemento a = ad ∈ A com ad ∈ Ad pertence a a se, e somente se, cada
componente homogênea ad pertence a a;
3. a é gerado por elementos homogêneos (possivelmente de graus diferentes).

Prova Claramente temos 2 ⇐⇒ 1 ⇒ 3. Para mostrar que 3 ⇒ 1, suponha que a é gerado por
elementos homogêneos
P fi e seja a ∈ a. Então podemos escrever a = gi1 · fi1 + · · · + gin · fin . Expandindo
cada gi = d gi,d como soma de suas componentes homogêneas, temos que o termo de grau d em a é
dado por ad = gi1 ,d−deg(fi1 ) · fi1 + · · · + gin ,d−deg(fin ) · fin e assim ad ∈ a.

Muitas propriedades de ideais homogênos podem ser verificadas testando-se apenas elementos ho-
mogêneos. Por exemplo, temos o seguinte
L
Lemma 3.5.3 Seja A = Ad um anel graduado. Um ideal homogêneo próprio p de A é primo se, e
só, ab ∈ p ⇐⇒ a ∈ p ou b ∈ p para todos os elementos homogêneos a, b ∈ A.
L
Prova P Sejam a, Pb ∈ A = Ad elementos arbitrários e escreva-os como soma de elementos homogêneos:
FT
a= ad , b = bd com ad , bd ∈ Ad . Suponha por absurdo que ab ∈ p mas a ∈ /peb∈ / p; sejam m e n
os menores inteiros para os quais am ∈ / p e bn ∈
/ p. O termo de grau m + n em ab é
)
28

· · · + am−2 bn+2 + am−1 bn+1 + am bn + am+1 bn−1 + am+2 bn−2 + · · ·


7:
(1
RA

Como p é homogêneo e ab ∈ p, temos que esta soma também pertence a p. Porém cada termo ai bj com
i < m ou j < n pertence a p, logo am bn ∈ p, uma contradição.
0
01

Definição 3.5.4 Seja k um corpo e seja A = k[x1 , . . . , xn ]. Seja M um A-módulo graduado finitamente
,2

gerado. A série de Hilbert de M é definida como


29

X
HM (t) = (dimk Md ) · td ∈ Z[[t]]
D
ct

d≥0
,O

Teorema 3.5.5 (Hilbert) Com a notação acima, temos


ET

p(t)
HM (t) =
(1 − t)d

para algum polinômio p(t) ∈ Z[t].


23

3.6 Conjuntos Algébricos Projetivos espaço projet

Dado um corpo k, o espaço projetivo Pnkde dimensão n sobre k é definido como o conjunto de
todas as direções no espaço afim k n+1 de dimensão n + 1. Em outras palavras, um ponto em Pnk
pode ser representado como um vetor não nulo (a0 , a1 , . . . , an ) ∈ k n+1 ; dois vetores (a0 , a1 , . . . , an ) e
(b0 , b1 , . . . , bn ) definem o mesmo ponto se eles são homotéticos, isto é, existe um λ ∈ k não nulo tal
que ai = λbi para i = 0, 1, . . . , n. Representamos o ponto definido pelo vetor (a0 , a1 , . . . , an ) através da
sugestiva notação (a0 : a1 : . . . : an ).
Por exemplo, temos que a reta projetiva pode ser decomposta como

P1k = {(1 : a1 ) | a1 ∈ k} ∪ {(0 : 1)}

pois se a0 6= 0 então (a0 : a1 ) = (1 : aa01 ) e se a0 = 0 então (0 : a1 ) = (0 : 1). Assim, a reta projetiva


consiste de uma “reta afim”, composta pelos pontos da forma (1 : a1 ), e mais um “ponto no infinito”
(0 : 1). Da mesma forma, temos que o plano projetivo

P2k = {(1 : a1 : a2 ) | (a1 , a2 ) ∈ k 2 } ∪ {(0 : a1 : a2 ) | a1 6= 0 ou a2 6= 0}

é a união de um “plano afim” (primeiro termo, já que (1 : a1 : a2 ) = (1 : a′1 : a′2 ) ⇐⇒ a1 = a′1 e
a2 = a′2 ) e uma reta projetiva no “infinito” (segundo termo). Note que a escolha de “reta no infinito” é
completamente arbitrária, pois poderı́amos tomar uma outra decomposição, por exemplo

P2k = {(a0 : a1 : 1) | (a0 , a1 ) ∈ k 2 } ∪ {(a0 : a1 : 0) | a0 6= 0 ou a1 6= 0}

e agora os pontos com a2 = 0 formam a “reta no infinito”.


Agora falaremos um pouco sobre curvas algébricas planas. No plano afim k 2 , temos que qualquer
polinômio p(x, y) ∈ k[x, y] define uma curva

C = {(a, b) ∈ k 2 | p(a, b) = 0}

(que pode eventualmente degenerar em um ponto, em todo o plano, ou mesmo no conjunto vazio, mas
não vamos nos preocupar com estes detalhes agora). Porém, no mundo projetivo só podemos considerar
polinômios homogêneos, isto é, polinômios cujos monômios têm todos o mesmo grau. De fato, se
p(x0 , x1 , x2 ) ∈ k[x0 , x1 , x2 ] é homogêneo de grau d então temos que

p(a0 , a1 , a2 ) = 0 ⇒ p(λa0 , λa1 , λa2 ) = λd p(a0 , a1 , a2 ) = 0.

Assim, faz sentido dizer quando um polinômio homogêneo p(x0 , x1 , x2 ) se anula em uma classe de vetores
FT
homotéticos e podemos considerar a curva projetiva definida por p(x0 , x1 , x2 ):
)
C = {(a0 : a1 : a2 ) ∈ P2k | p(a0 , a1 , a2 ) = 0}.
28
7:

Por exemplo temos que x0 = 0 é uma equação da “reta no infinito” descrita acima. Temos que para
(1
RA

qualquer (a, b, c) 6= (0, 0, 0) a equação ax0 + bx1 + cx2 = 0 define uma reta projetiva em P2k ; esta reta é
a união de uma reta afim de equação a + bx + cy = 0 (x0 6= 0) e de um “ponto no infinito” (0 : −c : b)
0

(supondo b 6= 0 ou c 6= 0), intersecção das retas ax0 + bx1 + cx2 = 0 e x0 = 0.


01

Em geral, duas retas distintas 


,2

ax0 + bx1 + cx2 = 0


29

dx0 + ex1 + f x2 = 0
D

possuem exatamente um ponto de intersecção, pois a solução do sistema linear homogêneo acima é
ct

1 dimensional (não pode ser 2 dimensional, pois neste caso as retas seriam coincidentes), logo define
,O

exatamente um ponto em P2k (ou seja, uma direção em k 3 ).


ET

Definição 3.6.1 Um conjunto algébrico projetivo é um subconjunto de Pn definido por zeros de


polinômios homogêneos, ou seja, um subconjunto da forma

{(a0 : . . . : an ) ∈ Pn | f (a0 , . . . , an ) = 0 para todo f ∈ a}

onde a é um ideal homogêneo de k[x0 , x1 , . . . , xn ].


24 Vamos ser amigos dos anéis!

Blow-up topologia de Zariski


Se a é a homogeneous ideal of R = k[X0 , . . . , Xn ], we can extend the definition of V (a) as the set
of common zeros of todo homogeneous polinômios in a. Então, as before, we may replace a por the
homogeneous ideal it generates. In the same way, the projective algébrico sets satisfy the axioms para
closed sets, e define a topology on Pn which we also call the Zariski topology. Algum things são
different, though: paraP instance, se e R+ é its irrelevant ideal, então V (R+ ) = ∅, even though R+ 6= R.
More generally, se a = d≥d0 Rd para algum d0 > 0, então V (a) = ∅, como a contains the homogeneous
polinômios X0d0 , . . . , Xnd0 , whose set of common zeros consists of the single tuple (0, . . . , 0), which é not
a ponto of Pn .

Definição 3.6.2 An irreducible projective algébrico set é called a projective variety.

4 Topologia de Zariski
Nos exemplos anteriores de “anéis de funções”, os ideais maximais puderam ser identificados com os
pontos do espaço considerado. Por analogia, para um anel qualquer A, queremos considerar os elementos
de A como “funções” no “espaço” Spec A de todos os ideais primos (e não só os maximais). Para uma
“função” f ∈ A, vamos apenas definir quando f se “anula” num “ponto” p ∈ Spec A: f se anula em p
se, e só se, f ∈ p (o ideal p é o conjunto de todas as “funções” que se anulam neste “ponto”). Neste
espı́rito, temos a seguinte

Definição 4.1 Seja a ⊂ A um ideal. Definimos a variedade cortada por a como sendo o subconjunto
de Spec A dado por
def
V (a) = {p ∈ Spec A | p ⊃ a}

Seja h ∈ A um elemento. Definimos o domı́nio de 1/h como o subconjunto de Spec A dado por

def
D(h) = {p ∈ Spec A | p 6∋ h}

Para aproximar Spec A um pouco mais de um objeto geométrico, definimos agora uma topologia
neste espaço:

Teorema 4.2 (Topologia de Zariski) Seja A um anel. Então


1. V ((0)) = Spec A e V ((1)) = ∅;
2. V (a) ∪ V (b) = V (ab);
T P
3. λ V (aλ ) = V ( λ aλ );
FT
Desta forma, os conjuntos da forma V (a) definem os fechados de uma topologia em Spec A, denominada
)
topologia de Zariski.
28

Prova O primeiro item é claro. Para o item 2, devemos mostrar que para p ∈ Spec A temos p ⊃ ab ⇐⇒
7:

p ⊃ a ou p ⊃ b. A implicação ⇐ é imediata; por outro lado, se p ⊃ ab mas p 6⊃ a e p 6⊃ b, então existem


(1
RA

a ∈ a \ p e b ∈ b \ p, mas como p é primo, ab ∈ ab T


\ p, uma contradição, o que completa a prova do item 2.
0

Finalmente, para P o item 3 basta notar


P que p ∈ λ V (aλ ) se, e só se, p ⊃ aλ para todo λ, o que ocorre
01

se, e só se, p ⊃ λ aλ ⇐⇒ p ∈ V ( λ aλ ).


,2

Vejamos algumas propriedades da topologia de Zariski.


29

Lemma 4.3 Seja A um anel. Temos


D
ct

1. Os conjuntos D(h), h ∈ A, formam uma base da topologia de Zariski.


,O

2. D(gh) = D(g) ∩ D(h).


ET

3. se φ: A → B é um morfismo de anéis, Spec(φ): Spec B → Spec A é um mapa contı́nuo.


4. se p ∈ Spec A, temos {p} = V (p) (fecho topológico). Em particular, m ∈ Spec A é um ponto
fechado se, e só se, m é um ideal maximal e se A é um domı́nio, (0) é um ponto denso.
5. Spec A é compacto.
25

S
Prova O primeiro item segue da identidade Spec A \ V (a) = h∈a D(h) (um primo p não contém a se, e
só se, p não contém algum elemento h ∈ a), enquanto que o segundo item é apenas uma reinterpretação
de gh ∈ / p ⇐⇒ g ∈ / peh∈ / p para um primo p. Para mostrar que Spec(φ) é contı́nuo, pelo primeiro
item basta mostrar que a pré-imagem de D(h) é um aberto em Spec A, o que segue de
−1  
Spec (φ) D(h) = D φ(h)
−1  
já que q ∈ Spec (φ) D(h) ⇐⇒ Spec (φ)(q) = φ−1 (q) ∈ D(h) ⇐⇒ q ∈ D φ(h) . Finalmente, o
item 4 segue de X 
\
{p} = V (a) = V a = V (p)
V (a)∋p a⊂p

Agora vamos provar o item 5. Pelo item 1, é suficiente provar que uma cobertura de Spec
PA por uma
famı́lia {D(hα )}, hα ∈ A, tem uma subcobertura finita. Como D(hα ) cobre P Spec A, o ideal α (hα ) não
está contido em
P nenhum ideal primo, logo não pode ser próprio e assim A = S α (hα ). Portanto podemos
escrever 1 = 1≤i≤n ai hαi para hαi e ai ∈ A. Mas isto implica Spec A = 1≤i≤n D(hαi ), o que encerra
a prova.

Exemplo 4.4 Seja k um corpo. Em Spec k × k = {(0) × k, k × (0)}, ambos os ideais são maximais e
portanto fechados, logo todos os subconjuntos de Spec k são fechados e a topologia de Zariski coincide
com a discreta. Por outro lado, em Spec k[[t]] = {(0), (t)} temos que (t) é um ponto fechado enquanto
que (0) é um ponto denso. Assim, os fechados de Spec k[[t]] são o vazio, todo o espectro e {(t)}.
Exemplo 4.5 Em C[t], todos os ideais primos são maximais, logo fechados, à exceção de (0), que é
denso (isto explica nossa escolha de representar o (0) pela linha cheia, “espalhado” por toda a reta).
Assim, os conjuntos fechados de Spec C[t] são os seus subconjuntos finitos e o espectro todo.
De maneira similar, os fechados próprios de Spec C[x, y]/(y 2 − x3 + x) no exemplo 3.4.4 também são
dados pelos subconjuntos finitos deste espectro. Os fechados do espectro Spec Z[α] do exemplo 3.2.4
também possuem a mesma descrição, o que explica a nossa escolha de representá-lo como uma “curva”,
em analogia aos dois casos geométricos anteriores.
Exemplo 4.6 Here é an amusing application: we mostrar que se A e B são n × n matrices com entries
in k, então the characteristic polinômios of AB e BA são equal. In fact, se A é invertible, this follows
from
det(xI − AB) = det(A−1 ) det(xI − AB) det(A) = det(xI − BA)
onde I é the n × n identity matrix. To extend the result to todo matrices, fix B e consider A as a ponto
FT
2
in An . The set
2
V = {A ∈ An | det(xI − AB) = det(xI − BA)}
)
28

é an algébrico set. This can be seen as follows: consider the n × n matrix M = (xij ) whose entries são
7:

indeterminates xij , 1 ≤ i, j ≤ n. Write


(1
RA

det(xI − M B) = xn + cn−1 (xij ) · xn−1 + · · · + c0 (xij )


0

det(xI − BM ) = xn + dn−1 (xij ) · xn−1 + · · · + d0 (xij )


01
,2

onde cl , dl são polinômios in k[xij ]. Então V = V (cn−1 − dn−1 , . . . , c0 − d0 ). Now V contains the non-
29

empty open set D(det M ) consisting of those matrices A com det A 6= 0, i.e., the invertible ones. But
2 2
open sets in the Zariski topology são huge, e in this case, como An é a variety, V é the whole set An .
D
ct

2 2
In fact, this follows from An = V ∪ V (det M ), portanto we must have V = An . This concludes the
,O

proof.
ET

Encerramos esta seção com a “versão projetiva” da topologia de Zariski.


L
Definição 4.7 Seja A = d≥0 Ad um anel N-graduado. O ideal homogêneo

def
M
A+ = Ad
d>0
26 Vamos ser amigos dos anéis!

é chamado de ideal irrelevante. Definimos ideal irrelevante

def
Proj A = {p ∈ Spec A | p é homogêneo e p 6⊃ A+ }

Para todo ideal homogêneo a ⊂ A, definimos a variedade projetiva cortada por a como o subcon-
junto de Proj A dado por
def
V+ (a) = {p ∈ Proj R | p ⊃ a}
Da mesma forma, para todo elemento homogêneo h ∈ A, definimos o domı́nio de 1/h como
def
D+ (h) = {p ∈ Proj R | p 6∋ h}

(esta definição é literalmente demais!)


Como antes, temos que
1. V+ ((0)) = Proj R e V+ ((1)) = ∅.
2. V+ (a) ∪ V+ (b) = V+ (ab)
T P
3. λ V+ (aλ ) = V+ ( λ aλ )
de modo que os conjuntos da forma V+ (a) são fechados de uma topologia em Proj A, também chamada
de topologia de Zariski. Mostra-se da mesma forma que os conjuntos D+ (h) formam uma base de
abertos desta topologia.

5 Exercı́cios
01. Seja A um anel e a ∈ A um elemento nilpotente. Mostre que 1 + a ∈ A× .
02. Encontre fórmulas explı́citas para as seguintes recursões:
(a) G0 = 0, G1 = 1 e Gn+2 = 5Gn+1 − 6Gn para n ≥ 0.
(b) P0 = P1 = 1, P2 = 0 e Pn+3 = 7Pn+1 − 6Pn para n ≥ 0.
03. Seja θ ∈ R e n um inteiro positivo. Calcule o resto da divisão do polinômio (cos θ + x sin θ)n ∈ R[x]
por x2 + 1.
04. Mostre que x2 = 1 + t possui solução x em R[[t]] (ou seja, 1 + t é um quadrado perfeito em R[[t]]!).
05. Considere o anel
Q[x, y]
A=
(x2 + y 2 − 1)
(a) Mostre que há uma bijeção entre o conjunto de todos os morfismos de anéis f : A → Q e pontos
FT
racionais (i.e. pontos com ambas as coordenadas racionais) do cı́rculo de equação x2 + y 2 = 1.
(b) Mostre que há uma bijeção entre o conjunto de todos os morfismos de anéis f : A → Q(i) e pontos
)
do cı́rculo de equação x2 + y 2 = 1 com ambas as coordenadas em Q(i).
28


(c) Determine (geradores para) ker f , onde f : A → C é o morfismo dado por x̄ 7→ i e ȳ 7→ 2.
7:

06. Seja p > 5 um número primo. Neste exercı́cio, mostraremos que p |√Fp2 −1 onde F√n denota o n-ésimo
(1
RA

n
−β n
número de Fibonacci. Lembre-se de que Fn = αα−β onde α = 1+2 5 e β = 1−2 5 são as raı́zes de
0

2
x − x − 1 = 0.
01

(a) Mostre que α, β ∈ Z[α]× .


,2

(b) Mostre que o quociente Z[α]/(n) é um anel finito com n2 elementos para todo inteiro positivo n.
Mostre ainda que a inclusão Z ֒→ Z[α] induz uma inclusão Z/(n) ֒→ Z[α]/(n).
29

2 2
(c) Mostre que (2α)p e 2α possuem a mesma imagem em Z[α]/(p). Conclua que αp −1 e 1 possuem a
D
ct

mesma imagem em Z[α]/(p).


,O

Hint: utilize o pequeno teorema de Fermat para inteiros: ap − a é um múltiplo de p para todo inteiro
a e primo p. Você se lembra da demonstração deste resultado?
ET

(d) Mostre que (α − β) · Fp2 −1 pertence ao ideal (p) ⊂ Z[α]. Conclua que Fp2 −1 é um múltiplo inteiro
de p.
07. Mostre que todo anel A 6= 0 possui um ideal primo minimal, ou seja, um ideal primo p tal que
q ∈ Spec A e q ⊂ p ⇒ q = p (por exemplo, se A é um domı́nio, então (0) é o único primo minimal de A).
Quais são os primos minimais de C[x, y]/(x2 − y 2 )? Dê uma interpretação geométrica.
Chapter 3
conjunto mul
mapa de loca

Manobras Básicas

Chegou a hora de aprender os “truques do ofı́cio”.

1 Localização
Definição 1.1 Seja A um anel. Um conjunto multiplicativo S ⊂ A é um subconjunto tal que 1 ∈ S
e que é fechado com relação ao produto: s, t ∈ S ⇒ st ∈ S.
As duas escolhas mais populares para S são:
1. S = { hn | n ≥ 0 }, o conjunto das potências de um elemento fixo h ∈ A.
2. S = A \ p, o complemento de um ideal primo p ∈ Spec A.
Dado um conjunto multiplicativo S ⊂ A, vamos construir um novo anel S −1 A onde todos os el-
ementos de S se tornem unidades. Fazemos isto invertendo formalmente os elementos de S, tomando
alguns cuidados para evitar problemas com os divisores de zero em A:
Definição 1.2 Seja S ⊂ A um conjunto multiplicativo. A localização S −1 A do anel A com relação a
S é o anel
 
−1 frações a/s com a ∈ A e s ∈ S, onde duas frações a1 /s1 e a2 /s2 são
S A=
identificadas se existe t ∈ S tal que t · (a1 s2 − a2 s1 ) = 0 em A
onde a soma e o produto são definidos da maneira usual:
a1 a2 a1 s2 + a2 s1 a1 a2 a1 a2
+ = · =
s1 s2 s1 s2 s1 s2 s1 s2
O anel S −1 A vem equipado de fábrica com um morfismo ρ: A → S −1 A dado por ρ(a) = a/1, que é
chamado de mapa de localização.
Devemos verificar que estas operações estão bem definidas, o que é fácil, mas deveras chato (e melhor
feito às escondidas, quando ninguém estiver olhando). Vejamos só que a soma está bem definida. Se
FT
a1 b1 a2 b2
= e = com a1 , a2 , b1 , b2 ∈ A, s1 , s2 , t1 , t2 ∈ S
s1 t1 s2 t2
)
28

então existem u, v ∈ S tais que u(a1 t1 − b1 s1 ) = v(a2 t2 − b2 s2 ) = 0. Devemos mostrar que


7:

a1 s2 + a2 s1 b 1 t2 + b 2 t1 
(1
RA

= ⇐⇒ w (a1 s2 + a2 s1 )t1 t2 − (b1 t2 + b2 t1 )s1 s2 = 0


s1 s2 t1 t2

0

⇐⇒ w (a1 t1 − b1 s1 )s2 t2 + (a2 t2 − b2 s2 )s1 t1 = 0


01

para algum w ∈ S. Basta tomar w = uv.


,2

Quando A é domı́nio e 0 ∈/ S temos o critério usual as = bt ⇐⇒ at = bs (para a, b ∈ A e s, t ∈ S)


29

e assim podemos interpretar S −1 A como subanel de Frac A, o corpo de frações de A. Em particular,


o mapa de localização é sempre injetivo. Em geral, na presença de divisores de zero isto é falso, como
D
ct

mostra o
,O

Exemplo 1.3 Seja A = Z/12Z e S = A− (2) = {1, 3, 5, 7, 9, 11}. Vamos determinar o kernel do mapa de
localização ρ: A → S −1 A. Utilizando o fato de que os únicos elementos em S que possuem divisores de
ET

zero em A são 3 e 9 (os demais são unidades) temos a1 = 01 ⇐⇒ sa = 0 para algum s ∈ S ⇐⇒ a ∈ (4).
Agora podemos “calcular” S −1 A. Note que ρ é sobrejetor: para mostrar isto, basta verificar que as
frações 1/s com s ∈ S estão na imagem. Se s 6= 3, 9 temos s ∈ A× , logo ρ(s−1 ) = 1/s. Por outro lado,
ρ(−1) = 1/3 pois −1/1 = 1/3 ⇐⇒ 3 · (−1 · 3 − 1 · 1) = 0 e portanto 1/9 = (1/3)2 = 1/1 também está
na imagem de ρ. Pelo teorema do isomorfismo, S −1 A ∼ = A/ ker ρ = A/(4) = Z/4Z.
28 Manobras Básicas

O anel S −1 A é o “maior” anel em que todos os elementos de S se tornam unidades, no seguinte propriedade universal
localização
sentido: se φ: A → B é um morfismo de anéis tal que φ(s) ∈ B × para todo s ∈ S então existe um único
φ fazendo o seguinte diagrama comutar (ρ é o mapa de localização):
φ -
A B

-
φ
∃!
ρ
?
S −1 A
Esta é a chamada propriedade universal da localização. Em termos categóricos, esta propriedade diz
que S −1 A representa o funtor HomS (A, −) dos morfismos que levam S em unidades, ou seja, composição
com ρ fornece uma bijeção natural
Hom(S −1 A, B) - HomS (A, B)

φ 7→ φ ◦ ρ
As provas de todos estes fatos seguem diretamente das definições e são deixadas a cargo do leitor.
Exemplo 1.4 Seja h ∈ A e S = {hn | n ∈ N}. Então temos um isomorfismo φ: S −1 A - A[x]/(1−hx).

n
Para construir φ, basta notar que o morfismo de A-álgebras A → A[x]/(1 − hx) leva h em uma unidade
pois hx̄ = 1̄ ⇒ hn x̄n = 1̄ em A[x]/(1 − hx), de modo que pela propriedade universal acima, temos um
morfismo φ: S −1 A → A[x]/(1 − hx) dado por φ(a/hn ) = ax̄n (x “faz o papel” de h−1 ). Para mostrar que
φ é um isomorfismo, basta construir o mapa inverso. Temos que o morfismo de A-álgebras A[x] → S −1 A
dado por x 7→ 1/h leva 1 − xh em 0, logo define um mapa ψ: A[x]/(1 − hx) → S −1 A, e uma verificação
imediata mostra que ψ ◦ φ = id e φ ◦ ψ = id, o que conclui a prova.
Podemos também localizar módulos (em particular, ideais) e álgebras: dado um A-módulo M , a
localização S −1 M de M com relação a S é o S −1 A-módulo
 
−1 frações m/s com m ∈ M e s ∈ S, onde duas frações m1 /s1 e m2 /s2 são
S M=
identificadas se existe t ∈ S tal que t · (s2 m1 − s1 m2 ) = 0 em M
onde as operações são dadas por
m1 m2 s 2 m1 + s 1 m2 a m am
+ = · =
s1 s2 s1 s2 t s ts
para todo a ∈ A, s, t ∈ S e m, m1 , m2 ∈ M . Novamente, verificações tediosas mostram que tudo funciona
como deveria funcionar.
Definição 1.5 Seja A um anel e M um A-módulo. Para h ∈ A, denotamos por Ah e Mh as localizações
FT
de A e M com relação ao conjunto das potências de h. Se p ∈ Spec A, Ap e Mp denotam as localizações
de A e M com relação a A − p.
)
28

Localização é na verdade um funtor da categoria de A-módulos para a categoria de S −1 A-módulos.


Se φ: M → N é um morfismo de A-módulos, temos um morfismo induzido de S −1 A-módulos
7:

S −1 φ: S −1 M → S −1 N
(1
RA

m φ(m) (m ∈ M e s ∈ S)
0

7→
s s
01

estando tudo bem definido como é fácil (e tedioso) verificar. Um fato notável é que este funtor é exato.
,2

Teorema 1.6 (Localização é um funtor exato) Seja A um anel, S um conjunto multiplicativo e


29

M
φ
- N - P
ψ
D
ct

uma sequência exata de A-módulos. Então


,O

S −1 φ S −1 ψ
S −1 M - S −1 N - S −1 P
ET

é uma sequência exata de S −1 A-módulos.


Prova Note que im S −1 φ ⊂ ker S −1 ψ já que S −1 ψ ◦ S −1 φ = S −1 (ψ ◦ φ) = 0. Para mostrar a inclusão
oposta, seja n/s ∈ ker S −1 ψ (onde n ∈ N e s ∈ S). Como ψ(n)/s = 0/1 em S −1 P existe t ∈ S tal que
t · ψ(n) = ψ(t · n) = 0. Pela exatidão da sequência de A-módulos, existe m ∈ M tal que φ(m) = t · n e
portanto (S −1 φ)(m/ts) = tn/ts = n/s, o que mostra que n/s ∈ im S −1 φ.
29

Sequências exatas podem ser utilizadas para codificar diversas relações “lineares” entre módulos. anulador
Por exemplo, o teorema anterior possui as seguintes consequências úteis
Corolário 1.7 Seja A um anel, S um conjunto multiplicativo e φ: M → N um morfismo de A-módulos.
1. se φ é injetor (respectivamente sobrejetor, bijetor) então o mesmo vale para S −1 φ.
2. localização comuta com kernels, cokernels e imagens: ker S −1 φ ∼ = S −1 ker φ, coker S −1 φ ∼
=
−1 −1 ∼ −1
S coker φ, im S φ = S im φ.
3. localização também comuta com quocientes: se M é um submódulo de N então S −1 (N/M ) ∼ =
S −1 N/S −1 M .

Prova O primeiro item é uma consequência do segundo, que pode ser mostrado localizando sequências
exatas adequadas. Por exemplo, localizando a sequência exata de A-módulos

0 - ker φ - M - N
φ

obtemos a sequência exata de S −1 A-módulos

S −1 φ
0 - S −1 ker φ - S −1 M - S −1 N

o que mostra que ker S −1 φ ∼ = S −1 ker φ. As provas para coker φ e im φ são análogas. Finalmente, o
terceiro item segue da localização da sequência exata

0 → M → N → N/M → 0

Temos ainda uma importante recı́proca para o teorema anterior:


Teorema 1.8 (Um princı́pio “local-global”) Seja A um anel.
1. Seja M um A-módulo. Então

M = 0 ⇐⇒ Mm = 0 para todos os ideais maximais m de A

2. Seja
M - N
φ ψ
- P

um complexo de A-módulos. Se as localizações


FT
Mm
φm
- Nm ψm
- Pm
)
28

são exatas para todos os ideais maximais m de A então o complexo anterior de A-módulos é
7:

exato. Em particular, um morfismo de A-módulos é injetor (respectivamente sobrejetor) se, e


(1
RA

só se, todas as localizações com relação aos ideais maximais possuem a mesma propriedade.
0

Prova O segundo item segue do primeiro aplicado ao A-módulo ker ψ/ im φ e do fato de que localização
01

é um funtor exato e portanto (ker ψ/ im φ)m = (ker ψ)m /(im φ)m = ker ψm / im φm .
,2

Resta provar o primeiro item, e a implicação ⇒ é clara. Para mostrar ⇐, seja m ∈ M ; vamos provar
que m = 0. Para isto, considere o anulador de m
29
D

def
ct

ann(m) = {a ∈ A | a · m = 0}
,O

que claramente é um ideal de A. Temos que mostrar que ann(m) = A e para isto basta mostrar que
ET

ann(m) 6⊂ m para todo ideal maximal m. Mas isto segue do fato de Mm = 0: como m/1 = 0/1, existe
um s ∈ A \ m tal que sm = 0 ⇐⇒ s ∈ ann(m) e portanto ann(m) 6⊂ m.

Como estamos acrescentando unidades ao anel A, um efeito colateral da localização é o “massacre de


ideais”. Isto simplifica o anel original e é um dos motivos que tornam a localização um dos instrumentos
mais úteis no estudo de anéis.
30 Manobras Básicas

Teorema 1.9 (Localização e Ideais) Seja A um anel e S um conjunto multiplicativo. Denote por
ρ: A → S −1 A o mapa de localização.
1. Todo ideal de S −1 A é da forma S −1 a para algum ideal a de A.
2. O mapa de espectros
Spec(ρ): Spec S −1 A ֒→ Spec A
é injetor e tem como imagem o conjuntos dos primos p de A tais que p ∩ S = ∅; a pré-imagem
de um tal p é dada por S −1 p.

Prova Primeiramente note que se a ⊂ A é um ideal de A, então S −1 a ⊂ S −1 A será um S −1 A-submódulo


de S −1 A, isto é, um ideal de S −1 A, pois localização, sendo exata, preserva injetividade. Reciprocamente,
def
se b é um ideal de S −1 A, defina a = ρ−1 b. É fácil ver que a é um ideal de A e que S −1 a ⊂ b. Por outro
lado, se b/s ∈ b (b ∈ A, s ∈ S) então multiplicando por s/1 obtemos ρ(b) = b/1 ∈ b, ou seja, b ∈ a. Logo
b ⊂ S −1 a e portanto devemos ter a igualdade, o que prova o item 1.
Para o item 2, seja DS ⊂ Spec A o conjunto dos primos p tais que p ∩ S = ∅. Note primeiramente
que a imagem de Spec(ρ) está contida em DS : se p = Spec(ρ)(q) ⇐⇒ p = ρ−1 q e há um elemento
s ∈ S ∩ p então ρ(s) ∈ q, mas como ρ(s) é unidade em S −1 A, isto é impossı́vel. Observe ainda que para
p ∈ DS , a ∈ A e s ∈ S temos
a
∈ S −1 p ⇐⇒ a ∈ p (∗)
s
A implicação ⇐ é óbvia; por outro lado, se a/s ∈ S −1 p, então existem p ∈ p e t ∈ S tais que a/s = p/t e
isto por sua vez implica que existe um r ∈ S tal que r(at − ps) = 0. Portanto rta ∈ p e, como p ∩ S = ∅,
a deve pertencer a p.
Agora mostremos que S −1 p ∈ Spec S −1 A quando p ∈ DS : primeiro, S −1 p é um ideal próprio pois

caso contrário terı́amos 1/1 ∈ S −1 p ⇒ 1 ∈ p por (∗); por outro lado, se as · as′ ∈ S −1 p (a, a′ ∈ A e
s, s′ ∈ S) então aa′ ∈ p novamente por (∗), assim ou a ou a′ pertence a p, isto é, ou a/s ou a′ /s′ deve
pertencer a S −1 p, como requerido.
Finalmente provemos que Spec(ρ): Spec S −1 A → DS e o mapa no sentido oposto p 7→ S −1 p, p ∈ DS ,
são inversos um do outro e portanto estabelecem uma bijeção entre Spec S −1 A e DS . Se p ∈ DS então
Spec(ρ)(S −1 p) = p por (∗). Agora, dado q ∈ Spec S −1 A, se p = Spec(ρ)(q) temos que q = S −1 p pelo
item 1. Isto completa a demonstração.

Exemplo 1.10 (Localização em um elemento) Seja h ∈ A e considere o anel localizado Ah com


mapa de localização ρ: A → Ah . Neste caso, o conjunto multiplicativo é o das potências de h e temos
que a imagem de Spec(ρ): Spec Ah ֒→ Spec A é justamente D(h), o domı́nio de 1/h: como h/1 é uma
unidade em Ah , todos os primos que contém h são mortos durante a localização.
FT
Exemplo 1.11 (Localização em um ideal primo) Seja p ∈ Spec A a e considere o anel localizado
Ap com mapa de localização ρ: A → Ap . Neste caso, temos que o conjunto multiplicativo sendo invertido
é o S = A \ p e a imagem de Spec(ρ): Spec Ap → Spec A consiste nos primos contidos em p; todos os
)
28

demais primos são “assinados” quando invertemos os elementos de S. Desta forma, Ap possui um único
ideal maximal S −1 p, que usualmente denotamos por pAp (o ideal gerado por ρ(p)). Como localização
7:

comuta com quociente e a imagem de S em A/p consiste nos elementos não nulos deste anel, temos que
(1
RA

o corpo
0

def
k(p) = Ap /pAp = S −1 (A/p) = Frac(A/p)
01

é o corpo de frações do domı́nio A/p (lembre-se de que p é primo!). Este corpo k(p) é chamado corpo
,2

residual de p.
29

Note os efeitos complementares da localização e do quociente: enquanto Spec(ρ): Spec Ap → Spec A


D

“filtra” os ideais primos contidos em p, o mapa de espectros Spec(q): Spec A/p ֒→ Spec A associado ao
ct

mapa quociente q: A ։ A/p tem como imagem V (p), a variedade cortada por p, que consiste em todos
,O

os ideais primos que contém p. Combinando localização e quociente, podemos desta forma selecionar
qualquer conjunto de primos que desejamos estudar. Por exemplo, o mapa f : Spec k(p) ֒→ Spec A
ET

associado à composição
A - Ap - - k(p)
ρ

tem como imagem exatamente o primo p pois f é a composição


Spec k(p) ֒→ Spec Ap ֒→ Spec A
31

e a imagem do primeiro mapa é pAp , que é levado em p pelo segundo mapa. Em outras palavras, o radical
nilradical
quociente e a localização filtram os primos que contêm e que estão contidos em p e o que sobre é apenas
o primo p.

2 Localização em Ação
Vejamos em alguns exemplos práticos como a utilização da localização pode simplificar o estudo de anéis.

2.1 Nilradical
Definição 2.1.1 Seja A um anel e a um ideal de A. O radical de a é definido como
√ def
a = {a ∈ A | an ∈ a para algum n ≥ 1}
p
O nilradical de A é definido como o conjunto de todos os elementos nilpotentes de A, i.e., como (0).
√ √ √
Observe que a√é de fato um ideal: se a ∈ a, então é claro que ra ∈ a para qualquer r ∈ A; por
outro lado, se a, b ∈ a, podemos escolher n grande o suficiente de modo que an e bn estão ambos em a.
Então
X 2n
2n
(a + b) = ai b2n−i ∈ a
i
0≤i≤2n

pois ou i ≥ n e portanto a ∈ a ou i ≤ n ⇐⇒ 2n − i ≥ n e portanto b2n−i ∈ a. Assim, a + b ∈ a.
i
p
Há uma conexão muito forte entre o nilradical e os primos de um anel. Se p ∈ Spec A e a ∈ (0),
então para algum n temos
an = 0 ⇒ an ∈ p ⇒ a ∈ p
Portanto um elemento nilpotente pertence a todo ideal primo de A:
p \
(0) ⊂ p
p∈Spec A

p
Na verdade, a inclusão acima é uma igualdade! Para mostrar isto, vamos provar que para todo h ∈ / (0)
existe p ∈ Spec A tal que h ∈ / p. Como o mapa Spec Ah ֒→ Spec A associado ao mapa de localização tem
imagem D(h), mostrar a existência de tal primo é equivalente a mostrar que Spec Ah 6= ∅ ⇐⇒ Ah 6= 0.
Mas Ah = 0 se, e só se, 1/1 = 0/1 em Ah , isto é, se, e só se, existe um n tal que hn (1 · 1 − 1 · 0) = 0 em
A, ou seja, se, e só se, h é nilpotente. Com isto, acabamos de provar o
p T
FT
Teorema 2.1.2 Para qualquer anel A, (0) = p∈Spec A p.
Aplicando o teorema acima para A/a no lugar de A e observando que o nilradical de A/a corresponde
)
ao radical de a na correspondência de ideais, obtemos o
28

√ T
7:

Corolário 2.1.3 Para qualquer ideal a do anel A, a = p∈V (a) p.


(1
RA

Exemplo 2.1.4 Seja k um corpo e considere o anel A = k[x1 , . . . , xn ], um domı́nio de fatoração única.
Se f ∈ A é um elemento não nulo e
0
01

f = u · pe11 . . . perr , u ∈ k×
,2
29

é a fatoração de f em potências de polinômios mônicos irredutı́veis p1 , . . . , pr ∈ A, então utilizando a


fatoração única concluı́mos que
D
ct

p
,O

(f ) = (p1 . . . pr ) = (p1 ) ∩ · · · ∩ (pr )


ET

Por outro lado, observe que se p ∈ Spec A então f ∈ p ⇒ pi ∈ p para algum i e, como os pi são irredutı́veis
em A, (pi ) já são ideais primos,
p donde concluı́mos que os (pi ) são os ideais primos minimais contendo
(f ). Pelo corolário acima, (f ) é a intersecção dos (pi ), o que concorda com a computação direta feita
acima. Assim, obtemos uma comprovação experimental do teorema anterior!
Vejamos agora algumas aplicações no estudo da topologia de Zariski.
32 Manobras Básicas

Lemma 2.1.5 Seja A um anel, a e b ideais e g, h ∈ A. elementos idempotent


√ √ √
1. V (a) ⊂ V (b) ⇐⇒ a ⊂ b. Em particular, V (a) = V (b) ⇐⇒ a = b.
p
2. D(h) ⊂ D(g) ⇐⇒ h ∈ (g).

Teorema 2.1.6 (Morfismo Dominante) Seja A ⊂ B uma inclusão de domı́nios. Então o mapa
f : Spec B → Spec A é dominante, i.e., sua imagem é densa.
Prova Devemos mostrar que a imagem de f intercepta qualquer aberto não vazio. Para isto, note que,
para um aberto básico D(h) ⊂ Spec A não vazio, isto é, com h 6= 0, temos

f −1 D(h) = D(h) ⊂ Spec B,

que também é não vazio.

Dizemos que um anel A é decomponı́vel se existe um isomorfismo A ∼ = B × C para dois anéis não
nulos B e C. Neste caso, os elementos e1 = (1, 0) e e2 = (0, 1) satisfazem as relações

e1 + e2 = 1 e21 = e1 e22 = e2 e1 e2 = 0

Dois elementos não nulos e1 e e2 de um anel qualquer A são chamados de idempotentes ortogonais
se as relações acima se verificam. Observe que as duas últimas identidades seguem das duas primeiras.
Temos que um anel A é decomponı́vel se, e só se, possui idempotentes ortogonais: já vimos uma das
direções; para mostrar a recı́proca observe que se e1 , e2 ∈ A são então são idempotentes ortogonais,
podemos definir subanéis B = Ae1 e C = Ae2 de A e é fácil verificar que os mapas

A

- B×C B×C

- A
a 7→ (ae1 , ae2 ) (b, c) 7→ b + c

são isomorfismos inversos um do outro.


p
Lemma 2.1.7 Seja A um anel e seja a um ideal tal que a ⊂ (0). Então

A/a é decomponı́vel ⇐⇒ A é decomponı́vel


FT
Prova (⇐) Sejam e1 , e2 ∈ A idempotentes ortogonais; então suas imagens e1 , e2 ∈ A/a também p são
idempotentes ortogonais: basta verificar que eles não são nulos. Mas se ei = 0 então ei ∈ a ⊂ (0), logo
)
eni = 0 para algum n. Aplicando a relação e2i = ei várias vezes, obtemos ei = 0, uma contradição.
28

(⇒) Sejam e1 , e2 ∈ A/a idempotentes ortogonais e seja e1 ∈ A umap pré-imagem de e1 . Defina e2 = 1−e1 ,
7:

que é uma pré-imagem de e2 . Como e1 e2 = 0, temos e1 e2 ∈ a ⊂ (0) e portanto (e1 e2 )n = 0 para algum
(1
RA

n. Da expansão de (e1 + e2 )2n , definimos dois novos elementos


0

     
01

def 2n 2n−1 2n 2n−2 2 2n


e′1 = e2n
+1 e1 e2 + e1 e2 + · · · + en+1 e2n−1
1 2 n−1 1
,2

     
′ def 2n 2n 2n 2n
29

2n−1 2 2n−2
e2 = e2 + e1 e2 + e e + ···+ en−1 en+1
2n − 1 2n − 2 1 2 n+1 1 2
D
ct

de modo que
,O

 
2n n n
e′1 + e′2 = e′1 + e′2 + e e = (e1 + e2 )2n = 1
n 1 2
ET

Além disso e′1 e′2 = 0 pois é uma soma de elementos da forma ei1 ej2 com i, j ≥ n. Como e1 e2 ∈ a, temos
que e′i também é uma pré-imagem de ei = e2n ′ ′
i para i = 1, 2 e como acima temos que ei 6= 0. Assim, e1 e

e2 são idempotentes ortogonais de A, que é portanto decomponı́vel.
33

Teorema 2.1.8 (Conexidade e Indecomponibilidade) Seja A um anel. Então Spec A é desconexo Fibra
se, e só se, A é decomponı́vel.
Prova Sendo B e C dois anéis não nulos, temos uma união disjunta

Spec B × C = V (a) ⊔ V (b) onde a = (0) × C e b = B × (0)

Como a e b são ideais próprios de A × B, os conjuntos fechados V (a) e V (b) são não vazios e assim
Spec B × C é desconexo.
Reciprocamente, se Spec A é desconexo, digamos Spec A = V (a) ⊔ V (b) para dois ideais próprios a
e b então
   
V (a) ∩ V (b) = ∅ V (a + b) = V (1)
 =∅ a + bp= (1)
⇐⇒ ⇐⇒
V (a) ∪ V (b) = Spec A V (ab) = V (0) = Spec A ab ⊂ (0)

Assim, pelo teorema chinês dos restos temos um isomorfismo

A A A A
= = ×
ab a∩b a b

Como a e b são próprios, os anéis A/a e A/b são não nulos e assim A/ab é decomponı́vel. Pelo lema
anterior, temos portanto que A é decomponı́vel.

2.2 Fibras
Agora vamos utilizar o par localização/quociente na descrição de fibras. Fibra é somente um nome
pomposo para a pré-imagem de um ponto. No nosso caso, estamos interessados em obter as fibras de
mapas entre espectros:
Teorema 2.2.1 (Fibras) Seja φ: A → B um morfismo de anéis e seja f : Spec B → Spec A o mapa
correspondente de espectros. Seja p ∈ Spec A e seja S = A − p. Então o mapa

Spec S −1 (B/pB) → Spec B

associado ao mapa natural B → S −1 (B/pB) estabelece uma bijeção

ideais primos em Spec S −1 (B/pB) ↔ ideais primos em f −1 (p)


FT
Observe que no teorema acima estamos interpretamos B como uma A-álgebra via φ, de modo que
pB denota o ideal de B gerado por φ(p) e assim por diante.
)
28

Vamos analisar separadamente o efeito do quociente e da localização. O primeiro dá origem aos
seguintes diagramas comutativos
7:
(1
RA

B/pB 
 B Spec B/pB ⊂ - Spec B
0

6 6
01

φ φ Spec φ Spec φ
,2

? ?
29

A/p 
 A Spec A/p ⊂ - Spec A
D
ct

onde φ é o mapa induzido por φ, ou seja, φ(a) = φ(a) para todo a ∈ A. Note que o mapa Spec A/p ֒→
,O

Spec A identifica Spec A/p com V (p). Vamos mostrar que a imagem de Spec B/pB ֒→ Spec B consiste
ET

justamente na pré-imagem (Spec φ)−1 V (p), de modo que no segundo diagrama Spec φ pode ser interpre-
tado como a “restrição” de Spec φ ao conjunto V (p) e à sua pré-imagem (as fibras de V (p)). De fato, um
primo q ∈ Spec B está na imagem de Spec B/pB ֒→ Spec B se, e só se, q ⊃ pB. Mas pB é o ideal de B ger-
ado por φ(p), assim a última condição é equivalente a q ⊃ φ(p) ⇐⇒ φ−1 q ⊃ p ⇐⇒ Spec(φ)(q) ∈ V (p),
que era o querı́amos provar.
Da mesma forma, temos diagramas comutativos
34 Manobras Básicas

S −1 B  B Spec S −1 B ⊂ - Spec B
6 6
S −1 φ φ Spec S −1 φ Spec φ
? ?
S −1
A A Spec S −1 A ⊂ - Spec A

onde os mapas horizontais no primeiro diagrama são os mapas de localização. Vamos mostrar que
Spec S −1 φ pode ser interpretado como o conjunto das fibras dos primos de A que estão contidos em p,
ou seja, vamos mostrar que a imagem de Spec S −1 B ֒→ Spec B é a pré-imagem por Spec φ do conjunto
dos primos de A contidos em p. A imagem de Spec S −1 B ֒→ Spec B consiste dos primos q de B tais que
q ∩ φ(S) = ∅ (lembre-se de que o produto de um elemento de A por um de B é feito via o morfismo base
φ e que portanto S −1 B denota na verdade a localização de B com relação ao conjunto multiplicativo
φ(S)). Porém q ∩ φ(S) = ∅ ⇐⇒ φ−1 q ∩ S = ∅, ou seja, Spec(φ)(q) ⊂ p, como desejado.
O teorema agora segue compondo os dois resultados acima:

S −1 (B/pB)  B/pB 
 B Spec S −1 (B/pB) ⊂- Spec B/pB ⊂ - Spec B
6 6 6
S −1 φ φ φ Spec S −1 φ Spec φ Spec φ
? ? ?
S −1 (A/p)  A/p 
 A Spec S −1 (A/p) ⊂ - Spec A/p ⊂ - Spec A

Note que S −1 (A/p) = k(p) é o corpo residual de p, logo Spec S −1 (A/p) consiste em um único ponto,
cuja imagem em Spec A é justamente p. Assim, o mapa Spec S −1 (B/pB) ֒→ Spec B tem como imagem
precisamente a fibra de p.
Exemplo 2.2.2 (Cálculo do Spec C[x, y]) Considere a inclusão φ: C[x] ֒→ C[x, y]. Vamos calcular as
fibras de Spec φ. Temos dois casos:
• Fibra de p = (x − a), a ∈ C. Seja S = C[x] \ p. Temos
 C[x, y]  x7→a
S −1 ∼
= C[y]
(x − a)

Assim, temos que os primos na fibra de p são da forma (x − a) e (x − a, y − b) com b ∈ C, que são
respectivamente as imagens de (0) e (y − b) pelo mapa Spec C[y] ֒→ Spec C[x, y].
FT
• Fibra de p = (0): seja S = C[x] \ {0}. Temos S −1 C[x, y] = C(x)[y], que é um domı́nio de ideais
principais visto que C(x) é um corpo. Assim, Spec C(x)[y] consiste no primo (0) e nos ideais principais
)
f (x, y) gerados por elementos irredutı́veis f (x, y) ∈ C(x)[y]. Sem perda de generalidade, podemos
28

supor que f (x, y) ∈ C[x, y] pois podemos multiplicá-lo por um polinômio não nulo em C[x] (que é
7:

uma unidade em C(x)[y], logo não altera o ideal). Mais ainda, podemos supor que os coeficientes de
f (x, y) ∈ C[x][y], visto como polinômio em y, são primos entre si (ou seja, f (x, y) é primitivo). Pelo
(1
RA

lema de Gauß, temos que f (x, y) é irredutı́vel em C(x)[y] se, e só se, é irredutı́vel em C[x, y]. Assim,
0


01

Spec C(x)[y] = {(0)} ∪ { f (x, y) | f (x, y) ∈ C[x, y] \ C[x] e é irredutı́vel em C[x, y]}
,2


A imagem de (0) por Spec C(x)[y]
 → Spec C[x, y] é (0), enquanto que a imagem de q = f (x, y) ⊂ C(x)[y]
29

é q ∩ C[x, y], que é f (x, y) ⊂ C[x, y]: se f (x, y)g(x, y)/s(x) ∈ C[x, y] ∩ q, g(x, y) ∈ C[x, y] e s(x) ∈ C[x],
pela fatoração única em C[x, y] temos s(x) | g(x, y) já que f (x, y) é primitivo, logo f (x, y)g(x, y)/s(x) é
D
ct

múltiplo de f (x, y) em C[x, y].


,O

Exemplo 2.2.3 (Blow-up) Considere o mapa f : Spec C[x, y, z]/(y −zx) → Spec C[x, y] correspondente
ET

ao morfismo de C-álgebras φ: C[x, y] → C[x, y, z]/(y − xz) dado por φ(x) = x e φ(y) = y. Note que φ
é injetor, de modo que podemos identificar A = C[x, y] como um subanel de B = C[x, y, z]/(y − zx).
Geometricamente, A é o anel de funções polinomiais do plano complexo C2 enquanto que B é o anel
de funções polinomiais na superfı́cie em C3 de equação y = zx, que é uma união de retas (para z = m
constante, y = mx é a reta de coeficiente angular m, desenhada na “altura” z = m). O mapa f
corresponde à projeção da superfı́cie no “plano xy”.
35

Vamos calcular a fibra de um ideal maximal da forma m = (x − a, y − b) ∈ Spec A. Temos uma bijeção anel local
corpo residua
de f −1 m com Spec S −1 (B/mB), onde S = A \ m e morfismo loca

 B   B   C[x, y, z]  x7y7→a
→b C[z]
S −1 = S −1 = S −1 ∼
=
mB (x − a, y − b) (x − a, y − b, y − xz) (b − az)

Note que como m consiste nos polinômios que se anulam em (a, b), temos que no último isomorfismo
todos os elementos de S são levados em constantes não nulas, que já são inversı́veis em C[z].
Agora temos alguns casos a analisar. Se a 6= 0, C[z]/(b − az) ∼ = C e a fibra consiste em um único ponto.
Para encontrá-lo, basta tomar a imagem de Spec C ֒→ Spec B associada ao morfismo B → C que mapeia
x 7→ a, y 7→ b, z 7→ b/a. Tal ponto é o ideal maximal (x − a, y − b, z − b/a) ⊂ B, que geometricamente
corresponde ao ponto (a, b, b/a) ∈ C3 da superfı́cie y = xz, que é o único que projeta no ponto (a, b) ∈ C2 .
Agora se a = 0 e b 6= 0, C[z]/(b − az) = 0 e a fibra é vazia. Finalmente, se a = b = 0, C[z]/(b − az) = C[z]
e a fibra é a imagem do mapa Spec C[z] ֒→ Spec B associado ao morfismo B → C[z] que mapeia x 7→ 0,
y 7→ 0, z 7→ z. Temos neste caso

f −1 (x, y) = {(x, y)} ∪ {(x, y, z − d) | d ∈ C}

onde o ideal (x, y) é a imagem de (0) ∈ Spec C[z] enquanto que (x, y, z − d) é a imagem de (z − d) ∈
Spec C[z]. Geometricamente esta fibra é uma reta, com um ponto denso correspondente a (x, y) e pontos
fechados correspondentes aos pontos (0, 0, d) ∈ C3 da superfı́cie y = xz, que são justamente os pontos
cuja projeção é a origem (0, 0) ∈ C2 .
Calculemos agora a fibra do ideal primo p = (y 2 − x2 (x + 1)) ∈ Spec A. Sendo S = A \ p, temos
 B   B   C[x, y, z] 
S −1 = S −1 2 2 = S −1
mB (y − x (x + 1)) (y 2 − x2 (x + 1), y − xz)
S −1 C[x, y, z] S −1 C[x, y, z] S −1 C[x, z] ∼ −1
= = = = S C[z]
((xz)2 − x2 (x + 1), y − xz) (z 2 − (x + 1), y − xz) (z 2 − (x + 1))

Aqui, no antipenúltimo isomorfismo utilizamos o fato de x ser unidade em S −1 C[x, y, z] já que x ∈ S.
O penúltimo isomorfismo é dado por x 7→ x, y 7→ xz, z 7→ z e o último, por x 7→ z 2 − 1, z 7→ z. Note
que S −1 C[z] = C(z), o corpo de frações de C[z], pois qualquer fração em C[z] pode ser escrita como
f (z) f (z)g(−z)
g(z) = g(z)g(−z) com denominador g(z)g(−z) que é uma função par, ou seja, só possui monômios cujos
expoentes são pares. Mas qualquer polinômio par é a imagem de algum polinômio h(x) ∈ S na variável
x apenas, já que a composição ψ dos isomorfismos acima leva x 7→ z 2 − 1.
FT
A fibra consiste portanto em um único elemento, que é pela imagem do mapa Spec C(z) → Spec B
associado ao morfismo ψ: B → C(z) que leva x 7→ z 2 − 1, y 7→ z(z 2 − 1), z 7→ z. Esta imagem é o ideal
)
primo ψ −1 (0) = (z 2 − x − 1) de B. De fato, é claro que ψ −1 (0) = ker ψ contém (z 2 − x − 1) e portanto
28

ψ induz um morfismo ψ: B/(z 2 − x − 1) → C(z). Assim, para mostrar a inclusão oposta basta verificar
7:

que ψ é injetor. Como B/(z 2 − x − 1) = C[x, y, z]/(z 2 − x − 1, y − xz) ∼ = C[z], onde o último isomorfismo
(1
RA

leva x 7→ z 2 − 1 e y 7→ z(z 2 − 1), seguindo os vários isomorfismos concluı́mos que ψ: C[z] ֒→ C(z) não é
nada além do mapa de inclusão, o que encerra a prova.
0

Geometricamente, este primo (z 2 − x − 1) corresponde a “abrir” a curva singular y 2 = x2 (x + 1): os dois


01

“ramos” que passam pela origem têm direções tangentes distintas, logo são levadas a alturas diferentes.
,2

Obtemos assim a curva não singular z 2 = x + 1 sobre a superfı́cie y = xz, como ilustra a figura a
seguir. Observe ainda na figura que cada ponto (a, b) ∈ C2 do plano de baixo tem uma única pré-imagem
29

(a, b, b/a) na superfı́cie, à exceção da origem, cuja pré-imagem é uma reta.


D
ct
,O

2.3 Anéis Locais


ET

Definição 2.3.1 Um anel local A é um anel que possui um único ideal maximal m. O quociente
k = A/m é chamado de corpo residual. Escrevemos (A, m, k) para denotar um anel local A com ideal
maximal m e corpo residual k.
Um morfismo φ: A → B entre dois anéis locais (A, m, k) e (B, n, l) é um morfismo local se φ−1 n = m
ou, equivalentemente, φ(m) ⊂ n. Note que um morfismo local induz um morfismo injetivo de corpos
residuais φ̄: k → l dado por a mod m 7→ φ(a) mod n para a ∈ A.
36 Manobras Básicas

truque do determinan
base minimal

Fibra de p = (y 2 − x2 (x + 1))

Observe que, em qualquer anel A,


u ∈ A× ⇐⇒ (u) = A ⇐⇒ u não pertence a nenhum ideal maximal de A (veja teorema II.2.8)
Desta forma, um anel A é local se, e somente se, A \ A× é um ideal. De fato, se A é local com ideal
maximal m, então u ∈ A× ⇐⇒ u ∈ / m pela observação acima e assim m = A \ A× . Por outro lado, se
A \ A é um ideal, todo ideal próprio de A está necessariamente contido em A \ A× , que é portanto o
×

único ideal maximal de A.


Em vista do princı́pio local-global (teorema 1.8), diversas questões sobre anéis gerais podem ser
reduzidas a anéis locais, que possuem portanto um papel central em álgebra comutativa. Um dos
principais resultados sobre anéis locais é o aparentemente inócuo
Lema 2.3.2 (Nakayama) Seja (A, m, k) um anel local, a um ideal próprio de A e M um A-módulo
finitamente gerado.
1. Se aM = M então M = 0.
2. Se N é um submódulo de M tal que M = N + aM então M = N .
Prova O segundo item segue diretamente do primeiro aplicado ao A-módulo M/N . Para provar o
primeiro item, utilizaremos o famoso truque do determinante. Seja ω1 , . . . , ωn geradores de M . Por
hipótese, existem aij ∈ a ⊂ m tais que
ω1 = a11 ω1 + a12 ω2 + · · · + a1n ωn
ω2 = a21 ω1 + a22 ω2 + · · · + a2n ωn
FT
..
.
)
ωn = an1 ω1 + a2n ω2 + · · · + ann ωn
28

Considere a matriz n × n dada por T = (aij ) e seja I a matriz identidade de ordem n. Em notação
7:

matricial, podemos escrever o “sistema linear” acima como (I − T ) · w = 0, onde 0 denota a matriz
(1
RA

coluna n × 1 nula e w é a matriz coluna cujas entradas são os ωi . Multiplicando pela matriz adjunta,
obtemos det(I − T ) · ωi = 0 para i = 1, . . . , n. Porém, como aij ∈ m, temos det(I − T ) ≡ det I = 1
0
01

(mod m), ou seja, det(I − T ) ∈ A× e assim ωi = 0 para todo i, mostrando que M = 0.


,2

Como aplicação, vamos mostrar que para um módulo finitamente gerado sobre um anel local, o
número de elementos em um conjunto minimal de geradores (com relação à inclusão) é o mesmo, inde-
29

pendentemente do conjunto escolhido:


D
ct

Corolário 2.3.3 (Bases Minimais) Seja (A, m, k) um anel local e M um A-módulo finitamente
,O

gerado. Então qualquer conjunto minimal de geradores de M possui dimk M/mM elementos.
Prova Seja d = dimk M/mM e seja ω1 , . . . , ωn um conjunto minimal de geradores de M . Temos que as
ET

imagens ω 1 , . . . , ωn ∈ M/mM dos ωi geram o k-espaço vetorial M/mM e portanto n ≥ d. Na verdade,


n = d, ou seja, os ωi formam uma base. Caso contrário, terı́amos m < n elementos ω 1 , . . . , ωm (digamos)
def
que gerariam M/mM . Mas neste caso o submódulo N = Aω1 + · · · + Aωm de M satisfaz M = N + mM
e por Nakayama terı́amos que M = N , ou seja, ω1 , . . . , ωm gerariam M , contrariando a minimalidade
do conjunto de geradores acima.
37

Mas atenção! Embora um conjunto minimal de geradores receba o nome de base minimal, este
conjunto não forma uma base em geral! Ou seja, estes elementos não são necessariamente linearmente
independentes.
Exemplo 2.3.4 Seja B = C[x, y]/(y 2 − x2 (x + 1)) e considere o ideal maximal n = (x, y) de B. Vamos
determinar o número mı́nimo de geradores do ideal maximal nBn do anel local Bn . Temos inicialmente
que o corpo residual de Bn é Frac B/n = C. Agora seja A = C[x, y] e considere o ideal maximal m = (x, y)
de A. Temos que Bn = Am /(y 2 − x2 (x + 1)) e como mAm ⊃ (y 2 − x2 (x + 1)) e (mAm )2 ⊃ (y 2 − x2 (x + 1)),
pela correspondência de ideais obtemos que o número mı́nimo de geradores de nBn é

nBn mAm
dimC 2
= dimC
(nBn ) (mAm )2

e este último espaço vetorial tem dimensão 2. De fato, vamos verificar que os elementos x e y de
mAm /(mAm )2 são linearmente independentes sobre C. Se a, b ∈ C são tais que ax + by = 0 ⇐⇒
ax + by ∈ mAm , então existem polinômios p(x, y), q(x, y), r(x, y) ∈ A e s(x, y) ∈ A \ m (isto é, s(0, 0) 6= 0)
tais que
p(x, y) · x2 + q(x, y) · xy + r(x, y) · y 2
ax + by =
s(x, y)
Substituindo y = 0, concluı́mos que a = p(x, 0) · x/s(x, 0), isto é, a pertence ao ideal maximal (x) do
anel local C[x](x) , o que só é possı́vel se a = 0. Analogamente, mostra-se que b = 0, o que completa a
prova da afirmação.
Agora seja p = (x + 1, y) ⊂ A e q = (x + 1, y) ⊂ B. Vamos determinar o número mı́nimo de geradores
do ideal maximal do anel local Bq . Como acima, temos que este número é

qBq pAp
dimC = dimC 2
(qBq )2 (y − x2 (x + 1)) + (pAp )2

(note que agora (pAp )2 6⊃ (y 2 − x2 (x + 1)), por isso o ideal correspondente em Ap muda). Como x é
unidade em Ap , temos que (y 2 − x2 (x + 1)) + (pAp )2 = (y 2 − x2 (x + 1), y 2 , (x + 1)y, (x + 1)2 ) = (y), logo
o último quociente acima é gerado apenas por x + 1, ou seja, é de dimensão 1 e, assim, qBq = (x + 1) é
um ideal principal.
Geometricamente, temos que B é o anel de funções polinomiais da curva complexa plana y 2 = x2 (x + 1)
e que n corresponde à origem (0, 0) ∈ C2 , enquanto que q corresponde ao ponto (−1, 0) ∈ C2 . Sendo uma
curva (dimensão 1), intuitivamente espera-se que, localmente, um ponto seja determinado pelos zeros de
um único elemento de B, como ocorre com o ponto regular (−1, 0), que é definido por x + 1 = 0 (note
FT
que x + 1 é justamente o gerador de nBn ). Entretanto, o ponto (0, 0) é singular, e portanto não pode
ser definido pelos zeros de um único elemento, precisamos de pelo menos dois elementos. Por exemplo,
)
a equação y = 0 define dois pontos (−1, 0) e (0, 0) sobre a curva, enquanto que x = 0 define um “ponto
28

duplo” (0, 0) pois o “anel de funções” correspondente Bn /(x) ∼


= C[y]/(y 2 ) não é exatamente C, mas uma
7:

álgebra de dimensão 2 sobre C.


(1
RA
0
01
,2
29

(−1, 0) (0, 0)
D
ct
,O
ET

A curva y 2 = x2 (x + 1)
38 Manobras Básicas

Observação 2.3.5 Se A é um anel qualquer e m é um ideal maximal de A, o corpo residual do anel mapa bilinear
produto tensorial
local Am é k = A/m e temos um isomorfismo de k-espaços vetoriais

m mAm
=
m2 (mAm )2

o que permite simplificar o cálculo do exemplo anterior. Para provar o isomorfismo acima, seja S = A\m.
Basta observar que as imagens dos elementos de S já são inversı́veis em k = A/m e assim, utilizando a
comutatividade entre localização e quociente, temos

mAm  
−1 m m
= S = 2
(mAm )2 m2 m

3 Produto Tensorial

3.1 Definição e Propriedades Básicas


Seja A um anel e sejam M , N e T A-módulos. Um mapa bilinear f : M × N → T é uma função que é
A-linear em cada entrada separadamente: para todo m, m′ ∈ M , n, n′ ∈ N e a ∈ A,
1. f (am, n) = af (m, n) = f (m, an)
2. f (m + m′ , n) = f (m, n) + f (m′ , n)
3. f (m, n + n′ ) = f (m, n) + f (m, n′ )
O conjunto de todos os mapas bilineares de M × N a T será denotado por BilA (M × N, T ).
O produto tensorial M ⊗A N de M e N (denotado também por M ⊗ N se A é claro pelo contexto)
é definido como o quociente L
def (m,n)∈M×N A · e(m,n)
M ⊗A N =
R
L
do A-módulo livre (m,n)∈M×N A · e(m,n) com base e(m,n) pelo submódulo R gerado pelos elementos da
forma
1. e(am,n) − a · e(m,n) ; e(m,an) − a · e(m,n)
2. e(m+m′ ,n) − e(m,n) − e(m′ ,n)
3. e(m,n+n′ ) − e(m,n) − e(m,n′ )
onde m, m′ ∈ M , n, n′ ∈ N e a ∈ A. Denotamos a imagem de e(m,n) em M ⊗A N por m ⊗ n (chamados
de tensores elementares). Assim, os tensores elementares geram M ⊗A N e um elemento arbitrário
de M ⊗A N pode ser escrito como uma soma finita m1 ⊗ n1 + · · · + mk ⊗ nk de tensores elementares.
FT
Em M ⊗A N , 1–3 acima dão origem às relações correspondentes
1. (am) ⊗ n = a(m ⊗ n) = m ⊗ (an)
)
28

2. (m + m′ ) ⊗ n = m ⊗ n + m′ ⊗ n
3. m ⊗ (n + n′ ) = m ⊗ n + m ⊗ n′
7:
(1
RA

e assim o produto tensorial vem equipado de fábrica com um mapa bilinear


0

⊗: M × N → M ⊗A N
01

(m, n) 7→ m ⊗ n
,2

A “raison d’être” da construção acima é a seguinte propriedade universal, que caracteriza o produto
29

tensorial a menos de isomorfismo: para qualquer “módulo de teste T ”, dado um mapa bilinear f ∈
D

BilA (M × N, T ), existe um único morfismo de A-módulos fˆ: M ⊗A N → T fazendo o seguinte diagrama


ct
,O

comutar:
f
ET

M ×N - T
-

∃!

?
M ⊗N
39

De fato, como f é bilinear, o kernel do mapa


M
A · e(m,n) → T
(m,n)∈M×N
X X
ai e(mi ,ni ) 7→ ai f (mi , ni )
i i

P
contém
P o submódulo A, logo define um único mapa fˆ: M ⊗A N → T dado por fˆ( i ai (mi ⊗ ni )) =
i ai f (mi , ni ) e que faz o diagrama anterior comutar. Em outras palavras, temos uma bijeção

HomA (M ⊗A N, T ) = BilA (M × N, T )
fˆ 7→ fˆ ◦ ⊗

Assim, em termos categóricos, o produto tensorial representa o funtor BilA (M × N, −). Note que a
propriedade universal acima diz que é muito fácil construir um morfismo partindo de M ⊗A N : basta
construir uma aplicação bilinear partindo de M × N .
O produto tensorial é uma ferramenta geral que inclui diversas outras construções como quocientes e
localizações. O seguinte teorema lista as propriedades básicas que permitem escrever o produto tensorial
em termos que nos são mais familiares:

Teorema 3.1.1 (Isomorfismos Básicos) Seja A um anel. Temos isomofismos


1. (associatividade) (M ⊗A N ) ⊗A P = M ⊗A (N ⊗A P ) dado por (m ⊗ n) ⊗ p 7→ m ⊗ (n ⊗ p);
2. (elemento neutro) A ⊗A M = M dado por a ⊗ m 7→ am;
3. (comutatividade) M ⊗A N = N ⊗A M dado por m ⊗ n 7→ n ⊗ m;
L  L
4. (distributividade) M ⊗A Ni = (M ⊗A Ni ) dado por m ⊗ (ni ) 7→ (m ⊗ ni );
5. (localização) para qualquer conjunto multiplicativo S de A, (S −1 A) ⊗A M = S −1 M dado por
(a/s) ⊗ m 7→ am/s;
6. (quociente) para qualquer ideal a de A, M ⊗A (A/a) = M/aM dado por m ⊗ ā 7→ am.
7. (produto tensorial comuta com limite direto) se Mi é um sistema direto de A-módulos e N é
um A-módulo qualquer então

lim Mi ) ⊗A N = −→
(−→ lim (Mi ⊗A N )
FT
Prova As provas são simples, consistindo em verificar que os mapas acima estão bem definidos utilizando
)
a propriedade universal e em construir morfismos inversos explı́citos para estes mapas. A tı́tulo de
28

exemplo, provemos (6). Observe que o mapa M × (A/a) → M/aM dado por (m, ā) 7→ am independe
7:

da escolha do representante de classe de ā; além disso, ele é A-bilinear, logo pela propriedade universal
(1
RA

temos que f : M ⊗A (A/a) → M/aM dado por m ⊗ ā 7→ am está bem definido. Para mostrar que este
mapa é um isomorfismo, vamos construir o seu P inverso. Considere o mapa M → M ⊗A (A/a) definido
0

por m →
7 m ⊗ 1̄. Se m ∈ aM , digamos m = i ai mi com ai ∈ a e mi ∈ M , o mapa anterior leva m em
01

P P
( i ai mi ) ⊗ 1̄ = i mi ⊗ ai = 0. Portanto o mapa anterior induz um morfismo g: M/aM → M ⊗A (A/a)
,2

tal que g(m) = m ⊗ 1̄. Agora basta checar que f ◦ g e g ◦ f são os mapas identidade, o que é fácil: temos
29

que f ◦ g(m) = f (m ⊗ 1̄) = m e g ◦ f (m ⊗ a) = g(am) = am ⊗ 1̄ = m ⊗ a.


D
ct

Exemplo 3.1.2 Seja k um corpo e V e W dois espaços vetoriais de dimensão finita sobre k. Sejam
,O

ω1 , . . . , ωm e τ1 , . . . , τn bases de V e W respectivamente. Usando a distributividade, temos que


 M   M 
ET

M
V ⊗k W = k · ωi ⊗ k k · τj = k · ωi ⊗ τj
1≤i≤m 1≤j≤n 1≤i≤m
1≤j≤n

Logo V ⊗k W é um espaço vetorial com base ωi ⊗ τj , 1 ≤ i ≤ m, 1 ≤ j ≤ n, e portanto dimk (V ⊗k W ) =


(dimk V ) · (dimk W ).
40 Manobras Básicas

Exemplo 3.1.3 Sejam m, n ∈ Z e d = gcd(m, n). Temos

Z/(m)
Z/(m) ⊗Z Z/(n) = = Z/(d)
(n) · (Z/(m))

Note que se f : M → M ′ e g: N → N ′ são dois morfismos de A-módulos, então o mapa M × N →


M ⊗A N ′ dado por (m, n) 7→ f (m) ⊗ g(n) é A-bilinear. Assim, pela propriedade universal temos um

mapa de A-módulos f ⊗ g: M ⊗A N → M ′ ⊗A N ′ . Em particular, para um módulo fixo N , temos um


funtor − ⊗A N da categoria de A-módulos para ela mesma, que leva a flecha f : M → M ′ na flecha
f ⊗ id: M ⊗A N → M ′ ⊗A N . O teorema a seguir traduz uma das propriedades mais importantes do
produto tensorial:
Teorema 3.1.4 O produto tensorial é um funtor exato à direita, i.e., se

M
f
-N g
- P →0

é uma sequência exata de A-módulos, então o mesmo vale para

M ⊗T - N ⊗T
f ⊗id
- P ⊗T →0
g⊗id

para qualquer A-módulo T .


Prova Em primeiro lugar, observe que g ⊗ id é sobrejetor: dado um gerador p ⊗ t de P ⊗ T onde
p ∈ P e t ∈ T , pela exatidão da primeira sequência podemos encontrar n ∈ N tal que g(n) = p e assim
(g ⊗ id)(n ⊗ t) = p ⊗ t.
Agora seja I ⊂ N ⊗ T a imagem de f ⊗ id. Como (g ⊗ id) ◦ (f ⊗ id) = (g ◦ f ) ⊗ id = 0, temos que
I está claramente contido no kernel de g ⊗ id, assim g induz um morfismo φ: (N ⊗ T )/I → P ⊗ T . Para
mostrar que I = ker(g ⊗ id), basta portanto mostrar que φ é um isomorfismo, e para isto construı́mos
explicitamente o mapa inverso. Considere o mapa P × T → (N ⊗ T )/I dado por p ⊗ t 7→ n ⊗ t + I onde
n ∈ N é qualquer elemento tal que g(n) = p (com p ∈ P e t ∈ T ). Observe que se g(n′ ) = p então
n′ = n + f (m) para algum m ∈ M , logo n′ ⊗ t = n ⊗ t + (f ⊗ id)(m ⊗ t), ou seja, n′ ⊗ t + I = n ⊗ t + I,
logo o mapa anterior não depende da escolha da pré-imagem n de p. É fácil ver que este mapa também
é bilinear, logo define um morfismo ψ: P ⊗ T → (N ⊗ T )/I. Uma verificação imediata mostra que φ ◦ ψ
e ψ ◦ φ são os mapas identidade, o que completa a demonstração.

Exemplo 3.1.5 Produtos tensoriais podem destruir a injetividade: considere a sequência exata de
Z-módulos
FT
0 - Z - Z - Z/(2) - 0
2
)
onde Z
2
- Z denota a multiplicação por 2. Como o funtor M 7→ M ⊗Z Z/(2) é isomorfo ao funtor
28

M 7→ M/2M , tensorizando a sequência acima com Z/(2) obtemos a sequência


7:
(1
RA

0 - Z/(2) - Z/(2)
0
- Z/(2)
id
- 0
0
01

que não é exata à esquerda.


,2

3.2 Mudança de Base e Produto Tensorial de Álgebras


29

Uma das interpretações mais úteis do produto tensorial é como mudança de base. Seja B uma A-
D

P então B ⊗A P
ct

álgebra; se M é um A-módulo, M pode ser visto como um B-módulo via multiplicação


na primeira coordenada: b · ( bi ⊗ mi ) = (bbi ) ⊗ mi para b, bi ∈ B e mi ∈ M . Como o mapa
,O

B × M → B ⊗A M dado por (b′ , m) 7→ bb′ ⊗ m é bilinear, temos que a operação anterior está bem
ET

definida.
Em outras palavras, temos um funtor − ⊗A B da categoria de A-módulos para a categoria de B-
módulos. Este é o chamado funtor mudança de base. Intuitivamente, a operação − ⊗A B consiste
em “trocar os coeficientes” de A para B. Por exemplo, se K ⊂ L é uma extensão de corpos e V é
um K-espaço vetorial de dimensão n e base ω1 , . . . , ωn , temos que V ⊗K L é um L-espaço vetorial de
dimensão n e base ω1 ⊗ 1, . . . , ωn ⊗ 1.
41

Note que neste contexto produtos tensorias comutam com a mudança de base: dados A-módulos M
e N , temos um isomofismo de B-módulos
(B ⊗A M ) ⊗B (B ⊗A N ) = B ⊗A (M ⊗A N )
definido por (b ⊗ m) ⊗ (b′ ⊗ n) 7→ (bb′ ) ⊗ (m ⊗ n). Além disso, se M é um A-módulo e P é um B-módulo,
temos um isomorfismo de B-módulos (transitividade da mudança de base)
P ⊗B (B ⊗A M ) = P ⊗A M
dado por p ⊗ (b ⊗ m) 7→ bp ⊗ m. Aqui P ⊗A M é visto como B-módulo via multiplicação na primeira
coordenada.
Exemplo 3.2.1 (Invariância de Posto) Como uma aplicação simples da mudança de base, vamos
mostrar que o posto de um módulo livre está bem definido. Se An ∼ = Am como A-módulos então n = m.
De fato, se m é qualquer ideal maximal de A e k = A/m, aplicando − ⊗A k e usando a distributividade,
obtemos k n ∼= k m , logo m = n já que a dimensão de um k-espaço vetorial está bem determinada. Se
você achou esta aplicação babaca, então talvez você se surpreenda com o fato de que existem anéis
não-comutativos A para os quais R ∼ = R2 ! Um exemplo é dado pelo anel R = HomC (CN , CN ) dos
endomorfismos do C-espaço vetorial de dimensão contável CN .
Podemos tensorizar álgebras também: dadas duas A-álgebras B e C, o produto tensorial B ⊗A C
admite estrutura de anel via
X X  X
b i ⊗ ci b′j ⊗ c′j = bi b′j ⊗ ci c′j , bi , b′j ∈ B, ci , c′j ∈ C
i j i,j

Como temos um mapa multilinear B × C × B × C → B ⊗A C dado por (b, c, b′ , c′ ) 7→ bb′ ⊗ cc′ , utilizando
a propriedade universal é fácil verificar que a operação acima está bem definida. Desta forma, B ⊗A C
admite a estrutura de uma A-álgebra: multiplicação por a ∈ A é dada por a(b ⊗ c) = (ab) ⊗ c = b ⊗ (ac).
Exemplo 3.2.2 (Mudança de Base de Anéis de Polinômios) Seja B uma A-álgebra, então
A[x] ⊗A B = B[x] como B-álgebras, pois A[x] é um A-módulo livre com base xn , n ≥ 0. Assim,
pela distributividade temos um isomorfismo de A-módulos A[x] ⊗A B = B[x] e é fácil verificar que
este isomorfismo preserva as estruturas de B-módulos correspondentes. Em particular, obtemos que
A[x] ⊗A A[y] = A[x, y].
Exemplo 3.2.3 Seja B um A-módulo e f (x) ∈ A[x]. Tensorizando por B a sequência exata

- A[x]
f (x)
- A[x] - 0
A[x] 
f (x)
FT
(o mapa da esquerda é a multiplicação por f (x)), obtemos a sequência exata
f (x)
- B[x] - A[x] - 0
)
B[x]  ⊗A B
28

f (x)

7:

e, assim, o isomorfismo de B-álgebras A[x]/(f (x)) ⊗A B = B[x]/(f (x)).


(1
RA


Exemplo 3.2.4 O que é Q( 2) ⊗Q C como C-álgebra? Temos
0
01

√ Q[x] C[x] C[x] C[x]


Q( 2) ⊗Q C = 2
⊗Q C = 2 = √ × √ = C×C
(x − 2) (x − 2)
,2

(x − 2) (x + 2)
29

pelo exemplo anterior e pelo teorema Chinês dos Restos.


D

Exemplo 3.2.5 (Fibras como mudança de base) Seja φ: A → B um morfismo de anéis e p ∈ Spec A.
ct

A fibra de p com relação a Spec φ: Spec B → Spec A pode ser calculada via uma mudança de base: seja
,O

k(p) = Frac A/p o corpo residual do anel local Ap . Temos diagramas comutativos
B ⊗A k(p)  - Spec B
ET

B Spec B ⊗A k(p) ⊂

6 6
φ ⊗ id φ Spec φ ⊗ id Spec φ
? ?
k(p) = A ⊗A k(p)  A Spec k(p) ⊂ - Spec A
42 Manobras Básicas

onde A → k(p) é o mapa natural e B → B ⊗A k(p) é o mapa obtido tensorizando por B este mapa A-plano
natural. Sendo S = A \ p, temos que

B ⊗A k(p) = (B ⊗A S −1 A) ⊗S −1 A k(p) = S −1 B ⊗S −1 A (S −1 A/S −1 p) = S −1 B/S −1 pB = S −1 (B/pB)

e portanto B → B ⊗A k(p) é isomorfo a B → S −1 (B/pB), de modo que o diagrama à direita representa


a restrição de Spec φ à fibra de p.
Além do cálculo de fibras, outros teoremas também podem ser enunciados na linguagem do produto
tensorial, que possui a vantagem de ser concisa e funtorial. Por exemplo, temos
Teorema 3.2.6 (Nakayama, segunda versão) Seja (A, m, k) um anel local e M um A-módulo fini-
tamente gerado. Temos
M ⊗A k = 0 ⇐⇒ M = 0

Geometricamente, para um anel A qualquer, podemos pensar em um A-módulo M como uma


“famı́lia” de espaços vetoriais M ⊗A k(p), um sobre cada “ponto” p ∈ Spec A (aqui k(p) = Frac A/p denota
o corpo residual de Ap ). Nakayama pode então ser entendido como um resultado de “continuidade”: se
M é finitamente gerado e “se anula” sobre um “ponto” p, então M se anula em uma vizinhaça de p. De
fato, como
M ⊗A k(p) = Mp ⊗Ap k(p) = 0 ⇒ Mp = 0
e M é finintamente gerado, digamos M = Aω1 + · · · + Aωn , como ωi /1 = 0/1 em Mp , existem si ∈ A \ p
tais que si ωi = 0 em M e portanto h = s1 . . . sn anula M . Desta forma, Mq = 0 ⇒ M ⊗A k(q) = 0 para
todo q na vizinhaça aberta D(h) de p.
Temos ainda a seguinte reformulação:
Teorema 3.2.7 (Bases Minimais, segunda versão) Seja (A, m, k) um anel local e M um A-módulo
finitamente gerado. O número mı́nimo de geradores de M é dado por

dimk M ⊗A k

3.3 Módulos e Álgebras Planas


A falta de exatidão à esquerda no produto tensorial faz com que mudanças de base arbitrárias facilmente
“destruam” as relações lineares entre módulos. Por exemplo, considere o mapa f : Spec C[t, x]/(tx) →
Spec C[t] associado ao morfismo de C-álgebras C[t] → C[t, x]/(tx) dado por t 7→ t. Pense nas fibras de f
como uma “famı́lia” de subconjuntos algébricos de C parametrizadas pelo “tempo” t. Quando t 6= 0, de
tx = 0 temos que o conjunto algébrico correspondente é apenas a “origem” x = 0; porém quando t = 0,
FT
temos que o conjunto algébrico correspondente é todo o C. Esta “descontinuidade” pode ser entendida
algebricamente da seguinte forma: ao calcular a fibra de (t − a), aplicamos − ⊗C[t] C[t]/(t − a) à sequência
)
exata
28

C[t, x]
0 - C[t, x] - C[t, x] - - 0
tx
(tx)
7:
(1
RA

obtendo uma nova sequência (exata à direita) isomorfa a


0

0 - C[x] - C[x]
ax
- C[x]/(ax) - 0
01
,2

Note que quando a 6= 0, esta sequência é exata também à esquerda, porém isto é falso para a = 0; neste
último caso a relação “tx = 0” no último termo “se perde”.
29

A fim de evitar o problema acima, definiremos álgebras planas A → B como as álgebras para as
D
ct

quais a mudança de base sempre preserva todas as estruturas “lineares” dos módulos originais, ou seja,
para as quais − ⊗A B é exato. Geometricamente, isto se traduz em um “comportamento contı́nuo” para
,O

as fibras de Spec B → Spec A.


ET

Definição 3.3.1 Seja A um anel e M um A-módulo. Dizemos que M é A-plano se o funtor M ⊗A − é


exato. Se B é uma A-álgebra, dizemos que B é A-plana se ela é plana vista como A-módulo.
Observação 3.3.2 Como qualquer sequência exata pode ser quebrada em sequências exatas curtas e o
produto tensorial é sempre exato à direita, temos que um A-módulo M será plano se, e somente se, o
funtor M ⊗A − preserva injetividade.
43

Exemplo 3.3.3 (Módulos livres) Qualquer A-módulo livre é A-plano. Se B é uma A-álgebra que é
livre como A-módulo então B é A-plano sobre A. Por exemplo, B = A[x] é A-plano para qualquer anel
A, e se k é um corpo, qualquer k-álgebra é plana sobre k.

Exemplo 3.3.4 (Localização) Para qualquer conjunto multiplicativo S de A, temos que S −1 A ⊗A −


é isomorfo ao funtor localização, que é exato. Assim, S −1 A é uma álgebra A-plana.

Exemplo 3.3.5 Se a é um ideal de A, então B = A/a é A-plano se, e só se, a = 0; afinal de contas,
nada melhor do que quocientes para destruir a injetividade! Formalmente, basta tomar um elemento
não nulo a ∈ a e considerar a inclusão (a) ֒→ A; tensorizando por A/a obtemos o mapa zero (a)/(a) · a =
(a) ⊗A A/a → A ⊗A A/a = A/a, o que mostra que A/a não pode ser A-plano.

Mais tarde, utilizando métodos homológicos, veremos diversos critérios que permitem decidir na
prática se uma álgebra é ou não plana. Nesta seção, nos contentamos em listar e provar algumas de suas
propriedades básicas.

Teorema 3.3.6 Seja A um anel e M um A-módulo. Sejam φ: A → B e ψ: B → C duas álgebras.


1. (Estabilidade por Composição) Se φ: A → B e ψ: B → C são planas, o mesmo vale para
ψ ◦ φ: A → C.
2. (Estabilidade por Mudança de base arbitrária) Se M é plano sobre A, então M ⊗A B é plano
sobre B. Em particular, se S é um conjunto multiplicativo de A, S −1 M é plano sobre S −1 A
quando M for plano sobre A.
3. (Natureza local) M é plano sobre A se, e somente se, Mm é plano sobre Am para todos os ideais
maximais m de A.
4. (Natureza microlocal) Seja N um B-módulo. Então N é plano sobre A se, e somente se, Nn é
plano sobre Aφ−1 (n) para todos os ideais maximais n de B.

Prova (1) segue imediatamente do isomorfismo

M ⊗A C = M ⊗A B ⊗B C

que mostra que o funtor − ⊗A C é a composição dos funtores exatos − ⊗A B e − ⊗B C, sendo portanto
também exato. Da mesma forma, (2) segue do isomorfismo de funtores − ⊗B (M ⊗A B) = − ⊗A M .
Mostremos agora (3). De (2), já sabemos que se M é A-plano, então Mm é Am -plano. Reciproca-
mente, seja N ֒→ N ′ uma injeção de A-módulos; se Mm é Am -plano para todo ideal maximal m, como
FT
′ ′
Nm ֒→ Nm é injetor (exatidão da localização), Nm ⊗Am Mm ֒→ Nm ⊗Am Mm também é injetor. Porém
)
28

Nm ⊗Am Mm = (N ⊗A Am ) ⊗Am (M ⊗A Am ) = (N ⊗A M ) ⊗A Am = (N ⊗A M )m
7:

Portanto (N ⊗A M )m ֒→ (N ′ ⊗A M )m é injetor para todo m; pelo princı́pio local-global (teorema 1.8),


(1
RA

temos que N ⊗A M ֒→ N ′ ⊗A M é injetor, logo M é A-plano. A prova de (4) é similar e é deixada para
0

o leitor.
01

Seja q be a maximal ideal of B. Se N é a Bq -módulo,


,2
29

N ⊗Aφ−1 q Bq = N ⊗A Bq = (N ⊗A B) ⊗B Bq
D
ct

Como localisation preserves exactness, the computation above shows that se B é flat sobre A então Bq
,O

é flat sobre Aφ−1 q . Reciprocamente, suponha que Bq é flat sobre Aφ−1 q para todo maximal ideals q of
B. Seja N ′ ֒→ N be an injection of A-módulos, e denote por K the kernel of N ′ ⊗A B → N ⊗A B.
ET

Localising at q, temos por the computation above

0 → Kq → N ′ ⊗Aφ−1 q Bq → N ⊗Aφ−1 q Bq

e portanto Kq = 0 para todo maximal ideals q. Por the local-global principle, K = 0.


44 Manobras Básicas

Exemplo 3.3.7 (Blow-up não é plano) Sejam A = C[x, y] e B = C[x, y, z]/(y − xz), visto como fielmente plano
A-álgebra da maneira usual. Então B não é plano sobre A: as fibras de Spec B → Spec A não variam
“continuamente” já que há um salto de dimensão na origem. De fato, aplicando − ⊗A B à inclusão
(x, y) ֒→ A, obtemos um mapa
ψ: a ⊗A B → B
que não é injetor: temos que ψ(x ⊗ z − y ⊗ 1) = 0, porém x ⊗ z − y ⊗ 1 6= 0 em a ⊗A B. De fato, considere
o morfismo de A-módulos
A⊕A
φ: a →
{a(y, −x) | a ∈ A}
α = p(x, y)x + q(x, y)y 7→ (p(x, y), q(x, y)) mod b

É fácil verificar que este mapa está bem definido, isto é, independe da representação de α como com-
binação A-linear de x e y. Assim, temos um mapa φ⊗ id: a ⊗A B → (B × B)/b. Now φ⊗ 1(x⊗ z − y ⊗ 1) =
(z, −1) + b é not 0, e portanto x ⊗ z − y ⊗ 1 6= 0, as required.
Agora vamos definir álgebras e módulos com relação aos quais a mudança de base é “reversı́vel”.
Definição 3.3.8 Um A-módulo M é fielmente plano sobre A se M é A-plano e se vale a “propriedade
de cancelamento”
M ⊗A N ⇒ N = 0 para todo A-módulo N
Uma A-álgebra B é fielmente plana se ela é plana como A-módulo.
Exemplo 3.3.9 Qualquer álgebra livre é fielmente plana. Em particular, qualquer álgebra sobre um
corpo é fielmente plana.
Observação 3.3.10 Uma álgebra φ: A → B fielmente plana é sempre injetora, de modo que podemos
considerar A como subanel de B. De fato, seja a = ker φ. Da planaridade e da sequência exata

0 → a → A → A/a → 0

obtemos outra sequência exata

0 → a ⊗A B → A ⊗A B → (A/a) ⊗A B → 0

Porém o mapa A ⊗A B → (A/a) ⊗A B corresponde ao mapa B → B/φ(a)B = B, que é um isomorfismo.


Portanto a ⊗A B = 0 e a = 0, como querı́amos.

Teorema 3.3.11 (Fidelidade) Seja M um A-módulo. As seguintes condições são equivalentes:


FT
1. M é fielmente plano sobre A.
2. Para qualquer sequência de A-módulos
)
28

N′ - N
f
- N ′′
g
(∗)
7:
(1
RA

a sequência
N ′ ⊗A M - N ⊗A M
f ⊗id
- N ′′ ⊗A M
g⊗id
0

(∗∗)
01

é exata se, e só se, (∗) é exata.


,2

3. M é plano e mM 6= M para todo ideal maximal m de A.


29

Prova (1 ⇒ 2) Se (∗) é exata, então (∗∗) é exata pois M é A-plano. Reciprocamente, se (∗∗) é exata,
D
ct

como M é A-plano, temos que o funtor − ⊗A M comuta com kernels, imagens e quocientes e, assim,
,O


0 = im (f ◦ g) ⊗ id = im(f ◦ g) ⊗A M ⇒ f ◦ g = 0
ET

de modo que (∗) é um complexo. Além disso,

ker(g ⊗ id) (ker g) ⊗A M  ker g  ker g


0= = = ⊗A M ⇒ =0
im(f ⊗ id) (im f ) ⊗A M im f im f

e portanto (∗) é exata.


45

(2 ⇒ 1) Claramente 2 implica que M é A-plano. Como por hipótese 0 → N → 0 é exato se, e só se,
0 → M ⊗A N → 0 é exato, temos que M ⊗A N = 0 ⇒ N = 0.
(1 ⇒ 3) Como A/m 6= 0, temos M ⊗A A/m = M/mM 6= 0.
(3 ⇒ 1) Seja N um A-módulo qualquer e suponha que N 6= 0; temos que mostrar que M ⊗A N 6= 0.
Seja n ∈ N um elemento não nulo; como M é A-plano, temos que M ⊗A An ֒→ M ⊗A N é injetor, assim
basta mostrar que
M ⊗A An 6= 0 ⇐⇒ M ⊗A A/ ann(n) = M/ ann(n)M 6= 0
Mas isto é fácil: seja m um ideal maximal de A tal que m ⊃ ann(n), então M ) mM por hipótese, logo
M ) ann(n)M .

Do critério 3, obtemos
Corolário 3.3.12 Seja A um anel local e B uma A-álgebra local. Então B é A-plano se, e só se, é
fielmente A-plano.
A ideia de trabalharmos com álgebras fielmente planas é que, para verificar que um certo módulo
ou álgebra possui uma certa propriedade, é em geral mais fácil fazê-lo após uma mudança de base. Por
exemplo, qualquer extensão de corpos é fielmente plana, então pode-se trabalhar sobre o fecho algébrico
k alg de k após uma mudança de base −⊗k k alg e depois tentar “descer” ao corpo original. Como ilustração
desta filosofia, vejamos o
Exemplo 3.3.13 Seja B uma A-álgebra fielmente plana. Vamos mostrar que um A-módulo M é plano
sobre A se, e só se, o B-módulo M ⊗A B é plano sobre B. Como planaridade é estável por mudança de
base arbitrária, basta mostrar que se M ⊗A B é plano sobre B então M é plano sobre A. Dada uma
sequência exata de A-módulos
N ′ → N → N ′′ (∗)
tensorizando por M sobre A, obtemos uma nova sequência

N ′ ⊗A M → N ⊗A M → N ′′ ⊗A M

que será exata se, e só se,

(N ′ ⊗A M ) ⊗A B → (N ⊗A M ) ⊗A B → (N ⊗A M ) ⊗A B

é exata. Mas esta sequência é isomorfa a


FT
(N ′ ⊗A B) ⊗B (M ⊗A B) → (N ⊗A B) ⊗A (M ⊗A B) → (N ′′ ⊗A B) ⊗B (M ⊗A B)
)
28

que é obtida a partir da sequência original (∗) aplicando-se primeiro o funtor exato −⊗A B e, em seguida,
7:

o funtor exato − ⊗B (M ⊗A B).


(1
RA

A “reversibilidade” da mudança de base fielmente plana possui uma expressão geométrica:


0
01

Teorema 3.3.14 Seja A → B um morfismo plano de anéis. Então A → B é fielmente plano se, e só
se, Spec B → Spec A é sobrejetor.
,2

Prova Temos que Spec B → Spec A é sobrejetor se, e só se, para todo p ∈ Spec A, a fibra Spec B ⊗A k(p)
29

de p é não vazia, ou seja, se, e só se, B ⊗A k(p) 6= 0.


D
ct

Assim, se Spec B → Spec A é sobrejetora então, para todos os ideais maximais m de A, B ⊗A k(m) =
,O

B/mB 6= 0, logo A → B é fielmente plano pelo critério 3 do teorema anterior. Reciprocamente, se A → B


é fielmente plano, como k(p) 6= 0 para todo p ∈ Spec A, temos B ⊗A k(p) 6= 0 e portanto Spec B → Spec A
ET

é sobrejetor.

4 Condições de Finitude
Nesta seção, veremos algumas condições de finitude sobre anéis e módulos que frequentemente são veri-
ficadas na prática e alguns resultados importantes que fazem uso destas condições de finitude.
46 Manobras Básicas

4.1 Anéis e Módulos Noetherianos noetheriano

Definição 4.1.1 Um anel A é noetheriano se satisfaz qualquer uma das seguintes propriedades equiv-
alentes:
1. todo ideal a de A é finitamente gerado;
2. toda cadeia ascendente de ideais estabiliza, isto é, dada uma cadeia de ideais

a0 ⊂ a1 ⊂ a2 ⊂ a3 ⊂ · · ·

então ai = ai+1 para i suficientemente grande;


3. todo conjunto não vazio I de ideais possui um ideal que é maximal em I com relação à inclusão.

Vamos verificar a equivalência das condições acima.


S
• (1) ⇒ (2) Tome a = i≥0 ai , que é um ideal de A: dados a, b ∈ a e r ∈ A, escolha i grande o
suficiente para que a, b ∈ ai , de modo que a + b ∈ ai ⊂ a e ra ∈ ai ⊂ a. Sejam a1 , . . . , an ∈ A
geradores de a. Então existe um i0 grande suficiente tal que a1 , . . . , an ∈ ai0 , logo a = ai0 e portanto
ai = ai+1 para todo i ≥ i0 .
• (2) ⇒ (1) Seja a um ideal e tome a1 ∈ a. Se (a1 ) 6= a, tome a2 ∈ a \ (a1 ). Se (a1 , a2 ) 6= a, tome
a3 ∈ a \ (a1 , a2 ). E assim por diante. Como a cadeia (a1 ) ⊂ (a1 , a2 ) ⊂ (a1 , a2 , a3 ) ⊂ · · · estabiliza,
temos que a = (a1 , . . . , an ) para algum n.
• (2) ⇒ (3) Suponha que I não possua elemento maximal e seja a0 ∈ I. Então existe a1 ∈ I tal que
a0 ( a1 . Repetindo este procedimento, obtemos uma cadeia ascendente estrita a0 ( a1 ( a2 ( · · ·,
o que é um absurdo.
• (3) ⇒ (2) Dada uma cadeia ascendente a0 ⊂ a1 ⊂ a2 ⊂ · · ·, tome I = {ai | i ≥ 0}. Se ai0 é um
elemento maximal de I então devemos ter ai = ai+1 para todo i ≥ i0 .

Quase todos os anéis que aparecem na natureza são noetherianos: por exemplo, corpos e PIDs (tais
como Z) são noetherianos e veremos a seguir que qualquer álgebra finitamente gerada sobre um anel
noetheriano também é noetheriano. O exemplo clássico (isto é, visto em classe) de anel não noetheriano
é o anel de polinômios k[x1 , x2 , . . .] em um número infinito de variáveis sobre um corpo k (isto é, cada
elemento deste anel é um polinômio usual cujas variáveis estão em um subconjunto finito de {x1 , x2 , . . .}).
De fato, neste anel temos a cadeia ascendente estrita de ideias

(x1 ) ( (x1 , x2 ) ( (x1 , x2 , x3 ) ( · · ·


FT
Os axiomas acima podem ser interpretados como substitutos para o “princı́pio de indução finita”
ou para o “princı́pio da boa ordem” dos números naturais. A “inversão” da ordem se deve ao fato que,
)
para ideais, “conter significa dividir”: por exemplo, em Z, o fato de não existir uma cadeia ascendente
28

estrita de ideais (d1 ) ( (d2 ) ( (d3 ) ( · · · corresponde ao fato de não existir uma “cadeia descrescente de
7:

divisibilidade” · · · | d3 | d2 | d1 . Vejamos como utilizar este “novo PIF noetheriano” no seguinte


(1
RA

Exemplo 4.1.2 (Indução Noetheriana) Seja A um anel noetheriano. Vamos demonstrar que todo
0

ideal a contém um produto finito de ideais primos. Suponha que isto seja falso e seja I o conjunto
01

de ideais que não contêm produtos finitos de ideais primos. Seja b um elemento maximal em I. Por
/ b tais que ab ∈ b. Como (a) + b ) b e (b) + b ) b, pela
hipótese, b não é primo, logo existem a, b ∈
,2

maximalidade de b temos que ambos os ideais (a) + b e (b) + b contém produtos finitos de ideais primos.
29

Mas neste caso, como ab ∈ b, temos que b ⊃ (a) + b · (b) + b e, assim, b também contém um produto
D

finito de ideais primos, uma contradição.


ct
,O

O exemplo anterior mostra que um anel noetheriano A possui apenas um número finito de ideais
primos minimais: como (0) ⊃ p1 · · · pn para certos pi ∈ Spec A, temos que para qualquer q ∈ Spec A,
ET

q ⊃ p1 · · · pn , logo q ⊃ pi para algum i. Assim, os primos minimais de A formam um subconjunto dos


pi ’s acima.
Talvez o mais importante dos resultados sobre anéis noetherianos é o famoso

Teorema 4.1.3 (Basissatz Hilberts) Se A é um anel noetheriano, então A[x] e A[[x]] também são
noetherianos.
47

Prova Seja a um ideal de A[x]. Para mostrar que a é finitamente gerado, considere para cada inteiro noetheriano
d ≥ 0 o conjunto cd dos coeficientes lı́deres polinômios em a de grau d, juntamente com o 0. Temos que
cada cd é um ideal de A e que cd ⊂ cd+1 . Como A é noetheriano, esta cadeia de ideais estabiliza, logo
existe D tal que cd = cD para todo d ≥ D. Para cada d = 0, 1, . . . , D, escolha um número finito de
polinômios de grau d em a tais que seus coeficientes lı́deres geram cd . Seja {p1 (x), . . . , pn (x)} o conjunto
de todos os polinômios escolhidos. Vamos mostrar que estes polinômios geram a. Seja f (x) ∈ a. Temos
que o coeficiente lı́der de f (x) é uma combinação linear de coeficientes lı́deres dos pi (x), logo é possı́vel
escolher monômios mi (x) = ci xei tais que

deg f (x) − m1 (x)p1 (x) − · · · − mn (x)p(x) < deg f (x)

Por indução no grau de f (x), podemos supor que f (x) − m1 (x)p1 (x) − · · · − mn (x)p(x) ∈ a pertence ao
ideal (p1 (x), . . . , pn (x)), logo f (x) ∈ (p1 (x), . . . , pn (x)) também, como desejado.
A demonstração de que A[[x]] é noetheriano é análoga e é deixada como exercı́cio para o leitor.

Vejamos agora como obter uma plétora de anéis noetherianos!

Lemma 4.1.4 Seja A um anel noetheriano. Então


1. A/a é noetheriano para todo ideal a.
2. Se S é um conjunto multiplicativo, então S −1 A é noetheriano.
3. Qualquer A-álgebra finitamente gerada B é noetheriana.

Prova (1) segue diretamente do teorema da correspondência de ideais, enquanto que (2) segue do
teorema 1.9. Por fim, (3) é consequência do Basissatz Hilberts: se B é gerado sobre A por ω1 , . . . , ωn ,
temos uma sobrejeção de A-álgebras
A[x1 , . . . , xn ] ։ B
xi 7→ ωi

Se a é o kernel desta sobrejeção, temos portanto que B é isomorfo a A[x1 , . . . , xn ]/a. Como A[x1 , . . . , xn ]
é noetheriano pelo teorema anterior, por (1) temos que B também é noetheriano.

Observação 4.1.5 O produto tensorial de álgebras noetherianas sobre um anel noetheriano nem sempre
é noetheriano.

Como para anéis e ideais, podemos definir módulos noetherianos de maneira completamente análoga:
FT
Definição 4.1.6 Seja A um anel qualquer (i.e. não necessariamente noetheriano). Um A-módulo é
noetheriano se satisfaz uma (e portanto todas) das seguintes condições equivalentes:
)
28

1. todo submódulo N de M é finitamente gerado;


7:

2. toda cadeia ascendente de submódulos estabiliza, isto é, dada uma cadeia de submódulos de M
(1
RA

N0 ⊂ N1 ⊂ N2 ⊂ N3 ⊂ · · ·
0
01

então Ni = Ni+1 para i suficientemente grande;


,2

3. todo conjunto não vazio N de submódulos de M possui um elemento que é maximal em N com
29

relação à inclusão.
D
ct

Teorema 4.1.7 Seja A um anel.


,O

1. Seja
ET

0 - M′ f
- M g
- M ′′ - 0

uma sequência exacta de A-módulos. Então M é noetheriano se, e só se, M ′ e M ′′ são noethe-
rianos.
2. Seja M um A-módulo finitamente gerado. Se A é noetheriano, então M é noetheriano.
48 Manobras Básicas

Prova (1) Sem perda de generalidade, podemos supor que M ′ é submódulo de M e que M ′′ = M/M ′ .
Assim, se M é noetheriano, claramente M ′ é noetheriano e M ′′ é noetheriano pelo teorema de corre-
spondência de submódulos. Reciprocamente, suponha que M ′ e M ′′ são noetherianos. Dada uma cadeia
ascendente de submódulos de M
M1 ⊂ M2 ⊂ M3 ⊂ · · ·
temos que as cadeias de submódulos, respectivamente de M ′ e M ′′ ,

M1 + M ′ M2 + M ′ M3 + M ′
M1 ∩ M ′ ⊂ M2 ∩ M ′ ⊂ M3 ∩ M ′ ⊂ · · · e ⊂ ⊂ ⊂ ···
M′ M′ M′
estabilizam para i ≫ 0. Assim, basta provar que

Mi + M ′ Mi+1 + M ′
Mi ∩ M ′ = Mi+1 ∩ M ′ e ′
= ⇒ Mi = Mi+1
M M′
Tome mi+1 ∈ Mi+1 . Como mi+1 ∈ Mi+1 + M ′ = Mi + M ′ , existe mi ∈ Mi e m′ ∈ M ′ tal que
mi+1 = mi + m′ . Mas então m′ = mi+1 − mi ∈ Mi+1 ∩ M ′ = Mi ∩ M ′ . Portanto mi+1 = mi + m′ ∈ Mi
e assim Mi+1 ⊂ Mi . Como já temos a inclusão oposta, o resultado segue.
(2) Note que A é noetheriano como A-módulo e, por (1), um módulo livre de posto finito An também
é noetheriano. Se M é finitamente gerado sobre A, existe uma sobrejeção An ։ M para algum n (basta
levar os elementos da base de An para os geradores de M ). Logo, novamente por (1), M é noetheriano.

Encerramos esta seção com uma aplicação menos trivial. O método de demonstração a seguir, “de-
vissagé” (desmantelamento), é extremamente útil e importante e subjaz os diversos métodos homológicos
que serão estudados mais tarde.
Teorema 4.1.8 Seja A um anel noetheriano e B um A-módulo plano. Seja M um A-módulo finitamente
gerado e N um A-módulo qualquer. Então o mapa canônico

HomA (M, N ) ⊗A B → HomB (M ⊗A B, N ⊗A B)


φ ⊗ b 7→ φ ⊗ mb

(onde mb denota a multiplicação por b) é um isomorfismo de B-módulos.


Prova O funtor HomA (−, N ) ⊗A B é exato à esquerda, pois é a composição do funtor exato à esquerda
HomA (−, N ) e o funtor exato − ⊗A B. Da mesma forma, HomB (− ⊗A B, N ⊗A B) também é exato à
esquerda. Note que o mapa natural
FT
HomA (T, N ) ⊗A B → HomB (T ⊗A B, N ⊗A B)
)
φ ⊗ b 7→ φ ⊗ mb
28

é um isomorfismo quando T = An é livre, já que neste caso o mapa acima se transforma no isomorfismo
7:

N n ⊗A B = (N ⊗A B)n .
(1
RA

Como M é finitamente gerado, existe uma sobrejeção An ։ M , cujo kernel também é finitamente
0

gerado pois An é noetheriano. Assim, podemos escrever uma sequência exata


01

Am - An - M - 0
,2
29

e obtemos o seguinte diagrama comutativo com linhas horizontais exatas:


D

- HomA (M, N ) ⊗A B - HomA (An , N ) ⊗A B - HomA (An , N ) ⊗A B


ct

0
,O

≈ ≈
ET

? ? ?
0 - HomB (M ⊗A B, N ⊗A B) - HomA (B n , N ⊗A B) - HomB (B n , ⊗A B)

As duas flechas verticais da direita são isomorfismos, logo a flecha vertical da esquerda também, por um
“easy diagram chase” (ou aplicação do “lema dos 5”).
49

4.2 Anéis e Módulos Artinianos artiniano


simples
“Invertendo” a definição de anel noetheriano, obtemos a noção de anel artiniano (não, não é anel onaire- irredutı́vel
hteon!) série de comp
comprimento
Definição 4.2.1 Um anel A é artiniano se satisfaz qualquer uma das seguintes propriedades equiva-
lentes:
1. toda cadeia descendente de ideais estabiliza, isto é, dada uma cadeia de ideais
a0 ⊃ a1 ⊃ a2 ⊃ a3 ⊃ · · ·
então ai = ai+1 para i suficientemente grande;
2. todo conjunto não vazio I de ideais possui um ideal que é minimal em I com relação à inclusão.
Ao contrário dos anéis noetherianos, que existem em grande profusão (olhe para o chão, garanto que
você acabou de achar um anel noetheriano!), anéis artinianos são mais “raros”. Intuitivamente, anéis
artinianos são “anéis muito pequenos”; por exemplo, veremos que seus espectros são sempre finitos.
Exemplo 4.2.2 Seja k um corpo. Então um k-módulo M (vulgo k-espaço vetorial) é artiniano se, e
somente se, dimk M < ∞. De fato, se dimk M < ∞ então uma cadeia descendente de subspaços vetoriais
de M estabiliza pois as dimensões destes subespaços formam uma sequência não crescente de naturais,
que é eventualmente constante. Por outro lado, se dimk M L= ∞, tome uma sequência ω1 , ω2 , . . . de
elementos linearmente independentes de M e defina Mn = i≥n k · ωi . Então M0 ) M1 ) M2 ) · · · é
uma cadeia estritamente descendente de submódulos de M , e portanto M não é artiniano.
Exemplo 4.2.3 Anéis finitos, tais como Z/n, são claramente artinianos. O anel Z, por sua vez, não é
artiniano, pois temos a cadeia estritamente decrescente de ideais (2) ) (22 ) ) (23 ) ) · · · por exemplo.
O anel C[t]/(tn ) é artiniano, pois possui apenas um número finito de ideais, correspondentes aos ideais
(p(t)) de C[t] tais que p(t) | tn , ou seja, (ti ) com i = 0, 1, . . . , n.
A demonstração do teorema seguinte é completamente análoga ao teorema para módulos noetheri-
anos e é deixado como exercı́cio para o leitor ( = estou com preguiça de escrever a prova).
Teorema 4.2.4 Seja A um anel.
1. Seja
0 → M ′ → M → M ′′ → 0
uma sequência exata de A-módulos. Então M é artiniano se, e só se, M ′ e M ′′ são artinianos.
2. Seja M um A-módulo finitamente gerado. Se A é artiniano, então M é artiniano.

4.3 Comprimento de módulos


Nesta seção, vamos estender a noção de dimensão de espaços vetoriais para módulos que são simultane-
FT
amente noetherianos e artinianos.
Definição 4.3.1 Seja A um anel. Um A-módulo M 6= 0 é simples ou irredutı́vel se os seus únicos
submódulos são 0 e M . Equivalentemente, M é simples/irredutı́vel se, e só se, M ∼
)
= A/m para algum
28

ideal maximal m de A.
7:

Verifiquemos a equivalência acima. Se m é um ideal maximal de A, então M = A/m é um A-módulo


(1
RA

irredutı́vel, pois os submódulos de M correspondem aos submódulos de A (popularmente conhecidos


como ideais) que contêm m. Reciprocamente, se M é simples e m ∈ M é qualquer elemento não nulo,
0

então devemos ter M = Am e assim M ∼


01

= A/ ann(m). Novamente pelo teorema de correspondência de


ideais, devemos ter que ann(m) deve ser maximal.
,2

Definição 4.3.2 Dado um A-módulo M , uma série de composição de M de tamanho n é uma


29

sequência de submódulos
D

M = Mn ⊃ Mn−1 ⊃ Mn−2 ⊃ · · · M1 ⊃ M0 = 0
ct
,O

tais que os quocientes consecutivos Mi+1 /Mi são todos A-módulos simples. O comprimento de M
sobre A, denotado lenA M , é o mı́nimo entre todos os tamanhos das séries de composição de M , ou ∞
ET

se M não admite série de composição finita.


Por exemplo, um k-espaço vetorial é irredutı́vel se, e só se, tem dimensão 1. Assim, uma série de
composição para um espaço vetorial V é uma sequência
V = Vn ⊃ Vn−1 ⊃ Vn−2 ⊃ · · · V1 ⊃ V0 = 0
onde dimk Vi = i. Assim, lenk V = n = dimk V .
50 Manobras Básicas

Lemma 4.3.3 M possui comprimento finito se, e só se, M é artiniano e noetheriano.
Prova Suponha que M seja artiniano e noetheriano. Podemos construir uma série de composição da
seguinte maneira: dentre os submódulos não nulos de M , tome um submódulo minimal M1 , que existe
pois M é artiniano; temos que M1 é necessariamente simples. Dentre os submódulos de M que contêm
M1 propriamente, tome M2 minimal; deste modo M2 /M1 será simples também. Procedendo desta forma,
teremos uma cadeia ascendente própria
0 ( M1 ( M2 ( · · ·
que eventualmente terminará em M pois este é noetheriano. Ou seja, M admite uma série de composição
finita.
Para mostrar a recı́proca, faremos uma indução sobre lenA M < ∞. Se lenA M = 0 então M = 0 e
o resultado é claro. Se n = lenA M > 0, existe uma série de composição
M = Mn ⊃ Mn−1 ⊃ Mn−2 ⊃ · · · ⊃ M1 ⊃ M0 = 0
Assim, M/M1 admite uma série de composição
M/M1 = Md /M1 ⊃ Md−1 /M1 ⊃ · · · ⊃ M1 /M1 = 0
De fato, note que o quociente entre dois termos consecutivos é um módulo simples pois temos o isomofismo
Mi /M1 Mi
=
Mi−1 /M1 Mi−1
Assim, temos que lenA M/M1 ≤ d − 1, logo M/M1 é noetheriano e artiniano por hipótese de indução.
Como M1 é claramente noetheriano e artiniano pois é simples. Portanto M é noetheriano e artiniano.

O seguinte teorema é um caso especial do teorema de Jordan-Hölder.


Teorema 4.3.4 Seja M um A-módulo de comprimento finito. Então todas as séries de composição de
M têm tamanho lenA M .
Prova Novamente faremos uma indução em d = lenA M . Se d = 0, então M = 0 e o resultado é claro.
Agora seja d > 0 e suponha que o teorema valha para todos os módulos de comprimento estritamente
menores do que d. Tome uma série de composição de tamanho d para M :
M = Md ⊃ Md−1 ⊃ Md−2 ⊃ · · · ⊃ M1 ⊃ M0 = 0
Como na demonstração do lema anterior, temos lenA M/M1 ≤ d − 1, logo todas as séries de composição
FT
de M/M1 têm tamanho lenA M/M1 = d − 1 por hipótese de indução. Agora seja
M = Me′ ⊃ Me−1
′ ′
⊃ Me−2 ⊃ · · · ⊃ M1′ ⊃ M0′ = 0
)
28

uma segunda série de composição de M de tamanho e ≥ d. Como M1 é simples, para cada i = 0, 1, . . . , e


7:

temos que Mi′ ∩ M1 = 0 ou Mi′ ∩ M1 = M1 . Seja r o menor ı́ndice para o qual Mr′ ∩ M1 = M1 ⇐⇒
Mr′ ⊃ M1 (que existe pois Mn′ ∩ M1 = M1 ). Então afirmamos que
(1
RA

M M′ M′ M′ M ′ + M1 M ′ + M1 M ′ + M1
0

= e ⊃ e−1 ⊃ · · · ⊃ r = r−1 ⊃ r−2 ⊃ ··· ⊃ 0 =0


01

M1 M1 M1 M1 M1 M1 M1
,2

é uma série de composição de M/M1 , logo tem comprimento d − 1 = e − 1 ⇒ e = d, o que encerra a


prova do teorema.
29

Para verificar a afirmação, observe inicialmente que Mr′ = Mr−1



+ M1 pois Mr′ /Mr−1

é simples,
D

′ ′ ′
ct

logo não há submódulos de M estritamente entre Mr−1 e Mr , e M1 6⊂ Mr . Por outro lado, temos que os
quocientes entre termos consecutivos da série acima são simples pois
,O

Mi′ + M1 M′
ET

= ′ i = Mi′ para i = 0, 1, . . . , r − 1
M1 Mi ∩ M1
e
Mi′ /M1 M′
′ = ′i para i = r + 1, r + 2, . . . , e
Mi−1 /M1 Mi−1
51

Teorema 4.3.5 (Aditividade em Sequências Exatas) Seja A um anel e


0 - M′ - M - M ′′ - 0

uma sequência exata de A-módulos. Então M possui comprimento finito sobre A se, e somente se, M ′
e M ′′ possuem comprimento finito sobre A. Neste caso,
lenA M = lenA M ′ + lenA M ′′

Prova Sem perda de generalidade podemos supor que M ′ ⊂ M e que M ′′ = M/M ′ . Temos que
M é artiniano (respectivamente noetheriano) se, e só se, M ′ e M ′′ são artinianos (respectivamente
noetherianos), logo M possui comprimento finito se, e só se, M ′ e M ′′ possuem comprimento finito.
Neste caso, para mostrar a aditividade dos comprimentos, dadas duas séries de composição
M ′ = Md′ ⊃ Md−1
′ ′
⊃ Md−2 ⊃ · · · M1′ ⊃ M0′ = 0
e
M ′′ = Me′′ ⊃ Me−1
′′ ′′
⊃ Me−2 ⊃ · · · M1′′ ⊃ M0′′ = 0
de M ′ e M ′′ , basta combiná-las em uma série de composição de M de tamanho d + e: se M̃i′′ denota o
submódulo de M contendo M1 correspondente ao submódulo Mi′′ de M ′′ , então
′′
M = M̃e′′ ⊃ M̃e−1 ′
⊃ · · · ⊃ M̃0′′ = M ′ = Md′ ⊃ Md−1 ⊃ · · · ⊃ M0′ = 0
é uma série de composição de M , como desejado.

Exemplo 4.3.6 Vamos calcular o comprimento ℓ(n) do C[x]-módulo C[x]/(xn ). Para n = 0 temos que
ℓ(0) = 0 e para n ≥ 1 temos uma sequência exata

- C[x] C[x]
n−1
0 - (x ) - - 0
(xn ) (xn ) (xn−1 )
Como a multiplicação por x induz um isomorfismo
C[x] - (x

n−1
)
(x) x (xn )
e lenC[x] C[x]/(x) = 1 pois C[x]/(x) é simples (o ideal (x) é maximal), temos pela aditividade do compri-
mento em sequências exatas curtas que ℓ(n) = ℓ(n − 1) + 1. Assim, ℓ(n) = n para todo n natural.
Exemplo 4.3.7 Vamos mostrar que o comprimento do C[x, y]-módulo C[x, y]/(xn , y m ) é mn. Isto é
FT
claro se m = 0 ou n = 0, então vamos supor que m, n > 0. Temos uma sequência exata

- C[x, y] C[x, y]
n−1 m
0 - (x ,y ) - - 0
)
(xn , y m ) (xn , y m ) (xn−1 , y m )
28
7:

Pela aditividade de comprimentos e por hipótese de indução em n, basta agora mostrar que o comprimento
de M = (xn−1 , y m )/(xn , y m ) é m. Temos uma sobrejeção C[x, y] ։ M dada por f (x, y) 7→ xn−1 f (x, y),
(1
RA

que induz um isomorfismo


C[y] ∼ C[x, y] ∼-
0

= M
01

(y m ) (x, y m )
,2

Note que como M é anulado por x, podemos vê-lo como um módulo sobre C[y] ∼ = C[x, y]/(x), assim
temos que o comprimento de M é o comprimento do C[y]-módulo C[y]/(y m ), que é m pelo exemplo
29

anterior.
D
ct

Observação 4.3.8 Nos exemplos acima, temos que os comprimentos das C-álgebras de dimensão finita
,O

C[x]/(xn ) e C[x, y]/(xn , y m ) coincidem com suas dimensões sobre C (uma base sobre C para a segunda
álgebra é dada pelos monômios da forma xi y j com 0 ≤ i ≤ n − 1 e 0 ≤ j ≤ m − 1). Porém, se k é um
ET

corpo e A uma k-álgebra de dimensão finita qualquer, em geral, lenA A 6= dimk A. Para ver isto, basta
tomar por exemplo A como um corpo que é extensão finita de k com dimk A > 1; como A é simples,
temos lenA A = 1.
(interpretação geométrica como multiplicidade de intersecção)
Utilizando o comprimento, podemos agora provar o importante
52 Manobras Básicas

Teorema 4.3.9 (Estrutura de Anéis Artinianos) Seja A um anel artiniano.


1. Spec A = {m1 , . . . , mn } é finito e todos os seus elementos são ideais maximais (e portanto ideais
primos minimais também).
2. dim A = 0.
3. A é noetheriano.
4. O mapa canônico
A - Am1 × · · · × Amn

é um isomorfismo.

Prova Em primeiro lugar, vamos mostrar que A possui um número finito de ideais maximais. Por
contradição, suponha que haja uma quantidade infinita de ideais maximais m1 , m2 , . . .. Temos uma
cadeia descendente de ideais
m1 ) m1 m2 ) m1 m2 m3 ) · · ·

Para chegar a uma contradição, basta verficar que a cadeia acima é estrita: se m1 . . . mn = m1 . . . mn+1 ,
então mn+1 ⊃ mi para algum i, 1 ≤ i ≤ n, e como ambos os ideais são maximais, devemos ter mn+1 = mi ,
uma contradição.
Agora seja a = m1 . . . mn = m1 ∩ · · · ∩ mn a intersecção de todos os ideais maximais m1 , . . . , mn de
A (o chamado radical de Jacobson de A). Vamos mostrar que am = 0 para algum m. Isto implicará
que todo ideal primo p de A é maximal, pois p ⊃ am e portanto p = mi para algum i. Desta forma,
concluiremos também que Spec A é finito e que dim A = 0.
Como as potências de a formam uma cadeia descendente, existe um m tal que am = am+1 ; provemos
que am = 0. Por contradição, suponha que am 6= 0 e seja S o conjunto de todos os ideais b tais que
bam 6= 0. Temos S 6= ∅ já que (1) ∈ S, assim existe um ideal minimal b em S. Isto significa que xam 6= 0
para algum x ∈ b e pela minimalidade de b temos que b = (x) é principal. Por outro lado,

(bam ) · am = ba2m = bam 6= 0

e portanto bam pertence a S também; novamente pela minimalidade de b, temos b = bam ⇐⇒ (x) =
xam . Consequentemente, x = xa ⇐⇒ (1 − a)x = 0 para algum a ∈ am . Mas 1 − a ∈ A× já que
a ∈ a e portanto 1 − a não pertence a nenhum mi . Assim, (1 − a)x = 0 ⇐⇒ x = 0, o que contradiz
bam = xam 6= 0. Portanto am = 0, como afirmado.
Para mostrar que A é noetheriano, vamos mostrar que A tem comprimento finito sobre si mesmo.
FT
Considere a cadeia
A ⊃ m1 ⊃ m1 m2 ⊃ m1 m2 m3 ⊃ · · · ⊃ a = m1 . . . mn
)
⊃ am1 ⊃ am1 m2 ⊃ · · · ⊃ a2
28

⊃ a2 m1 ⊃ a2 m1 m2 ⊃ · · ·
7:

⊃ am−1 ⊃ am−1 m1 ⊃ · · · ⊃ am = 0
(1
RA
0

Sejam M e mi M dois termos consecutivos desta cadeia. Basta mostrar que M/mi M tem comprimento
01

finito sobre A, ou seja, que M/mi M é um espaço vetorial de dimensão finita sobre A/mi . Mas isto é
,2

claro, pois o ideal M é um A-módulo artiniano, logo M/mi M é artiniano sobre A e também sobre A/mi
e um espaço vetorial sobre um corpo é artiniano se, e só se, é de dimensão finita.
29

Finalmente, para mostrar que o mapa natural (produto dos mapas de localização)
D
ct

A → Am1 × · · · × Amn
,O
ET

é um isomorfismo, basta mostrar que todas as suas localizações com relação aos ideais maximais de A
são isomorfismos, o que é claro.

4.4 Álgebras e Módulos de Presentação Finita


Nesta seção, apresentamos certas condições “relativas” de finitude.
53

Definição 4.4.1 Seja A um anel. Um A-módulo M é de presentação finita sobre A se existem m e presentação fi
n e uma sequência exata de A-módulos
Am - An - M - 0

Uma A-álgebra B é de presentação finita se ela se escreve como um quociente


A[x1 , . . . , xn ]
B∼
= fi ∈ A[x1 , . . . , xn ]
(f1 , . . . , fm )
Se A é um anel noetheriano, então todo módulo finitamente gerado e toda álgebra finitamente gerada
são de presentação finita. Por exemplo, se B é gerado como A-álgebra por ω1 , . . . , ωn , então temos uma
sobrejeção de A-álgebras
A[x1 , . . . , xn ] ։ B
xi 7→ωi
Mas como A[x1 , . . . , xn ] é noetheriano pelo Basissatz Hilberts, temos que o kernel deste mapa é finita-
mente gerado, logo B é de fato de presentação finita sobre A. Isto mostra que “ser de presentação finita”
é uma espécie de “noetherianidade relativa”, já que A não é, necessariamente, noetheriano.
Teorema 4.4.2 (Morfismos de presentação finita)
1. (Estabilidade por composição) Se A → B e B → C são álgebras de presentação finita, o mesmo
vale para a composição A → C.
2. (Estabilidade sob mudança de base arbitrária) Seja B uma A-álgebra de presentação finita e A′
uma A-álgebra arbitrária. Então B ⊗A A′ é de presentação finita sobre A′ .
3. A localização Ah de um anel A em um elemento h ∈ A é de presentação finita sobre A, a saber
Ah ∼
= A[x]/(xh − 1).
4. Se φ: B ։ C é um morfismo sobrejetor de A-álgebras de presentação finita então ker φ é um
ideal finitamente gerado de B.

Prova Os itens (1)–(3) são deixados como exercı́cios para o leitor; aqui provaremos apenas o item (4).
Como B é o quociente de uma álgebra polinomial A[x1 , . . . , xn ] por um ideal finitamente gerado, basta
provar o teorema para B = A[x1 , . . . , xn ]. Escreva C = A[y1 , . . . , ym ]/a onde a é um ideal finitamente
gerado de A[y1 , . . . , ym ]. Como
A[y1 , . . . , ym , x1 , . . . , xn ]
C=
a + (x1 , . . . , xn )
é de presentação finita sobre B, podemos reduzir a prova para o caso B = A. A prova agora é por
indução no número de variáveis m e é suficiente provar a afirmação para m = 1: pois neste caso, como
FT
φ: A ։ C se fatora como
ψ A[y1 , . . . , ym ]
A → A[y1 , . . . , ym−1 ] ։ C =
)
a
28

com ψ sobrejetor, temos que ker ψ é finitamente gerado e podemos escrever C = A[y1 , . . . , ym−1 ]/ ker ψ.
7:

Portanto temos uma sobrejeção φ: A ։ A[y]/a de A-álgebras onde a é um ideal finitamente gerado de
(1
RA

A[y] e temos que mostrar que ker φ também é finitamente gerado. Mas como y está na imagem de φ existe
a ∈ A tal que y − a ∈ a. Logo se f1 (y), . . . , fr (y) são geradores de a então C ∼ = A/ f1 (a), . . . , fr (a) , i.e.,
0
01

o kernel de φ pode ser gerado por f1 (a), . . . , fr (a).


,2

O seguinte resultado explicita a relação entre álgebras de presentação finita e anéis noetherianos.
Este teorema (e seus companheiros) são frequentemente utilizados para reduzir demonstrações que en-
29

volvam álgebras gerais de presentação finita ao caso noetheriano. Como não utilizaremos este teorema,
D
ct

omitimos sua prova (que é um pouco longa, mas não é particularmente difı́cil), referindo o leitor a
,O

EGAIVc , 8.9.1, pág. 34.


Teorema 4.4.3 (“Redução Noetheriana”) Seja A um anel e B uma A-álgebra de presentação finita.
ET

Então existe um anel noetheriano A0 , um mapa A0 → A e uma A0 -álgebra B0 finitamente gerada tal
que B = B0 ⊗A0 A.
A ideia central da prova é escrever A como união (limite direto) de suas subálgebras finitamente
geradas sobre Z e utilizar o fato que as relações que definem B sobre A podem ser expressas em termos
de elementos de algum destes subanéis.
54 Manobras Básicas

5 Anéis completos álgebra do blow-up


álgebra de Rees

5.1 Topologia a-ádica e o teorema de Artin-Rees


Seja A um anel e a um ideal. O conjunto das potências an de a formam uma base de abertos do 0; a
topologia definida em A é chamada de topologia a-ádica. É fácil mostrar que as operações
T de soma e
produto são contı́nuas com relação a esta topologia e que A é Hausdorff se, e somente se, n≥0 an = (0).
Da mesma forma, dado um A-módulo M , o conjunto an M é uma base de abertos de 0 que define a
topologia a-ádica de M .
O próximo teorema diz que a topologia a-ádica de um submódulo N de um módulo M coincide com
a topologia induzida pela topologia a-ádica de M .
Teorema 5.1.1 (Artin-Rees) Seja a um ideal de um anel noetheriano A e seja M um A-módulo
finitamente gerado. Seja N um submódulo de M . Então existe um r tal que para todo n ≥ r

(an M ) ∩ N = an−r · (ar M ) ∩ N
Em particular, para n grande o suficiente, an N ⊂ (an M ) ∩ N ) ⊂ an−r N , logo a topologia a-ádica em N
é a mesma que a induzida pela topologia a-ádica de M .
Prova Considere o anel graduado Ba (A)
def
Ba (A) = A ⊕ a ⊕ a2 ⊕ · · ·
Este anel é chamado de álgebra do blow-up ou álgebra de Rees de a. Temos também o Ba (R)-módulo
graduado Ba (M )
def
Ba (M ) = M ⊕ aM ⊕ a2 M ⊕ · · ·
Afirmamos que o anel Ba (A) é noetheriano e o módulo Ba (M ) é finitamente gerado sobre Ba (A). De
fato, sejam a1 , . . . , an geradores de a. Temos um morfismo sobrejetor
φ : A[x1 , . . . , xn ] ։ Ba (A)
xi 7→ (0, ai , 0, 0, . . .)
de A-álgebras graduadas, logo Ba (A) é finitamente gerada sobre A e portanto é noetheriana também. Por
outro lado, se M = Am1 +· · ·+Amd , então Ba (M ) é generada sobre Ba (A) pelos elementos (mi , 0, 0, . . .),
1 ≤ i ≤ d. Desta forma, Ba (M ) é um módulo noetheriano.

A inclusão (an M ) ∩ N ⊃ an−r · (ar M ) ∩ N é clara. Para mostrar a inclusão oposta, considere o
submódulo de Ba (M )
def
M
P = (an M ) ∩ N
FT
n≥0

Como Ba (M ) é noetheriano, P é finitamente gerado sobre Ba (A). Fixe um conjunto de geradores


)
homogêneos e seja r o grau máximo de um elemento deste conjunto. Então, para n ≥ r,
28

X
7:

Pn = Ba (A)n−i · Pi ⇒
(1
RA

0≤i≤r
X  
n
(a M ) ∩ N = an−i · (ai M ) ∩ N ⊂ an−r · (ar M ) ∩ N
0
01

0≤i≤r
,2
29

Observação 5.1.2 Geometricamente, Proj Ba (A) corresponde ao blow-up de Spec A com centro no
subesquema fechado Spec A/a, daı́ o nome de álgebra de blow-up. Por exemplo, se A = C[x, y] e
D
ct

a = (x, y), o ideal correspondendo à “origem” de C2 , temos um isomorfismo de C[x, y]-álgebras graduadas
,O

(exercı́cio!)
C[x, y, w, z] ∼-
ET

B(x,y) (C[x, y])


(yw − xz)
w 7→ (0, x, 0, 0, . . .)
z 7→ (0, y, 0, 0, . . .)
e a variedade definida por yw − xz = 0 é o blow-up usual do plano C2 na origem.
55

Teorema 5.1.3 (Intersecção de Krull) Seja A um domı́nio noetheriano e a um ideal próprio de A. sequência de
completo
Então \ completamen
an = (0)
n≥0

T
Prova Seja b = n≥0 an . Pelo teorema de Artin-Rees, existe c tal que, para n ≫ 0, b = b ∩ an+c ⊂ ban ,
isto é, b = ba. Localizando em um ideal maximal m ⊃ a e aplicando Nakayama, temos que bm = 0.
Como A é domı́nio, temos que isto implica b = 0.

5.2 Completamento
Seja A um anel e a ⊂ A um ideal qualquer. Uma sequência an ∈ A é uma sequência de Cauchy se
dado d > 0 existe um n0 > 0 tal que

m, n ≥ n0 ⇒ am − an ∈ ad

Dizemos que uma sequência (an ) em A converge para um elemento a ∈ A se dado d > 0 existe um
n0 > 0 tal que
n ≥ n0 ⇒ an − a ∈ a d

Um anel A é completo com relação à topologia a-ádica se toda a sequência de Cauchy em A converge.
Podemos construir o completamento de A como o limite projetivo
n Y o

lim A/an = (an ) ∈
 = ←− A/an an ≡ am (mod am ) para todo n ≥ m
n≥1

Lemma 5.2.1 Seja a be an ideal of a noetherian anel A. Seja M be an A-módulo e N be a finite


submódulo of M . Então
lim N/an N = ←−
←− lim N/(N ∩ an M )

Prova temos natural maps fn : N/an N → N/(N ∩ an M ) which determines a mapa f : ←− lim N/an N =
n n
lim N/(N ∩ a M ). Seja us mostrar que f é an isomorfismo. Seja (Xn ) ∈ ←−
←− lim N/a N be tal que
fn (Xn ) = 0 para todo n. Então fn+r (Xn+r ) = 0 implies Xn = φn+r,n (Xn+r ) = 0 por the Artin-Rees
theorem. Portanto f é injetivo. Now seja (yn ) ∈ ←−lim N/(N ∩ an M ). Choose X̃n tal que fn (X̃n ) =
FT
yn e define Xn = φn+r,n (X̃n+r ). Como fn+r (φn+r+1,n+r (X̃n+r+1 ) − X̃n+r ) = 0, again por Artin-
lim N/an N e
Rees 0 = φn+r,n (φn+r+1,n+r (X̃n+r+1 ) − X̃n+r ) = φn+1,n (Xn+1 ) − Xn . Portanto (Xn ) ∈ ←−
)
f (Xn ) = (yn ), proving that f é surjective.
28
7:

d = A[[x]]
Exemplo 5.2.2 A[x]
(1
RA

 P 1
Observação 5.2.3 Frac Z[[x]] =6 Q((x)). Por exemplo, se pn denota o n-ésimo primo, então n≥0 ·
0

 pn
01

tn ∈ Q((x)) \ Frac Z[[x]] .


,2

We can now mostrar que under our assumptions completion preserve exactness.
29

Teorema 5.2.4 Seja a be an ideal of a noetherian anel A. Se


D
ct

- M′ - M
f g
- M ′′ - 0
,O

0
ET

é an exact sequence of finite módulos, então the corresponding sequence of a-adic completions


0 - M̂ ′ - M̂ - M̂ ′′

- 0

ˆ
é also exact. In particular, temos (M/M ′ ) = M̂ /M̂ ′ .
56 Manobras Básicas

Prova We may assume that M ′ é a submódulo of M e that f é the inclusão mapa. Então the exact
sequence
M′ - M - M
′′
0 - - 0
f g
M ′ ∩ an M an M an M ′′
determines an exact sequence of inverse systems

- lim M′ - lim M
f
- lim M
g
′′
- 0
0 ←− M ′ ∩ an M ←− an M ←− an M ′′

which we will prove to be exact. Exactness at the first two módulos é easy, seja us just check the last
one.
lim M ′′ /an M ′′ , we shall construct elements Xn ∈ M/an M inductively de modo que
Dado (yn ) ∈ ←−
(Xn ) ∈ ←− lim M/an M e gn (Xn ) = yn . suponha que Xn−1 has already been determined, choose X̃n ∈
M/a M tal que gn (X̃n ) = yn . Now gn−1 (Xn−1 − φn,n−1 (X̃n )) = 0, portanto existe a z ∈ f (M ′ )/f (M ′ )∩
n

an M com in−1 ◦ φn,n−1 (z) = Xn−1 − φn,n−1 (X̃n ). Defining Xn = z + X̃n , temos que φn,n−1 (Xn ) = Xn−1
e gn (X̃n ) = yn . This shows that g é surjective.
Now combining the above com the previous corollary finishes the proof.

Teorema 5.2.5 Seja A be a noetherian anel, a e b be ideals of A e M be a finite A-módulo. Seˆdenotes


the a-adic completion, temos
1. M ⊗A Â = M̂
2. (bM )ˆ= bM̂ = b̂M̂

Prova 1. Consider the exact sequence Am → An → M → 0. temos the seguinte diagrama comutativo

Am ⊗ Â - An ⊗ Â - M ⊗ Â - 0

? ? ?
Âm - Ân - M̂ - 0

The top row é exact como the tensor product é right exact; the bottom row é exact por the previous
theorem e the isomorfismo Âm = (Am )ˆ. Now the first two vertical flechas são isomorfismo, portanto so
é the third.
FT
2. Seja b = Ab1 + · · · Abn e define f : M n → M por f (m1 , . . . , mn ) = b1 m1 + · · · + bn mn . temos an
exact sequence
)
28

M n - M - M/bM - 0
f g
7:

e portanto
(1
RA

M̂ n
f
- M̂ - (M/bM )ˆ
g
- 0
0

é also exact. Portanto bM̂ = im(fˆ) = ker(ĝ). Como (M/bM )ˆ = M̂ /(bM )ˆ, ker(ĝ) = (bM )ˆ e portanto
01

bM̂ = bM )ˆ. Finally, applying this last result to M = A, temos b = b̂, e portanto b̂M̂ = bM̂ , completing
,2

the proof.
29
D
ct

Teorema 5.2.6 Seja a be an ideal of a noetherian anel A. Se a = Aa1 + · · ·+ Aan , the a-adic completion
,O

of A é isomorphic to A[[X1 , . . . , Xn ]]/(X1 − a1 , . . . , Xn − an ). In particular, Â é noetherian.


Prova Seja A = A[X1 , . . . , Xn ], m = Ax1 + · · · + Axn e n = A(X1 − a1 ) + · · · + A(Xn − an ). Denoting
ET

por ˆ the m-adic completion, temos que A ∼ = A/n implies  ∼


= Â/n̂ = Â/n = A[[X1 , . . . , Xn ]]/(X1 −
a1 , . . . , Xn − an ). However, the m-adic completion of A as an A-módulo coincides com the a-adic com-
pletion of A as a anel, e the result follows. Como A é noetherian, A[[X1 , . . . , Xn ]] é also noetherian e
portanto so é Â.
57

Lema 5.2.7 (Hensel) Seja (A, m, k) um anel local completo. Se f (x) ∈ A[x] é um polinômio mônico tal

que sua imagem f (x) ∈ k[x] = A[x]/mA[x] possui uma raiz simples α em k (i.e. f (α) = 0 e f (α) 6= 0)
então f (x) possui uma raiz a ∈ A tal que a mod m = α.

Lema 5.2.8 (Nakayama Completo) Seja (A, m, k) um anel local T completo e M um A-módulo (não
necessariamente finitamente gerado sobre A) que é m-separado, i.e., n≥0 mn M = 0. Então

M ⊗A k = 0 ⇐⇒ M = 0

Teorema 5.2.9 (Preparação de Weierstraß)  Seja (A, m, k) um anel local completo e f (x) ∈ A[[x]]
tal que f (x) ∈
/ mA[[x]]. Então A[[x]]/ f (x) é uma extensão finita de A.
Prova Temos
A[[x]] k[[x]]
 ⊗A k = 
f (x) f (x)
é finita sobre k. Utilizando o Nakayama completo, podemos levantar uma base desta k-álgebra.

Teorema 5.2.10 Seja A um anel noetheriano, a um ideal de A e M um A-módulo finitamente gerado.


Denote por  e M̂ os completamentos a-ádicos de A e M , respectivamente. Então
1. Temos um isomorfismo M ⊗A Â = M̂ .
2. Â é plano sobre A.
3. se a = (a1 , . . . , an ) então
A[[x1 , . . . , xn ]]
 ∼
=
(x1 − a1 , . . . , xn − an )
e portanto  também é noetheriano.

Lemma 5.2.11 Seja (A, m) be a local noetherian anel, e M be a finitely generated A-module. Se N é a
submodule of M generated por ω1 , . . . , ωd , temos

N̂ = Âω1 + · · · + Âωd

as a submodule of M̂ , onde the hat denotes the completion com respect to the m-adic topology. In
particular, se a é an ideal of A, então â = aÂ.
FT
Prova Dado an element x̂ = â1 ω1 + · · · + âd ωd of Âω1 + · · · + Âωd , âi ∈ Â, choose Cauchy sequences
(ain )n≥1 in A converging to âi , i = 1, . . . , d; então xn = a1n ω1 + · · · + adn ωd são the terms of a Cauchy
)
28

sequence in N converging to x̂, portanto x̂ ∈ N̂ . Reciprocamente, suponha que x̂ ∈ N̂ , e seja (xn )n≥1
be a Cauchy sequence converging to x̂ tal que xn+1 − xn ∈ mn N para todo n. Define x0 = 0 e choose
7:

elements ain ∈ A tal que


(1
RA

xn+1 − xn = a1n ω1 + · · · + adn ωd , ain ∈ mn


0
01

Então X  X   X 
,2

xn = (xj+1 − xj ) = a1j ω1 + · · · + adj ωd


29

0≤j<n 0≤j<n 0≤j<n


P
D

Como cada sum 0≤j<n aij converges in  as n → ∞, concluı́mos que (xn )n≥1 converges to an element
ct

of Âω1 + · · · + Âωd , e portanto x̂ ∈ Âω1 + · · · + Âωd .


,O
ET

6 Exercı́cios
01. Seja A um anel e seja Y
A→ Ap
p∈Spec A

o produto dos mapas de localização. Prove que este mapa é injetor.


58 Manobras Básicas

02. Sejam a e bpideais de um anel A. coprimos


√ √ √ √
(a) Prove que a = a e ab = a ∩ b.
(b) Mostre que se a e b são coprimos (i.e. a + b = (1)), então o mesmo vale para am e bn para todos
m, n ≥ 0.
√ √
(c) Suponha que a é finitamente gerado. Mostre que existe um inteiro n tal que a ∈ a ⇒ an ∈ a
globalmente (i.e., independentemente do a).

03. Seja A um anel graduado e a um ideal homogêneo de A. Mostre que a também é um ideal
homogêneo.
04. Considere o anel A = C[x, y](x,y) /(xy). Determine Spec A, seus abertos e fechados e calcule explici-
tamente a localização Ax . Dê uma interpretação geométrica.
05. Para cada um dos anéis A a seguir, descreva o mais explicitamente possı́vel
1. Spec A e os ideais maximais de A
2. os abertos e fechados de Spec A
Dê também descrições geométricas destes anéis (por exemplo, faça um desenho) no maior número
de casos que você conseguir.
(a) C (b) C × C
(c) C[z] (d) C[[z]]
(e) C[z]/(z 2010
) (f) C[[z]]/(z 2010 )
(g) C[x, y]/(x2010 , y 2010 ) (h) C[x, y]/(xy)2010
(i) R[z]/(z 2 + 1) (j) C[z]/(z 2 + 1)
(k) F5 [z]/(z 2 − 2) (l) F5 [z]/(z 5 − 2)
(m) C[x, y]/(y 2 − x3 ) (n) C[x, y]/(y 2 − x2 (x + 1))
(o) C[x, y]/(y 2 − x3 + 1) (p) C[x, y, z]/(y − xz, z 2 − x − 1)
06. Seja φ: C[x, y] → C[x, y, z]/(y − xz) o morfismo de C-álgebras dado por φ(x) = x̄ e φ(y) = ȳ. Seja
f = Spec(φ): Spec C[x, y, z]/(y − xz) → Spec C[x, y] o morfismo de espectros associado a φ.
(a) Determine f (ma,b,c ) onde ma,b,c = (x̄ − a, ȳ − b, z̄ − c) é o ideal maximal associado ao ponto (a, b, c) ∈
C3 da superfı́cie y = xz.
(b) Calcule as fibras f −1 ma,b onde ma,b = (x − a, y − b) ∈ Spec C[x, y] é o ideal maximal associado ao
ponto (a, b) ∈ C2 .
FT
(c) Calcule as fibras f −1 pa,b e f −1 (0) onde pa,b = (ax+by) ∈ Spec C[x, y] denota o ideal primo associado
ao ponto genérico das retas em C2 .
)
28

07. Mostre que Spec Z[x] consiste nos seguintes ideais primos:
7:

1. (0);
(1
RA

2. (f (x)), onde f (x) ∈ Z[x] é um polinômio irredutı́vel;


0

3. (p), onde p ∈ Z é um número primo;


01

4. (p, f (x)), onde p ∈ Z é um número primo e f (x) ∈ Z[x] é tal que sua imagem em Fp [x] é um
,2

polinômio irredutı́vel.
29

08. Seja A o anel de todas as funções contı́nuas f : [0, 1] → R. Mostre que existe um primo p ∈ Spec A
D

que não é da forma


ct

IP = {f ∈ A | f (P ) = 0}
,O

com P ∈ [0, 1].


ET

Hint: Localize.
09. Use o “truque do determinante” para mostrar o seguinte:
(a) (Cayley-Hamilton) Seja K um corpo e M uma matriz n × n com entradas em K. Se pM (x) ∈ K[x] é
o polinômio caracterı́stico de M , então pM (M ) = 0. Para provar este teorema, considere V = K n como
def
um K[x]-módulo onde multiplicação de f (x) ∈ K[x] por v ∈ K n é dada por f (x) · v = f (M )v.
59

10. Encontre geradores minimais para os ideais maximais dos seguintes anéis locais A = Bm onde
(a) B = C[x, y]/(y 2 − x3 ) e m = (x, y).
(b) B = C[x, y]/(y 2 − x2 (x + 1)) e m = (x, y).
(c) B = C[x, y]/(y 2 − x2 (x + 1)) e m = (x + 1, y).
(d) B = C[x, y, z]/(y − xz, z 2 − x − 1) e m = (x, y, z − 1)
(e) B = Z[x] e m = (3, x)
(f) B = Z[x]/(x2 − 45) e m = (3, x).
Dê interpretações geométricas.
11. Seja (A, m, k) um anel local e sejam f (x), g(x) ∈ A[x] polinômios mônicos. Denote por f (x), g(x) ∈
k[x] as imagens de f (x) e g(x) em A[x]/mA[x] = k[x], respectivamente. Mostre que se (f (x), g(x)) = (1),
então
A[x] ∼ A[x] A[x]
= ×
(f (x)g(x)) (f (x)) (g(x))
12. Sejam V e W espaços vetoriais de dimensão finita sobre um corpo K com bases ωi e τj respectiva-
mente. Mostre que os elementos ωi ⊗ τj formam uma base do K-espaço vetorial V ⊗K W e conclua que
dimK (V ⊗K W ) = dimK V · dimK W .
13. Mostre os seguintes isomorfismos de anéis:
(a) Z/n ⊗Z Z/m = Z/(m, n)
(b) C[x] ⊗C C[y] = C[x, y].
14. Seja φ: A → B uma A-álgebra.
15. Seja L ⊃ K uma extensão finita galoisiana de corpos com G = Gal(L/K). Mostre que

L ⊗K L ∼
= Maps(G, L)

onde Maps(G, L) é o anel de todas as funções de G em L, ou seja, Maps(G, L) ∼


= L|G|, o anel de tuplas
com entradas em L indexadas por elementos de G (produto e soma componente a componente).
16. Seja A um anel noetheriano e φ: A ։ A um morfismo sobrejetor. Mostre que φ é um isomorfismo.
17. Prove que se todos os elementos de Spec A são finitamente gerados então o anel A é noetheriano.
18. Calcule o comprimento do A-módulo M onde
(a) A = C[x] e M = A/(xn ) com n ∈ N.
(b) A = Z e M = A/(pn ) com n ∈ N e p primo.
FT
(c) A = C[x, y] e M = A/(x, y)n com n ∈ N.
(d) A = C[x, y]/(y 2 − x3 + 1) e M = A/(x̄, ȳ)n com n ∈ N.
A = C[x, y]/(y 2 − x3 ) e M = A/(x̄, ȳ)n com n ∈ N.
)
(e)
28

19. Prove that a noetherian domain A é a PID se, e só se, todo its prime ideals são principal. Hint: suponha
7:

que the result é false e seja a be an ideal of A, maximal among the non-principal ones. Então a é not
(1
RA

prime; seja x, y ∈
/ a but xy ∈ a. Então (x) + a = (a) e (y) + a = (b) para algum a e b. Consider the ideal
b = (a : a).
0
01

20. (Chomp, o jogo) Há um chocolate em cada ponto (m, n) de Z2≥0 , com exceção do ponto (0, 0), no
,2

qual há um morango envenenado. Dois jogadores se alternam: cada movimento deste jogo consiste em
escolher um chocolate em um ponto (m, n) e papar todos os chocolates acima e à direita de (m, n), ou
29

seja, todos os chocolates nos pontos (x, y) com x ≥ m e y ≥ n (sim, cada jogador é um tremendo glutão
D
ct

capaz fagocitar infinitos chocolates em um só movimento!) O jogo acaba quando um deles morre (os
jogadores são highlanders, somente o morango envenenado é capaz de lhes subtrair a vida). Mostre que
,O

o jogo acaba após um número finito de movimentos.


ET

Hint: Utilize Basissatz Hilberts.


D
ET
,O
ct
29
RA
,2
01
0
(1
FT
7:
28
)
Chapter 4
integral
finita

Extensões Finitas e Integrais

Extensões finitas e integrais de anéis generalizam os conceitos de extensões finitas e algébricas de corpos,
sendo portanto conceitos muito importantes no estudo de anéis, com diversas aplicações em Geometria
Algébrica e Teoria dos Números.

1 Definições e Propriedades Básicas


Definição 1.1 Seja A ⊂ B uma extensão de anéis. Um elemento b ∈ B é integral sobre A se ele satisfaz
um polinômio mônico com coeficientes em A:

bn + an−1 · bn−1 + · · · + a0 = 0, ai ∈ A

Dizemos que o anel B é integral sobre A se todo elemento de B é integral sobre A.


Definição 1.2 Uma A-álgebra B é dita finita se B é um A-módulo finitamente gerado.
As definições acima são generalizações para anéis dos conceitos familiares de elemento algébrico
e extensões finitas e algébricas de corpos. Por exemplo, o teorema seguinte é uma generalização do
conhecido fato de que um elemento θ é algébrico sobre um corpo K se, e somente se, [K(θ) : K] < ∞.
Teorema 1.3 (Caracterização Intrı́nseca de Integralidade) Seja A ⊂ B uma extensão de anéis
e seja b ∈ B. As seguintes condições são equivalentes:
1. b é integral sobre A;
2. A[b] é uma A-álgebra finita;
3. A[b] ⊂ C para alguma A-subálgebra finita C de B.

Prova (1 ⇒ 2) Seja bn + an−1 bn−1 + · · · + a0 = 0, ai ∈ A, uma equação mônica para b. Então, para
FT
i ≥ 0, temos
bn = −an−1 bn−1 − · · · − a0 ⇒ bn+i = −an−1 bn−1+i − · · · − a0 bi
)
28

o que nos permite recursivamente expressar qualquer potência bj com j ≥ n em termos de combinações
7:

A-lineares de 1, b, . . . , bn−1 . Assim,


(1
RA

A[b] = A + Ab + · · · + Abn−1
0
01

é uma A-álgebra finita.


,2

(2 ⇒ 3) Óbvio: basta tomar C = A[b].


29

(3 ⇒ 1) Aqui usamos o nosso velho conhecido, o truque do determinante: se ω1 , . . . , ωn são geradores


D

de C sobre A, como b · ωi ∈ C para todo i, temos o seguinte “sistema linear” nas “variáveis” ωi e
ct

“coeficientes” aij ∈ A:
,O

b · ω1 = a11 ω1 + · · · + a1n ωn
ET

..
.
b · ωn = an1 ω1 + · · · + ann ωn
Assim, b é raiz do polinômio caracterı́stico da matriz (aij ), que é mônico e possui coeficientes em A.
62 Extensões Finitas e Integrais

Corolário 1.4 Sejam A ⊂ B ⊂ C extensões de anéis. fecho integral


normal
1. (Finito ⇒ Integral) Se B é uma A-álgebra finita então B é integral sobre A. Reciprocamente, integralmente fechado
temos
finitamente gerado + integral = finito
2. (Transitividade de Finitude e Integralidade) Se C é finito (resp. integral) sobre B e B é finito
(resp. integral) sobre A então C é finito (resp. integral) sobre A.
3. (Normalização é anel) Seja à ⊂ B o subconjunto de todos os elementos de B que são integrais
sobre A. Então à é um subanel de B, chamado de fecho integral ou normalização de A em
B.

Prova (1) é consequência imediata do teorema. Para provar (2), suponha que B seja finito sobre A,
gerado por ωi , 1 ≤ i ≤ n, e que C seja finito sobre B, gerado por τj , 1 ≤ j ≤ m. Então C é finito sobre
A, gerado pelos mn elementos ωi τj , 1 ≤ i ≤ n e 1 ≤ j ≤ m. Agora suponha que C ⊃ B e B ⊃ A sejam
extensões integrais e seja c ∈ C; devemos mostrar que c é integral sobre A. Por hipótese, c satisfaz uma
relação mônica
cn + bn−1 cn−1 + · · · + b0 = 0 bi ∈ B
Por outro lado, cada bi é integral sobre A. Assim, pelo teorema A[b0 ] ⊃ A é uma extensão finita. Como
b1 é integral sobre A, também é integral sobre A[b0 ], de modo que A[b0 , b1 ] ⊃ A[b0 ] é finita, bem como
A[b0 , b1 ] ⊃ A pelo que acabamos de mostrar. Procedendo desta forma, concluı́mos que

A[b0 , b1 , . . . , bn−1 , c] ⊃ A

é uma extensão finita e c ∈ A[b0 , b1 , . . . , bn−1 , c], de modo que c é integral sobre A, como desejado.
Finalmente, para provar (3), sejam b1 , b2 ∈ Ã; devemos mostrar que b1 ± b2 e b1 b2 também estão
em Ã, ou seja, são integrais sobre A. Mas ambos estes elementos pertencem ao subanel A[b1 , b2 ], que é
finito sobre A (veja a prova do item anterior), o que encerra a demonstração.

Definição 1.5 Um domı́nio A é normal ou integralmente fechado se A é seu próprio fecho integral
em Frac A.

Teorema 1.6 UFDs são normais.


Prova Seja A um UFD e seja θ ∈ Frac A um elemento integral sobre A, digamos raiz do polinômio
mônico f (x) = xn + cn−1 xn−1 + · · · + c0 ∈ A[x]. Podemos escrever θ = a/b com a, b ∈ A primos entre si.
Limpando os denominadores em f (θ) = 0 obtemos
FT
an + cn−1 an−1 b + cn−2 an−2 b2 + · · · + c0 bn = 0
)
28

Como b divide todos os termos a partir do segundo, temos que b divide an também. Mas como a e b são
7:

primos entre si temos que a única possibilidade para que isto ocorra é b ∈ A× , logo θ ∈ A.
(1
RA
0

Lema 1.7 (“Limpando denominadores”) Seja A um domı́nio com corpo de frações K = Frac A e
01

seja L ⊃ K uma extensão algébrica de corpos. Para todo θ ∈ L, existe d ∈ A não nulo tal que dθ é
integral sobre A.
,2

Prova Suponha que an θn + an−1 θn−1 + · · · + a0 = 0 com ai ∈ A, an 6= 0. Multiplicando por ann−1


29

obtemos (an θ)n + an−1 (an θ)n−1 + · · · + ann−1 a0 = 0, logo podemos tomar d = an .
D
ct
,O

Teorema 1.8 Seja A um domı́nio normal com corpo de frações K = Frac A. Seja L ⊃ K uma extensão
algébrica de corpos e seja B o fecho integral de A em L.
ET

1. Um elemento β ∈ L pertence a B se, e só se, seu polinômio minimal p(x) sobre K pertence a
A[x].
2. Se L ⊃ K é uma extensão finita, então TrL/K (β), NL/K (β) ∈ A para todo β ∈ B.
3. Se L ⊃ K é uma extensão finita separável e A é noetheriano então B é finito sobre A.
63

Prova 1. Se o polinômio minimal (que é mônico por definição) de β ∈ L pertence a A[x] então
claramente β ∈ B. Reciprocamente, seja β ∈ B e seja f (x) ∈ K[x] o polinômio minimal de β. Então
toda raiz de f (x) (no fecho algébrico de K) é integral sobre A: se f (β) = f (γ) = 0 seja σ o isomorfismo
de K-álgebras dado pela composição

K[β] - K[x]
β7→x
- K[γ]
x7→γ
≈ f (x) ≈

Se p(x) ∈ A[x] é um polinômio mônico tal que p(β) = 0 então p(γ) = p(σ(β)) = σ(p(β)) = 0 e portanto
γ também é integral sobre A.
Assim, como os coeficientes de f (x) são polinômios simétricos elementares em suas raı́zes, temos
que os coeficientes de f (x) são integrais sobre A e pertencem a K. Como A é normal, isto implica que
f (x) ∈ A[x].
2. Pelo item anterior, o polinômio minimal p(x) = xn + an−1 xn−1 + · · · + an de β sobre K pertence
a A[x]. Como TrL/K (β) é um múltiplo inteiro de an−1 , temos que TrL/K (β) ∈ A. Da mesma forma,
NL/K (β) é, a menos de sinal, uma potência de a0 , logo NL/K (β) ∈ A também.
3. Como A é noetheriano, basta mostrar que B está contido em algum A-submódulo finitamente
gerado de L. Seja ω1 , . . . , ωn uma base de L sobre K. Como os ωi são algébricos sobre K, “limpando
denominadores” podemos supor que sem perda de generalidade que ωi ∈ B para i = 1, . . . , n. Vamos
mostrar a existência de um D ∈ A não nulo tal que
ω1 ωn
B ⊂A· + ···+ A ·
D D
Tome β ∈ B. Como os ωi formam uma base de L sobre K, podemos escrever β = a1 ω1 + · · · + an ωn
com ai ∈ K. Vamos aplicar novamente o “truque do determinante”: multiplicando a relação anterior
por ωj e tomando traços, obtemos o “sistema linear” nos ai ’s:

TrL/K (βω1 ) = a1 TrL/K (ω1 ω1 ) + · · · + an TrL/K (ωn ω1 )


TrL/K (βω2 ) = a1 TrL/K (ω1 ω2 ) + · · · + an TrL/K (ωn ω2 )
..
.
TrL/K (βωn ) = a1 TrL/K (ω1 ωn ) + · · · + an TrL/K (ωn ωn )

Note que como βωi e ωi ωj são todos integrais sobre o anel normal A, D = det(TrL/k (ωi ωj )), o discrim-
inante da base ωi , pertence a A. Como L ⊃ K é separável, temos que D 6= 0. Pela regra de Cramer
temos que ai ∈ A · D−1 , o que mostra que β ∈ A · ωD1 + · · · + A · ωDn , como desejado.
FT
Exemplo 1.9 Seja d um inteiro
√ livre de quadrados (i.e. nenhum quadrado de primo divide d). Então
)
28

o fecho integral de Z em Q( d) é dado por Z[ω] = Z + Zω, onde


(√
7:

d se d ≡ 2, 3 (mod 4)
(1
RA

ω = 1+√d
2 se d ≡ 1 (mod 4)
0
01

Teorema 1.10 (Localização × Integralidade) Seja B ⊃ A uma extensão de anéis e seja S um


,2

conjunto multiplicativo de A.
29

1. Se B é integral sobre A então S −1 B é integral sobre S −1 A.


D

2. Se A é um domı́nio normal então S −1 A é normal também.


ct
,O

3. Um domı́nio A é normal se, e só se, Am é normal para todos os ideais maximais m de A.
ET

Prova Para mostrar (1), seja b/s ∈ S −1 B (com b ∈ B e s ∈ S). Como B é integral sobre A, b satisfaz
uma relação mônica bn + an−1 bn−1 + · · · + a0 = 0 com ai ∈ A. Assim, “dividindo por sn ”, temos em
S −1 B  b n a  b n−1 a  b n−2
n−1 n−2 a0
+ · + 2 · + ···+ n = 0
s s s s s s
o que mostra que b/s é integral sobre S −1 A.
64 Extensões Finitas e Integrais

Agora, suponha que A seja um domı́nio normal e considere um elemento x ∈ Frac S −1 A = Frac A
integral sobre S −1 A. Então existem ai ∈ A e t ∈ S (tomando um “denominador comum”) tais que
an−1 n−1 a1 a0
xn + x + ···+ x + = 0 ⇒ (tx)n + an−1 (tx)n−1 + · · · + a1 tn−2 (tx) + a0 tn−1 = 0
t t t
Assim, tx ∈ Frac A é integral sobre A e logo tx ∈ A. Portanto x ∈ S −1 A, o que prova (2).
Por (2), já sabemos que se A é normal então Am também é normal para todo ideal primo m.
Reciprocamente, suponha que Am é normal para todos os ideais maximais m de A e seja x ∈ Frac A um
elemento integral sobre A. Como A ⊂ Am (vistos como subanéis de Frac A), temos que x é integral sobre
cada Am e portanto \
x∈ Am = A
m

pelo princı́pio local-global (teorema III.1.8), o que mostra que A é também normal.

Teorema 1.11 Se A é um domı́nio normal, o mesmo vale para A[x].



Prova Seja K = Frac A e seja f (x) ∈ Frac A[x] um elemento integral sobre A[x]. Então f (x) é integral
sobre K[x]. Mas como K[x] é um PID, logo um UFD, é um domı́nio normal e portanto f (x) ∈ K[x].
Escreva
f (x) = αn xn + · · · + α0 , αi ∈ K
Vamos agora mostrar por indução no grau n de f (x) que f (x) ∈ A[x]. É suficiente mostrar que αn ∈ A,
já que neste caso teremos que f (x) − αn xn também é integral sobre A[x] e, aplicando a hipótese de
indução, que f (x) − αn xn ∈ A[x] e assim f (x) ∈ A[x].
Vamos agora mostrar que αn é integral sobre A e assim αn ∈ A pois A é normal. A ideia é utilizar
a filosofia de “redução noetheriana” (c.f. teorema III.4.4.3). Vamos construir um subanel noetheriano
subanel A0 ⊂ A e um A0 -módulo finito M0 ⊂ K tal que A0 [αn ] ⊂ M0 . Como A0 é noetheriano, temos
que A0 [αn ] será uma A0 -álgebra finita contendo αn , logo este elemento será integral sobre A0 e a fortiori
sobre A também.
Agora vamos construir A0 e M0 . Seja

F (T ) = T d + cd−1 (x) · T d−1 + · · · + c0 (x), ci (x) ∈ A[x]



um polinômio mônico tal que F f (x) = 0. Seja A0 a Z-subálgebra de A gerada por todos os coeficientes
dos ci (x)’s. Claramente A0 é noetheriana. Agora sejam ω1 , . . . , ωr todos os coeficientes dos polinômios
2 d−1
FT
1, f (x), f (x) , · · · , f (x)
 i
Defina M0 = A0 · ω1 + · · · A0 · ωr . Observe que como F (T ) ∈ A0 [x] [T ] todas as potências f (x) de
)
28

f (x) podem ser escritas como A0 [x]-combinações lineares das primeiras d potências acima. Logo todos
i
7:

os coeficientes de f (x) pertencem a M0 , em particular todas as potências αin de αn pertencema a M0


i
(1
RA

já que αin é o coeficiente lı́der de f (x) . Isto mostra que A0 [αn ] ⊂ M0 , o que encerra a prova.
0

Observação 1.12 Em geral, A normal não implica A[[x]] normal.


01
,2

2 Fibras de Extensões Integrais e Going-up


29

Lema 2.1 (“Corpos descem”) Seja B ⊃ A uma extensão integral de anéis.


D
ct

1. Suponha que A e B sejam domı́nios. Então


,O

A é corpo ⇐⇒ B é corpo
ET

2. Seja P ∈ Spec B e seja p = P ∩ A ∈ Spec A a imagem de P pelo mapa Spec B → Spec A


induzido pela inclusão A ֒→ B. Então

p é maximal ⇐⇒ P é maximal
65

Prova Suponha que A é corpo e tome b ∈ B não nulo. Como B é integral sobre A, b é algébrico sobre
A e satisfaz um polinômio

bn + an−1 bn−1 + · · · + a0 = 0 ⇐⇒ b · (bn−1 + an−1 bn−2 + · · · + a1 ) = −a0

com ai ∈ A e a0 6= 0 (se a0 = 0, podemos cancelar b pois B é domı́nio). Assim, como a0 ∈ A× ⊂ B × ,


temos que b ∈ B × também, o que mostra que B é um corpo.
Reciprocamente, suponha que B seja um corpo. Se a ∈ A é um elemento não nulo, então a−1 ∈ B
é integral sobre A, logo existem ai ∈ A tais que

(a−1 )n + an−1 · (a−1 )n−1 + an−2 · (a−1 )n−2 + · · · + a0 = 0

Multiplicando por an−1 , temos

a−1 = −an−1 − an−2 · a · · · − a0 · an−1 ∈ A

o que mostra que A é um corpo.


Finalmente, aplicando o resultado provado à extensão integral de domı́nios A/p ֒→ B/P, obtemos
o item (2).

Teorema 2.2 Seja B ⊃ A uma extensão integral de anéis.


1. (Recobrimento) Spec B ։ Spec A é sobrejetor.
2. (Incomparabilidade) Os primos de uma fibra de Spec B ։ Spec A são dois a dois incomparáveis,
i.e., P1 ∩ A = P2 ∩ A ⇒ P1 = P2 para Pi ∈ Spec B.
3. (Finitude) Se B é finito sobre A então as fibras de Spec B ։ Spec A são conjuntos finitos.

Prova Seja p ∈ Spec A e S = A \ p. Temos que a extensão S −1 B ⊃ S −1 A é integral (lembre-se de que


localização é um funtor exato, logo preserva inclusões) e é finita se B ⊃ A é finita. Logo, substituindo A
e B por suas localizações com relação a S, podemos supor que A é local com ideal maximal p. Seja k o
corpo residual de A.
Para mostrar (1), devemos mostrar que a fibra Spec B ⊗A k de p é não vazia, ou seja, que B ⊗A k 6= 0.
Suponha por absurdo que B ⊗A k = 0 ⇐⇒ B = pB. Se a extensão B ⊃ A é finita temos por Nakayama
(teorema III.3.2.6) que B = 0, absurdo. No caso geral, de B = pB podemos escrever

1 = p1 b 1 + · · · + pn b n com pi ∈ p, bi ∈ B
FT
Seja B ′ = A[b1 , . . . , bn ] a A-subálgebra de B gerada pelos bi ’s. Temos que B ′ é finita sobre sobre A e
B ′ = pB ′ pois multiplicando a relação acima por bi vemos que bi ∈ pB ′ . Logo o caso geral segue do caso
especial que acabamos de provar.
)
28

Para provar (2), basta mostrar que todo primo P de B ⊗A k é maximal. Temos que (B ⊗A k)/P
é um domı́nio, que é uma extensão integral de k, pois B ⊗A k é integral sobre k. Pelo lema anterior,
7:

(B ⊗A k)/P é corpo, isto é, P é maximal.


(1
RA

Finalmente, se B ⊃ A é uma extensão finita, temos que dimk B ⊗A k < ∞, logo B ⊗A k é artiniano e
0

portanto Spec B ⊗A k é finito pelo teorema III.4.3.9. Isto prova (3). Note que, neste caso finito, sabemos
01

ainda que todos os primos B ⊗A k são maximais, logo dois a dois incomparáveis, o que fornece uma
,2

“outra” prova de (2) neste caso.


29

Corolário 2.3 (“Going-up”) Seja B ⊃ A uma extensão integral de anéis. Sejam p ( p′ ideais primos
D
ct

de A. Se P ∈ Spec B é um ideal primo sobre p (i.e. p = P ∩ A) existe P′ ∈ Spec B sobre p′ e tal que
,O

P ( P′ .
P ( ∃P′ ( B
ET

| | |
p ( p′ ( A

Prova A inclusão A ֒→ B induz uma inclusão de domı́nios A/p ֒→ B/P, que é integral pois B é integral
sobre A. Assim, o resultado segue do teorema anterior (recobrimento) aplicado a esta última inclusão e
o ideal primo p′ /p de Spec A/p.
66 Extensões Finitas e Integrais

Corolário 2.4 Seja A ⊂ B uma extensão integral de anéis. Então dim A = dim B.
Prova Se
p0 ( p1 ( · · · ( pn ( A (∗)
é uma cadeia de ideais primos em A, então pelo going-up existe uma cadeia de ideais primos em B

P0 ( P1 ( · · · ( Pn ( B (∗∗)

com Pi ∩ A = pi e portanto dim B ≥ dim A. Reciprocamente, dada uma cadeia como (∗∗), temos que os
def
primos pi = Pi ∩ A são todos distintos pela incomparabilidade dos primos em uma fibra e assim definem
uma cadeia com em (∗). Logo dim A ≥ dim B.

3 Normalização de Noether e Nullstellensatz Hilberts

Teorema 3.1 (Normalização de Noether) Seja A um domı́nio finitamente generado sobre um corpo
k e seja r = tr. degk Frac A. Então existe uma base de transcendência x1 , . . . , xr ∈ A tal que A é finito
sobre o subanel k[x1 , . . . , xr ].
Prova A prova é por indução no número n de geradores de A sobre k. Se A = k[a1 , . . . , an ] e os ai ’s
são algebricamente independentes sobre k, então n = r = tr. degk Frac A e podemos tomar xi = ai . Caso
contrário, n > r e existe uma relação polinomial não trivial entre os geradores ai :
X
be1 ,...,en · ae11 ae22 . . . aenn = 0 be1 ,...,en ∈ k (∗)
e1 ,...,en

A ideia é reescrever (∗) de modo a obter uma equação mônica para a1 . Isto pode ser feito da seguinte
i−1
maneira: para algum natural N , fazemos a “mudança de coordenadas” a′i = ai − aN 1 , i = 2, . . . , n.
Como e2 n−1 en
ae11 ae22 . . . aann = ae11 a′2 + aN
1 . . . a′n + aN
1
n−1
= ae11 +e2 N +···+en N + termos de grau baixo em a1 ,

escolhendo N grande o suficiente podemos fazer com que todos os expoentes e1 + e2 N + · · · + en N n−1
“escritos na base N ” fiquem distintos. Desta maneira, (∗) fornece um polinômio mônico em a1 com
coeficientes no subanel A′ = k[a′2 , . . . , a′n ] de A. Então a1 será integral sobre A′ , bem como todos os
i−1
ai = a′i + aN
1 . Resumindo, A é integral sobre A′ ; por outro lado, por hipótese de indução, A′ é integral
sobre um subanel R = k[y1 , . . . , yr ] para alguma base de transcendência yi sobre k, portanto A é também
FT
integral sobre A, o que encerra a prova.
)
28

Geometricamente, a normalização de Noether corresponde à existência de uma projeção Spec A ։


Spec k[x1 , . . . , xr ] com fibras finitas de uma variedade qualquer sobre o espaço afim k r .
7:
(1
RA

Corolário 3.2 Se A é um domı́nio finitamente generado sobre um corpo k então dim A = tr. degk Frac A.
0
01

Prova Pelo teorema, temos que A é finito sobre k[x1 , . . . , xr ] onde r = tr. degk Frac A. Assim, do
corolário 2.4, temos que dim A = dim k[x1 , . . . , xr ] ≥ r, pois temos a cadeia de primos de tamanho r
,2
29

(0) ( (x1 ) ( (x1 , x2 ) ( · · · ( (x1 , . . . , xr ) ( k[x1 , . . . , xr ]


D
ct

Por outro lado, se temos uma cadeia de ideais primos


,O

(0) = p0 ( p1 ( · · · ( pd ( k[x1 , . . . , xr ]
ET

temos que B = k[x1 , . . . , xr ]/p1 é um domı́nio finitamente gerado sobre k e tr. degk Frac B ≤ r − 1 pois
qualquer polinômio não nulo em p1 fornece uma relação de dependência algébrica entre os xi ’s. Por
indução temos dim B ≤ r − 1, assim d − 1 ≤ dim B ⇒ d ≤ r, ou seja, dim A ≤ r.
67

Teorema 3.3 (Nullstellensatz Hilberts, versão I) Seja A um domı́nio finitamente generado sobre
um corpo k. Seja m ∈ Spec A. Então

m é um ideal maximal de A ⇐⇒ dimk A/m < ∞

Prova Substituindo A por A/m, temos que mostrar que um corpo A que é finitamente gerado como
k-álgebra é necessariamente algébrico sobre k. Pela normalização de Noether, A é uma extensão finita
de um anel de polinômios k[x1 , . . . , xr ] onde r = tr. degk A. Mas pelo lema 2.1, k[x1 , . . . , xr ] também é
um corpo, o que só ocorre se r = 0, ou seja, se A é algébrico e portanto uma extensão finita de k, já que
A é finitamente gerado sobre k.

Corolário 3.4 Seja A um domı́nio finitamente generado sobre um corpo k. Então os ideais maximais
de A formam um conjunto denso em Spec A. Além disso, se a é um ideal de A, temos
√ \
a= m
m∈V (a)
m maximal

Corolário 3.5 Seja φ: A → B um morfismo de k-álgebras finitamente geradas. Então Spec(φ) leva
ideias maximais de B em ideais maximais de A.

Prova Seja q um ideal maximal de B e p = φ−1 (q) ∈ Spec A. O morfismo φ induz uma injeção de
k-álgebras A/p ֒→ B/q. Como q é maximal, pelo Nullstellensatz, dimk B/q < ∞ e assim dimk A/p < ∞
o que, novamente pelo Nullstellensatz, implica que p é maximal.

Observação 3.6 Em geral, o mapa entre espectros não preserva ideais maximais. É fácil achar contra-
exemplos: o mapa Spec Q → Spec Z associado à inclusão Z ֒→ Q leva o ideal maximal (0) de Q no ideal
primo não maximal (0) de Z.

Teorema 3.7 (Nullstellensatz Hilberts, versão II) Seja k um corpo algebricamente fechado. Os
ideais maximais de k[x1 , . . . , xn ] são precisamente os da forma (x1 − a1 , . . . , xn − an ) com ai ∈ k.

Prova Como k[x1 , . . . , xn ]/(x1 −a1 , . . . , xn −an ) é isomorfo a k via xi 7→ ai , temos que (x1 −a1 , . . . , xn −
an ) é maximal. Reciprocamente, se m é um ideal maximal de k[x1 , . . . , xn ], pelo teorema anterior o corpo
FT
k[x1 , . . . , xn ]/m é uma extensão finita de k e portanto isomorfo a k já que este é algebricamente fechado.
Assim, existem elementos ai ∈ k tais que xi ≡ ai (mod m), ou seja, (x1 − a1 , . . . , xn − an ) ⊂ m. Mas
)
como ambos os ideais são maximais, devemos ter a igualdade.
28
7:

Teorema 3.8 (Nullstellensatz Hilberts, versão III) Seja k um corpo algebricamente fechado.
(1
RA


Então I(V (a)) = a para todos os ideais a de k[x1 , . . . , xn ].
0


01

Prova É claro que a ⊂ I(V (a)). Para provar a inclusão oposta, tome f ∈ I(V (a)) e considere o ideal
a + (yf − 1) de k[x1 , . . . , xn , y]. Afirmamos que a + (yf − 1) = (1): de fato, se existisse um ideal maximal
,2

(x1 − a1 , . . . , xn − an , y − b) contendo a + (yf − 1), então (a1 , . . . , an ) ∈ V (a) e b · f (a1 , . . . , an ) = 1, o


29

que é um absurdo pois f (a1 , . . . , an ) = 0 para todo (a1 , . . . , an ) ∈ V (a) por definição.
D

Assim, existem g, gi ∈ k[x1 , . . . , xn , y] e pi ∈ a tais que


ct

 X
,O

1 = g(x1 , . . . , xn , y) · yf (x1 , . . . , xn ) − 1 + gi (x1 , . . . , xn , y) · pi (x1 , . . . , xn )


ET

Agora substitua y = 1/f na expressão acima e multiplique por uma potência suficientemente grande f m
m
√ f para “limpar” os denominadores. Obtemos uma nova expressão que mostra que f ∈ a ⇐⇒ f ∈
de
a.
68 Extensões Finitas e Integrais

Corolário 3.9 (Anéis artinianos revisitados) Seja k um corpo e A uma k-álgebra finitamente ger- valorização discreta
domı́nio de valorizaçã
ada. As seguintes condições são equivalentes: valorização p-ádica
1. dimk A < ∞;
2. A é artiniano;
3. dim A = 0;
4. Spec A é discreto;
5. Spec A é um conjunto finito.
Prova Claramente 1 ⇒ 2 e já sabemos pelo teorema III.4.3.9 que 2 ⇒ 3.
(3 ⇒ 4) Como dim A = 0, todo ideal primo de A é simultaneamente maximal e minimal, assim todo
ponto de Spec A é fechado. Além disso, como A é finitamente gerado sobre k, A é noetheriano e portanto
possui apenas um número finito de ideais primos minimais, logo Spec A é finito e portanto discreto.
(4 ⇒ 5) Seja p ∈ Spec A. Como todo ponto é fechado, temos {p} = {p} ⇐⇒ V (p) = {p}, ou seja,
p é maximal. Assim, todo ideal primo de A é simultaneamente maximal e minimal e como acima isto
implica que Spec A é finito.
(5 ⇒ 1) Se Spec A é finito então A possui um número finito de ideais maximais m1 , . . . , mn . Pela prova do
teorema III.4.3.9, temos uma cadeia finita A ⊃ m1 ⊃ m1 m2 ⊃ · · · ⊃ 0 tal que se M e mi M denotam dois
termos consecutivos nesta cadeia então o quociente M/mi M é finito sobre A/mi . Assim, para mostrar
que A é finito sobre k basta mostrar que cada A/mi é finito sobre k, o que segue do Nullstellensatz
(teorema 3.3).

4 Valorizações discretas
Na definição seguinte, ∞ denota um sı́mbolo sujeito às condições a + ∞ = ∞ e min{a, ∞} = a para todo
a ∈ Z ∪ {∞}.
Definição 4.1 Seja K um corpo. Uma valorização discreta v: K → Z∪{∞} é uma função sobrejetora
que satisfaz os seguintes três axiomas:
1. v(a) = ∞ ⇐⇒ a = 0;
2. v(ab) = v(a) + v(b) para todo a, b ∈ K (isto é, v: K × → Z é um morfismo de grupos);
3. v(a + b) ≥ min{v(a), v(b)} para todo a, b ∈ K.
O conjunto
def
Ov = {a ∈ K | v(a) ≥ 0}
é um anel local com ideal maximal
def
FT
mv = {a ∈ K | v(a) > 0}
já que
)
Ov× = {a ∈ K | v(a) = 0} = Ov \ mv
28

O anel Ov é chamado de domı́nio de valorização discreta associado à valorização v. Um domı́nio


7:

para A para o qual existe uma valorização discreta v: Frac A → Z ∪ {∞} tal que A = Ov também será
(1
RA

chamado de domı́nio de valorização discreta.


0

Exemplo 4.2 Seja p um número primo. Associado a p, temos a chamada valorização p-ádica vp : Q →
01

Z ∪ {∞}, definida da seguinte forma: para cada racional z 6= 0, escreva


,2

a
z = pn · com a, b ∈ Z e p ∤ a e p ∤ b
b
29

Defina vp (z) = n se z = 0 e vp (0) = ∞, ou seja, vp (z) é o expoente da maior potência de p que divide z.
D
ct

É fácil ver que v é uma valorização discreta de Q e que Ov = Z(p) .


,O

Exemplo 4.3 Seja M o corpo de todas as funções meromorfas de C em C. Considere a função



ET

 ∞ se f (z) = 0;

n se f (z) tem um zero de ordem n em z = 0;
v0 (f (z)) =

 0 se f (z) se f (z) é holomorfa em z = 0 e f (0) 6= 0;
−n se f (z) tem um polo de ordem n em z = 0.
Então v0 : M → Z ∪ {∞} é uma valorização discreta de M .
69

Lemma 4.4 Seja v: K → Z ∪ {∞} uma valorização discreta. Então


1. v(±1) = 0 e v(−a) = v(a) para todo a ∈ K;
2. v(a/b) = v(a) − v(b) para todo a, b ∈ K com b 6= 0;
3. v(a + b) = v(b) se v(a) > v(b).
4. se a1 + · · · + an = 0 então existem i 6= j tais que v(ai ) = v(aj ).
5. se a ∈ K × então a ∈ Ov ou a−1 ∈ Ov .

Lemma 4.5 Seja A um domı́nio noetheriano normal com K = Frac A. Seja a um ideal de A. Se f ∈ K
é tal que f · a ⊂ a então f ∈ A.
Prova Truque do determinante. Se a = (ω1 , . . . , ωn ), temos que
X
f ωi = aij ωj aij ∈ A i = 1, 2, . . . , n
1≤j≤n

e assim f é raiz do polinômio caracterı́stico da matriz (aij ), que é mônico e tem coeficientes em A. Como
A é normal, temos que f ∈ A.

Teorema 4.6 Seja (A, m, k) um domı́nio local com K = Frac A. As seguintes condições são equivalentes:
1. A é um domı́nio de valorização discreta;
2. A é um domı́nio de ideais principais que não é um corpo;
3. A é noetheriano e o ideal maximal m = (π) de A é principal;
4. A é um domı́nio de fatoração única com um único elemento irredutı́vel π a menos de unidades
(π é chamado de uniformizador de A).
5. A é noetheriano, de dimensão 1 e normal;

Prova (1 ⇒ 2) Seja v a valorização discreta associada a A = Ov . Como v é sobrejetor, temos que m 6= 0


e portanto A não é corpo. Agora tome a um ideal não nulo. Como v assume valores não negativos em A,
podemos escolher um elemento t ∈ a com valorização mı́nima dentre os elementos de a; vamos mostrar
que a = (t). A inclusão a ⊃ (t) é clara pois t ∈ a. Agora seja a ∈ a; temos que v(a) ≥ v(t) pela escolha
de t, logo v(a/t) = v(a) − v(t) ≥ 0, ou seja, a/t ∈ A e portanto a ∈ (t).
(2 ⇒ 3) Se A é um PID então é noetheriano e m é principal. Além disso, os ideais primos não nulos de
FT
A são todos maximais, logo dim A = 1.
(3 ⇒ 4) Devemos mostrar que qualquer elemento não nulo a ∈ A é, a menos de unidade, uma potência
)
28

do elemento primo π. Como T A é um domı́nio noetheriano, pelo teorema T de intersecção de Krull (teo-
rema III.5.1.3) temos que n≥0 (π n ) = (0). Alternativamente, sendo a = n≥0 (π n ), um ideal finitamente
7:

gerado já que A é noetheriano, é fácil ver que (π) · a = a, logo por Nakayama temos a = 0. Em todo
(1
RA

caso, dado a 6= 0 existe n ≥ 0 tal que a ∈ (π n ) \ (π n+1 ), ou seja, a = uπ n com u ∈/ (π) = m ⇐⇒ u ∈ A× .


0

(4 ⇒ 1) Todo elemento a ∈ K × pode ser unicamente escrito na forma a = uπ n com u ∈ A× e n ∈ Z.


01

Basta definir v(a) = n e é fácil verificar que v define uma valorização discreta com Ov = A.
,2

(2 ⇒ 5) Um domı́nio de ideais principais é noetheriano e é de fatoração única, logo é normal também.


29

Se não for corpo, sua dimensão é 1.


D

(5 ⇒ 3) Temos que mostrar que m é principal. Tome a ∈ m não nulo. Como dim A = 1, temos que m é o
ct

único ideal primo diferente de (0). Assim, como A é um domı́nio noetheriano, temos que m ⊃ (a) ⊃ mn
,O

para algum n. Podemos supor que n é mı́nimo com esta propriedade, ou seja, que (a) 6⊃ mn−1 . Tome
def
b ∈ mn−1 \ (a). Temos que f = b/a ∈ K é tal que f m ⊂ A mas f ∈
ET

/ A. Pelo lema anterior, não podemos


ter f m ⊂ m, logo f m = A e portanto m = (f −1 ) é principal. (observe que pelas escolhas de n e b,
“secretamente” sabemos que a possui valorização n enquanto que b possui valorização n − 1, portanto
f −1 terá valorização 1 como esperado.)

Da condição 5 acima, temos imediatamente os seguintes


70 Extensões Finitas e Integrais

Corolário 4.7 Seja A um domı́nio noetheriano normal e p um ideal primo de A com altura 1. Então
Ap é um domı́nio de valorização discreta.

Corolário 4.8 Seja A um domı́nio noetheriano com dim A = 1. Então A é normal se, e só se, mAm é
principal para todo ideal maximal m de A.

Corolário 4.9 Seja f (x, y) ∈ C[x, y] um polinômio irredutı́vel. Seja (a, b) ∈ C2 um ponto da curva
f (x, y) = 0 e seja m = (x − a, y − b) o ideal maximal correspondente. Então
C[x, y]m
A= 
f (x, y)
é um domı́nio de valorização discreta se, e só se, (a, b) é um ponto não singular de f (x, y) = 0, isto é,
∂f ∂f
(a, b) 6= 0 ou (a, b) 6= 0
∂x ∂y
Em particular, C[x, y]/(f (x, y)) é um domı́nio normal se, e só se, f (x, y) = 0 é uma curva não singular.
Prova Temos que A é um anel noetheriano local com dim A = 1. Mostremos que o ideal maximal
(x − a, y − b) de A é principal. Temos a seguinte expansão (finita) de Taylor
∂f ∂f
f (x, y) = f (a, b) + (a, b) · (x − a) + (a, b) · (y − b)
∂x ∂y
∂2f ∂2f ∂2f
+ 2
(a, b) · (x − a)2 + (a, b) · (x − a)(y − b) + 2 (a, b) · (y − b)2 + · · ·
∂x ∂x∂y ∂y
∂f
Suponha que 6= 0. Como f (a, b) = 0, a imagem em A da igualdade acima tem a forma
∂x (a, b)
 ∂f ∂2f 
0 = (x − a) · (a, b) + (a, b) · (x − a) + · · · + múltiplo de (y − b)
| ∂x ∂x2 {z }
unidade em A
Logo (x − a, y − b) = (y − b) é principal.

Exemplo 4.10 Vamos mostrar que C[x, y]/(y 2 − x3 + x) é um domı́nio normal. Sendo f (x, y) =
y 2 − x3 + x, temos que um ponto singular (a, b) ∈ C2 da curva f (x, y) = 0 satisfaz
( 2
∂f ∂f b = a3 − a
f (a, b) = (a, b) = (a, b) = 0 ⇐⇒ 3a2 − 1 = 0
∂x ∂x
2b = 0
que não tem solução. Logo f (x, y) = 0 é não singular e C[x, y]/(y − x3 + x) é normal.
2

Exemplo
√ 4.11 Vamos mostrar que A = Z[ 3 2] é normal e portanto é igual ao fecho integral de Z em
FT
Q( 3 2). Para isto, temos
√ que mostrar que mAm é principal para todo ideal maximal m de A. Por cálculo √
de fibras de Spec Z[ 3 2] ։ Spec Z, temos que os ideais maximais de A são da forma m = (p, f ( 3 2)),
onde p ∈ Z é um número primo e f (x) ∈ Z[x] é um polinômio tal que sua imagem f (x) ∈ Fp [x] é um
)
28

fator irredutı́vel de x3 − 2 ∈ Fp [x].


7:

Temos o seguinte “critério de não singularidade aritmético” obtido calculando-se a derivada de x3 − 2


módulo p: se gcd(3x2 , x3 − 2) = 1 em Fp [x], então x3 − 2 não possui raı́zes múltiplas e portanto temos
(1
RA

uma fatoração
0

x3 − 2 = f (x) · g(x)
01

com f (x) irredutı́vel e gcd(f (x), g(x)) = 1



Sejam f (x), g(x) ∈ Z[x] pré-imagens de f (x), g(x) ∈ Fp [x] e m = (p, f ( 3 2)). Então existem polinômios
,2

a(x), b(x), j(x), h(x) ∈ Z[x] tais que


29

√3
√3
a(x)f (x) + b(x)g(x) = 1 + p · j(x) ⇒ b( 2)g( 2) ≡ 1 (mod m)
D
ct

e √ √ √
,O

x3 − 2 − f (x)g(x) = p · h(x) ⇒ f ( 2)g( 2) = p · h( 2)


3 3 3

√ √
Portanto, no anel local Am , temos que g( 3 2) ∈ A× 3
m e assim f ( 2) é múltiplo de p, ou seja, mAm = (p)
ET

é principal.
Agora, suponha que gcd(3x2 , x3 − 2) 6= 1 em Fp [x], o que ocorre se p = 2 ou p = 3 (em todos os outros
casos, 2 e 3 são inversı́veis e gcd(3x2 , x3 − 2) =
√ gcd(x2 , x3 − 2) = gcd(x2 , 2) = 1). Mas por cálculo de
3
fibras, o√único primo de A sobre p = 2 é m = ( 2), que já é principal,
√ e o√único √
primo de A sobre p = 3
é m = ( 3 2 + 1), que também é principal (note que 3 = 2 + 1 = ( 3 2 + 1)( 3 4 − 3 2 + 1)).
71

4.1 Anéis de valorização discretos completos domı́nio de D

(incluir independência de valorizações)

4.2 Extensões de anéis de valorização discretos


P
i ei fi = n.

5 Domı́nios de Dedekind
Definição 5.1 Um domı́nio A é um domı́nio de Dedekind se satisfaz os seguintes axiomas:
1. A é noetheriano;
2. A é normal;
3. dim A = 1.
Exemplo 5.2 Z e k[x], k corpo, são domı́nios de Dedekind. Qualquer domı́nio de valorização discreta
é de Dedekind.

Teorema 5.3 Um domı́nio noetheriano A é de Dedekind se, e somente se, todas as suas localizações
Am em primos maximais m são domı́nios de valorização discreta. Em particular, se dim A = 1, A é de
Dedekind se, e só se, os ideais mAm são principais.

Teorema 5.4 (Fatoração Única) Seja A um domı́nio de Dedekind. Todo ideal não nulo de A se
escreve como produto de ideais primos de maneira única, a menos da ordem dos fatores.
Prova

Exemplo 5.5 suponha que char k 6= 2, 3 e seja C = V (Y 2 − X 3 − 10) ⊂ A2k . Por the last example,
k[C] é a Dedekind domain, so we may apply the discussion above. Consider para instance the function
f = x − 2y + 7 ∈ k[C], onde x e y são the images of X e Y in k[C] as usual. Então the zeros of f são
P = (−1, 3) e Q = (9/4, 37/8), the pontos of intesection of the line L = V (X − 2Y + 7) e C.
Seja mP = (x + 1, y − 3) e mQ = (x − 9/4, y − 37/8) be the corresponding maximal ideals. We already
know that both mP e mQ divide (f ), but we suspect that the order of vanishing of f at P é greater than
1 como L é tangent to C at P . In fact, m2P | (f ), como x−2 2 1 2
3 · (x + 1) − 3 · (y − 3) = x − 2y + 7. Portanto
2 2 2
mP mQ | (f ). We now mostrar que mP mQ = (f ). First observe that mP mQ é gerado por

(x + 1)2 (x − 49 ), (x + 1)(y − 3)(x − 49 ), (y − 3)2 (x − 94 )


37 37 37
(x + 1)2 (y − 8 ), (x + 1)(y − 3)(y − 8 ), (y − 3)2 (y − 8 )
FT
But we saw that f ∈ m2P mQ , e como f = (x + 1) − 2(y − 3) = (x − 94 ) − 2(y − 37
8 ), temos m2P mQ =
(f, (y − 37 2
8 )(y − 3) ). Finally
)

28

(f ) = (x − 2y + 7, y 2 − x3 − 10) = x − 2y + 7, (y − 3)2 · (y − 37
8 ) ,
7:

portanto (f ) = m2P mQ .
(1
RA
0

Teorema 5.6 Seja A um domı́nio de Dedekind com K = Frac A. Se L é uma extensão finita e separável
01

de K e B é o fecho integral de A em L então B também é um domı́nio de Dedekind.


,2

Teorema 5.7 Seja A um domı́nio de Dedekind. Um A-módulo M é plano se, e só se, é livre de torção,
29

isto é, ann(m) = 0 para todo m ∈ M não nulo.


D
ct

Teorema 5.8 Seja A um domı́nio normal noetheriano. Então


,O

\
Ap = A
ET

ht p=1

M
0 - A× - K× -
div
Z - 0
ht p=1

onde a soma percorre todos os primos de altura 1.


72 Extensões Finitas e Integrais

6 Primos Associados divisor de zero


anulador
suporte
Vamos estudar o “anel de definição” de um módulo.

Definição 6.1 Seja A um anel e M um A-módulo. Um elemento a ∈ A é um divisor de zero de M se


existe m ∈ M , m 6= 0, tal que a · m = 0. O anulador de um elemento m ∈ M é o ideal de A

def
ann(m) = {a ∈ A | a · m = 0}

Por sua vez, o anulador do módulo M é definido como o ideal de A dado por

def
\
ann M = ann(m) = {a ∈ A | a · m = 0 para todo m ∈ M }
m∈M

Observe que se um A-módulo M pode ser visto como A/(ann M )-módulo via

def
a·m = a·m a ∈ A, m∈M

multiplicação esta que está bem definida uma vez que (a + r) · m = a · m se r ∈ ann M .

Definição 6.2 O suporte supp M de M é definido (em analogia com o suporte de uma função em
Análise) como o conjunto
def
supp M = {p ∈ Spec A | Mp 6= 0}

Lema 6.3 (Anulador e Suporte) Para todo A-módulo temos supp M ⊂ V (ann M ), com igualdade
se M é finitamente gerado sobre A.

Prova Temos

p ∈ supp M ⇐⇒ Mp 6= 0 ⇐⇒ sm 6= 0 para algum m ∈ M e todo s ∈ A \ p


⇒ ann M ⊂ p ⇐⇒ p ∈ V (ann M )

Por outro lado, se M é finitamente gerado, digamos M = Aω1 + · · · + Aωn , então temos que ann M =
T
1≤i≤n ann(ωi ). Assim, se p ∈ V (ann M ), temos que p ⊃ ann(ωi ) para algum i, logo s · ωi 6= 0 para todo
s ∈ A \ p e portanto vale a recı́proca da única implicação acima que não é uma equivalência.
FT
No estudo dos divisores de zero, módulos da forma M = A/p, p ∈ Spec A, são particularmente
fáceis de se entender: temos ann M = p, supp M = V (p) e os divisores de zero de M são exatamente os
)
28

elementos de p. Isto nos leva à seguinte


7:

Definição 6.4 Seja M um A-módulo. Um ideal primo p de A é dito associado a M se satisfaz uma
(1
RA

das (e portanto todas) seguintes condições equivalentes:


1. M contém um submódulo isomorfo a A/p;
0
01

2. p = ann(m) para algum elemento m ∈ M .


,2

O conjunto de primos associados a M é denotado por Ass M (eu sei que soa feio, mas esta é a notação
29

oficial ⌢).
D

Mostremos que as duas condições acima são, de fato, equivalentes. Se p = ann(m), m ∈ M , então
ct

temos uma injeção de A-módulos


,O

A/p ֒→ M
ET

a 7→ a · m

E reciprocamente, se temos uma injeção φ: A/p ֒→ M de A-módulos, sendo m = φ(1) temos p = ann(m).
Primos associados são muito importantes no estudo dos divisores de zero de módulos finitamente
gerados sobre anéis noetherianos. O próximo teorema mostra que primos associados realmente existem
de verdade.
73

Teorema 6.5 Seja A um anel noetheriano e M um A-módulo.


1. Se M 6= 0 então Ass(M ) 6= ∅;
2. O conjunto dos divisores de zero de M é a união dos primos em Ass M .

Prova 1. Considere a famı́lia de ideais

F = {ann(m) | m ∈ M, m 6= 0}

Como A é noetheriano, existe um elemento maximal ann(m) que, como mostraremos, é um ideal primo,
logo associado de M . Suponha que ab ∈ ann(m) mas a ∈ / ann(m). Então am 6= 0 e portanto ann(am) ∈
F . Mas ann(am) ⊃ ann(m), logo devemos ter ann(am) = ann(m). Assim, b ∈ ann(am) = ann(m),
mostrando que o ideal ann(m) é de fato primo.
S
2. Claramente p∈Ass M p consiste de divisores de zero em M . Reciprocamente, dado um divisor de
zero a em M , considere a famı́lia de ideais

G = {ann(m) | m ∈ M, m 6= 0, am = 0}

Como em (1), um elemento maximal p desta famı́lia é um ideal primo, logo p ∈ Ass(M ) e a ∈ p.

Teorema 6.6 (Cadeia Primária) Seja A um anel noetheriano e seja M um A-módulo finitamente
gerado. Então M admite uma cadeia de submódulos

0 = M0 ⊂ M1 ⊂ · · · ⊂ Mn = M

tal que os quocientes Mi+1 /Mi são isomorfos a A/pi com pi ∈ Spec A.
Prova O resultado é verdadeiro para M = 0. Se M 6= 0 então pelo teorema anterior Ass M 6= ∅
e portanto M contém um submódulo M1 ∼ = A/p1 com p1 ∈ Spec A. Se M1 6= M , podemos repetir
o procedimento com M/M1 no lugar de M , obtendo um submódulo M2 de M tal que M2 ⊃ M1 e
M2 /M1 ∼= A/p2 com p2 ∈ Spec A. Este processo eventualmente termina pois M é noetheriano, fornecendo
a decomposição pedida.

Lemma 6.7 Seja


0 - M′ - M - M ′′ - 0

uma sequência exata de A-módulos. Então


FT
Ass M ⊂ Ass M ′ ∪ Ass M ′′
)
28

Prova Sem perda de generalidade podemos supor que M ′ ⊂ M e M ′′ = M/M ′ . Tome p ∈ Ass M ,
7:

digamos p = ann(m), m ∈ M , de modo que N = Am ⊂ M é um submódulo isomorfo a A/p. Se


(1

N ∩ M ′ = 0 então N ∼
RA

= (N + M ′ )/M ′ ⊂ M ′′ e portanto p ∈ Ass M ′′ . Por outro lado, se N ∩ M ′ 6= 0,


vamos mostrar que N ∩ M ′ contém um submódulo N ′ isomorfo a N e que, portanto, p ∈ Ass M ′ . Seja
0

b ∈ A tal que 0 6= bm ∈ N ∩ M ′ . De fato, como b ∈ / ann(m) = p, temos que b não é divisor de zero em
01

N e portanto multiplicação por b estabelece um isomorfismo entre N e N ′ = bN = Abm ⊂ N ∩ M ′ . Isto


,2

encerra a prova.
29
D

Corolário 6.8 Seja A um anel noetheriano e M um A-módulo finitamente gerado. Então Ass M é um
ct

conjunto finito.
,O

Prova Indução no comprimento n da cadeia do teorema anterior. Se M = 0 então Ass M = ∅ enquanto


ET

que se M = A/p, p ∈ Spec A, então Ass M = {p}. Se n > 1, temos uma sequência exata

0 - M1 - M - M/M1 - 0

e M/M1 admite uma cadeia de comprimento n − 1. Assim, por hipótese de indução, Ass M1 e Ass M/M1
são finitos, logo Ass M é finito pelo lema anterior.
74 Extensões Finitas e Integrais

Teorema 6.9 Seja A um anel noetheriano, p ∈ Spec A e seja M um A-módulo. Então

AssAp Mp = {qAp | q ∈ AssA M, q ⊂ p}

Prova Se q ∈ AssA M e q ⊂ p, say q = ann(m), m ∈ M , então qAp = ann(m/1), so qAp ∈ AssAp Mp .


Reciprocamente, se qAp ∈ AssAp Mp para algum prime ideal q ⊂ p, we may write qAp = ann(m/s) para
algum m ∈ M e s ∈ A\p. Como q é finitamente gerado, existe a t ∈ A\p tal que tqm = 0 para todo q ∈ q.
Portanto q ⊂ ann(tm), e reciprocamente se x ∈ ann(tm), então x/1 ∈ ann(tm/1) = ann(m/s) = qAp ,
i.e., x ∈ q. Portanto q ∈ AssA M .

Teorema 6.10 (Suporte e Primos Associados) Seja A um anel noetheriano e M um A-módulo


finitamente gerado. Então [
Supp M = V (q)
q∈Ass M

Além disso, todo primo minimal em supp M é associado.


Prova Temos

p ∈ Supp M ⇐⇒ Mp 6= 0 ⇐⇒ AssRp Mp 6= ∅ ⇐⇒ p ⊃ q para algum q ∈ Ass M

Lemma 6.11 Seja A be a noetherian normal domain, e seja x ∈ A be a nonzero element. Então para
qualquer p ∈ Ass A/x, ht p = 1.
Prova Como p ∈ Ass A/x implies pAp ∈ Ass Ap /x, we may assume that A é local com maximal ideal
/ (x). Observe that xy · p é an ideal of A.
p. Por definition of associated prime, p = (x : y) para algum y ∈
But y/x ∈ y
/ A, portanto the previous lemma implies that x · p 6= p, e portanto xy · p = A ⇐⇒ p = (y/x).
Como p é principal, concluı́mos que ht p = 1.

Teorema 6.12 Seja A um domı́nio noetheriano normal. Então


\
A= Ap
ht p=1
FT
T T
Prova É claro que A ⊂ ht p=1 Ap . Reciprocamente, tome f = x/y ∈ ht p=1 Ap , x, y ∈ A. We need to
)
28

mostrar que f ∈ A. Suponha not; então the image x of x in A/y é not zero. Amongst todo the ideals of
7:

the form ann(rx), r ∈ A, pick a maximal one, say p = ann(ax). Então p ∈ Ass(A/y), e por the lemma,
ht p = 1. But f ∈ Ap , so há m, s ∈ A, s ∈
/ p, tal que f = m/s. Então
(1
RA
0

x m
f= = ⇒ sx = my ⇒ sax = may ⇒ s ∈ ann(ax) = p,
01

y s
,2

a contradiction. This finishes the proof.


29
D

7 Ação de Grupo e Going-down


ct
,O

Seja B um anel e G ⊂ Aut(B) um grupo de automorfismos de B. Denote por A = B G o subanel fixo


por G, isto é,
ET

def
A = B G = {b ∈ B | σ(b) = b para todo σ ∈ G}
Note que se S é um conjunto multiplicativo de A, então a localização de σ ∈ G fornece um automorfismo
S −1 σ de S −1 B que fixa cada elemento de S −1 A. Temos

S −1 A = (S −1 B)G
75

onde ainda denotamos por G o subgrupo de Aut(S −1 B) obtido localizando-se os automorfismos de


G ⊂ Aut(B). De fato, temos uma sequência exata
Y
0 - A - B -
f
B
σ∈G

b 7→ (σ(b) − b)σ∈G

Localizando-a com relação a S, obtemos uma nova sequência exata

S −1 f Y
0 - S −1 A - S −1 B - S −1 B
σ∈G

donde (S −1 B)G = S −1 A. Resumindo: a ação de grupo é compatı́vel com localização arbitrária, o que
será frequentemente usado para reduzir o caso geral para o caso local.
Agora se G é finito, então B é integral sobre A, pois b ∈ B é raiz do polinômio mônico
Y 
p(x) = x − σ(b) ∈ A[x]
σ∈G

cujos coeficientes são expressões simétricas elementares em σ(b), σ ∈ G, e portanto são fixos por G. Agora
seja p ∈ Spec A e S = A\p. Temos uma extensão integral de anéis S −1 A ⊂ S −1 B com (S −1 B)G = S −1 A.
Além disso, se P ∈ Spec B pertence à fibra de p com relação ao mapa Spec B → Spec A induzido pela
inclusão, temos pelo lema 2.1 que S −1 P é um ideal maximal de S −1 B. Assim, quando G é finito,
localizando poderemos sempre supor que A é local e que B é semi-local (i.e., possui um número finito
de ideais maximais).
Teorema 7.1 (Ação transitiva) Sejam B um anel, G um grupo finito de automorfismos de B e
A = B G . Então G age transitivamente nas fibras de Spec B ։ Spec A.
Prova Sejam P, P′ ∈ Spec B dois primos da fibra de p ∈ Spec A. Localizando com relação a S = A \ p,
podemos assumir que A é local com ideal maximal p e que P, P′ são ideais maximais em B. Suponha
por absurdo que P, P′ possuam órbitas disjuntas:

{σ(P) | σ ∈ G} ∩ {σ(P′ ) | σ ∈ G} = ∅

Como os ideais σ(P) e σ(P′ ) são maximais (logo comaximais), pelo teorema chinês dos restos, existe um
elemento b ∈ B tal que
FT
 
b ≡ 0 (mod σ −1 (P)) σ(b) ≡ 0 (mod P)
⇐⇒ para todo σ ∈ G
)
b ≡ 1 (mod σ −1 (P′ )) σ(b) ≡ 1 (mod P′ )
28
7:

Da primeira congruência, temos que a “norma” de b é tal que


(1
RA

def
Y
NG (b) = σ(b) ∈ P ∩ A = p
0
01

σ∈G
,2

Porém, como NG (b) ∈ p ⊂ P′ também, terı́amos σ(b) ∈ P′ para algum σ ∈ G, o que contradiz a segunda
congruência. Logo a ação de G é transitiva na fibra de p.
29
D
ct

Corolário 7.2 Seja A um domı́nio normal com K = Frac A. Seja L ⊃ K uma extensão normal de
,O

corpos e seja B o fecho integral de A em L. Então G = AutK (L) age transitivamente nas fibras de
Spec B ։ Spec A.
ET

Prova Observe inicialmente que todo σ ∈ G se restringe a um A-automorfismo de B: de fato, se b ∈ B


é raiz de um polinômio mônico p(x) ∈ A[x] então p(σ(b)) = σ(p(b)) = 0, o que mostra que σ(b) ∈ B.
Primeiro, tratamos do caso em que L ⊃ K é uma extensão finita e separável e portanto Galois com
grupo de Galois G. Neste caso, A ⊂ B G ⊂ K mas como B G ⊂ B e A é normal, devemos ter A = B G .
O resultado agora segue do teorema anterior.
76 Extensões Finitas e Integrais

O caso em que L ⊃ K é uma extensão Galois infinita pode ser reduzido ao anterior por uma “casa
dos pombos infinita”. Dados dois primos P e P′ de B, para cada corpo M tal que L ⊃ M ⊃ K e M ⊃ K
é Galois finito, considere o conjunto

F (M ) = {σ ∈ G | σ(P ∩ M ) = P′ ∩ M }

Temos que F (M ) é não vazio (pelo já demonstrado) e fechado em G na topologia de Krull. Assim, como
G é compacto, temos
\
F (M ) 6= ∅
M

onde M percorre todas as subextensões Galois finitas de L ⊃ K. Um elemento σ nesta intersecção é um


lim AutK (M ) tal que σ(P) = P′ , como desejado.
automorfismo de G = ←−
M
Agora, para L ⊃ K é arbitrário, seja M = LG . Temos que L ⊃ M é Galois enquanto que M ⊃ K
é puramente inseparável. Assim, basta tratar do caso em que L ⊃ K é puramente inseparável com
p = char K > 0. Note que neste caso G é trivial, assim temos que mostrar que a fibra de um primo
p ∈ Spec A possui um único elemento. De fato, como para qualquer b ∈ B existe um natural n > 0 tal
n
que bp ∈ K ∩ B = A, o único primo de B na fibra de p é

def n
P = {b ∈ B | bp ∈ p para algum n > 0}

Exemplo 7.3 Seja τ (z) = z a conjugação complexa. Seja B = Z[i] e G = {id, τ } ⊂ Aut(B). Então
A = Z. Se p é um número primo da forma 4k + 3, então (p) é primo em B e G fixa (p). Por outro lado,
se p é um número primo da forma 4k + 1, então p se fatora como p = (a + bi)(a − bi), a, b ∈ Z, de modo
que (a + bi) e (a − bi) são os primos da fibra de (p), que são claramente permutados por τ .

Teorema 7.4 (“Going-down”) Seja B ⊃ A uma extensão integral de domı́nios com A normal. Então
se P′ ∈ Spec B está sobre p′ então existe P ∈ Spec B sobre p tal que P ( P′ .

∃P ( P′ ( B
| | |
p ( p′ ( A
FT
Prova Sejam K = Frac A, L = Frac B e M o fecho normal de L. Seja C o fecho integral de A em M .
)
28


σ(Q) Q ( σ(Q′ ) = Q′0 Q ( C ⊂ M
7:

| | | |
(1
RA

def
P = σ(Q) ( P = Q′0 ∩ B

( B ⊂ L
| | | |
0
01

p ( p′ ( A ⊂ K
,2

Pelo going-up, existem primos Q ( Q′ de C tais que Q ∩ A = p e Q′ ∩ A = p′ . Temos Q ∩ B ( Q′ ∩ B,


29

de modo que é tentador definir P como Q ∩ B, mas infelizmente pode ocorrer um “desalinhamento”
Q′ ∩ B 6= P′ , mas que pode ser facilmente corrigido utilizando-se um automorfismo de AutK (M ), já que
D
ct

este grupo age transitivamente sobre a fibra de p′ . “Going-up once more”, tome Q′0 ∈ Spec C tal que
,O

Q′0 ∩ B = P′ . Existe σ ∈ AutK (M ) tal que σ(Q′ ) = Q′0 . Basta tomar agora P = σ(Q) ∩ B.
ET

Teorema 7.5 (“Going-down Plano”) Seja B uma A-álgebra plana. Dada uma inclusão de ideais
primos p ⊂ q de A e um ideal primo Q de B sobre q, existe P ∈ Spec B tal que P ⊂ Q e P está sobre p.

Prova Como B é plano sobre A, BQ é fielmente plano sobre Aq , logo Spec BQ → Spec Aq é sobrejetor.
Assim, existe P ∈ Spec B tal que a imagem de PBQ é pAq . Mas então P ⊂ Q e P está sobre p.
77

Definição 7.6 Sejam B um anel e G ⊂ Aut(B) um grupo de automorfismos de B. Temos que G induz grupo de deco
grupo de inér
uma ação em Spec B; dado P ∈ Spec B, definimos o grupo de decomposição GP de P (com relação
a G) é como o estabilizador de P com relação a esta ação, ou seja,

def
GP = {σ ∈ G | σ(P) = P}

Um automorfismo σ ∈ GP induz um automorfismo σ ∈ Aut(B/P) dado por

def
σ(b) = σ(b) para todo b ∈ B

Assim, temos um morfismo de grupos


GP → Aut(B/P)
σ 7→ σ
cujo kernel IP é chamado de grupo de inércia de P.
Exemplo 7.7 Seja B = C[x] e G o grupo de “rotações”

G = {id, ρ, ρ2 , . . . , ρn−1 }

onde ρ denota o automorfismo de C-álgebras

ρ: B → B
x 7→ x · e2πi/n

Temos A = B G = C[xn ] ∼
= C[y]. Se a ∈ C é não nulo, temos que a órbita de (x − a) consiste em n primos
distintos:
{(x − a · e2πik/n ) ∈ Spec B | k = 0, 1, . . . , n − 1}
de modo que o grupo de decomposição G(x−a) = {id} é trivial. Por outro lado, temos que G(x) = G.
Como ρ = id ∈ Aut(C), temos também que I(x) = G.

Na notação da definição, seja S um conjunto multiplicativo de A tal que S ∩ P = ∅. É fácil mostrar


que os grupos de decomposição e inércia são compatı́veis com a localização em S:

GS −1 P = GP e IS −1 P = IP
FT
Além disso, os corpos residuais dos primos não se alteram com a localização. Isto nos permitirá localizar
)
com relação a p = P ∩ A ∈ Spec A, de modo que poderemos supor A local e B semi-local.
28

Setup 7.8 Seja B um anel, seja G ⊂ Aut(B) um grupo finito de automorfismos e seja A = B G . Seja
7:

q ∈ Spec B e seja p = q ∩ A ∈ Spec A. Sejam k e l os corpos residuais de p e q respectivamente.


(1
RA

Finalmente, sejam
B ′ = B Gq q′ = q ∩ B ′ ∈ Spec B ′
0

e
01

B ′′ = B Iq q′′ = q ∩ B ′′ ∈ Spec B ′′
,2

Denote ainda por l′′ o corpo residual de q′′ . Pictoriamente:


29

q ⊂ B l
D
ct

| |
,O

q′′ ⊂ B ′′ l′′
| |
ET

q′ ⊂ B′ k
| k
p ⊂ A k

(a igualdade dos corpos residuais de p e q′ será justificada no próximo lema).


78 Extensões Finitas e Integrais

Teorema 7.9 (Anel de decomposição) Na notação do setup 7.8, temos:


1. q é o único primo de B sobre q′ .
2. A inclusão k(p) ֒→ k(q′ ) de corpos residuais é um isomorfismo.
3. pBq′ ′ = q′ Bq′ ′ .

Prova 1. Como Gq age transitivamente sobre a fibra de q′ e Gq estabiliza q, o resultado é claro.


Para os próximos itens, podemos localizar com relação a S = A \ p e portanto supor que (A, p, k) é
local e que q e q′ são maximais em B e B ′ , respectivamente.
2. Vamos mostrar que para todo b′ ∈ B ′ existe a ∈ A tal que a ≡ b′ (mod q). Seja σ1 , . . . , σg um
sistema de representantes de classes laterais à esquerda de Gq em G com σ1 = 1. Como G permuta
transitivamente os primos na fibra de p, esta fibra consiste nos ideais maximais σ1 (q), . . . , σg (q). Se i 6= 1
então σi−1 ∈
/ Gq e portanto σi−1 (q) 6= q. Então (1) implica que σi−1 (q) ∩ B ′ 6= q′ e portanto pelo teorema
chinês dos restos existe x ∈ B ′ tal que
 
x ≡ b′ (mod q′ ) x ≡ b′ (mod q)

x ≡ 1 (mod (σi−1 q) ∩ B ′ ) para i 6= 1 σi (x) ≡ 1 (mod q) para i 6= 1

Então a “norma” de x
def
Y
a = σi (x) ≡ b′ (mod q)
1≤i≤g

é um elemento com a propriedade desejada. De fato, vejamos que a ∈ A. Dado σ ∈ G, escrevendo


σσi = σji τi com 1 ≤ ji ≤ g e τi ∈ Gq , temos que ji1 6= ji2 se i1 6= i2 : caso contrário, σσi1 τi−1
1
=
−1 −1 −1 ′
σσi2 τi2 ⇐⇒ σi1 τi1 = σi2 τi2 , o que é impossı́vel já que σi1 Gq 6= σi2 Gq . Assim, como x ∈ B é fixo
por Gq , temos
Y Y Y
σ(a) = σσi (x) = σji τi (x) = σi (x) = a
1≤i≤g 1≤i≤g 1≤i≤g

o que prova que a é fixo por todo elemento de G, isto é, a ∈ A.


3. Como p ⊂ q′ , basta mostrar que q′ ⊂ pBq′ ′ . Sejam q′1 = q′ , q′2 , . . . , q′s os ideais maximais de B ′ . Note
que, mutatis mutandis, a mesma prova do item anterior mostra que qualquer x ∈ q′1 \ (q′2 ∪ · · · ∪ q′s )
pertence a pBq′ ′ . De fato, se σ ∈
/ Gq então σ −1 (q′ ) = q′i para algum i 6= 1; isto implica que σ(x) ∈ / q′ .
Q ′
Portanto 2≤i≤g σi (x) ∈ / q onde σi são como em (2). Portanto
FT
Y
)
x· σi (x) ∈ q ∩ A = p ⇒ x ∈ pBq′
28

2≤i≤g
7:
(1
RA

Em seguida, observe que como a fibra de p é finita, temos que Spec B ′ /pB ′ é um conjunto finito,
logo a k-álgebra B ′ /pB ′ é um anel artiniano (corolário 3.9). Portanto temos um isomorfismo
0
01

B′ Bq′ ′ Bq′ ′s
,2

= 1
× · · · ×
pB ′ pBq′ ′ pBq′ ′s
29

1
D
ct

Para provar que pBq′ ′ = q′ Bq′ ′ precisamos mostrar que o ideal maximal q′1 Bq′ ′ /pBq′ ′ do primeiro fator é
,O

1 1
0. Seja t ∈ q′1 Bq′ ′ /pBq′ ′ e seja x ∈ B ′ uma pré-imagem do elemento (t, 1, . . . , 1) no produto acima. Então
1 1
x ∈ q′1 \ (q′2 ∪ · · · ∪ q′s ) e pelo que já provamos x ∈ pBq′ . Mas isto implica t = 0, como desejado.
ET

Observação 7.10 Note que pelo item 1 do teorema anterior, Spec B → Spec B ′ induz uma bijeção entre
os primos da fibra de p em B e B ′ , respectivamente, ou seja, o primo p “se decompôs” totalmente em
B ′ , o anel fixo por Gq , daı́ o nome grupo de decomposição para este último grupo.
79

Teorema 7.11 (Anel de Inércia) Na notação do setup 7.8, temos


1. q′′ é o único primo de B ′′ sobre q′ .
2. l ⊃ k é uma extensão normal de corpos, i.e., qualquer polinômio irredutı́vel f (x) ∈ k[x] que
possui uma raiz em l se fatora completamente.
3. O morfismo Gq ։ Aut(l/k) é sobrejetor. Assim, temos um isomorfismo

Gq /Iq = Aut(l/k)

4. l′′ ⊃ k é a máxima subextensão separável de l ⊃ k, de modo que Aut(l/k) = Aut(l′′ /k).


5. pBq′′′′ = q′′ Bq′′′′ .

Prova 1. Segue de (1) do teorema anterior juntamente com o fato de que Spec B ։ Spec B ′′ é
sobrejetor (B ⊃ B ′′ é uma extensão integral).
2. Seja f (x) ∈ k[x] um polinômio mônico irredutı́vel e suponha que a imagem b ∈ l de b ∈ B é uma
raiz de f (x). Temos que b é raiz do polinômio mônico
Y
m(x) = (x − σ(b)) ∈ A[x]
σ∈G

cuja imagem m(x) em k[x] se fatora completamente em l[x]. Mas como f (x) é o polinômio minimal de
b e m(b) = 0, temos que f (x) | m(x). Assim, f (x) também se fatora completamente em l[x].
3. Observe que pelo teorema anterior p e q′ têm mesmo corpo residual e que pBq′ ′ = q′ Bq′ ′ . Portanto,
substituindo A por Bq′ ′ e B pela localização com relação a B ′ \ q′ , podemos supor que todos os anéis A,
B e B ′′ são locais e que Gq = G.
Seja ks ⊃ k a máxima subextensão separável de l ⊃ k. Pela prova do item anterior, qualquer
elemento b ∈ ks é raiz um polinômio m(x) de grau |G|. Assim, pelo teorema do elemento primitivo,
[ks : k] ≤ |G| é finito e podemos escrever ks = k(b) para algum b ∈ B. Seja f (x) ∈ k[x] o polinômio
minimal de b. Sendo m(x) como no item anterior, temos que f (x) | m(x), logo as raı́zes de f (x) são
todas da forma σ(b) = σ(b), σ ∈ G = Gq . Assim, dado um automorfismo φ ∈ Aut(l/k) = Gal(ks /k),
como φ permuta as raı́zes de f (x), temos que existe σ ∈ G tal que φ(b) = σ(b), o que implica φ = σ já
que b gera ks sobre k. Portanto G ։ Aut(l/k) é sobrejetor, como querı́amos demonstrar.
4. Observe que aplicando os itens anteriores a B ′′ no lugar de A temos que l′′ ⊃ l é uma extensão
normal de corpos com Aut(l′′ /l) é trivial (pois Iq ։ Aut(l′′ /l) é sobrejetor). Logo l ⊃ l′′ é puramente
inseparável e portanto ks ⊂ l′′ (mantemos as notações e reduções do item anterior). Queremos mostrar
que ks = l′′ . Para isto, substituindo B ′′ por B, l′′ por l e G = Gq por Gq /Iq , podemos assumir que Iq é
FT
trivial e que, portanto, G = Aut(l′′ /k).
Observe inicialmente que qualquer elemento b ∈ l′′ é raiz de um polinômio em k[x] de grau menor
Q
)
′′
ou igual a |G| (tome a imagem de σ∈G (x − σ(b)) ∈ A[x]). Agora seja k0 = (l′′ )Aut(l /k) . Temos que
28

l′′ ⊃ k0 é uma extensão Galois de grau |G| e que k0 ⊃ k é puramente inseparável. Pelo teorema do
7:

elemento primitivo, podemos escrever l′′ = k0 (θ) para algum θ ∈ l′′ . Temos que θ é separável sobre k:
(1
RA

como θ possui |G| conjugados distintos, temos


0

[k(θ) : k]sep ≥ |G| ≥ [k(θ) : k] ≥ [k(θ) : k]sep


01
,2

e assim temos igualdade em todos os lugares, o que mostra que k(θ) ⊃ k é uma extensão separável de
grau |G|.
29

Para concluir que l′′ ⊃ k é separável, devemos mostrar que k0 = k. Como k0 é puramente inseparável
D

sobre k é suficiente mostrar que, dado um elemento λ ∈ k0 , λ ∈ k(θ), sendo portanto separável sobre k.
ct

n
Denote por p = char k > 0. Existe um n natural tal que λp ∈ k. Assim,
,O

n n n n
(λ − θ)p = λp − θp ⇒ k(θp ) ⊂ k(λ − θ)
ET

n
Mas como θ é separável sobre k temos que k(θp ) = k(θ). Por outro lado, [k(λ − θ) : k] ≤ |G| = [k(θ) : k],
logo k(λ − θ) = k(θ). Finalmente, isto implica que λ = θ + (λ − θ) ∈ k(θ) e portanto λ é separável sobre
k, como querı́amos.
5.
80 Extensões Finitas e Integrais

Teorema 7.12 (Completamento e Grupo de Decomposição) Seja B um anel noetheriano, G um discriminante


grupo finito de automorfismos de B e A = B G . Seja p ∈ Spec A e sejam P1 , . . . , Pg ∈ Spec B os primos
da fibra de p com relação a Spec B ։ Spec A. Então temos um isomorfismo
Y
B ⊗A Âp = B̂Pi
1≤i≤g

onde o chapéu denota completamentos com relação aos ideais maximais correspondentes. Temos que G
age transitivamente sobre B ⊗A Âp , permutando seus fatores. O estabilizador do i-ésimo fator B̂Pi é
GPi e
GPi
B̂Pi = Âp

8 Exercı́cios
01. (Nilva’s problem) Seja A um anel noetheriano e m um ideal maximal. Mostre que A/mn é artiniano
para todo n ≥ 0.
02. Seja d um inteiro livre de quadrados (i.e.,
√ d não é divisı́vel por nenhum quadrado de primo). Mostre
que o anel A dos inteiros algébricos em Q( d) é A = Z + Zω onde
 √
1+ d
ω= se d ≡ 1 (mod 4)
√ 2
d caso contrário

03. Seja A = C[x, y]/(y 2 − x2 (x + 1)). Mostre que as localizações Am são normais para todos os ideais
maximais m de A com exceção de m = (x̄, ȳ).
04. Mostre que C[x, y]/(y 2 − x3 + x) é normal.
05. Seja G um grupo finito de automorfismos de um anel B e seja

def
A = B G = {b ∈ B | σ(b) = b para todo σ ∈ G}

o anel fixo por G.


(a) Mostre que B é integral sobre A.
(b) Seja B = C[x, y] e considere G = {id, σ} onde σ: B → B é o automorfismo de C-álgebras definido
por σ(x) = −x e σ(y) = −y. Mostre A = B G = C[x2 , xy, y 2 ] e determine explicitamente a ação de G
sobre as fibras de Spec B → Spec A sobre os ideais maximais de A.
FT
06. Seja A um domı́nio com K = Frac A. Seja L ⊃ K uma extensão finita separável de corpos de grau
)
n = [L : K].
28

(a) Mostre que existe uma base ω1 , . . . , ωn ∈ L de L sobre K tal que cada ωi é integral sobre A.
7:

(b) Seja B ⊂ L o subanel dos elementos integrais sobre A. Utilize o truque do determinante para
(1
RA

mostrar que
ω1 ωn
A · ω1 + · · · + A · ωn ⊂ B ⊂ A · + ··· + A·
0

D D
01

onde D ∈ A é o determinante da matriz (TrL/K (ωi ωj ))1≤i,j≤n (D é chamado de discriminante da base


,2

ωi ).
(c) Conclua que se A é noetheriano então B é finito sobre A e portanto é noetheriano. Além disso, se
29

A é um domı́nio de ideais principais então B é um A-módulo livre de posto n.


D
ct

07. Mostre que √ √ √


,O

3 3 3
Z[ 2] = {a + b 2 + c 4 | a, b, c ∈ Z}
ET

é normal (sugestão: localize ou utilize o exercı́cio anterior).


08. Seja A uma k-álgebra finitamente gerada onde k é um corpo.
(a) Mostre que o conjunto de todos os ideais maximais é denso em Spec A.

(b) Seja a um ideal de A. Mostre que a é igual à intersecção de todos os ideais maximais de A contendo
a.
81

09. Seja A um domı́nio finitamente gerado sobre Z. Mostre que um ideal m de A é maximal se, e somente
se, A/m é um corpo finito.
Dica: seja S = Z \ {0} e aplique o teorema de normalização de Noether para a Q-álgebra finitamente
gerada S −1 A.
10. Seja A um dvr e seja p(x) ∈ R[x] um polinômio de Eisenstein. Mostre que R[x]/(p(x)) também é
um dvr.
11. Mostre que um domı́nio de Dedekind com um número finito de primos é um PID.
12. Mostre que qualquer ideal em um domı́nio de Dedekind pode ser gerado por dois elementos.
13. Seja
0 - M′ - M - M ′′ - 0

uma sequência exata de A-módulos. Mostre que

supp M = supp M ′ ∪ supp M ′′

14. Seja A = C[x, y] e M = C[x, y]/(x3 y 4 ), visto como A-módulo. Determine Ass(M ) e escreva sua
cadeia primária.
15. Seja θ uma raiz do polinômio irredutı́vel f (x) = x3 − x2 − 2x − 8 ∈ Q[x] e seja K = Q[θ].
(a) Mostre que ν = (θ2 + θ)/2 é um inteiro algébrico.
(b) Mostre que o fecho integral de Z em K é Z + Zθ + Zν.

FT
)
28
7:
(1
RA
0
01
,2
29
D
ct
,O
ET
D
ET
,O
ct
29
RA
,2
01
0
(1
FT
7:
28
)
Chapter 5
dimensão de
altura
caternário
polinômio bin

Dimensão

1 Dimensão de Krull
Definição 1.1 Seja A um anel. A dimensão de Krull dim A de A é definida como o maior comprimento
n de uma cadeia (numerada a partir do zero)
p0 ( p1 ( p2 ( p3 ( · · · ( pn
de ideais primos em A.
Definição 1.2 Seja A um anel e p ∈ Spec A. A altura ht p de p é definida como dim Ap ou, equivalen-
temente, o maior comprimento n de uma cadeia (numerada a partir do zero)
p0 ( p1 ( p2 ( p3 ( · · · ( pn = p
de ideais primos em A contidos em p.
Exemplo 1.3 Se k é um corpo, temos dim k = 0. Como todo ideal primo não nulo de k[t] é maximal,
temos que dim k[t] = 1. Da mesma forma, temos também que dim Z = 1 e dim k[[t]] = 1.
Diretamente das definições, temos
dim A ≥ ht p + dim A/p
para todo p ∈ Spec A. Se ocorre a igualdade para todos os ideais primos de A, dizemos que A é
caternário. Os anéis do exemplo anterior são todos caternários.

2 Algumas Identidades Binomiais


Seja d ≥ 0 um inteiro. Definimos o polinômio binomial de grau d como o polinômio em Q[x] dado por
 
x def x(x − 1)(x − 2) . . . (x − d + 1)
=
d d!
FT
 
Observe que se n ∈ Z então nd ∈ Z. Para n ≥ d é um inteiro positivo, nd = d!(n−d)! n!
coincide com o
coeficiente binomial usual. Podemos utilizar este fato para dar provas combinatórias de certas identidades
)
28

de polinômios binomiais, por exemplo, a identidade


     
7:

x x x+1
+ =
(1
RA

d d+1 d+1
pode ser demonstrada da seguinte forma: se x = n ≥ d é um inteiro positivo, temos que ambos os lados
0
01

da igualdade contam o número de maneira  de escolhermos d + 1 dentre n + 1 objetos; no lado esquerdo,


esta contagem é feita considerando as nd maneiras que incluem o (n + 1)-ésimo objeto (basta escolher os
,2

n
d objetos que faltam dentre os n primeiros objetos) e as d+1 maneiras que não incluem o (n + 1)-ésimo
29

objeto. Agora, como ambos os lados são polinômios em Q[x] que concordam para um número infinito
D

de valores, ou seja, a diferença é um polinômio com um número infinito de raı́zes, temos que eles devem
ct

ser iguais como polinômios em Q[x].


,O

Seja ∆ o operador “derivada discreta”


ET

def
∆f (n) = f (n + 1) − f (n)
Por exemplo, a identidade binomial acima pode ser reescrita como
   
x x
∆ =
d d−1
84 Dimensão

Lemma 2.1 função de Hilbert

1. Um polinômio p(x) ∈ Q[x] de grau d é tal que p(n) ∈ Z para todo inteiro n ≫ 0 se, e só se, é
da forma        
x x x x
p(x) = ad + ad−1 + · · · + a1 + a0
d d−1 1 0
com ai ∈ Z e ad 6= 0.
2. Seja f : N → N uma função. Suponha que, para n ≫ 0, f (n+1)−f (n) = q(n) para um polinômio
q(x) ∈ Q[x] de grau d − 1. Então, para n ≫ 0, f (n) = p(n) para um polinômio p(x) ∈ Q[x] de
grau d.
Prova 1. Pela interpretação combinatória, é claro que qualquer combinação Z-linear de polinômios
binomiais assume valores inteiros para valores inteiros positivos de x. Para mostrar a recı́proca, faremos
uma indução em d, sendo o caso d = 0 claro. Como os polinômios xd formam uma base de Q[x] sobre
Q, podemos escrever
       
x x x x
p(x) = ad + ad−1 + · · · + a1 + a0
d d−1 1 0
com ai ∈ Q. Para mostrar que estes coeficientes são, de fato, inteiros, considere
     
x x x
∆p(x) = ad + ad−1 + · · · + a1
d−1 d−2 0
Como ∆p(n) = p(n + 1) − p(n) ∈ Z para todo inteiro n ≫ 0, por hipótese de indução temos que
a1 , . . . , ad ∈ Z. Mas então
     
x x x
p(x) − ad − ad−1 − · · · − a1
d d−1 1
é um polinômio constante a0 ∈ Z.
2. Por (1), podemos escrever
     
x x x
q(x) = ad + ad−1 + · · · + a1 ai ∈ Z
d−1 d−2 0
Seja      
x x x
p(x) = ad + ad−1 + · · · + a1
d d−1 1
Temos que ∆(f − p)(n) = 0 para todo inteiro n ≫ 0. Em outras palavras, f (n) − p(n) = a0 ∈ Z é
constante para n ≫ 0 e o resultado segue.

3 Polinômio de Hilbert-Samuel
FT
L
Seja A = d≥0 Ad um anel graduado. Suponha que k = A0 é um corpo e que A é finitamente gerado
sobre k por elementos de grau 1 (por exemplo, o anel de polinômios k[x1 , . . . , xd ] possui esta propriedade).
)
28

Seja M um A-módulo graduado finitamente gerado. Definimos a função de Hilbert de M por


7:

hM (n) = dimk Mn
(1
RA

Teorema 3.1 Nas condições acima, existe um polinômio p(x) ∈ Q[x] tal que hM (n) = p(n) para todo
0

inteiro n ≫ 0.
01

Prova Faremos uma indução no número de geradores d de grau 1 de A sobre k. Se d = 0, temos que
,2

A = k e M é um k-espaço vetorial de dimensão finita, logo hM (n) é uma função constante.


Agora suponha que d > 0 e seja g ∈ A1 um gerador de A sobre k. Temos uma sequência exata de
29

A-módulos graduados
D

0 - N - M - M [1] - P - 0
g
ct
,O

onde N e P são respectivamente o kernel e o cokernel da multiplicação por g e M [1] denota o módulo
M com graduação “deslocada por 1”, ou seja, M [1]d = Md+1 para d ≥ 0. Olhando para a dimensão da
ET

parte de grau n obtemos


dimk Nn − dimk Mn + dimk Mn+1 − dimk Pn = 0 ⇐⇒ hM (n + 1) − hM (n) = hP (n) − hN (n)
Como g anula P e N , podemos vê-los como A/(g)-módulos finitamente gerados. Por hipótese de indução,
hP (n) e hN (n) são funções polinomiais para n ≫ 0, logo o mesmo vale para hM (n) pelo lema anterior.
85

Exemplo 3.2 Seja A = k[x1 , . . . , xd ], k corpo. Então função de Hil


polinômio de
  função de Hil
n+d−1 polinômio de
hA (n) = sistema de pa
d

é um polinômio de grau d.
Pelo exemplo anterior, temos que o grau da polinômio de Hilbert mede, de certa forma, o “número
de parâmetros independentes” de A. Agora utilizaremos este fato no estudo da dimensão de um anel
local noetheriano. Até o final deste capı́tulo, (A, m, k) denotará um anel local noetheriano.
Definição 3.3 A função de Hilbert-Samuel hA (n) do anel A é definida como

def A
hA (n) = lenA
mn

Teorema 3.4 Existe um polinômio χA (x) ∈ Q[x] tal que hA (n) = χA (n) para n suficientemente grande.
Este polinômio é chamado de polinômio de Hilbert-Samuel.
Prova Basta mostrar que hA (n+1)−hA(n) é uma função polinomial para n ≫ 0. Temos uma sequência
exata
mn - A - A
0 - n+1 n+1
- 0
m m mn
de modo que hA (n + 1) − hA (n) = lenA mn /mn+1 = dimk mn /mn+1 . Portanto esta diferença é a função
de Hilbert da k-álgebra graduada
M mn
mn+1
n≥0

que é gerada sobre k = A/m pelos geradores de m em grau 1.

Até o final deste capı́tulo, (A, m, k) denotará um anel local noetheriano.


Definição 3.5 A função de Hilbert-Samuel hA (n) do anel A é definida como

def A
hA (n) = lenA
mn

Teorema 3.6 Existe um polinômio χA (x) ∈ Q[x] tal que hA (n) = χA (n) para n suficientemente grande.
FT
Este polinômio é chamado de polinômio de Hilbert-Samuel.

4 Teorema de dimensão de Krull


)
28

p m, k) um anel local. Um conjunto {a1 , . . . , an } ⊂ A é chamado de sistema de


Definição 4.1 Seja (A,
7:

parâmteros de A se (a1 , . . . , an ) = m.
(1
RA

Teorema 4.2 (Prime avoidance) Seja a um ideal arbitrário e p1 , . . . , pn be ideais primos em um


0

anel. Então
01

[
a⊂ pi ⇒ a ⊂ pi para algum i
,2

1≤i≤n
29

S A prova é por indução em n. Se n = 1 o resultado é claro. Agora seja n > 1 e suponha que
Prova
D
ct

a ⊂ 1≤i≤n pi . Mostremos que a está contido na união de n − 1 primos pi ’s, de modo que o resultado
S S
,O

segue por indução. Suponha, por absurdo, que a ⊂ 1≤j≤n pj mas que a 6⊂ j6=i pj para todo i. Para
S
cada i, tome ai ∈ a \ j6=i pj , de modo que ai ∈ pi mas ai ∈ / pj se i 6= j. Considere o elemento
ET

a = a1 + a2 a3 . . . an ∈ a. Então a não pertence a nenhum primo pi :


na ∈ p 
1 1 a1 ∈/ pi
⇒a∈ / p1 e ⇒a∈ / pi para i = 2, 3, . . . , n
a2 a3 . . . an ∈
/ p1 a2 a3 . . . an ∈ p i
S
Isto contradiz a ⊂ 1≤i≤n pi .
86 Dimensão

Lemma 4.3 Seja (A, m, k) um anel local. Seja a ∈ m, B = A/(a) e denote por dA e dB os graus de χA
e χB respectivamente. Então
1. dB ≥ dA − 1;
2. se a é regular (i.e, não é divisor de zero em A), então dB = dA − 1.

Prova Denote por m a imagem de m em B. O anel B é um anel local com ideal maximal m e corpo
residual B/m = A/m = k. Temos isomorfismos

B A (a) + mn (a) a A
n = e = =
m (a) + mn m (a) ∩ m n n
(m : a)

onde o último isomorfismo é induzido pela multiplicação por a e (mn : a) denota o ideal {b ∈ A | ab ∈ mn }.
Assim, obtemos a seguinte sequência exata:

A A B
0→ → n → n →0
(mn : a) m m

Portanto, como mn−1 ⊂ (mn : a), temos


 A B   A 
lenA = lenB n + lenA
mn m (mn : a)
B   A 
≤ lenB + lenA
mn mn−1
Assim, para n grande o suficiente, temos que χB (n) ≥ χA (n) − χA (n − 1) ⇒ dB ≥ dA − 1.
Agora assuma que a é regular. Então, pelo teorema de Artin-Rees, temos que existe uma constante
r para a qual (mn : a) ⊂ mn−r para todo n grande o suficiente. De fato, basta observar que

b ∈ (mn : a) ⇒ ba ∈ (a) ∩ mn = mn−r · ((a) ∩ mr ) ⊂ amn−r

e, como a é regular, ba ∈ amn−r ⇒ b ∈ mn−r . Assim,


 A B   A 
lenA = lenB n + lenA
mn m (mn : a)
B   A 
≥ lenB + lenA
mn mn−r
FT
Isto implica que, para n grande o suficiente, temos χB (n) ≤ χA (n) − χA (n − r) ⇒ dB ≤ dA − 1 ⇒ dB =
dA − 1.
)
28

Teorema 4.4 (Krull) Seja (A, m, k) um anel local noetheriano. Então dim A é finito. Além disso,
7:

dim A é igual ao grau dA do polinômio de Hilbert-Samuel χA (n) e à menor cardinalidade δA de um


(1
RA

sistema de parâmetros de A.
0

Prova Vamos mostrar uma sequência de desigualdades dim A ≤ dA ≤ δA ≤ dim A. Observe que a
01

primeira desigualdade mostra que dim A é finita.


,2

(i) dim A ≤ dA : vamos mostrar, por indução em dA , que dada uma cadeia de ideais primos p0 ( p1 (
· · · ( pn de tamanho n, temos n ≤ dA . Se dA = 0, então χA (n) = lenA A/mn é constante para
29

n suficientemente grande. Mas como χA (n + 1) = χA (n) + lenA mn /mn+1 , temos que mn ⊗A k =


D

mn /mn+1 = 0 para n grande. Por Nakayama, temos que mn = 0 para algum n, logo Spec A = {m}
ct

e portanto n ≤ 0.
,O

Agora suponha que dA > 0. Como dA/p0 ≤ dA , basta mostrarmos que n ≤ dA/p0 . Assim, substi-
ET

tuindo A por A/p0 , podemos assumir que p0 = (0) e que A é um domı́nio. Seja a ∈ p1 um elemento
não nulo e seja B = A/(a). Como a é regular, temos pelo lema que dB = dA − 1. Por outro lado, as
imagens de p1 , . . . , pn em B formam uma cadeia de tamanho n − 1, assim por hipótese de indução
temos que n − 1 ≤ dB = dA − 1 ⇒ n ≤ dA , como desejado. p
(ii) dA ≤ δA : vamos utilizar indução em δA . Se δA = 0, então m = (0) e portanto lenA A/mn = lenA A
é constante para n suficientemente grande, provando que dA = 0. Agora suponha que δA > 0. Seja
87

a ∈ A um elemento pertencente a um sistema de parâmetros de cardinalidade δA e seja B = A/(a).


Então δB ≤ δA − 1 e por hipótese de indução δB = dB . Assim, pela parte (a) do lema, temos que
dA − 1 ≤ dB = δB ≤ δA − 1 ⇒ dA ≤ δA .
(iii) δA ≤ dim A: vamos fazer umap indução em dim A (que já sabemos ser finito por (i)). Se dim A = 0,
então Spec A = {m}, logo (0) = m e portanto δA = 0. Agora suponha que dim A > 0. Como
A é noetheriano, A possui apenas um número finito de ideais primos minimais. Assim, utilizando
o “prime avoidance”, podemos escolher a ∈ m que não pertence a nenhum primo minimal, de
modo que B = A/(a) é tal que dim B ≤ dim A − 1. Por hipótese de indução, temos portanto
δB ≤ dim B. Note ainda que se a1 , . . . , ar ∈ A são elementos cujas imagens em B formam um sistema
de parâmetros de B, então a, a1 , . . . , ar forma um sistema de parâmetros de A, logo δA ≤ δB + 1.
Assim δA − 1 ≤ δB ≤ dim B ≤ dim A − 1 ⇒ δA ≤ dim A.

Corolário 4.5 (Teorema do Ideal Principal de Krull) Seja A um domı́nio noetheriano e seja
a 6= 0 um elemento de A. Então qualquer ideal primo que é minimal dentre os que contêm (a) tem altura
1. Mais geralmente, qualquer ideal primo que é minimal dentre os que contêm (a1 , . . . , an ) tem altura
menor ou igual a n.

Prova Seja p ∈ Spec A que é minimal dentre os primos contendo (a1 , . . . , an ).


p Portanto pAp =
(a1 , . . . , an ), logo ht p = dim Ap = δAp ≤ n.

Como δA é menor ou igual ao número mı́nimo de geradores do ideal maximal m, temos

Corolário 4.6 Seja (A, m, k) um anel local noetheriano. Então

m
dimk ≥ dim A
m2

Definição 4.7 Um anel local noetheriano (A, m, k) é regular se m pode ser gerado por dim A elementos.
Um anel noetheriano B qualquer é regular se todas as suas localizações Bn com relação aos seus ideais
maximais n são regulares.

Exemplo 4.8 Seja A = Z[x] e seja m = (3, x). Então dim Am = 2 e Am é regular. De fato, como temos
uma cadeia de ideais primos (0) ( (x) ( (3, x), concluı́mos que dim Am ≥ 2. Por outro lado, mAm pode
ser gerado por dois elementos, assim dim Am = δAm ≤ 2. Em suma: dim Am = 2, que também é igual ao
número mı́nimo de geradores de mAm , assim Am é regular.
FT
Exemplo 4.9 Seja A = Z[x]/(x2 − 18) e seja m = (3, x). Então dim Am = 1 e Am não é regular. De
fato, como temos uma cadeia
p de ideais primos (0) ( (3, x), concluı́mos que dim Am ≥ 1. Por outro lado,
)
2
28

como x = 18 ⇒ x ∈ (3), temos que {3} é um sistema de parâmetros e portanto δAm ≤ 1. Logo
dim Am = δAm = 1. Por outro lado, temos isomorfismos de F3 -espaços vetoriais
7:
(1
RA

mAm m (3, x) (3, x)


= 2 = 2 = (9, 3x, x2 )
0

(mAm )2 m (9, 3x, x )


01

que tem dimensão 2 sobre F3 , portanto o número mı́nimo de geradores de mAm é 2. Assim, Am não é
,2

regular.
29
D

Teorema 4.10 Um anel local noetheriano regular (A, m, k) é um domı́nio.


ct
,O

Prova Indução em dim A. Se dim A = 0, temos que m = (0) e portanto A é um corpo. Agora suponha
que n = dim A > 0 e sejam a1 , . . . , an geradores de m. Então B = A/(a1 ) é um anel noetheriano local
ET

cujo ideal maximal pode ser gerado por n − 1 geradores. Mas como dim B ≥ dim A − 1 = n − 1, devemos
ter dim B = n − 1 e portanto B é regular, logo um domı́nio por hipótese de indução. Assim, (a1 ) é um
ideal primo. Dados x, y ∈ A com xy = 0, se ambos estes elementos são não nulos então dividindo x e y
por a1 , podemos supor x, y 6∈ (a1 ), o que é um absurdo.
88 Dimensão

Teorema 4.11 (Dimensão das Fibras) Sejam (A, m, k) e (B, n, l) anéis locais noetherianos e seja
φ: A → B um morfismo local (i.e., φ−1 (n) = m). Então

dim B ≤ dim A + dim B ⊗A k

com igualdade se B é (fielmente) plano sobre A.


Prova Sejam m = dim A e n = dim B ⊗A k. Seja a1 , . . . , am um sistema de parâmetros de A
e sejam b1 , . . . , bn ∈ B pré-imagens de um sistema de parâmetros de B ⊗A k = B/φ(m)B. Para
provar que dim B ≤ m + n, pelo teorema de dimensão de Krull é suficiente mostrar que os m + n
elementos
p b1 , . . . , bn , φ(a1 ), . . . , φ(am ) formam um sistema de parâmetros de B. r Para mostrar que
n = (b1 , . . . , bn , φ(a1 ), . . . , φ(am )), tome b ∈ n. Existe um r natural tal que b ∈ (b1 , . . . , bn ) em
B ⊗A k = B/φ(m)B, ou seja, br ∈ (b1 , . . . , bn ) + φ(m)B. Como ms ⊂ (a1 , . . . , am ) para algum natural s,
temos
s
brs ∈ (b1 , . . . , bn ) + φ(m)B ⊂ (b1 , . . . , bn ) + φ(ms )B ⊂ (b1 , . . . , bn , φ(a1 ), . . . , φ(am ))

como desejado.
Agora suponha que B seja plano sobre A, de modo que o going-down vale. Escolha uma cadeia de
primos em A
p0 ( p1 ( · · · ( pm = m
e uma cadeia de primos em B contendo φ(m), ou seja, primos da fibra de m em B:

φ(m) ⊂ qm ( qm+1 ( · · · ( qm+n = q

Pelo going-down, podemos estender a cadeia anterior para uma cadeia de primos em B

q0 ( · · · ( qm−1 ( qm ( · · · ( qm+n = q

com qi sobre pi para i = 0, 1, . . . , m. Assim, dim B ≥ m + n e portanto dim B = m + n.

Corolário 4.12 Seja A um anel noetheriano. Então

dim A[x] = dim A + 1

Prova Dada uma cadeia de ideais primos de A


FT
p0 ( p1 ( · · · ( pn
)
28

temos uma cadeia de ideais primos em A[x]


7:
(1
RA

p0 · A[x] ( p1 · A[x] ( · · · ( pn · A[x] ( pn + (x)


0

de modo que dim A[x] ≥ dim A + 1. Para mostrar a desigualdade oposta, seja q um ideal maximal de
01

A[x] tal que dim A[x] = dim A[x]q . Seja p = q ∩ A ∈ Spec A e seja k = Frac(A/p) seu corpo residual.
,2

Como A[x] é plano sobre A, temos que A[x]q é plano sobre Ap , logo
29

dim A[x] = dim A[x]q = dim Ap + dim A[x]q ⊗A k = dim Ap + dim k[x]q ≤ dim A + 1
D
ct
,O
ET

Lemma 4.13 Seja A um anel noetheriano. Se A é regular, então A[x] também é regular.
Prova Seja q ∈ Spec A[x] e seja p = q ∩ A. Seja k = Frac A/p o corpo residual de p. A fibra de p é
isomorfa a Spec k[x], logo temos duas possibilidades: q = pA[x] ou q = (p, f (x)) onde f (x) ∈ A[x] é tal
que sua imagem em k[x] é irredutı́vel. No segundo caso, como dim A[x]q = dim Ap + 1 e pAp é gerado
por dim Ap elementos, temos que A[x]q é regular.
89

5 Exercı́cios
01. Para cada um dos anéis locais noetherianos a seguir, determine:
(i) um sistema de parâmetros minimal;
(ii) o polinômio de Hilbert-Samuel;
(iii) a dimensão de Krull.
Diga ainda se cada um destes anéis é regular ou não.
(a) um corpo K (b) Z(p) , p primo
(c) Q[t](t) (d) Q[t](t2 +1)
(e) C[x, y](x,y) (f) C[x, y, z](x,y,z)
(g) Z[x](3,x) (h) C[x, y](y2 −x3 )
(i) Am onde A = C[x, y]/(y 2 − x2 (x + 1)) e m = (x + 1, y)
(j) Am onde A = C[x, y]/(y 2 − x2 (x + 1)) e m = (x, y)
(k) Am onde A = Z[x]/(x2 − 15) e m = (3, x)
(l) Am onde A = Z[x]/(x2 − 45) e m = (3, x)

FT
)
28
7:
(1
RA
0
01
,2
29
D
ct
,O
ET
D
ET
,O
ct
29
RA
,2
01
0
(1
FT
7:
28
)
Chapter 6
complexo
exato
morfismo
homologia
homotopia

Métodos Homológicos

Em Topologia, um fato surpreendente é que a “falha” ou “obstrução” de certa propriedade pode ser
quantificada pela falha de exatidão de certas sequências. Por exemplo, o grupo de homologia H1 (X, Z)
de um espaço topológico X pode ser entendido como uma medida da “falha” de X ser simplesmente
conexo. Neste capı́tulo, veremos como esta filosofia oriunda da Topologia se aplica no estudo de anéis
regulares e de planaridade.

1 Complexos e Homologia
Definição 1.1 Dado um anel A, uma sequência de morfismos de A-módulos

··· - Mi+1 - Mi
di+1
- Mi−1
di di−1
- ···

é um complexo se a composição de duas flechas consecutivas é 0, ou seja,

di ◦ di+1 = 0 ⇐⇒ im di+1 ⊂ ker di para todo inteiro i

Se im di+1 ⊂ ker di para todo i, dizemos que o complexo é exato.


Um morfismo entre dois complexos (M• , d• ) e (N• , e• ) de A-módulos é uma coleção de morfismos de
A-módulos φi : Mi → Ni tais que o seguinte diagrama comuta:

··· - Mi+1 di+1 - Mi di -


Mi−1
di−1 -
···

φi+1 φi φi−1
? ei+1 - ? ei - ? ei−1 -
··· - Ni+1 Ni Ni−1 ···
FT
Definição 1.2 Dado um complexo (M• , d• ), definimos a sua i-ésima homologia como o A-módulo
)
28

def ker di
Hi (M• , d• ) =
7:

im di+1
(1
RA

Observe que (M• , d• ) é exato se, e só se, Hi (M• , d• ) = 0 para todo i. Assim, a homologia de um complexo
0

quantifica a “falta de exatidão” de um complexo.


01

Dado um morfismo do complexos φ• : (M• , d• ) → (N• , e• ), temos um morfismo induzido em homolo-


,2

gias
29

Hi (φ• ): Hi (M• ) → Hi (N• )


z ∈ ker di
D

z mod im di+1 7→ φi (z) mod im ei


ct
,O

É fácil verificar que este mapa está bem definido e que ele preserva identidade e composições. Temos
portanto que cada Hi é um funtor da categoria de complexos de A-módulos para a categoria de A-
ET

módulos.
Definição 1.3 Sejam dois complexos (M• , d• ) e (N• , e• ) de A-módulos e sejam φ• e ψ• dois morfismos
entre eles. Uma homotopia entre φ• e ψ• é um coleção de mapas ki : Mi → Ni+1 tais que

φi − ψi = ei+1 ◦ ki + ki−1 ◦ di
92 Métodos Homológicos

para todo i. Dois morfimos são homotópicos se existe alguma homotopia entre eles. quase-isomorfismo
sequência exata curta
morfismos conectores
- Mi+1 di+1 - di - di−1 -
··· Mi Mi−1 ···

1
φi+1 , ψi+1 k i φi , ψi k i− φi−1 , ψi−1
? ei+1 - ? ei - ? ei−1 -
··· - Ni+1 Ni Ni−1 ···

A importância de homotopias se deve ao seguinte


Lemma 1.4 Se φ• , ψ• : (M• , d• ) → (N• , e• ) são homotópicos, então eles induzem a mesma homologia.
Prova Seja k• uma homotopia entre φ• e ψ• . Temos que mostrar que Hi (φ• ) = Hi (ψ• ), ou seja, que
Hi (φ• − ψ• ) = 0. Mas k• “constrói pré-imagens” para anular a homologia: se z ∈ ker di , temos que

φi (z) − ψi (z) = ei+1 ◦ ki (z) + ki−1 ◦ di (z) = ei+1 ◦ ki (z) ∈ im ei+1

que portanto possui imagem 0 em Hi (N• , e• ).

Definição 1.5 Um morfismo de complexos φ• : (M• , d• ) → (N• , e• ) é um quase-isomorfismo se os


morfismos induzidos em homologia

Hi (φ• ): Hi (M• ) - Hi (N• )


são todos isomorfismos.


Uma sequência exata curta de morfismos de complexos

0 - M′ - M•
f•
- M ′′
g•
- 0
• •

é uma sequência de morfismos de complexos para a qual

0 - M′ - Mi
fi
- M ′′
gi
- 0
i i

é exata para cada i. O próximo resultado é a ferramenta mais importante no estudo e cálculo da
FT
homologia de complexos.
Teorema 1.6 (A sequência exata longa) Dada uma sequência exata curta de complexos
)
28

0 - M•′ - M•
f•
- M•′′
g•
- 0
7:
(1
RA

existem morfismos δi : Hi (M•′′ , d′′• ) → Hi−1 (M•′ , d′• ), chamados de morfismos conectores, tais que a
0

sequência
01

··· - Hi (M•′ ) Hi (f- •)


Hi (M• ) - Hi (M•′′ ) δ-
Hi (g• ) i
,2

Hi−1 (M•′ ) - Hi−1 (M• )


Hi−1 (f• ) Hi−1 (g• )
- Hi−1 (M ′′ ) - ···
δi−1

29

é exata.
D
ct

Prova Vamos primeiro definir δi . Considere o seguinte diagrama commutativo com linhas exatas:
,O

- M′ fi - gi - ′′ - 0
ET

0 i Mi Mi

d′i di d′′i
? f ? ?
0 - Mi−1
′ - Mi−1 gi−1- Mi−1
i−1 ′′ - 0
93

Dado um elemento z ∈ Hi (M•′′ ) representado por z ∈ ker d′′i , tome y ∈ Mi tal que gi (y) = z. Como M -regular

gi−1 ◦ di (y) = 0, existe um único x ∈ Mi−1 para o qual fi−1 (x) = di (y).

gi -
- z ∈ M ′′
y ∈ Mi i

di

fi−1 ?

x ∈ Mi−1 ⊂ - fi−1 (x) = di (y) ∈ Mi−1

É fácil verificar que d′i−1 (x) = 0 e que δi (z) = x + im d′i é independente das escolhas de z ou y.
Um “diagram chase” simples (mas longo e tedioso, você pode fazer quando ninguém estiver olhando)
mostra que a sequência longa acima definida é realmente exata.

Observação 1.7 Os mapas δi são transformações naturais: dado um diagrama comutativo com linhas
exatas
0 - M•′ - M• - M•′′ - 0

? ? ?
0 - N•′ - N• - N•′′ - 0

temos um diagrama comutativo

δiM
Hi (M•′′ ) - Hi−1 (M•′ )

? δN ?
Hi (N•′′ ) -
i
Hi−1 (N•′ )

Uma observação final. Às vezes, trabalharemos com complexos com numeração crescente:
dp−1 dp dp+1
··· - M p−1 - Mp - M p+1 - ···
FT
Mutatis mutandis, todos os resultados e definições anteriores também se aplicam para essas “sequências
crescentes”; neste caso, convencionamos escrever os ı́ndices como superescritos. Por exemplo, para a
)
sequência anterior, sua homologia é definida por
28
7:

ker dp
H p (M • ) =
(1
RA

im dp−1
0

2 Sequências Regulares e o Complexo de Koszul


01

Como primeiro exemplo de aplicação prática, vejamos como as técnicas homológicas são úteis no estudo
,2

de sequências regulares.
29

Definição 2.1 Seja A um anel e M um A-módulo. Dizemos que um elemento a ∈ A é M -regular se a


multiplicação por este elemento M - M é injetora, ou seja, am = 0 ⇒ m = 0 para todo m ∈ M .
D

a
ct

Dizemos que uma sequência a1 , . . . , an ∈ A é uma M -sequência regular (ou apenas M -sequência) se
,O

as seguintes condições são satisfeitas:


ET

1. ai é M/(a1 , . . . , ai−1 )M -regular para cada i = 1, . . . , n;


2. (a1 , . . . , an )M 6= M .
Exemplo 2.2 Seja (A, m, k) um anel local noetheriano regular de dimensão n, de modo que m =
(a1 , . . . , an ). Temos que a1 , . . . , an é uma sequência A-regular pois A/(a1 , . . . , ai−1 ) é um anel noetheriano
local regular, logo um domı́nio, de modo que ai não é divisor de zero neste anel.
94 Métodos Homológicos

No estudo de sequências regulares, o seguinte complexo terá papel central. complexo de Koszul

Definição 2.3 Seja M um A-módulo e a = (ai )1≤i≤n uma sequência de elementos em A. O complexo
de Koszul (K• (a, M ), d• ) associado a M e a é definido da seguinte maneira:

0 se p < 0
Kp (a, M ) = L M se p = 0
 M · e se p > 0
0≤i1 <···<ip ≤n i1 ...ip

Ou seja, para p > 0 o módulo Kp (a, M ) é o produto tensorial de M com o A-módulo livre de posto np
e base {ei1 i2 ...ip | 1 ≤ i1 < i2 < · · · < ip ≤ n}. Finalmente, definimos, para p > 1,

dp : Kp (a, M ) → Kp−1 (a, M )


X
m · ei1 ...ip 7→ (−1)k−1 aik m · ei1 ...îk ...ip
1≤k≤p

onde ei1 ...îk ...ip é o elemento da base obtido omitindo-se o ı́ndice ik . Para p = 1, d1 (m · ei ) = ai m,
1 ≤ i ≤ n.
Note que realmente temos um complexo:
dp−1 ◦ dp (m · ei1 ...ip )
X
= (−1)k−1 aik dp−1 (m · ei1 ...îk ...ip )
1≤k≤p
!
X X X
k−1 j−1 j
= (−1) aik (−1) aij m · ei1 ...îj ...îk ...ip + (−1) aij m · ei1 ...îk ...îj ...ip
1≤k≤p 1≤j≤k−1 k+1≤j≤p

=0
pois cada elemento da base ei1 ,...,ep com dois ı́ndices ij e ik omitidos aparece duas vezes na soma acima
com coeficientes de sinais opostos.
Exemplo 2.4 Para n = 1, o complexo de Koszul é simplesmente

0 - K 1 = M e1 - K0 = M
d1 =a1
- 0

onde d1 é simplesmente a multiplicação por a1 . Assim, a sequência a1 é M -regular se, e só se,
H1 (K• ) = 0 e H0 (K• ) 6= 0
Exemplo 2.5 Para n = 2, o complexo de Koszul é dado por
FT
 
−a2
d2 =
)
a1
0 - K2 = M e12 - K 1 = M e1 ⊕ M e2 - K0 = M - 0
d1 =(a1 ,a2 )
28
7:

onde os mapas di são dados por


(1
RA

d1 (m1 e1 + m2 e2 ) = a1 m1 + a2 m2 d2 (me12 ) = a1 me2 − a2 me1


0
01

Vejamos a relação entre este complexo e a M -regularidade de a1 , a2 . Para isto, considere a seguinte
sequência exata de complexos de Koszul:
,2

- 0 - M e12 me12 7→ me1 - - 0


29

0 M e1
D

 
ct

−a2
a1
,O

a1
? ? ?
- M e1 me1 7→ me- m1 e1 + m2 e2 7→ m-
ET

0
1
M e1 ⊕ M e2
2
M - 0

a1 (a1 , a2 )
? id ? ?
0 - M -M - 0 - 0
95

Aqui as linhas são exatas, a coluna do meio é o complexo de Koszul para a1 , a2 e as colunas externas
são os complexos de Koszul para a1 , sendo o da direita “deslocado” em uma posição. Temos portanto
uma sequência exata (não muito) longa

0 - H2 (K• (a1 , a2 , M )) - H1 (K• (a1 , M ))


δ2
- H1 (K• (a1 , M )) - H1 (K• (a1 , a2 , M )) - H0 (K• (a1 , M ))
δ1
- H0 (K• (a1 , M )) - H0 (K• (a1 , a2 , M )) - 0

Vamos descrever explicitamente os morfismos de conexão. Seja me1 , m ∈ M , um representante de um


elmento de H1 (K• (a1 , M )), de modo que a1 m = 0. Temos

δ2 (me1 ) = d2 (me12 ) = −a2 me1

Assim, δ2 é induzido pela multiplicação por −a2 . Da mesma forma mostra-se que δ1 é é induzido pela
multiplicação por +a2 .
Se a1 , a2 é M -regular, então H1 (K• (a1 , M )) = 0 e δ1 é injetor, pois a2 é regular em M/a1 M =
H0 (K• (a1 , M )). Portanto, da sequência exata acima, temos H1 (K• (a1 , a2 , M )) = H2 (K• (a1 , a2 , M )) = 0
enquanto H0 (K• (a1 , a2 , M )) = M/(a1 , a2 )M 6= 0.
Agora suponhamos adicionalmente que (A, m, k) é um anel noetheriano local, que a1 , a2 ∈ m e que M é
finitamente gerado sobre A. Vejamos o que ocorre quando H1 (K• (a1 , a2 , M )) = 0 e H0 (K• (a1 , a2 , M )) 6=
0. Neste caso, a2 anula H1 (K• (a1 , M )). Mas como M é noetheriano, temos que esta homologia é um
módulo é finitamente gerado sobre A, logo por Nakayama concluı́mos que H1 (K• (a1 , M )) = 0, ou seja, a1
é regular. Assim, temos também que H2 (K• (a1 , a2 , M )) = 0. Isto implica que a2 é regular em M/a1 M :
se a2 m2 = a1 m1 para mi ∈ M , temos que m1 e1 − m2 e2 ∈ ker d1 = im d2 , logo m2 ∈ a1 M .
Resumindo: se (A, m, k) é um anel noetheriano local, a1 , a2 ∈ m e M 6= 0 é um módulo finitamente
gerado sobre A, então
a1 , a2 é M -regular ⇐⇒ H1 (K• (a1 , a2 , M )) = 0

já que, por Nakayama, H0 (K• (a1 , a2 , M )) = M/(a1 , a2 )M = 0 ⇐⇒ M = 0. Note que, em particular,
a1 , a2 é M -regular se, e só se, a2 , a1 é M -regular, pois a condição H1 (K• (a1 , a2 , M )) = 0 é claramente
independente da ordem dos ai ’s.

No exemplo anterior, temos que a homologia do complexo de Koszul é uma “medida de falha” de
regularidade de uma sequência. Podemos generalizar este exemplo, para isto provemos inicialmente o
seguinte
FT
Lemma 2.6 Seja A um anel, M uma A-módulo e a = (ai )1≤i≤n uma sequência de elementos em A.
Denote Hp (K• (a, M )) por Hp (a, M ).
)
28

1. Temos uma sequência exata de complexos


7:
(1
RA

0 → K• (a1 , . . . , an−1 , M ) - K• (a1 , . . . , an , M )


f g
- K• (a1 , . . . , an−1 , M )[−1] → 0
0
01

onde [−1] denota “deslocamento” por −1, isto é,


,2

def
Kp (a1 , . . . , an−1 , M )[−1] = Kp−1 (a1 , . . . , an−1 , M )
29
D
ct

Desta sequência, obtemos uma sequência exata longa da forma


,O

(−1)p an
··· - Hp (a1 , . . . , an−1 , M ) → Hp (a1 , . . . , an , M ) → Hp−1 (a1 , . . . , an−1 , M )
ET

p−1
(−1)
- Hp−1 (a1 , . . . , an−1 , M ) → Hp−1 (a1 , . . . , an , M ) → · · ·
an

2. Para todo p, an Hp (a1 , . . . , an , M ) = 0. Em particular, Hp (a1 , . . . , an , M ) é anulada por qual-


quer elemento do ideal (a1 , . . . , an ).
96 Métodos Homológicos

Prova 1. Para cada p, defina fp : Kp (a1 , . . . , an−1 , M ) → Kp (a1 , . . . , an , M ) por

fp (mei1 i2 ...ip ) = mei1 i2 ...ip m∈M

e gp : Kp (a1 , . . . , an , M ) → Kp (a1 , . . . , an−1 , M )[−1] por



mei1 i2 ...ip−1 se ip = n
gp (mei1 i2 ...ip ) = m∈M
0 se ip < n

Temos portanto sequências exatas

0 - Kp (a1 , . . . , an−1 , M ) - Kp (a1 , . . . , an , M )


fp
- Kp (a1 , . . . , an−1 , M )[−1]
gp
- 0

e é fácil verificar que os mapas fp e gp comutam com os diferenciais dp dos complexos de Koszul.
Assim, temos uma sequência exata de complexos. Falta ainda mostrar que os morfismos de conexão
δp : Hp (a1 , . . . , an−1 , M )[−1] → Hp−1 (a1 , . . . , an−1 , M ) são dados por multiplicação por (−1)p−1 an .
P
Tome um representante ...ip−1 ei1 ...ip−1 ∈ ker dp de um elemento de Hp−1 (a1 , . . . , an−1 , M ).
mi1P
O resultado segue observando que mi1 ...ip−1 ei1 ...ip−1 n ∈ Kp (a1 , . . . , an , M ) é uma pré-imagem deste
elemento por gp e que
X 
dp mi1 ...ip−1 ei1 ...ip−1 n
X  X 
k−1 p−1
= (−1) aik mi1 ...ip−1 ei1 ...îk ...ip−1 n + (−1) an mi1 ...ip−1 ei1 ...ip−1
i1 ,...,ip−1 1≤k≤p−1
X
p−1
= (−1) an mi1 ...ip−1 ei1 ...ip−1
P
utilizando o fato que mi1 ...ip−1 ei1 ...ip−1 ∈ ker dp .
2. Podemos escrever
 
Kp (a1 , . . . , an , M ) = ⊕ip <n M ei1 ...ip ⊕ ⊕ip =n M ei1 ...ip

= Kp (a1 , . . . , an−1 , M ) ⊕ Kp−1 (a1 , . . . , an−1 , M )

e, nesta decomposição, dp : Kp (a1 , . . . , an , M ) → Kp−1 (a1 , . . . , an , M ) se escreve como



dp (x, y) = dp (x) + (−1)p−1 an y, dp−1 (y)
FT
para x ∈ Kp (a1 , . . . , an−1 , M ) e y ∈ Kp−1 (a1 , . . . , an−1 , M ).
)

28

Assim, se dp (x, y) = 0 então dp (x) = (−1)p an y e dp−1 (y) = 0, logo an · (x, y) = dp+1 0, (−1)p x .
Isto mostra que an Hp (a1 , . . . , an , M ) = 0. Como temos um isomorfismo de complexos
7:
(1
RA

K• (a1 , . . . , an , M ) ∼
= K• (aσ(1) , . . . , aσ(n) , M )
0
01

para qualquer permutação σ de 1, 2, . . . , n, temos também um isomorfismo das homologias. Assim, por
simetria, todo ai anula Hp (a1 , . . . , an , M ).
,2
29

Agora podemos generalizar o critério do segundo exemplo.


D
ct

Teorema 2.7 Seja M 6= 0 um módulo finitamente gerado sobre um anel noetheriano local (A, m, k).
,O

Sejam a1 , . . . , an ∈ m. As seguintes condições são equivalentes:


1. a1 , . . . , an é uma M -sequência regular.
ET

2. Hp (a1 , . . . , an , M ) = 0 para todo p > 0.


3. H1 (a1 , . . . , an , M ) = 0.

Prova Observe inicialmente que M/(a1 , . . . , an )M 6= 0 automaticamente por Nakayama (lema III.2.3.2).
A prova é por indução em n, sendo que os casos n = 1 e n = 2 seguem dos exemplos. Seja n > 1.
97

(1 ⇒ 2) A sequência exata longa do lema anterior, juntamente com a hipótese de indução, mostra que
Hp (a1 , . . . , an , M ) = 0 se p > 1. Para p = 1, temos o seguinte fragmento da sequência exata longa:
0 =H1 (a1 , . . . , an−1 , M ) - H1 (a1 , . . . , an , M ) f
- H0 (a1 , . . . , an−1 , M )
- H0 (a1 , . . . , an−1 , M )
an

Como an é regular em H0 (a1 , . . . , an−1 , M ) = M/(a1 , . . . , an−1 )M , concluı́mos que f = 0. Portanto


H1 (a1 , . . . , an , M ) = 0 e o resultado vale para p = 1 também.
(2 ⇒ 3) Claro.
(3 ⇒ 1) Temos uma sequência exata:
H1 (a1 , . . . , an−1 , M )
−an
- H1 (a1 , . . . , an−1 , M ) - H1 (a1 , . . . , an , M ) = 0
Como an ∈ m, por Nakayama (lema III.2.3.2) temos H1 (a1 , . . . , an−1 , M ) = 0. Por hipótese de indução,
temos que a1 , . . . , an é M -regular. Por outro lado, a exatidão de
0 = H1 (a1 , . . . , an , M ) - H0 (a1 , . . . , an−1 , M ) - H0 (a1 , . . . , an−1 , M )
an

mostra que an é regular em H0 (a1 , . . . , an−1 , M ) = M/(a1 , . . . , an−1 )M .

Novamente como temos um isomorfismo de complexos K• (a1 , . . . , an , M ) ∼


= K• (aσ(1) , . . . , aσ(n) , M )
para qualquer permutação σ de 1, 2, . . . , n, obtemos o seguinte
Corolário 2.8 Nas condições do teorema anterior, uma permutação de uma sequência M -regular é
M -regular.
Continuamos nas condições do teorema anterior. Queremos agora considerar M -sequências que são
maximais; elas certamente existem pois para toda M -sequência a1 , a2 , . . ., temos uma cadeia ascendente
estrita (a1 ) ( (a1 , a2 ) ( · · ·, que estabiliza pois A é noetheriano. O fato surpreendente é que todas as
M -sequências possuem o mesmo comprimento:
Teorema 2.9 Seja (A, m, k) um anel local noetheriano. Sejam b1 , . . . , bn geradores de m e seja
q = max{i | Hi (b1 , . . . , bn , M ) 6= 0}
Então toda M -sequência maximal tem tamanho n − q.
Prova Seja a1 , . . . , as uma M -sequência maximal, provemos por indução
S em s que s = n − q. Se s = 0,
então todo elmento de m é um divisor de zero em M , logo m ⊂ p∈Ass(M) p. Pelo “prime avoidance”
(teorema V.4.2), temos m ∈ Ass(M ), assim existe v 6= 0 em M tal que m = ann(v). Portanto
v ∈ Hn (b1 , . . . , bn , M ) = {m ∈ M | b1 m = · · · = bn m = 0}
o que mostra que neste caso q = n.
FT
Agora seja s > 0. Vamos mostrar que
max{i | Hi (b1 , . . . , bn , M/a1 M ) 6= 0} = q + 1 (∗)
)
Deste modo, por hipótese de indução aplicada à M/a1 M -sequência maximal a2 , . . . , as de tamanho s − 1,
28

teremos s − 1 = n − (q + 1) ⇐⇒ s = n − q.
7:

Para provar (∗), considere a sequência exata


(1
RA

0 - M - M
a1
- M/a1 M - 0
0

Temos uma sequência exata de complexos de Koszul


01

- K• (b1 , . . . , bn , M ) a1
- K• (b1 , . . . , bn , M ) - K• (b1 , . . . , bn , M/a1 M ) - 0
,2

0
e portanto uma sequência exata longa
29

- Hp (b1 , . . . , bn , M ) a1
- Hp (b1 , . . . , bn , M ) - Hp (b1 , . . . , bn , M/a1 M )
D

···
ct

- Hp−1 (b1 , . . . , bn , M ) - Hp−1 (b1 , . . . , bn , M )


a1
- Hp−1 (b1 , . . . , bn , M/a1 M ) - ···
,O

Mas como a1 ∈ m = (b1 , . . . , bn ) anula estes módulos (lemma 2.6), esta sequência longa se parte em
ET

sequências menores da forma


0 - Hp (b1 , . . . , bn , M ) - Hp (b1 , . . . , bn , M/a1 M ) - Hp−1 (b1 , . . . , bn , M ) - 0
Se p > q + 1, temos Hp (b1 , . . . , bn , M ) = Hp−1 (b1 , . . . , bn , M ) = 0 e portanto Hp (b1 , . . . , bn , M/a1 M ) = 0
também para o termo do meio. Finalmente, para p = q + 1, obtemos Hq+1 (b1 , . . . , bn , M/a1 M ) ∼ =
Hq (b1 , . . . , bn , M ) 6= 0, o que encerra a prova de (∗) e do teorema.
98 Métodos Homológicos

Definição 2.10 Nas condições do teorema anterior, o número de elementos depth M em qualquer M - Cohen-Macauley
exato à esquerda
sequência maximal é chamado de profundidade de M . exato à direita
exato
Exemplo 2.11 Seja k um corpo e A = k[x, y](x,y) /(y 2 − x3 ). O complexo de Koszul de A com relação
à sequência x, y é  
−y
x
0 - Ae12 - Ae1 ⊕ Ae2 (x,y) - A - 0
Como A é um domı́nio e x, y não são 0, temos que H2 (x, y, A) = 0. Por outro lado, temos que o elemento
−x2 e1 + ye2 ∈ ker d1 mas não pertence a im d2 , já que y ∈
/ (x) (caso contrário, este anel seria regular, o
que já sabemos não ser verdade). Isto mostra que H1 (x, y, A) 6= 0. Assim, depth A = 2 − 1 = 1.
A profundidade é um importante invariante de módulos e anéis. Temos a seguinte relação com a
dimensão:
Teorema 2.12 Seja (A, m, k) um anel local noetheriano. Então
depth A ≤ dim A
p
Prova Indução na dimensão. Se dim A = 0 então m = (0) e portanto depth M = 0. Se dim A > 0
e a1 , . . . , as é uma sequência A-regular maximal (s = depth A) então depth A/a1 A = s − 1, tanto como
A-módulo e como A/a1 A-módulo. Por outro lado, dim A/a1 A = dim A − 1 pois a1 não é divisor de zero.
Por indução, temos portanto que
s − 1 = depth A/a1 A ≤ dim A/a1 A = dim A − 1 ⇐⇒ depth A ≤ dim A

Exemplo 2.13 Seja k um corpo e A = k[x, y](x,y) /(xy, x2 ). Temos que dim A = 1 pois y é um sistema
de parâmetros. Por outro lado, como (x, y) = ann(x) é associado, temos que todo elemento deste ideal
maximal é divisor de zero e portanto depth A = 0, mostrando que a desigualdade acima pode ser estrita.
Definição 2.14 Um anel noetheriano local A é Cohen-Macauley se depth A = dim A.
Veremos mais tarde que a condição acima é uma espécie de “regularidade fraca.” Em particular,
anéis regulares são Cohen-Macauley, já que uma base minimal para o ideal maximal é uma sequência
regular.

3 Resoluções e Funtores Derivados


Após este breve interlúdio com o complexo de Koszul, voltemos à situação abstrata geral. Vamos estudar
FT
agora a “falha de exatidão” de certos funtores, o que nos leva ao conceito de funtor derivado.

3.1 Módulos Projetivos e Injetivos


)
28

Definição 3.1.1 Seja F um funtor da categoria de A-módulos para a categoria de grupos abelianos.
7:

Dizemos que F é exato à esquerda se para toda sequência exata curta de A-módulos
(1
RA

0 - M′ - M - M ′′ - 0
0
01

a sequência
0 - FM′ - FM - F M ′′
,2

também é exata. Por outro lado, o funtor F é dito exato à direita se para toda sequência exata curta
29

de A-módulos como acima, a sequência


D

- FM - F M ′′ - 0
ct

FM′
,O

é exata. O funtor F é exato se é simultaneamente exato à esquerda e à direita, ou seja, preserva


sequências exatas.
ET

Se G é um funtor contravariante, dizemos que ele é exato à esquerda se


0 - GM ′′ - GM - GM ′

é exato para toda sequência exata curta de A-módulos como acima. Analogamente define-se funtor
contravariante exato à direita e funtor contravariante exato.
99

Exemplo 3.1.2 Seja N um A-módulo qualquer. Então − ⊗A N é exato à esquerda, sendo exato se, e projetivo
injetivo
só se, N é A-plano. grupo divisı́ve

Exemplo 3.1.3 Para qualquer A-módulo N , temos um funtor HomA (N, −) que associa a cada A-módulo
M o grupo abeliano dos morfismos de A-módulos φ: N → M . Para cada flecha f : M1 → M2 , este funtor
associa a flecha
HomA (N, f ): HomA (N, M1 ) → HomA (N, M1 )
φ 7→ f ◦ φ

Agora é fácil checar que HomA (N, −) é um funtor exato à esquerda. Da mesma forma, define-se o funtor
contravariante HomA (−, N ), que também é exato à esquerda.

Definição 3.1.4 Um A-módulo P é projetivo se o funtor HomA (P, −) é exato. Um A-módulo I é


injetivo se o funtor HomA (−, I) é exato.
L
Exemplo 3.1.5 Todo módulo livre é projetivo: se M = i∈I Aei é livre, então
Y Y
HomA (M, T ) = HomA (Aei , T ) = T ei
i∈I i∈I

e portanto o funtor HomA (M, −) é exato.

Teorema 3.1.6 Um A-módulo I é injetivo se, e só se, para qualquer ideal a de A

HomA (A, I) ։ HomA (a, I)

é sobrejetor, i.e., qualquer morfismo f : a → I se estende para um morfismo f˜: A → I sobre todo A.

Prova A necessidade é clara. Para mostrar que a condição é suficiente, seja N ⊂ M uma inclusão de
A-módulos e tome φ ∈ HomA (N, I); temos que obter uma extensão φ̃ ∈ HomA (M, I), o que é mais uma
tarefa para o lema de Zorn: considere a coleção das extensões (T, ψ) de φ, isto é,
n o

(T, ψ) T é um A-módulo tal que N ⊂ T ⊂ M e ψ ∈ HomA (T, I) é tal que ψ|N = φ

parcialmente ordenado da maneira usual:

(T1 , ψ1 ) ≤ (T2 , ψ2 ) ⇐⇒ T1 ⊂ T2 e ψ2 |T1 = ψ1


FT
É fácil verificar que toda cadeia é limitada superiormente, de modo que o conjunto acima admite um
)
28

elemento maximal (Ñ , φ̃). Devemos mostrar que Ñ = M . Suponha por absurdo que não e tome
m ∈ M \ Ñ . Considere o ideal a = {a ∈ A | am ∈ Ñ }. Por hipótese, existe f˜: A → I estendendo o mapa
7:

f : a → I dado por f (a) = φ̃(am), a ∈ a. Podemos agora definir N̂ = Ñ + Am e


(1
RA
0

φ̂: N̂ → I
01

a ∈ A, ñ ∈ Ñ
ñ + am 7→ φ̃(ñ) + f˜(a)
,2
29

Observe que φ̂ está bem definido, i.e., independe da particular representação de um elemento em N̂ . Mas
então (N̂ , φ̂) é estritamente maior que (Ñ , φ̃), o que é absurdo.
D
ct
,O

Utilizando o critério anterior, podemos dar agora exemplos de módulos injetivos. Relembrando: um
grupo abeliano G é dito divisı́vel se a multiplicação por n em G é sobrejetora para todo inteiro n > 0.
ET

Por exemplo, Q e Q/Z são grupos abelianos divisı́veis.

Lemma 3.1.7 Qualquer grupo abeliano divisı́vel G é um Z-módulo injetivo.


Prova Todo ideal de Z é principal. Assim, dado d ∈ Z e um morfismo f : (d) → G, basta definir uma
extensão f˜: Z → G, o que é fácil: se y ∈ G é tal que dy = f (d), basta por f˜(1) = y.
100 Métodos Homológicos

Definição 3.1.8 Uma sequência exata curta de A-módulos cinde


seção

0 - M′ - M
i
- M ′′
p
- 0

cinde se a injeção i admite uma seção s, isto é, um morfismo de A-módulos s: M → M ′ tal que s ◦ i = id
ou, equivalentemente, a projeção p admite uma seção t, isto é, morfismo de A-módulos t: M ′′ → M tal
que p ◦ t = id. Neste caso, temos isomorfismos

M - M ′ ⊕ M ′′

M ′ ⊕ M ′′ - M

e
m 7→ (s(m), p(m)) (m , m ) 7→ i(m′ ) + t(m′′ )
′ ′′

Exemplo 3.1.9 Sobre um corpo k, toda sequência exata curta de k-espaços vetoriais cinde. A sequência
de Z-módulos
0 - Z/2 - Z/4 - Z/2 - 0
2

não cinde, pois Z/4 ∼


6 Z/2 ⊕ Z/2.
=

Lemma 3.1.10 Seja


0 - M′ - M - M ′′ - 0

uma sequência exata curta de A-módulos.


1. Se M ′ é injetivo ou se M ′′ é projetivo então a sequência acima cinde.
2. Se a sequência acima cinde, então para qualquer funtor F exato à esquerda ou à direita a
sequência
0 - FM′ - FM - F M ′′ - 0

é exata.

Prova Se M ′ é injetivo, qualquer extensão s: M → M ′ da identidade id: M ′ → M ′ é uma seção de


M ′ ֒→ M , logo a sequência cinde.

Teorema 3.1.11 (Projetivo como Somando de Livres) Um módulo M é projetivo se, e só se,
existe M ′ tal que M ⊕ M ′ é livre.
Prova Existe um módulo livre F e uma sobrejeção F ։ M . Seja N o kernel deste mapa; temos uma
sequência exata
FT
0 - N - F - M - 0

Assim, como M é projetivo, a sequência acima cinde e F ∼


)
= M ⊕ N.
28
7:

Lemma 3.1.12 Seja M um A-módulo. Então


(1
RA
0

M é livre ⇒ M é projetivo ⇒ M é plano


01
,2

Prova Já sabemos que todo módulo livre é projetivo. Agora suponha que M é projetivo e seja M ′ tal
29

que M ⊕ M ′ é livre. Seja i: N ′ ֒→ N uma injeção qualquer de A-módulos. Então


D
ct

(M ⊕ M ′ ) ⊗ N ′ ⊂ -
id ⊗i
(M ⊕ M ′ ) ⊗ N
,O

k k
(M ⊗ N ′ ) ⊂ -
id ⊗i
(M ⊗ N )
ET

⊕ ⊕
(M ′ ⊗ N ′ ) ⊂ -
id ⊗i
(M ′ ⊗ N )

é injetor, logo o mesmo vale para M ⊗ N ′ ֒→ M ⊗ N , provando que M é plano.


101

Teorema 3.1.13 (“Abundância Projetiva e Injetiva”) Seja M um A-módulo.


1. existe um A-módulo projetivo P e uma sobrejeção P ։ M .
2. existe um A-módulo injetivo I e uma injeção M ֒→ I.

Prova O primeiro item é uma consequência direta do fato que todo módulo é quociente de um módulo
livre. Para o segundo item, vamos primeiro mostrar o resultado quando A = Z. Para qualquer grupo
abeliano M , defina M ∨ = HomZ (M, Q/Z), o grupo dual de M . Temos
1. O dual F ∨ de um grupo abeliano livre F é Z-injetivo (i.e., é um grupo divisı́vel)
2. Há uma injeção canônica de M no duplo dual
M ֒→ M ∨∨
m 7→ (φ 7→ φ(m); φ ∈ M ∨ )
Para mostrar que o mapa acima é de fato injetor, observe que dado m 6= 0 existe um mapa não zero
φ: Zm → Q/Z e como Q/Z é injetivo, φ se estende a um mapa não zero φ̃ ∈ M ∨ .
Agora dado um grupo abeliano M , tome F um grupo abeliano livre e uma sobrejeção φ: F ։ M ∨ .
Dualizando, obtemos uma injeção M ∨∨ ֒→ F ∨ . Assim, a composição M ֒→ M ∨∨ ֒→ F ∨ é uma imersão
de M em um Z-módulo injetivo F ∨ .
Vejamos agora o caso geral. Em primeiro lugar, para um grupo abeliano G, note que podemos ver
HomZ (A, G) como A-módulo da seguinte forma: dados f ∈ HomZ (A, G) e a ∈ A, af ∈ HomZ (A, G) é o
mapa x 7→ f (xa), x ∈ A. Denotando por M0 o grupo abeliano subjacente de um A-módulo M , temos
1. Há uma injeção de A-módulos
M ֒→ HomZ (A, M0 )
m 7→ (a 7→ am; a ∈ A)
2. Para todo grupo abeliano G, temos um isomorfismo de A-módulos

HomA (M, HomZ (A, G))



- HomZ (M0 , G)
φ 7→ (m 7→ φ(m)(1); m ∈ M0 )
O mapa inverso é dado por

HomZ (M0 , G)

- HomA (M, HomZ (A, G))
ψ 7→ (m 7→ ψm ; m ∈ M )
onde ψm ∈ HomZ (A, G) é dado por ψm (a) = ψ(am).
Agora dado um A-módulo M , seja M0 ֒→ I0 uma imersão de M0 em Z-módulo injetivo I0 . Temos
FT
portanto uma injeção HomZ (A, M0 ) ֒→ HomZ (A, I0 ). Mas como HomA (−, HomZ (A, I0 )) = HomZ (−, I0 )
é exato, HomZ (A, I0 ) é um A-módulo injetivo, logo a composição M ֒→ HomZ (A, M0 ) ֒→ HomZ (A, I0 ) é
)
28

a imersão pedida.
7:

3.2 Resoluções Projetivas e Injetivas


(1
RA

Definição 3.2.1 Um complexo (M• , d• ) é limitado superiormente se Mi = 0 para i ≫ 0 e é lim-


0

itado inferiormente se Mi = 0 para i ≪ 0. O complexo é limitado se é limtado superiormente e


01

inferiormente.
,2

Teorema 3.2.2 (Extensão Projetiva) No seguinte diagrama, a linha superior é um complexo com
29

Pi projetivos e a linha inferior é exata.


D

d2 - d1 - ǫ -
ct

··· - P2 P1 P0 M - 0
,O
ET

∃f2 ∃f1 ∃f0 f


? d′2 - ? d′1 - ? ǫ′ - ?
··· - Q2 Q1 Q0 N - 0
Então qualquer morfismo f : M → N pode ser estendido a um morfismo f• : P• → Q• entre os dois
complexos. Além disso, esta extensão é única a menos de homotopia.
102 Métodos Homológicos

Prova Vamos construir os fi ’s indutivamente. Como P0 é projetivo e Q0 ։ N é sobrejetor, por definição


de módulo projetivo existe f0 : P0 → Q0 que é um “levantamento” da composição f ◦ ǫ: P0 → M → N .
Agora, utilizando a comutatividade do quadrado mais à esquerda, temos

ǫ′ ◦ (f0 ◦ d1 ) = f ◦ (ǫ ◦ d1 ) = 0 ⇒ im(f0 ◦ d1 ) ⊂ ker ǫ′ = im d′1

Logo temos um mapa f0 ◦ d1 : P1 → im d′1 e uma sobrejeção d′1 : Q1 ։ im d′1 . Como P1 é projetivo, existe
um levantamento f1 : P1 → Q1 de f0 ◦ d1 . Procedendo desta forma, obtemos indutivamente fi : Pi → Qi
como levantamento de fi−1 ◦ di : Pi → im d′i .
Agora temos que mostrar que duas extensões de f são homotópicas. Para isto, basta mostrar que
qualquer extensão f• de f = 0 é homotópica a 0. Vamos construir os mapas de homotopia ki : Pi → Qi+1
indutivamente. Como f = 0, temos ǫ′ ◦ f0 = f ◦ ǫ = 0 ⇒ im f0 ⊂ ker ǫ′ = im d′1 . Assim, temos um mapa
f0 : P0 → im d′1 e como P0 é projetivo e d′1 : Q1 ։ im d′1 é sobrejetor, existe um levantamento k0 : P0 → Q1
de f0 . Assim,
f0 = d′1 ◦ k0
Da mesma forma, temos

d′1 ◦ (f1 − k0 ◦ d1 ) = d′1 ◦ f1 − (d′1 ◦ k0 ) ◦ d1 = f0 ◦ d1 − f0 ◦ d1 = 0 ⇒ im(f1 − k0 ◦ d1 ) ⊂ ker d′1 = im d′2

Assim, temos um mapa (f1 − k0 ◦ d1 ): P1 → im d′2 , que pode ser levantado para k1 : P1 → Q2 . Portanto

f1 = k0 ◦ d1 + d′2 ◦ k1

Procedendo indutivamente desta forma, definimos ki : Pi → Qi+1 como o levantamento de (fi − ki−1 ◦
di ): Pi → im d′i+1 de modo que
fi = ki−1 ◦ di + d′i+1 ◦ ki

Teorema 3.2.3 (Resoluções Projetivas) Dado um complexo (M• , d• ) limitado inferiormente, existe
um complexo (P• , d• ) limitado inferiormente quase-isomorfo a (M• , d• ) com Pi projetivo.
Prova Sem perda de generalidade vamos supor que Mi = 0 para i < 0. Seja P0 ։ M0 uma sobrejeção
com P0 projetivo.
FT
Definição 3.2.4 A injetivo resolução (I • , d• ) of M é a complex of injetivo módulos together com an
injection ǫ: M ֒→ I 0 de modo que the augmented complex
)
28

d0 d1
0 - M - I0
ǫ
- I1 - I2 - ···
7:
(1
RA

é exact. The resolução é said to be finite se I p = 0 para todo sufficiently large p.


0

We define the injetivo dimensão of a módulo as the length of the shortest injetivo resolução, in the
01

same way as we did para projetivo dimensão. por exemplo,


,2

0 → Z → Q → Q/Z → 0
29
D

é an injetivo resolução of the Z-módulo Z, which portanto has injetivo dimensão 1; it cannot be 0 como
ct

Z é not injetivo.
,O

We now mostrar que todo módulo has an injetivo resolução. The proof of the corresponding result
ET

para projetivo resoluçãos was based on the fact that todo módulo é the quociente of a free módulo.
Similarly in order to show the existence of an injetivo resolução, todo that we need é to mostrar que
qualquer módulo can be embedded into an injetivo módulo, para once temos an embedding M ֒→ I0 of
M into an injetivo módulo I0 , we can embed the cokernel of this mapa into another injetivo módulo I1
to obtain an exact sequence 0 → M → I0 → I1 ; iterating this process, obtemos an injetivo resolução of
M.
103

Lema 3.2.5 (Resoluções) Dado um complexo (M• , d• ) de A-módulos limitado inferiormente, existe aditivo
funtor derivad
um outro complexo (P• , e• ) quase-isomorfo a (M• , d• ) e tais que os módulos Pi são todos projetivos.
Definição 3.2.6 Um funtor F é aditivo se para qualquer f, g ∈ Hom(M, N ), F (f + g) = F (f ) + F (g).
Definição 3.2.7 Seja F um funtor aditivo exato à esquerda. O funtor derivado à esquerda LF de
F.

3.3 Tor e Ext


Exemplo 3.3.1 (Tor sobre Z) Seja M e N be finitamente gerado Z-módulos (a.k.a finitamente gerado
abelian groups). We can write a surjection ǫ: F0 ։ M com F0 free of finite rank. Como Z é a PID, ker ǫ
é also free of finite rank, so obtemos a projetivo resolução

0 - F1 - F0
d1 ǫ
- M - 0

of M . This shows that TorZp (M, N ) = 0 para p > 1. Como TorZ0 (M, N ) = M ⊗ N , the só thing left to be
computed é TorZ1 (M, N ) = ker d1 ⊗ 1. But como M e N são direct sums of free Z-módulos, which são
flat, e finite cyclic groups, everything boils down to the computation of TorZ1 (Z/(m), Z/(n)).
A free resolução of Z/(m) é
0 - Z - Z - Z/(m) - 0
m ǫ

Deleting the term Z/(m) from this sequence e tensoring it com Z/(n), concluı́mos que TorZ1 (Z/(m), Z/(n))
é the kernel of the multiplication por m in Z/(n), i.e., TorZ1 (Z/(m), Z/(n)) = Z/(d) onde d = gcd(m, n).

Teorema 3.3.2
depth M = min{i | Exti (k, M ) 6= 0}

4 Planaridade

Lemma 4.1 Seja


0 - M′ - M - M ′′ - 0
uma sequência exata de A-módulos com M ′′ plano. Então, para qualquer A-módulo N , a sequência
0 - M′ ⊗ N - M ⊗N - M ′′ ⊗ N - 0

é exata.
Prova Temos uma sequência exata longa
- M′ ⊗ N - M ⊗N - M ′′ ⊗ N - 0
FT
0 = Tor1 (M ′′ , N )
onde o termo da esquerda se anula pois M ′′ é plano.
)
28

Teorema 4.2 Seja (A, m, k) um anel noetheriano local e M um A-módulo finitamente gerado. Então
7:
(1
RA

M é livre ⇐⇒ M é projetivo ⇐⇒ M é plano


0

Prova Basta mostrar que, nas condições do enunciado, todo módulo plano é livre. Seja M um A-módulo
01

plano finitamente gerado e seja {ω1 , . . . , ωn } um conjunto minimal de geradores de M ; sabemos que as
,2

imagens dos ωi em M ⊗ k = M/mM formam uma base deste k-espaço vetorial. Defina
29

φ: An ։ M
D
ct

(a1 , . . . , an ) 7→ a1 ω1 + · · · + an ωn
,O

e seja N = ker φ, de modo que temos uma sequência exata curta


ET

0 - N - An φ
- M - M - 0

Queremos mostrar que N = 0. Como M é plano sobre A, pelo lema anterior temos que esta sequência
permanece exata quando a tensorizamos por k. Mas como φ ⊗ k é um isomorfismo por construção, temos
que N ⊗A k = 0. Temos ainda que A é noetheriano e M é finitamente gerado, logo N é finitamente
gerado também. Portanto N = 0 ⇐⇒ N ⊗A k = 0 por Nakayama (lema III.2.3.2), como desejado.
104 Métodos Homológicos

O próximo resultado mostra que, para módulos finitamente gerados sobre anéis noetherianos,

projetivo = localmente livre

Corolário 4.3 Seja A um anel noetheriano e seja M um A-módulo finitamente gerado. Então

M é projetivo ⇐⇒ Mm é livre para todo ideal maximal m de A

Prova Pelo teorema anterior, devemos mostrar que M é projetivo se, e só se, cada Mm é projetivo. Se
M é projetivo, então é somando de um módulo livre, propriedade que é preservada sob localização, o
que mostra que cada Mm é projetivo.
Reciprocamente, suponha que cada Mm é projetivo. Seja N • uma sequência exata de A-módulos.
Pelo princı́pio local-global (teorema III.1.8), temos que HomA (M, N • ) será uma sequência exata de A-
módulos se, e só se, HomA (M, N )m for exata para todo m maximal. Mas como M é finitamente gerado
e Am é plano sobre A, temos um isomorfismo (teorema III.4.1.8)

HomA (M, N )m = HomAm (Mm , Nm )

para qualquer A-módulo N . Como localização é um funtor exato, temos que cada sequência de Am -
• •
módulos Nm é exata. Assim, como Mm é projetivo, temos que HomAm (Mm , Nm ) é exata e portanto M
é projetivo, como querı́amos.

Exemplo 4.4 Todo ideal a em um √ domı́nio de Dedekind


√ A é um módulo projetivo. Por exemplo, já
sabemos que o ideal a = (3, 1 + 2i 5) de A = Z[i 5] não é principal (isto é, não é um A-módulo livre),
mas que ele é localmente principal.

Teorema 4.5 Seja M um A-módulo. Então as seguintes condições são equivalentes:


1. M é A-plano;
2. TorA
n (M, N ) = 0 para todo n ≥ 1 e todo A-módulo N ;

3. TorA
1 (M, N ) = 0 para todo A-módulo N ;
4. o mapa natural a ⊗A M ։ aM é um isomorfismo para todo ideal a de A.

Prova 1 ⇒ 2 ⇒ 3 é claro. Vamos mostrar que 3 ⇒ 4. Considere a sequência exata curta


FT
0 - a - A - A/a - 0
)
28

Temos uma sequência exata longa


7:

0 = TorA - M ⊗A a - M ⊗A A - M ⊗A (A/a) - 0
1 (M, A/a)
(1
RA
0

que é isomorfa a
01

0 - M ⊗A a - M - M/aM - 0
,2

e portanto a imagem de M ⊗A a → M é igual a aM , o que mostra (4).


29

Agora vamos provar que 4 ⇒ 1. Seja N ′ ⊂ N uma inclusão de A-módulos; temos que mostrar que

D

N ⊗ M → N ⊗ M é injetor.
ct

Primeiro, vejamos que sem perda de generalidade podemos assumir que N/N ′ é finitamente gerado.
,O

Temos
N ⊗ M = (−→lim N ′′ ) ⊗ M,
ET

N ′′

onde N ′′ percorre todos os módulos tais que N ′ ⊂ N ′′ ⊂ M e N ′′ /N ′ é finitamente gerado.


P P ′ Assim, se
′ ′ ′′
i n i ⊗ m i ∈ N ⊗ M tem imagem 0 em N ⊗ M , então existe N como acima tal que i ni ⊗ mi tem
imagem 0 em N ′′ ⊗ M , ou seja, se existe um contra-exemplo N ′ ⊂ N para a planaridade de M , existe um
com N/N ′ finitamente gerado. Podemos até mesmo supor que N/N ′ é gerado por um único elemento,
105

pois se a injetividade é preservada neste caso especial, então para o caso geral N = N ′ + Aω1 + · · · + Aωn
podemos escrever N ′ ⊗ M → N ⊗ M como composição de injeções
(N ′ + Aω1 + · · · + Aωi ) ⊗ M ֒→ (N ′ + Aω1 + · · · + Aωi+1 ) ⊗ M
Suponha portanto que N/N ′ = Aω de modo que N/N ′ = A/a para a = ann(ω). Da sequência exata
curta
0 - a - A - A/a - 0,
obtemos uma sequência exata longa
0 = Tor1 (M, A) - Tor1 (M, A/a) - M ⊗a - M ⊗A=M

onde o termo da esquera é zero pois A é plano sobre si mesmo. Como M ⊗a ֒→ M é injetivo por hipótese,
da sequência acima concluı́mos que Tor1 (M, A/a) = 0. Finalmente, da sequência exata
0 - N′ - N - A/a - 0

obtemos a sequência exata


0 = Tor1 (M, A/a) - M ⊗ N′ - M ⊗N

que mostra que M ⊗ N ′ ֒→ M ⊗ N é de fato injetor.

Corolário 4.6 Seja


0 - M′ - M - M ′′ - 0
uma sequência exata de A-módulos. Se M ′ e M ′′ são A-planos, o mesmo vale para M .
Prova Para todo A-módulo N , temos uma sequência exata
0 = Tor1 (M ′ , N ) - Tor1 (M, N ) - Tor1 (M ′′ , N ) = 0

onde os extremos, e portanto o meio, se anulam pois M ′ e M ′′ são A-planos. Assim, M é A-plano pelo
teorema anterior.

Teorema 4.7 (Critério Local de Planaridade) Seja (A, m, k) uma anel local noetheriano. Seja
B uma A-álgebra noetheriana local e M um B-módulo finito. Para n ≥ 1, escreva An = A/mn e
Mn = M ⊗A An = M/mn M . As seguintes condições são equivalentes:
1. M é plano sobre A;
2. TorA
1 (k, M ) = 0;
FT
3. Mn é plano sobre An para todo n ≥ 1.

Prova (1 ⇒ 2) é claro.
)
28

(2 ⇒ 3) Provemos inicialmente que TorA 1 (N, M ) = 0 para todo A-módulo N de comprimento finito por
indução em lenA N : se lenA N = 0 então N = 0 e se lenA N = 1 então N ∼
7:

= A/m = k e em ambos os
A
(1
RA

casos Tor1 (k, M ) = 0. Para lenA N > 1, podemos escrever uma sequência exata curta
- N′ - N - N/N ′ - 0
0

0
01

onde N ′ é um submódulo simples de N , de modo que lenA N ′ e lenA N/N ′ são estritamente menores do
,2

que lenA N . O resultado segue da hipótese de indução e da sequência exata


29

0 = TorA ′ - TorA (N, M ) - TorA (N/N ′ , M ) = 0


1 (N , M ) 1 1
D
ct

Agora, dado um an ideal de An , precisamos mostrar que o mapa natural an ⊗An Mn → Mn é injetor.
,O

Temos uma sequência exata de A-módulos


TorA - an ⊗A M - An ⊗A M - (An /an ) ⊗A M - 0
ET

1 (An /an , M )

Mas como An /an é anulado por mn , seu comprimento sobre A é finito e portanto TorA
1 (An /an , M ) = 0.
Assim,
an ⊗An Mn = an ⊗A An ⊗A M = an ⊗A M ֒→ An ⊗A M = Mn
é injetor, como querı́amos.
106 Métodos Homológicos

(3 ⇒ 1) Dado um ideal a ⊂ A, devemos mostrar que o mapa natural a ⊗A M → M é injetor. Seja n o


ideal maximal de B. Como a ⊗A M é um módulo finitamente gerado sobre o anel noetheriano local B e
mB ⊂ n, temos
\ \
nn (a ⊗A M ) = 0 ⇒ mn (a ⊗A M ) = 0
n≥1 n≥1

Assim, é suficiente mostrar que ker(a ⊗A M → M ) ⊂ mn (a ⊗A M ) para todo n. É até mesmo suficiente
mostrar que
 a  
ker(a ⊗A M → M ) ⊂ ker a ⊗A M → ⊗ A M para todo n (∗)
a ∩ mn

De fato, da sequência exata

- a ∩ mn - a - a - 0
0
a ∩ mn

temos uma sequência exata


 a 
(a ∩ mn ) ⊗A M - a ⊗A M - ⊗A M - 0
a ∩ mn

Além disso, pelo teorema de Artin-Rees (teorema III.5.1.1), existe uminteiro positivo
 r tal que a ∩ mn ⊂
n−r a
m a para todo n ≥ r. Portanto  se x ∈ ker a ⊗A M → a∩mn ⊗A M para todo n ≥ 0 então
x ∈ im (mn−r a) ⊗A M → a ⊗A M = mn−r (a ⊗A M ) para todo n ≥ r.
Tensorizando o diagrama comutativo

- a
a
∩ a ∩ mn

?
? A
A - = An
mn

com M , obtemos o diagrama comutativo


 a   a 
-
FT
a ⊗A M n
⊗A M == ⊗ An M n
a∩m a ∩ mn
∩ ∩
)
28

? ? ?
7:

M - An ⊗A M ============ Mn
(1
RA

Mas como Mn é plano sobre An por hipótese e a/(a ∩ mn ) ֒→ A/mn é injetor, a flecha vertical à direita
0
01

no último diagrama também é injetora. Assim, (∗) se verifica, o que completa a prova.
,2
29

Corolário 4.8 Sejam A → B → C morfismos locais entre anéis locais noetherianos. Seja M um C-
módulo finitamente gerado. Suponha que B seja plano sobre A. Seja k o corpo residual de A. São
D
ct

equivalentes:
,O

1. M é plano sobre B
ET

2. M é plano sobre A e M ⊗A k é plano sobre B ⊗A k.

5 Teorema de Serre

5.1 Dimensão Global e Resoluções Minimais Livres


107

Definição 5.1.1 Seja M um A-módulo. Dizemos que M tem dimensão projetiva finita se M admite dimensão pro
dimensão inje
uma resolução projetiva finita: dimensão glob

0 - Pn - Pn−1 - Pn−2 - ··· - P0 - M - 0

Neste caso, definimos proj. dim M como o menor comprimento n dentre todas estas resoluções. Se todas
as resoluções projetivas de M são infinitas, escrevemos proj. dim M = ∞. Da mesma forma, definimos a
dimensão injetiva inj.dim M de um módulo M .

Definição 5.1.2 Seja A um anel. A dimensão global de A é o supremo de proj. dim M quando M
percorre todos os A-módulos finitamente gerados.

Observação 5.1.3 A dimensão global pode também ser definida como o supremo de todas as dimensões
projetivas de todos os módulos, finitamente gerados ou não.

Exemplo 5.1.4 Como todo módulo sobre um corpo é livre, temos que a dimensão global de qualquer
corpo é 0. Por outro lado, temos que a dimensão global de Z ou de qualquer PID A é 1: para A-módulos
da forma A/(a) 6= 0, temos uma resolução projetiva de tamanho 1

0 - P1 = A - P0 = A
a
- A/(a) - 0

Como A/(a) não é projetivo, temos assim que proj. dim A/(a) = 1 e portanto a dimensão global de A
também é 1, já que qualquer A-módulo finitamente gerado é soma direta de cópias de A e de módulos
da forma A/(a).

Temos a seguinte caracterização das dimensões projetivas e injetivas de um módulo.

Lemma 5.1.5 Seja A um anel e M um A-módulo.


1. proj. dim M ≤ n ⇐⇒ Exti (M, N ) = 0 para todo A-módulo N e todo i ≥ n + 1;
2. inj.dim M ≤ n ⇐⇒ Exti (N, M ) = 0 para todo A-módulo N e todo i ≥ n + 1.

Prova Se proj. dim M ≤ n, utilizando uma resolução projetiva de M de comprimento menor ou igual a
n para calcular Exti (M, N ), obtemos imediatamente Exti (M, N ) = 0 para i ≥ n + 1. Reciprocamente,
suponha que Exti (M, N ) = 0 para todo A-módulo N e todo i ≥ n + 1. Mostremos por indução em n
que proj. dim M ≤ n. Para n = 0, dada uma sequência exata de A-módulos

0 - N′ - N - N ′′ - 0
FT
obtemos uma sequência exata
)
28

0 - HomA (M, N ′ ) - HomA (M, N ) - HomA (M, N ′′ ) - Ext1A (M, N ′ ) = 0


7:
(1
RA

e portanto HomA (M, −) é exato, ou seja, M é projetivo e portanto proj. dim M = 0. Para n > 0, faremos
0

um “dimension shift”: considere uma sequência exata


01

0 - Q - P - M - 0 (∗)
,2
29

com P projetivo. Para qualquer A-módulo N , temos uma sequência exata


D
ct

Exti (M, N ) - Exti (P, N ) - Exti (Q, N ) - Exti+1 (M, N )


,O

Para i ≥ n, temos que Exti+1 (M, N ) = 0 por hipótese e Exti (P, N ) = 0 também, pois P é projetivo
ET

e i ≥ 1. Assim, Exti (Q, N ) = 0, logo por indução temos que Q admite uma resolução projetiva de
comprimento menor ou igual a n − 1. Desta forma, podemos estender (∗) para uma resolução projetiva
de M de comprimento no máximo n, como querı́amos.
A demonstração do segundo item é análoga e é deixada como (mais um!) exercı́cio para o leitor.
108 Métodos Homológicos

Agora nos restringiremos ao caso local. Nesta situação, um tipo especial de resolução projetiva nos
será especialmente importante no estudo das dimensões projetivas e globais.
Definição 5.1.6 Seja (A, m, k) um anel noetheriano local. Uma resolução livre minimal de M é uma
resolução projetiva
··· - Fi di
- Fi−1 - ···
di−1
- F0
d1
- M
ǫ
- 0
satisfazendo as seguintes propriedades:
1. Todos os Fi são A-módulos livres de posto finito.
2. Para todo i, os mapas di ⊗ id: Fi ⊗ k → Fi−1 ⊗ k são zero. Em outras palavras, im di ⊂ mFi−1 .
3. O mapa ǫ ⊗ id: F0 ⊗ k → M ⊗ k é um isomorfismo.
Vamos mostrar que todo A-módulo finitamente gerado M possui uma resolução minimal. Tome
uma base minimal ω1 , . . . , ωn de M e defina a sobrejeção
def
ǫ: F0 = An ։ M
(a1 , . . . , an ) 7→ a1 ω1 + · · · + an ωn

Por construção, temos que ǫ ⊗ id: F0 ⊗ k



- M ⊗ k é um isomorfismo. Agora seja L0 = ker ǫ,
que é um A-módulo finitamente gerado pois A é noetheriano. Repetindo o procedimento com L0 no
lugar de M , obtemos um A-módulo livre F1 de posto finito e um mapa sobrejetor d′1 : F1 ։ L0 tal que
d′1 ⊗ id: F1 ⊗ k - L0 ⊗ k é um isomorfismo. Assim, definindo d1 : F1 → F0 como a composição de d′1 e

da inclusão i0 : L0 ֒→ F0 , temos que d1 ⊗ id: F1 ⊗ k → F0 ⊗ k é o mapa zero: da exatidão de


L0 ⊗ k - F0 ⊗ k
i0 ⊗id
- M ⊗k
ǫ⊗id
- 0

temos que i0 ⊗ id = 0, logo d1 ⊗ id = (d′1 ⊗ id) ◦ (i0 ⊗ id) = 0. Procedendo indutivamente desta forma,
obtemos a resolução minimal de M desejada.
Exemplo 5.1.7 (Resoluções Minimais Livres e Complexo de Koszul) Seja (A, m, k) um anel
noetheriano local e a1 , a2 , . . . , an ∈ m uma A-sequência regular, de modo que pelo teorema 2.7 o complexo
de Koszul K• (a1 , . . . , an , A) fornece uma resolução livre de M = A/(a1 , . . . , an ):
A
0 - Kn (a1 , . . . , an , A) - · · · - K0 (a1 , . . . , an , A) ǫ- M = - 0
dn d1
(a1 , . . . , an )
Esta resolução é minimal, pois cada termo Kp (a1 , . . . , an , A) é livre de posto finito e, além disso, im dp ⊂
mKp−1 (a1 , . . . , an , A) pois ai ∈ m. Finalmente, temos isomorfismos
K0 (a1 , . . . , an , A) ⊗ k = A ⊗ k = k e M ⊗k =k
e com estas identificações ǫ ⊗ id: k → k é o mapa identidade.
FT
O próximo lema permite-nos restringir a resoluções livres minimais no lugar de resoluções projetivas
arbitrárias.
)
28

Lemma 5.1.8 Seja (A, m, k) um anel noetheriano local e seja M um A-módulo finitamente gerado. Se
7:

n = proj. dim M < ∞, M admite uma resolução livre minimal


(1
RA

0 - Fn - Fn−1
dn
- ···
dn−1
- F0
d1
-M
ǫ
- 0
0

de comprimento n.
01

Prova Indução em proj. dim M . Se proj. dim M = 0, M é projetivo, logo livre de posto finito e o
,2

resultado é claro. Agora se proj. dim M > 0, escreva a sequência exata


29

0 - L0 - F0 - M
ǫ
- 0 (∗)
D
ct

como na construção da resolução minimal de M descrita acima. Para todo A-módulo N , temos uma
,O

sequência exata
Exti (M, N ) - Exti (F0 , N ) - Exti (L0 , N ) - Exti+1 (M, N )
ET

Se i ≥ proj. dim M , temos que Exti+1 (M, N ) = 0. Além disso, como F0 é projetivo e i ≥ 1, Exti (F0 , N ) =
0, o que mostra que Exti (L0 , N ) = 0 para todo i ≥ n. Portanto proj. dim L0 ≤ n − 1 e por indução
L0 terá uma resolução livre minimal de tamanho no máximo n − 1, o que permite estender (∗) a uma
resolução livre minimal de M de tamanho menor ou igual a n (na verdade igual pois proj. dim M = n).
109

O teorema a seguir traduz o fato que, para calcular a dimensão global de um anel noetheriano local
(A, m, k), basta calcular a dimensão projetiva de k, o “menos livre” dentre todos os A-módulos.
Teorema 5.1.9 Seja (A, m, k) um anel noetheriano local e seja M um A-módulo. Suponha que
0 - Fn - Fn−1
dn dn−1
- ··· - F0
d1
-M
ǫ
- 0
é uma resolução minimal livre finita de M com Fi 6= 0 para todo i. Então
1. proj. dim M = n = max{i | Tori (M, k) 6= 0}
2. proj. dim M ≤ proj. dim k.
Em particular, temos que a dimensão global de A é igual a proj. dim k.
Prova 1. Tensorizando a resolução acima por k, obtemos Tori (M, k) = Fi ⊗ k 6= 0 para todo i.
Portanto n = max{i | Tori (M, k) 6= 0}. Por outro lado, temos
max{i | Tori (M, k) 6= 0} ≤ proj. dim M ≤ n
onde a primeira desigualdade segue do cálculo de Tori (M, k) utilizando-se uma resolução de tamanho
proj. dim M . Assim, devemos ter igualdade em todos os lugares e o resultado segue.
2. Calculando Tori (M, k) com uma resolução projetiva de tamanho proj. dim k, vemos que
max{i | Tori (M, k) 6= 0} ≤ proj. dim k
o que, combinado com o primeiro resultado, fornece a desigualdade desejada.

Podemos relacionar agora a dimensão projetiva e a profundidade de um módulo no importante


Teorema 5.1.10 (Auslander, Buchsbaum) Seja (A, m, k) um anel noetheriano local e M um A-
módulo de dimensão projetiva finita. Então
proj. dim M = depth A − depth M
Prova A prova é por indução em proj. dim M . Se proj. dim M = 0, M é livre e portanto depth M =
depth A. Agora faremos um “dimension shift”. Sejam a1 , . . . , an geradores de m; pelo teorema 2.9
podemos calcular profundidades utilizando o complexo de Koszul K• (a1 , . . . , an , M ). Por legibilidade,
denotaremos Hi (K• (a1 , . . . , an , M )) simplesmente por Hi (M ).
Suponha inicialmente que proj. dim M = 1. Temos que mostrar que depth M = depth A − 1, isto é,
que
n − max{i | Hi (M ) 6= 0} = depth A − 1 ⇐⇒ max{i | Hi (M ) 6= 0} = n − depth A + 1
Para isto, seja
0 - F1 - F0
d1
- M
ǫ
- 0
uma resolução livre minimal de M . Temos uma sequência exata longa
· · · - Hi (F1 ) - Hi (F0 ) - Hi (M )
FT
- Hi−1 (F1 ) - Hi−1 (F0 ) - · · ·
)
Temos ainda
28

depth F0 = depth F1 = depth A = n − max{i | Hi (F0 ) 6= 0} = n − max{i | Hi (F1 ) 6= 0}


7:

Assim, temos que Hi (M ) = 0 para i ≥ n − depth A + 2, logo basta mostrar que Hi (M ) 6= 0 para
(1
RA

i = depth A + 1. Para isto, basta mostrar que Hi (F1 ) → Hi (F0 ) é zero, pois isto implicará Hi (M ) ∼ =
0

Hi−1 (F1 ) 6= 0 para i = depth A + 1. Porém Hi (F1 ) → Hi (F0 ) é induzido por d1 ; como a resolução acima
01

é minimal, im d1 ⊂ mF0 . Mas já sabemos que m anula Hi (F1 ) e o resultado segue.
,2

Para p = proj. dim M > 1, temos uma resolução livre minimal


- Fp dp
- ··· - F1 d1
- F0 - M - 0
29

0
D

Sendo L = im d1 , temos uma sequência exata curta


ct

0 - L - F0 - M - 0
,O

com proj. dim L = proj. dim M −1. Por hipótese de indução, depth L = depth A−proj. dim L < depth F0 .
ET

Da sequência exata longa


··· - Hi (L) - Hi (F0 ) - Hi (M ) - Hi−1 (L) - · · ·
temos como antes depth M = depth L − 1. Assim,
proj. dim M = proj. dim L + 1 = depth A − depth L + 1 = depth A − depth M
110 Métodos Homológicos

5.2 Anéis Locais Regulares

Teorema 5.2.1 (Serre) Um anel local noetheriano (A, m, k) é regular se, e só se, sua dimensão global
é finita.
Prova Se A é regular de dimensão de Krull n, seu ideal maximal m pode ser gerado por n elementos
a1 , . . . , an , que formam uma sequência A-regular. Assim, o complexo de Koszul K• (a1 , . . . , an , M ) é exato
e fornece uma resolução livre minimal de k, de modo que A tem dimensão global proj. dim k = n < ∞.
Reciprocamente, suponha que a dimensão global de A é finita, ou seja, proj. dim k < ∞. Pela
fórmula de Auslander-Buchsbaum, proj. dim k = depth A − depth k = depth A. Como

dimk m/m2 ≥ dim A ≥ depth A = proj. dim k

se mostrarmos que proj. dim k ≥ dimk m/m2 teremos igualdade em todos os pontos, de modo que
dimk m/m2 = dim A, i.e, A é regular.
Seja n = dimk m/m2 e sejam a1 , . . . , an geradores de m. Seja (F• , d• ) uma resolução livre mini-
mal finita de k (que existe pois proj. dim k < ∞). Considere ainda o complexo obtido do de Koszul
K• (a1 , . . . , an , A) apensando o mapa K0 (a1 , . . . , an , A) → k → 0. Como cada Ki (a1 , . . . , an , A) é livre,
temos um mapa f : K• (a1 , . . . , an , A) → F• que estende a identidade em k. Vamos mostrar que f é
injetivo e que ela “cinde” F• , ou seja, que f identifica K• (a1 , . . . , an , A) com um somando direto de F• .
Isto provará que proj. dim k ≥ n. Precisamos de um
Lemma 5.2.2 Seja f : K → F um mapa entre A-módulos livres de posto finito. Se f ⊗ id: K ⊗ k → F ⊗ k
é injetivo, então existe g: F → K tal que g ◦ f = id.
Prova Escolha α1 , . . . , αn ∈ K e β1 , . . . , βm ∈ F tais que α1 ⊗ 1, . . . , αn ⊗ 1 e β1 ⊗ 1, . . . , βn ⊗ 1 são
bases dos k-espaços vetoriais K ⊗ k e F ⊗ k, respectivamente, e tais que f ⊗ id(αi ⊗ 1) = βi ⊗ 1 para
1 ≤ i ≤ n, o que é possı́vel pois f ⊗ id é injetivo. Por Nakayama lema III.2.3.2, α1 , . . . , αn e β1 , . . . , βm
são bases dos módulos livres K e F . Podemos portanto definir

αi para 1 ≤ i ≤ n
g(βi ) =
0 para n < i ≤ m

Assim, basta mostrar que os mapas fp ⊗ k: Kp (a1 , . . . , an , A) ⊗ k → Fp ⊗ k são injetivos. Para p = 0,


isto segue do diagrama comutativo com linhas que são isomorfimos:

≈ -
F0 ⊗ k k
FT
w
w
6 w
w
w
w
)
f0 ⊗ k w
28

w
w
w
7:

≈ -
K0 (a1 , . . . , an , A) ⊗ k k
(1
RA

Para p > 0, temos um diagrama comutativo


0
01

Fp - mFp−1
Fp ⊗ k ∼
=
,2

mFp m2 Fp−1
6 6
29

fp ⊗ k f p−1
D
ct

Kp (a1 , . . . , an , A) dp - mKp−1 (a1 , . . . , an , A)


,O

Kp (a1 , . . . , an , A) ⊗ k ∼
= ⊂
mKp (a1 , . . . , an , A) m2 Kp−1 (a1 , . . . , an , A)
ET

onde dp e f p−1 são induzidos por dp e fp−1 , respectivamente. Afirmamos que dp e f p−1 são injetivos, o
que é suficiente para mostrar que fp ⊗ k é injetivo.
Por indução, fp−1 ⊗ k é injetor, logo pelo lema existe uma seção gp−1 : Fp−1 → Kp−1 (a1 , . . . , an , A)
para f p − 1. Assim, existe um mapa g p−1 : mFp−1 /m2 Fp−1 → mKp−1 (a1 , . . . , an , A)/m2 Kp−1 (a1 , . . . , an , A)
tal que g p−1 ◦ f p−1 = 1, provando que f p−1 é injetor.
111

P
Para mostrar que dp é injetor, suponha que i1 ...ip ai1 ...ip ei1 ...ip ⊗ 1 + mKp (a1 , . . . , an , A) pertença irredutı́vel
associados
a ker dp . Então
X X
(−1)k−1 aik ai1 ...ip ei1 ...îk ...ip ⊗ 1 ∈ m2 Kp−1 (a1 , . . . , an , A)
i1 ...ip 1≤k≤p

Como a1 , . . . , an é uma base minimal de m, temos que a1 + m2 , . . . , an + m2 é uma base do k-espaço


vetorial m/m2 . Portanto a identidade anterior implica que ai1 ...ip ∈ m para todo 1 ≤ i1 < . . . < ip ≤ n.
Logo dp é injetor.

Um importante corolário é o seguinte:


Corolário 5.2.3 Se (A, m, k) é um anel local noetheriano regular, então Ap é também regular para todo
p ∈ Spec A.
Prova Como A é regular, proj. dim A/p < ∞ e assim A/p admite uma resolução livre minimal finita.
Localizando esta resolução em p, obtemos uma resolução livre finita do corpo residual de Ap , mostrando
que a dimensão global de Ap é finita, logo Ap é regular.

6 Teorema de Auslander-Buchsbaum
Seja A um domı́nio. Recordando: dizemos que um elemento p ∈ A \ (A× ∪ {0}) é irredutı́vel se
p = ab ⇒ a ∈ A× ou b ∈ A× . Dizemos que p é primo se o ideal (p) é primo. Note que todo elemento
primo é irredutı́vel, mas em geral não vale a recı́proca. Dois elementos a, b ∈ A são associados se geram
o mesmo ideal, ou seja, se existe u ∈ A× tal que a = ub.
Temos que A é um UFD se cada elemento não nulo de A se escreve como produto de elementos
primos. A “unicidade” é automática: se up1 . . . pr = vq1 . . . qs com u, v ∈ A× e pi , qj primos, então como
q1 . . . qs ∈ (p1 ), temos q1 ∈ (p1 ), digamos, e como p1 e q1 são irredutı́veis existe w ∈ A× tal que q1 = wp1 .
Portanto up2 . . . pr = vwq2 . . . qs e por indução no número de fatores temos que r = s e que, após uma
permutação, cada pi é associado a qi .
Se A é noetheriano, por “PIF noetheriano” temos que todo elemento em A \ (A× ∪ {0}) é produto
de irredutı́veis. Assim, um domı́nio noetheriano é um UFD se, e só se, todo irredutı́vel é primo. Neste
caso, sendo K = Frac A, temos uma sequência exata
M
0 - A× - K × - Z - 0
div

ht p=1

onde a soma percorre todos os primos de altura 1.


Lemma 6.1 Um domı́nio noetheriano A é um UFD se, e somente se, todo ideal primo ideal p de altura 1
FT
é principal.
Prova Suponha primeiro que A é um UFD e seja p ∈ Spec A um ideal primo de altura 1. Temos que
)
28

p contém algum elemento irredutı́vel p (basta tomar qualquer elemento não nulo de p e escrevê-lo como
produto de irredutı́veis, um dos quais deve pertencer a p). Porém como (p) é primo e ht p = 1, devemos
7:

ter p = (p), mostrando que este ideal é principal.


(1
RA

Reciprocamente, suponha que todo ideal primo de altura 1 em A é principal. Devemos mostrar que
todo irredutı́vel q é primo. Um ideal primo minimal p contendo q possui altura 1 pelo teorema do ideal
0
01

principal de Krull (corolário V.4.5), logo por hipótese p = (p). Mas como (p) ⊃ (q) ⇐⇒ p | q e q é
irredutı́vel, temos que p e q são associados, logo q é primo.
,2
29

Lemma 6.2 Seja A um domı́nio noetheriano e seja p ∈ A um elemento primo. Se Ap é um UFD então
D

o mesmo vale para A.


ct
,O

Prova Seja q ∈ Spec A de altura de 1. Se p ∈ q então q = (p) é principal, caso contrário, qAp ∈ Spec Ap
tem altura 1 e pelo lema anterior é principal, digamos qAp = (q). Como p é unidade em Ap , multiplicando
ET

por uma potência de p, podemos supor que q ∈ A. Tome q ∈ A tal que q gera qAp e qA é maximal em
A com essa propriedade (A é noetheriano). Temos q ∈ q = qAp ∩ A, logo (q) ⊂ q. Reciprocamente, dado
a ∈ q ⊂ qAp , temos que a = qb/pn ⇐⇒ apn = qb para algum b ∈ A e n ∈ N. Como p é primo temos
p | q ou p | b. O primeiro caso não pode ocorrer pela maximalidade de q. Então p | b; cancelando o fator
p e repetindo o argumento eventualmente chegaremos a a = qb′ para algum b′ ∈ A, ou seja, a ∈ (q), o
que prova a inclusão oposta q ⊂ (q).
112 Métodos Homológicos

Queremos mostrar que ideais primos de altura 1 em são principais, ou seja, livres de posto 1 como
A-módulos. Isto é feito em dois tempos:
Lemma 6.3 Seja A um anel e M um A-módulo projetivo que admite uma resolução livre finita. Então
M é “estavelmente livre”, ou seja, existe um A-módulo livre F tal que M ⊕ F é livre.
Prova Seja
0 - Fn - · · · → F1
dn
- F0
d1
- M
ǫ
- 0

uma resolução livre finita de M . A prova é por indução no comprimento desta resolução. Se n = 0,
M∼= F0 é livre. Se n > 0, seja L = im d1 de modo que temos uma sequência exata

0 - L - F0 - M - 0

Como M é projetivo, esta sequência cinde e portanto M ⊕ L ∼ = F0 . Assim L é projetivo e admite uma
resolução finita livre de comprimento n − 1, logo por indução L é estavelmente livre, i.e., existe F ′ livre
def
tal que F = L ⊕ F ′ é livre e portanto M ⊕ F ∼ = F0 ⊕ F ′ também é livre.

Para ideais, estavelmente livre é o mesmo que livre.


Lemma 6.4 Seja A um domı́nio e seja a um ideal tal que a ⊕ An ∼
= An+1 . Então a é principal.
Prova Seja ω0 , . . . , ωn uma base de An+1 e identifiquemos a ⊕ An e A ⊕ An como submódulos de

A ⊕ An = An+1 = Aω0 ⊕ Aω1 ⊕ · · · ⊕ Aωn

da maneira natural (i.e. inclusão componente a componente). Seja M = (aij )0≤i,j≤n a matriz corre-
spondente ao isomorfismo φ: An+1 - a ⊕ An nesta base, i.e.,

X
φ(ωi ) = aij ωj
0≤j≤n

Vamos mostrar que a = (d) onde d = det M . Como ai0 ∈ a para 0 ≤ i ≤ n, temos claramente que d ∈ a.
Reciprocamente, dado a ∈ a, considere a matriz
 
a 0 0 ··· 0
0 1 0 ··· 0
 
N =

0 0 1 ··· 0

 .. 
FT
.
0 0 0 ··· 1
)
28

Como aω0 , ω1 , . . . , ωn ∈ im φ, podemos encontrar uma matriz P tal que M P = N . Tomando determi-
nantes, temos
7:

det M · det P = det N ⇐⇒ d det C = a ⇒ a ∈ (d)


(1
RA

e portanto a ⊂ (d), o que encerra a prova.


0
01

Teorema 6.5 (Auslander-Buchsbaum) Todo anel local noetheriano regular (A, m, k) é um UFD.
,2

Prova A prova é por indução em dim A. Se dim A = 0, A é um corpo e o resultado é claro. Suponha
29

agora que dim A > 0 e escolha p ∈ m − m2 . Então p é primo (c.f. prova do teorema V.4.10). Pelo que já
D

provamos, precisamos mostrar que Ap é um UFD, ou seja, que todos os ideais em Spec Ap de altura 1
ct

são principais. Para isto, basta mostrar que estes ideais são projetivos e admitem resolução livre finita.
,O

Um ideal de Spec Ap de altura 1 é da forma qAp para algum q ∈ D(p) ⊂ Spec A de altura 1. Como
ET

A é regular, temos que sua dimensão global é finita, logo q, e portanto qAp , admitem resoluções livres
finitas.
Falta mostrar que qAp é projetivo, ou seja, que ele é localmente livre. Seja n um ideal maximal
de Ap ; note que (Ap )n é um anel local com dimensão estritamente menor do que dim A (já p ∈ m) e é
regular pelo teorema de Serre. Por hipótese de indução, temos que (Ap )n é um UFD. Portanto (qAp )n ,
que é ou igual a (Ap )n ou um primo de altura 1, é principal, logo livre de posto 1.
113

7 Álgebra Multilinear álgebra tenso


álgebra exter
O produto tensorial pode ser utilizado para construir certas álgebras não comutativas que são frequente- álgebra simét

mente utilizadas em Álgebra e, especialmente, em Geometria e Topologia.


Definição 7.1 Seja M um A-módulo. A álgebra tensorial de M é a A-álgebra graduada (não comu-
tativa em geral) dada por

def
M
T (M ) = M ⊗n = A ⊕ M ⊕ (M ⊗ M ) ⊕ (M ⊗ M ⊗ M ) ⊕ · · ·
n≥0

onde o produto de um elemento homogêneo m1 ⊗ · · · ⊗ mr ∈ M ⊗r de grau r por outro n1 ⊗ · · · ns ∈ M ⊗s


de grau s é dado por

(m1 ⊗ · · · ⊗ mr ) · (n1 ⊗ · · · ns ) = m1 ⊗ · · · ⊗ mr ⊗ n1 ⊗ · · · ns ∈ M ⊗(r+s)

A álgebra exterior de M é A-álgebra graduada (não comutativa em geral) dada pelo quociente
^ def T (M )
M =
(m ⊗ m | M )
⊗2
V gerado por elementos da forma m ⊗ m ∈ M . A imagem do
de T (M ) pelo ideal homogêneo bilateral
⊗r
elemento m1 ⊗ · · · ⊗ mr ∈ M em M será denotada por m1 ∧ · · · ∧ mr .
Finalmente, a álgebra simétrica de M é A-álgebra graduada comutativa definida como o quociente

def T (M )
S(M ) =
(m ⊗ n − n ⊗ m | m, n ∈ M )

de T (M ) pelo ideal homogêneo bilateral gerado por elementos da forma m ⊗ n − n ⊗ m ∈ M ⊗2 .


Note que como, em geral, m ⊗ n 6= n ⊗ m ∈ M ⊗2 , T (M ) não é comutativa. Por outro lado, como
S(M ) é gerado em grau 1 pelas imagens dos elementos de MV, que comutam por definição, temos que
S(M ) é sempre comutativa. Finalmente, a álgebra exterior M é anti-comutativa: para m, n ∈ M
temos

(m + n) ∧ (m + n) = 0 ⇐⇒ m ∧ m + m ∧ n + n ∧ m + n ∧ n = 0 ⇐⇒ m ∧ n = −n ∧ m
L
Exemplo 7.2 Para o A-módulo livre M = 1≤i≤n Axi de posto n com base x1 , . . . , xn , como
FT
M
M ⊗d = Axi1 ⊗ xi2 ⊗ . . . ⊗ xid ,
(i1 ,...,id )∈{1,...,n}d
)
28

é um A-módulo livre de posto nd , é fácil ver que temos um isomorfismo de A-álgebras graduadas
7:
(1
RA

Ahx1 , . . . , xn i

- T (M )
0

xi 7→ (0, xi , 0, 0, . . .)
01

onde Ahx1 , . . . , xn i é a álgebra dos polinômios nas variáveis não comutativas x1 , . . . , xn , ou seja, a A-
,2

álgebra associativa livre em x1 , . . . , xn . Por outro lado, neste isomorfismo temos que o ideal de T (M )
29

gerado por m⊗n−n⊗m, m, n ∈ M , corresponde ao ideal de Ahx1 , . . . , xn i pelos comutadores xi xj −xj xi ,


D

1 ≤ i, j ≤ n, e assim temos o isomorfismo


ct
,O

S(M ) = A[x1 , . . . , xn ]
ET

da álgebra simétrica com a álgebra comutativa usual dos polinômios. Finalmente, utilizando a anti-
Vd V
comutatividade, temos que a parte de grau d M de M é dada por
d
^ M
M= Axi1 ∧ . . . ∧ xid
1≤i1 <···<id ≤n
114 Métodos Homológicos

 Vd Vn
que é um A-módulo livre de posto nd . Note que M = 0 para d > n. O termo de grau mais alto M determinante
caracterı́stica de Eule
tem posto 1 com base x1 ∧ . . . ∧ xn e é às vezes chamado de determinante de M pela seguinte razão:
dada uma matriz quadrada (aij ) de ordem n com entradas em A, consideremos os elementos “coluna”
V1
em M

def def def


v1 = a11 x1 + · · · + an1 xn , v2 = a12 x1 + · · · + an2 xn , ..., vn = a1n x1 + · · · + ann xn
Vn
Utilizando a anticomutatividade, temos que o produto exterior v1 ∧ . . . ∧ vn ∈ M destes elementos é
X
v1 ∧ . . . ∧ vn = sgn σ · a1σ(1) . . . anσ(n) · x1 ∧ . . . ∧ xn = det(aij ) · x1 ∧ . . . ∧ xn
σ∈Sn

onde Sn denota o grupo simétrico de ordem n (grupo de todas as permutações de 1, . . . , n) e sgn σ é a


assinatura de σ ∈ Sn , i.e., −1 se σ é uma permutação ı́mpar e +1 se for par.
O exemplo acima mostra
V que, para um módulo livre M de posto finito, as álgebras tensorial T (M ),
simétrica S(M ) e exterior M são generalizações das álgebras de polinômios (tanto comutativos como
não comutativos) e do determinante, mas elas são definidas de modo completamente “intrı́nseco”, não
dependendo da escolha de uma particular base de M . Como veremos mais tarde, isto é muito útil no caso
em que queremos construir “famı́lias” de tais álgebras a partir de módulos M que são localmente livres
(i.e. tais que Mp é um módulo Ap livre para todo p ∈ Spec A), mas não necessariamente “globalmente”
livres.
Teorema 7.3 Seja
0 - M - N - P - 0

uma sequência exata de módulos livres de postos m, n e p, respectivamente. Então temos um isomorfismo
canônico
^n m
^ ^p
N= M⊗ P

8 Exercı́cios
01. (5-lemma) No seguinte diagrama de A-módulos, as linhas são exatas, a flecha vertical mais à esquerda
é sobrejetora, a mais à direita é injetora e as duas interiores marcadas com ≈ são isomorfismos. Mostre
que a flecha vertical central remanescente é um isomorfismo.

M - N - P - Q - R

FT
≈ ≈
)
?
? ? ? ? ?
28

M′ - N′ - P′ - Q′ - R′
7:
(1
RA

02. (Caracterı́stica de Euler) Seja (M• , d• ) um complexo de A-módulos de comprimento finito e com um
número finito de componentes Mi 6= 0. A sua caracterı́stica de Euler é definida como
0
01

def
X
χ(M• , d• ) = (−1)i lenA Hi (M• , d• )
,2

i
29

Seja
D

- M′ - M• - M ′′ - 0
ct

0 • •
,O

uma sequência exata de complexos como acima. Mostre que


ET

χ(M• , d• ) = χ(M•′ , d′• ) + χ(M•′′ , d′′• )

03. Sejam a e b ideais de um anel A. Mostre que that TorA


1 (A/a, A/b) = (a ∩ b)/ab.

04. Mostre que a dimensão global de um anel A é igual ao supremo de proj. dim M onde M percorre
todos os A-módulos (e não só os finitamente gerados).
115

Teorema 8.1 (Equational characterisation of flatness) An A-módulo M é flat se, e só se, it has the
seguinte propriedade: dado an m × n matrix A com entries in A e a vector (x1 , . . . , xn )T ∈ M n tal que
   
x1 0
 .  .
A .
. 
= .. 
xn 0

existe a n × p matrix B com entries in A tal que AB = 0 e

   
x1 y1
 ...  = B  . 
 .. 
xn yp

para algum vector (y1 , . . . , yp )T ∈ M p .

Prova Suponha that M é flat sobre A. The matrix A defines an A-linear mapa φ: Rn → Rm ; seja
K = ker φ. Então (x1 , . . . , xn )T é in the kernel of Rn ⊗ M = M n → Rm ⊗ M = M m , which é K ⊗ M ,
há y1 , . . . , yp ∈ M e (b1,1 , . . . , b1,n ), . . . , (bp,1 , . . .P
portanto P , bp,n ) ∈ K tal que (x1 , . . . , xn ) can be written in
the form 1≤i≤p (bi,1 , . . . , bi,n )·yi , which é the image of 1≤i≤p (bi,1 , . . . , bi,n )⊗ yi under the isomorfismo
Rn ⊗ M = M n . We may então take B = (bij ).
Reciprocamente, suponha that the above propriedade holds; temos to show thatPpara todo ideal
a of A, a ⊗ M → M é injetivo. Seja a1 , . . . , an ∈ a e x1 , . . . , xn ∈ M be tal que 1≤i≤n ai ⊗ xi é
P
in the kernel of a ⊗ M → M , i.e., 1≤i≤n ai xi = 0. Então há elements y1 , . . . , yp ∈ M e a n × p
P P
matrix B = (bij ) com 1≤i≤n ai bij = 0 para todo j e tal que xi = 1≤j≤p bij yj . This implies that
P P P 
1≤i≤n ai ⊗ xi = 1≤j≤p 1≤i≤n ai bij yj = 0, as was to be shown.

Teorema 8.2 Seja A be a noetherian anel e seja B be a noetherian flat A-álgebra. Se b ∈ B é tal que
its image in B/mB é regular para todo maximal ideals m of A, então B/(b) é flat sobre A.

Prova

Corolário 8.3 Se P (X1 , . . . , Xn ) ∈ R[X1 , . . . , Xn ] é such that the ideal gerado por its coefficients é (1),
então R[X1 , . . . , Xn ]/(P ) é flat sobre A.
FT
Seja M be an A-módulo. We write M ∗ para the dual of M :
)
28

M ∗ = HomR (M, R)
7:
(1
RA

Two observations about duals. First, se A é a domain e M é torsion-free, então so é M ∗ , as can be easily
checked. Second, se A é noetherian e M é finitamente gerado sobre A, so é M ∗ . To see this, write an
0
01

exact sequence F → M → 0, onde F é a free módulo of finite rank; dualising, obtemos an exact sequence
0 → M ∗ → F ∗ . Como F ∗ ∼ = F (non-canonically) e A é noetherian, concluı́mos que the submódulo M ∗
,2


of F é indeed finitamente gerado.
29

Also, existe always a canonical mapa M → M ∗∗ taking m ∈ M to the mapa φ 7→ φ(m), φ ∈ M ∗ Se


M → M ∗∗ é an isomorphism, we say that M é reflexive.
D
ct

Recall that se (R, m, k) é a local anel, então


,O
ET

proj. dim M = sup{n | Torn+1 (M, k) = 0}

Portanto se F é free e
0→N →F →M →0

é exact, então proj. dim M = proj. dim N − 1.


116 Métodos Homológicos

Lemma 8.4 Seja (R, m, k) be a noetherian regular local anel of dimension at most 2. Então qualquer
finitamente gerado reflexive A-módulo M é free.
Prova Como A é noetherian, M ∗ é finitamente gerado, e we may write an exact sequence

0 → N → F → M∗ → 0

para algum free módulo F of finite rank e algum finitamente gerado módulo N . Dualising, obtemos an
exact sequence
0 → M ∗∗ → F ∗ → N ∗
Seja Q be the image of F ∗ → N ∗ . Por hypothesis, M é reflexive, so we can write an exact sequence

0 → M → F∗ → Q → 0 (1)

Como A é regular, it é a domain; seja K = Frac R. Now N , as a submódulo of the free módulo F ,
é torsion-free. Portanto N ∗ é also torsion-free, e so é its submódulo Q. Portanto Q → Q ⊗R K é an
injection (notice that Q ⊗R K é just the localisation of Q com respect to the multiplicative set R \ {0}).
Por (1), Q é finitamente gerado, portanto Q⊗R K é a finite dimensional K-espaço vetorial; seja ω1 , . . . , ωn
be a base. Multiplying the ωi por a convenient element of K, we may assume that todo the generators
of Q mapa to elements of the free módulo P = Rω1 + · · · + Rωn . Portanto temos an exact sequence

0 → Q → P → P/Q → 0 (2)

Como A has global dimension at most 2, proj. dim P/Q ≤ 2, e (1) e (2) imply that proj. dim Q ≤ 1 e
proj. dim M = 0. Portanto M é projective, but como it é finitamente gerado sobre a local anel A, it
must be free.

FT
)
28
7:
(1
RA
0
01
,2
29
D
ct
,O
ET
Chapter 7
A-derivação

Aspectos Diferenciais de Anéis

1 Derivações e Diferenciais de Kähler

1.1 Definições e Exemplos


Definição 1.1.1 Sejam A um anel, B uma A-álgebra e M um B-módulo. Uma A-derivação de B em
M é uma função D: B → M satisfazendo os seguintes axiomas: para todo a1 , a2 ∈ A e b1 , b2 ∈ B,
1. (A-linearidade) D(a1 b1 + a2 b2 ) = a1 D(b1 ) + a2 D(b2 );
2. (Regra de Leibniz) D(b1 b2 ) = b1 D(b2 ) + b2 D(b1 ).
O conjunto de todas as A-derivações de B em M é denotado por DerA (B, M ). No lugar de DerA (B, B),
escreveremos simplesmente DerA (B).
Note, em particular, que D(1 · 1) = D(1) + D(1) ⇒ D(1) = 0 e portanto D(a) = a · D(1) = 0 para
todo a ∈ A. Ou seja, A pode ser visto como o “anel dos escalares” ou “anel das constantes”.
Exemplo 1.1.2 Considere a A-álgebra B = A[x1 , . . . , xn ] dos polinômios em n variáveis. Temos que as
def

derivadas parciais Di = ∂x i
são A-derivações de B em B. E é fácil mostrar que qualquer B-combinação
linear dos Di ’s é uma derivação:

D = q1 · D1 + · · · + qn · Dn ∈ DerA (B) qi ∈ B

Reciporcamente, qualquer elemento D ∈ DerA (B) é da forma acima: pela “regra da cadeia”, temos
 X 
D p(x1 , . . . , xn ) = Di p(x1 , . . . , xn ) · D(xi )
1≤i≤n
FT
e assim, basta tomar qi = D(xi ) na igualdade acima.
Exemplo 1.1.3 (Espaço Tangente) Seja (B, n, R) a R-álgebra que é a localização do anel das funções
)
28

infinitamente diferenciáveis f : Rn → R com relação ao ideal maximal {f | f (0, . . . , 0) = 0} das funções


que se anulam na origem. Consideremos M = R como um A-módulo via
7:
(1
RA

def
f · m = f (0, . . . , 0) · m f ∈ A, m∈M
0
01

Temos que um vetor v = (v1 , . . . , vn ) do espaço tangente de Rn na origem define uma R-derivação de B
,2

em M , a saber, a derivada direcional


29

f (tv) − f (0, . . . , 0) X ∂f
D

Dv (f ) = lim = vi · (0, . . . , 0)
ct

t→0 t ∂xi
1≤i≤n
,O

Reciprocamente, qualquer derivação D ∈ DerR (B, M ) é da forma acima, de modo que temos uma
ET

identificação natural de DerR (B, M ) com o espaço tangente de Rn na origem! Para provar isto, observe
que dado f ∈ B, temos
Z 1 X Z 1
df ∂f
f (x1 , . . . , xn ) − f (0, . . . , 0) = (tx1 , . . . , txn ) dt = xi · (tx1 , . . . , txn ) dt
0 dt 0 ∂xi
1≤i≤n
118 Aspectos Diferenciais de Anéis

Ou seja, temos diferenciais de Kähler


X
f (x1 , . . . , xn ) = f (0, . . . , 0) + xi · φi (x1 , . . . , xn ) com φi ∈ B
1≤i≤n
∂f
e φi (0, . . . , 0) = ∂xi (0, . . . , 0). Expandindo cada φi como acima e substituindo, temos portanto que
X ∂f
f (x1 , . . . , xn ) = f (0, . . . , 0) + xi · (0, . . . , 0) + g(x1 , . . . , xn ) com g ∈ n2
∂xi
1≤i≤n

Note que se p, q ∈ n, temos D(pq) = p · D(q) + q · D(p) = 0, logo D(g) = 0 (g é uma soma de termos da
forma pq). Portanto
 X ∂f
D f (x1 , . . . , xn ) = D(xi ) · (0, . . . , 0)
∂xi
1≤i≤n

e assim temos D = Dv para o vetor v como componentes vi = D(xi ).


Em seguida, vamos definir o análogo algébrico de uma forma diferenciável.
Definição 1.1.4 Seja B uma A-álgebra. O módulo de diferenciais de Kähler ΩB/A é o quociente
M/N do B-módulo livre M
M= B · db
b∈B
gerado pelos sı́mbolos db, um para cada elemento b ∈ B, pelo submódulo N gerado pelas expressões

d(ab + a′ b) − a · db − a′ · db′
a, a′ ∈ A, b, b′ ∈ B
d(bb′ ) − b · db′ − b′ · db
Por construção, ΩB/A vem equipado de fábrica com uma A-derivação
d: B 7→ ΩB/A
cujos elementos da imagem db geram ΩB/A . Temos ainda a seguinte propriedade universal: dado um B-
módulo de “teste” T e uma A-derivação D: B → T , existe um único morfismo de B-módulos φ: ΩB/A → T
de modo que o seguinte diagrama comuta:
D -
B T
-
φ

d
∃!

?
ΩB/A
FT
Em outras palavras, temos uma bijeção natural
)
- DerA (B, T )

28

HomB (ΩB/A , T )
φ 7→ φ ◦ d
7:
(1
RA

Exemplo 1.1.5 Seja B = A[x1 , . . . , xn ]. Temos ΩB/A é um B-módulo livre com base dxi :
M
0

ΩB/A = B · dxi
01

1≤i≤n
,2

De fato, como ΩB/A é gerado por elementos da forma df , f ∈ B, e


29

∂f ∂f
· dx1 + · · · +
df = · dxn
D

∂x1 ∂xn
ct

temos que ΩB/A = Bdx1 + · · · + Bdxn e só resta mostrar que os dxi são linearmente independentes sobre
,O


B. Pela propriedade universal, cada ∂x i
∈ DerA (B) dá origem a um morfismo φi ∈ HomB (ΩB/A , B) tal
ET

que n
1 se i = j
φi (dxj ) =
0 caso contrário
Portanto
b1 dx1 + · · · + bn dxn = 0 ⇒ b1 φi (dx1 ) + · · · + bn φi (dxn ) = 0 ⇒ bi = 0
para todo i, como querı́amos.
119

Exemplo 1.1.6 (Espaço cotangente) Mantemos a notação do exemplo 1.1.3. Temos uma bijeção

HomB (ΩB/R , M ) - DerR (B, M )


Por outro lado, vimos que DerR (B, M ) pode ser identificado com o espaço tangente de Rn na origem, um

espaço vetorial de dimensão n com base ∂x 1
, . . . , ∂x∂ n . Como n anula M , temos que HomB (ΩB/R , M ) =
HomR (ΩB/R /nΩB/R , M ), que é dual a ΩB/R /nΩB/R como R-espaço vetorial. Assim, a “fibra na origem”
ΩB/R ⊗B B/n = ΩB/R /nΩB/R de ΩB/R é dual a DerR (B, M ) e pode ser interpretado como o espaço
cotangente de Rn na origem.
Raı́zes múltiplas de polinômios podem ser detectadas utilizando-se o critério da derivada. Não
surpreedentemente, diferenciais podem ser utilizados para reconhecer extensões separáveis de corpos.
Vejamos o
Exemplo 1.1.7 Seja L ⊃ K uma extensão separável de corpos (possivelmente infinita). Vamos mostrar
que ΩL/K = 0. Seja θ ∈ L e f (x) ∈ K[x] seu polinômio minimal. Em ΩL/K temos

d f (θ) = 0 ⇒ f ′ (θ)dθ = 0
Mas como L ⊃ K é separável por hipótese, temos f ′ (θ) 6= 0 e portanto dθ = 0. Como elementos desta
forma geram ΩL/K , o resultado segue.
Seja B uma A-álgebra. Terminamos esta seção com uma segunda construção para o módulo de
diferenciais ΩB/A . Considere o mapa de multiplicação
m: B ⊗A B ։ B
b ⊗ b′ 7→ bb′
e seja I = ker m. Como m é sobrejetor, temos
P um isomorfismo (B ⊗A B)/I ∼= B e portanto podemos ver
I/I como um B-módulo: para b ∈ B e 1≤i≤n bi ⊗ b′i ∈ I, temos
2

X X X
b·( bi ⊗ b′i mod I 2 ) = bbi ⊗ b′i mod I 2 = bi ⊗ bb′i mod I 2
1≤i≤n 1≤i≤n 1≤i≤n

pois m(b ⊗ 1) = m(1 ⊗ b) = b. Definimos agora uma A-derivação


d: B → I/I 2
b 7→ (1 ⊗ b − b ⊗ 1) mod I 2
Como 1 ⊗ a − a ⊗ 1 = a ⊗ 1 − a ⊗ 1 = 0 para a ∈ A, temos que da = 0 para todo a ∈ A. Por outro lado,
d(b1 b2 ) − b1 db2 − b2 db1 = 1 ⊗ b1 b2 − b1 b2 ⊗ 1 − (b1 ⊗ b2 − b1 b2 ⊗ 1) − (b2 ⊗ b1 − b1 b2 ⊗ 1) mod I 2
= (1 ⊗ b1 − b1 ⊗ 1)(1 ⊗ b2 − b2 ⊗ 1) mod I 2 = 0
FT
mostra que d satisfaz a regra de Leibniz, logo temos que d é de fato uma A-derivação. É fácil checar
que o par (I/I 2 , d) possui a propriedade universal do módulo de diferenciais de Kähler, sendo portanto
)
28

isomorfo ao módulo anteriormente construı́do.


7:

1.2 Propriedades Funtoriais


(1
RA

Vejamos agora as propriedades funtoriais do módulo de diferenciais. Seja


0

ψ -
01

B0 B
6 6
,2
29
D
ct

A0 - A
,O

um diagrama comutativo de anéis. Então temos um mapa de B-módulos


ET

ΩB0 /A0 ⊗B0 B → ΩB/A


b 0 ∈ B0 , b∈B
db0 ⊗ b 7→ b · dψ(b0 )
que corresponde à A0 -derivação b0 7→ dψ(b0 ), b0 ∈ B0 , na bijeção natural
DerA0 (B0 , ΩB/A ) = HomB0 (ΩB0 /A0 , ΩB/A ) = HomB (ΩB0 /A0 ⊗B0 B, ΩB/A )
120 Aspectos Diferenciais de Anéis

Lema 1.2.1 (Mudança de Base) Sejam A′ e B duas A-álgebras e seja B ′ = B ⊗A A′ . Temos


isomorfismos
ΩB/A ⊗A A′ = ΩB/A ⊗B B ′ = ΩB ′ /A′

Em particular, se S é um conjunto multiplicativo de A então

S −1 ΩB/A = ΩS −1 B/S −1 A = ΩS −1 B/A

Prova O primeiro isomorfismo é apenas uma consequência do isomorfismo

M ⊗A A′ = M ⊗B (B ⊗A A′ )

válido para qualquer B-módulo M . Pela discussão acima, temos um morfismo natural ΩB/A ⊗B B ′ →
ΩB ′ /A′ , assim para provar o segundo isomorfismo basta construir o mapa inverso. Note que o mapa

B ′ = B ⊗A A′ → ΩB/A ⊗B B ′
X X
bi ⊗ a′i 7→ dbi ⊗ (1 ⊗ a′i ) a′i ∈ A′ , bi ∈ B
i i

é uma A′ -derivação, logo define um mapa ΩB ′ /A′ → ΩB/A ⊗B B ′ . Agora é fácil verificar que este mapa
é inverso do anterior.
Por fim, o isomorfismo ΩS −1 B/S −1 A = ΩS −1 B/A é uma consequência imediata do fato que temos
uma bijeção DerS −1 A (S −1 B, M ) = DerA (S −1 B, M ) para todo S −1 B-módulo M .

A principal ferramenta no cálculo do módulo de diferenciais é o

Teorema 1.2.2 (Sequências Fundamentais) Sejam A → B e B → C duas álgebras.


1. A sequência
ΩB/A ⊗B C - ΩC/A - ΩC/B - 0

é exata.
2. Seja b um ideal de B e suponha que C = B/b, visto como B-álgebra pelo mapa quociente. Então
a sequência
FT
b/b2 - ΩB/A ⊗B C - ΩC/A - 0
δ
)
é exata, onde δ é dado por
28

δ(b mod b2 ) = db ⊗ 1 b∈b


7:
(1
RA

Prova Primeiro, observe que em ΩB/A ⊗B C temos, para b, b′ ∈ b,


0
01


d(bb′ ) ⊗ 1 = (b · db′ + b′ · db) ⊗ 1 = db′ ⊗ b + db ⊗ b = 0
,2
29

de modo que δ está bem definido. É fácil verificar que ambas as sequências são complexos e que ΩC/A ։
D

ΩC/B e ΩB/A ⊗B C ։ ΩC/A são sobrejetores. Falta apenas verificar a exatidão nos termos do meio das
ct

duas sequências.
,O

Para a primeira sequência, seja I a imagem de ΩB/A ⊗B C → ΩC/A , que é gerada por elementos
ET

da forma db, b ∈ B. Defina M = ΩC/A /I. Devemos mostrar que o morfismo M → ΩC/B induzido por
ΩC/A → ΩC/B é um isomorfismo. Mas como c 7→ dc mod I, c ∈ C, define uma derivação em DerB (C, M ),
temos um morfismo ΩC/B → M . Agora é fácil verificar que este mapa é o inverso de M → ΩC/B .
A prova para a segunda sequência é similar. Seja M = (ΩB/A ⊗B C)/ im δ; o mapa inverso de
M → ΩC/A é o correspondente à derivação em DerA (C, M ) dada por b mod b 7→ db ⊗ 1 mod im δ, b ∈ B.
121

Exemplo 1.2.3 Seja A um anel. Considere a A-álgebra de presentação finita matriz Jacobi

A[x1 , . . . , xn ]
C= fi ∈ A[x1 , . . . , xn ]
(f1 , . . . , fm )

Para calcular ΩC/A , vamos utilizar a segunda sequência fundamental com B = A[x1 , . . . , xn ] e b =
(f1 , . . . , fm ). Do exemplo
L 1.1.5, já sabemos que ΩB/A é um B-módulo livre com base dx1 , . . . , dxn e
portanto ΩB/A ⊗B C = 1≤i≤n C · dxi . Por outro lado, a imagem de δ é gerada por

X  ∂fi 
dfi ⊗ 1 = · dxj ⊗ 1 para i = 1, . . . , m
∂xj
1≤j≤n

Assim, ΩC/A é isomorfo ao cokernel do mapa C-linear C m → C n dado pela matriz Jacobiana
 ∂f1 ∂fm 
∂x1 mod b · · · ∂x1 mod b
 .. .. 
 . . 
∂f1 ∂fm
∂xn mod b · · · ∂xn mod b

Em particular, para C = A[x]/f (x) temos um isomorfismo

C A[x] ∼
- ΩC/A
= 
(f ′ (x)) f (x), f ′ (x)
p(x) 7→ p(x) · dx

Teorema 1.2.4 (Espaço Cotangente) Sejam k um corpo, A uma k-álgebra e m um ideal maximal
de A tal que o mapa natural k - A/m é um isomorfismo. Então temos um isomorfismo

m ∼-
δ: ΩA/k ⊗A k
m2 (m ∈ m)
m 7→ dm ⊗ 1

Prova Nas condições do enunciado, a segunda sequência fundamental se escreve como

m δ
- ΩA/k ⊗A k - Ωk/k - 0
FT
m2

Como Ωk/k = 0, basta mostrar que δ é injetor. Seja s: A → k a composição A ։ A/m ∼


)
= k (pense em
28

s(a) como o “valor” da “função” a no “ponto” m). Como a − s(a) ∈ m para todo a ∈ A, podemos definir
7:

m
(1
RA

D: A →
m2 
0

a 7→ a − s(a) mod m2
01
,2

É fácil mostrar que D é uma k-derivação, logo define um morfismo de A-módulos ΩA/k → m/m2 . Assim,
temos um morfismo de A-módulos
29

m
D

ǫ: ΩA/k ⊗A k →
ct

m2  (a, b ∈ A)
,O

da ⊗ b 7→ b · a − s(a) mod m2
ET

Como ǫ ◦ δ = id, temos que δ é injetor e portanto um isomorfismo, como querı́amos demonstrar.

1.3 Separabilidade
Uma das principais aplicações dos diferenciais de Kähler é testar a separabilidade de corpos. Mais
geralmente, temos
122 Aspectos Diferenciais de Anéis

Teorema 1.3.1 Seja k um corpo e A uma k-álgebra de dimensão finita. Denote por k alg o fecho algébrico
de k. As seguintes condições são equivalentes:
1. A é um produto de um número finito de corpos que são extensões finitas separáveis de k.
2. A ⊗k k alg é um produto de um número finito de cópias de k alg .
3. A ⊗k k alg é um anel reduzido.
4. ΩA/k = 0.
Prova (1 ⇒ 2) Podemos supor que A é um corpo que é uma extensão finita separável de k. Pelo
teorema do elemento primitivo, podemos escrever A = k[α], onde α satisfaz um polinômio mônico
irredutı́vel f (x) ∈ k[x] com raı́zes distintas α = α1 , . . . , αn ∈ k alg . Pelo teorema Chinês dos restos, temos
portanto
k[x] k alg [x] k[x] k[x] ∼ alg
B∼=  ⊗k k alg = = × ···× = k × · · · × k alg
f (x) f (x) (x − α1 ) (x − αn )
(2 ⇒ 3) Trivial.
(3 ⇒ 1) Como A é artiniano, ele é um produto de anéis artinianos
plocais, assim sem perda de generalidade
podemos assumir que A é local com um único ideal primo m = (0). Como k alg é fielmente plano sobre
k (k alg é um k-módulo livre), temos que m ⊗k k alg ֒→ A ⊗k k alg é injetor e como A ⊗k k alg é reduzido,
temos m ⊗k k alg = 0 ⇒ m = 0, ou seja, A é um corpo.
Mostremos que A ⊃ k é uma extensão separável. Suponha por absurdo que não e seja p > 0 a
caracterı́stica de k. Então existe um elemento em A cujo polinômio minimal f (x) ∈ k[x] é da forma
f (x) = xpn + an−1 xp(n−1) + · · · + a1 xp + a0 , ai ∈ k
Observe que f (x) é uma p-ésima potência em k alg [x]; seja g(x) ∈ k alg [x] tal que g(x)p = f (x). Então
A ⊗k k alg contém uma subálgebra (k alg é fielmente plano sobre k) que é isomorfa a
k[x] k alg [x] k alg [x]
 ⊗k k alg =  = 
f (x) f (x) g(x)p
que não é reduzida, absurdo.
(2 ⇒ 4) Seja B = A ⊗k k alg . Como k alg é fielmente plano sobre k, basta mostrar que ΩA/k ⊗k k alg = 0.
Mas isto segue da mudança de base ΩA/k ⊗k k alg = ΩB/kalg = 0.
(4 ⇒ 2) Como dimkalg A ⊗k k alg = dimk A < ∞ e, por mudança de base, Ω(A⊗k kalg )/kalg = ΩA/k ⊗k k alg =
0, substituindo k por k alg e A por A ⊗k k alg podemos assumir que k é algebricamente fechado. Mais
ainda, como A é artiniano, logo um produto de anéis locais, é suficiente mostrar que se (A, m, k) é uma
álgebra local de dimensão finita sobre um corpo algebricamente fechado k e ΩA/k = 0 então A = k, ou
seja, m = 0. Porém, pelo teorema 1.2.4, temos ΩA/k = m/m2 , logo por Nakayama, m = 0.
FT
Definição 1.3.2 Uma k-álgebra A satisfazendo as condições 1–4 do teorema anterior é dita separável.
Definição 1.3.3 Seja L ⊃ K uma extensão de corpos. Um subconjunto B ⊂ L é chamado de base
)
28

de transcendência separante de L sobre K se B é uma base de transcendência de L sobre K e L é


separável sobre K(B), o subcorpo de L gerado por B sobre K.
7:
(1
RA

Teorema 1.3.4 Seja K um corpo perfeito e seja L ⊃ K uma extensão finitamente gerada de corpos.
Então L admite uma base de transcendência separante x1 , . . . , xn sobre K.
0
01

Prova Podemos assumir que p = char K > 0. Seja x1 , . . . , xn uma base de transcendência de L sobre
,2

K tal que o grau de separabilidade [L : K(x1 , . . . , xn )]sep de L sobre K(x1 , . . . , xn ) é mı́nimo. Vamos
mostrar que x1 , . . . , xn é uma base de transcendência separante. Suponha por absurdo que não; então
29

existe um elemento θ ∈ L \ K(x1 , . . . , xn ) que é raiz de um polinômio da forma


D

f (y) = ad · y pd + ad−1 · y p(d−1) + · · · + a0 ,


ct

ai ∈ K[x1 , . . . , xn ]
,O

que é irredutı́vel em K[x1 , . . . , xn ][y] e portanto em K(x1 , . . . , xn )[y] pelo lema de Gauß. Temos também
que f ∈ / K[xp1 , . . . , xpn , y], caso contrário como K é perfeito f seria uma p-ésima potência. Portanto
ET

podemos supor que xn é separável sobre K(x1 , . . . , xn−1 , θ). Mas então x1 , . . . , xn−1 , θ seria uma base
de transcendência para a qual
[L : K(x1 , . . . , xn )]sep = [L : K(x1 , . . . , xn , θ)]sep · [K(x1 , . . . , xn , θ) : K(x1 , . . . , xn )]sep
> [L : K(x1 , . . . , xn−1 , θ)]sep
o que contradiz a minimalidade de [L : K(x1 , . . . , xn )]sep .
123

Corolário 1.3.5 Nas condições do teorema, temos não-ramificad

M
ΩL/K = L · dxi (n = tr. degK L)
1≤i≤n

Prova Como no exemplo 1.1.5, mostra-se que ΩK(B)/K é um K(B)-espaço vetorial com base {dx | x ∈
B}. Agora observe que toda derivação D ∈ DerK K(B) se estende unicamente a uma derivação em
DerK (L). De fato, seja θ ∈ L e seja f (x) = an xn + · · · + a0 seu polinômio minimal sobre K(B)[x], que é
separável por hipótese, de modo que f ′ (θ) 6= 0. Então qualquer extensão de D deve satisfazer

 D(an ) · θn + D(an−1 ) · θn−1 + · · · D(a0 )


D f (θ) = 0 ⇒ D(θ) =
f ′ (θ)

o que prova a unicidade. Para mostrar a existência, utilize a expressão acima para definir a extensão.

Definição 1.3.6 Seja L ⊃ K uma extensão de corpos de caracterı́stica p > 0. Um subconjunto B ⊂ L é


chamado de p-base de L sobre K se, B gera L sobre o compósito Lp · K e, para todo subconjunto finito
{x1 , . . . , xn } ⊂ B, o conjunto
{xe11 . . . xenn | 0 ≤ ei < p}
é linearmente independente sobre Lp · K.
Uma aplicação simples do lema de Zorn (exercı́cio!) mostra que p-bases sempre existem. A im-
portância de p-bases em nosso estudo de derivações é que qualquer mapa D: B → L se estende unicamente
a uma derivação D ∈ DerK (L) por
X
D(xe11 . . . xenn ) = ei · xe11 . . . xei i −1 . . . xenn · Dxi
1≤i≤n

para todo {x1 , . . . , xn } ⊂ B. Portanto ΩL/K possui base {dx | x ∈ B} sobre L.

1.4 Discriminante e Diferente


d = ann ΩB/A

2 Morfismos não-ramificados
Definição 2.1 Uma A-álgebra φ: A → B é não-ramificada se é de presentação finita e ΩB/A = 0.
FT
Exemplo 2.2 Seja A um anel e a um ideal qualquer de A. Então o morfismo quociente A ։ A/a é não
ramificado.
)
28

Teorema 2.3 Seja k um corpo e A uma k-álgebra finitamente gerada. Então A é não ramificada sobre
7:

k se, e só se, A ∼


= l1 × · · · × ln , onde li são corpos que são extensões finitas separáveis de k.
(1
RA

Prova Como A é finitamente gerado sobre k, A é noetheriano e portanto automaticamente de pre-


0

sentação finita sobre k. Portanto A é não ramificado sobre k se, e só se, ΩA/k = 0.
01

Se A ∼ = l1 × · · · × ln , onde li são corpos que são extensões finitas separáveis de k, temos que
,2

ΩA/k = 0, logo A é não ramificado sobre k. Reciprocamente, suponha que ΩA/k = 0. Seja p um
ideal primo minimal de A e seja K = Frac(A/p). Então K é finitamente gerado (como corpo) sobre
29

k; além disso, ΩA/k = 0 ⇒ ΩK/k = 0. Portanto K é uma extensão finita separável de k e assim
D
ct

dim A/p = tr. degk K = 0. Portanto dim A = 0, i.e., A é uma k-álgebra de dimensão finita. O resultado
,O

agora segue do teorema da seção anterior.


ET

Lemma 2.4
1. (Composição) A composição de dois morfismos não ramificados é não ramificado;
2. (Mudança de Base) Se φ: A → B é não ramificado e A′ é uma A-álgebra qualquer, então a
mudança de base φ ⊗ id: A′ → B ⊗A A′ também é não ramificada.
124 Aspectos Diferenciais de Anéis

Teorema 2.5 Seja φ: A → B um morfismo de presentação finita. As seguintes condições são equiva-
lentes.
1. φ é não-ramificado;
2. O morfismo fibra φ ⊗ id: k(p) → B ⊗A k(p) é não-ramificado para todo ideal primo p ∈ Spec A;
3. Para todo ideal primo q ∈ Spec B, sendo p = φ−1 (q) ∈ Spec A, temos que qBq = pBq e
k(q) ⊃ k(p) é uma extensão separável de corpos.

Prova Como B é of finite type sobre A, ΩB/A é a finite B-módulo, e portanto (ΩB/A )q = 0 se, e só se,
existe h ∈ B − q tal que (ΩB/A )h = 0, which é equivalent to Bh being unramified sobre A. Portanto
1 ⇔ 2. Furthermore, como (ΩB/A )q = ΩBq /Ap , temos que 2 ⇔ 3.
Now we prove that 4 ⇔ 5. Como f −1 (y) = Spec B ⊗A k(p) é of finite type sobre Spec k(p), we can
apply the last theorem, e 4 holds se e só se Bq ⊗A k(p) é a finite separable corpo extensão of k(p). But
that means that pBq equals the maximal ideal qBq of Bq , e that k(q) = Bq /qBq = Bq /pBq é separable
over k(p), which é precisely condição 5.
Finally, we mostrar que 2 ⇔ 4. We already know that 4 holds se, e só se, Of −1 (y),x = Bq ⊗A k(p)
é formally unramified sobre k(p). Write k = k(p), B = Bq ⊗A k(p) = Bq /pBq, e l para the common
residue corpo of B e Bq . Então 4 holds se, e só se, ΩB/k = 0, while 2 holds se, e só se, ΩBq /Ap = 0. But
como ΩB/k e ΩBq /Ap são finite módulos over B e Bq , respectively, Nakayama’s lemma implies that

ΩBq /Ap = 0 ⇐⇒ ΩBq /Ap ⊗Bq l = 0 ⇐⇒ (ΩBq /Ap ⊗Bq B) ⊗B l = 0


⇐⇒ ΩB/k ⊗B l = 0 ⇐⇒ ΩB/k = 0

3 Morfismos étales

3.1 Condições Abertas

Lemma 3.1.1 Seja A um domı́nio noetheriano e seja B ⊃ A com B finitamente gerado sobre A. Seja
f : Spec B → Spec A o mapa correspondente à inclusão A ֒→ B. Então existe um elemento não nulo
h ∈ A tal que D(h) ⊂ f (Spec B).
Prova Escreva B = A[x1 , . . . , xn ] em que x1 , . . . , xd são algebricamente independente sobre Frac A e os
demais xi satisfazem relações algébricas
FT
(i) (i)
cr(i)
i
(x1 , . . . , xd ) · xri i + cri −1 (x1 , . . . , xd ) · xri i −1 + · · · + c0 (x1 , . . . , xd ) = 0,
(i)
cj (x1 , . . . , xd ) ∈ A[x1 , . . . , xd ] c(i)
ri (x1 , . . . , xd ) 6= 0
)
28
7:

para i = d + 1, . . . , n. Considere o produto dos coeficientes lı́deres:


(1
RA

Y
t(x1 , . . . , xd ) = c(i)
ri (x1 , . . . , xd ) 6= 0
0

d+1≤i≤n
01
,2

Localizando com relação ao elemento t(x1 , . . . , xd ), temos que Bt é integral sobre A[x1 , . . . , xd ]t , portanto
Spec Bt ։ Spec A[x1 , . . . , xd ]t é sobrejetor. Seja h ∈ A qualquer não nulo de t(x1 , . . . , xd ). Temos que
29

D(h) ⊂ f (Spec B). De fato, dado p ∈ D(h) temos que t ∈ / p · A[x1 , . . . , xd ] e portanto p · A[x1 , . . . , xd ]t é
D

um ideal primo; tome P ∈ Spec B tal que PBt é levado em p · A[x1 , . . . , xd ]t . Então f (P) = p.
ct
,O

Teorema 3.1.2 (Morfismos Planos são Abertos) Seja A um anel noetheriano e seja B uma A-
ET

álgebra plana finitamente gerada. Então o mapa associado de espectros f : Spec B → Spec A é um mapa
aberto.

Prova Temos que mostrar que f D(h) é aberto para todo h ∈ B. Substituindo B por Bh , é suficiente
mostrar que f (Spec B) é aberto. Seja F = Spec A \ f (Spec B); temos que mostrar que F é fechado em
Spec A. Como A é noetheriano, podemos escrever o fecho F de F como união finita Z1 ∪ · · · ∪ Zn de
125

fechados irredutı́veis Zi = V (pi ) com pi ∈ SpecSA. Basta mostrar que pi ∈ F para todo i: de fato, como étale
B é A-plano, pelo going-down temos que F ⊃ i Zi , i.e., F = F é fechado.
Seja p ∈ f (Spec B) e seja q ∈ Spec B tal que f (q) = p. Afirmamos que existe um aberto W ⊂ Spec A
tal que W ∩V (p) ⊂ f (Spec B). De fato, isto do fato que V (p) é homeomorfo a Spec A/p e do lema anterior
aplicado a A/p ֒→ B/q.
Provemos que pi ∈ F . Suponha por absurdo que não. Neste caso, temos pi ∈ f (Spec B) e portanto
f (Spec B) ⊃ W ∩ Zi para algum aberto W ⊂ Spec A. Mas como pi ∈ / Zj para j 6= i, temos
[ c
pi ∈ W ∩ Zj ⊂ W ∩ (f (Spec B) ∪ Zi ) ⊂ f (Spec B)
j6=i

S c
Portanto W ∩ j6=i Zj é uma vizinhança aberta de pi disjunta de F , contradizendo pi ∈ F .

3.2 Morfismos étale


Definição 3.2.1 Uma A-álgebra φ: A → B é étale se é de presentação finita, não-ramificada e plana.
Seja C be an A-álgebra that é a free A-módulo of finite rank. Para qualquer c ∈ C, multiplication
por c gives rise to an A-linear mapa Tc : C → C; se ω1 , . . . , ωn é a base of C, we can express Tc in terms
of a matrix (aij )1≤i,j≤n com entries aij ∈ A satisfazendo
X
c · ωi = aij ωj , i = 1, . . . , n
1≤j≤n

The trace of c, written TrC/A (c), é defined to be the trace of the matrix (aij ); it é easy to see that
TrC/A (c) é independent of the choice of base above. Notice that para qualquer base change φ: A → A′
temos TrC⊗A A′ /A′ (c ⊗ 1) = φ(TrC/A (c)).
Lemma 3.2.2 Seja C be an A-álgebra that é free of finite rank as an A-módulo.
1. Se c ∈ C é nilpotent, então TrC/A (c) ∈ A é also nilpotent.
2. Suponha that A é a normal domain com Frac A = K. Se c ∈ C ⊗A K é integral sobre C, então
TrC⊗A K/K (c) ∈ A.
3. Seja f ∈ A[x] be a monic polinômio of grau n, e suponha that C has the form A[x]/f . The
determinant of the n × n matrix (TrC/A (xi+j ))1≤i,j≤n é equal to the discriminant of f , i.e., to
the resultant of f e f ′ .

Prova 1. We need to mostrar que TrC/A (c) ∈ p para todo p ∈ Spec A, i.e., that TrC/A (c) maps to 0 in
FT
k(p), or equivalently, that TrC⊗A k(p)/k(p) (c) = 0. But c é nilpotent in C, so c ⊗ 1 é nilpotent in C ⊗A k(p).
But that means that the k(p)-linear mapa Tc⊗1 é nilpotent, e portanto its trace é 0, as required.
)
28

Teorema 3.2.3 Seja B be an étale A-álgebra.


7:

1. Se A é reduced, então B é also reduced.


(1
RA

2. Se A é normal, então B é normal.


0
01

Prova Todo properties são local, so we may assume that (A, m, k) é local, e that B = Cq , onde C =
A[x]/f para algum f é monic polinômio of grau n, q ∈ Spec A[x] contains f e lies sobre m, e f ′ é a unit
,2

in B.
29

1. Se b = c/s é in the nilradical of B, então existe t ∈ C − q tal que ct é nilpotent in C. Now write
D
ct

ct = a0 + a1 x + · · · + an−1 xn−1 , ai ∈ A
,O

Então, para i = 0, . . . , n − 1, temos


ET

X
TrC/A (tcxi ) = aj TrC/A (xi+j )
0≤j<n

Now cada TrC/A (tcxi ) = 0 como it é nilpotent in A. Moreover, det(TrC/A (xi+j )) é invertible in A como
f ′ é a unit in B. Portanto aj = 0 para todo j, i.e., tc = 0, e portanto b = c/s = 0 in B.
126 Aspectos Diferenciais de Anéis

2. First we mostrar que we may take f irredutı́vel sobre K, e portanto C é a domain. In fact, se f = gh
com monic polinômios f, g ∈ K[x], então como the coefficients of g e h são polinômios in the roots of f ,
they são integral sobre A, e portanto they must lie in A. Portanto g, h ∈ A[x]. We cannot have both
g e h in q, como otherwise f ′ ∈ q. Então por the Chinese remainder theorem, B decomposes, which é
impossible como it é a local anel. Portanto f must be irredutı́vel in K[x].
Now L = C ⊗A K é a finite separable corpo extensão of K, como f ′ é a unit in B; it é the quociente
corpo of B e C. Seja c ∈ L é integral sobre C, e write

c = a0 + a1 x + · · · + an−1 xn−1 , ai ∈ K

As before, D = det(TrC/A (xi+j )) é tal que Daj ∈ A, e portanto Dc ∈ C. But D é invertible in B, e the
result follows.

Exemplo 3.2.4 Seja k be an algébricoally closed corpo com char k 6= 2, e seja A = k[x, y]/(y 2 −x2 (x+1)).
The maximal ideal m = (x + 1, y), corresponding to the ponto (−1, 0) of the curve Spec A, defines an
invertible sheaf: para the open cobertura D(x) ∪ D(x + 1) of Spec A, temos mx = (y) e mx+1 = (1).
temos que m2 = (x + 1), e it can be checked that m é not principal; portanto m defines an element of
order 2 in Pic A.
Now we use m to construct an étale A-álgebra B. Seja µ: m2 → A be the isomorphism of A-módulos
dado por µ(x + 1) = 1. As an A-módulo, seja B = A ⊕ m, e define the multiplication in B por

(a1 , t1 ) · (a2 , t2 ) = (a1 a2 + µ(t1 t2 ), a1 t2 + a2 t1 ), a1 , a2 ∈ A, t1 , t2 ∈ m

Now we check that B é étale sobre A. In D(x), temos an isomorphism

Ax [t]
→ Bx , t 7→ (0, y)
t2 − x2

while in D(x + 1) temos the isomorphism

Ax+1 [t]
1 → Bx+1 , t 7→ (0, 1)
t2 − x+1

Portanto B é locally isomorphic to standard étale álgebras.


We can write an explicit presentation of B. Consider the polinômio álgebra A[w, z] e the surjective
morphism of A-álgebras
A[w, z] → B w 7→ (0, x + 1), z 7→ (0, y)
FT
Clearly the elements
)
28

−yz + x2 w, yw − (x + 1)z, w2 − (x + 1), wz − y, z 2 − x2


7:

mapa to 0, e it é easy to check that they actually generate the kernel. E como x2 w − yz = z(wz − y) −
(1
RA

w(z 2 − x2 ), we can do com just 4 polinômios: seja a be the ideal generated por these 4 polinômios, e
consider the ideals p = a+(x−z) = (x−z, w2 −(x+1), wx−y) e q = a+(x+z) = (x+z, w2 −(x+1), wx+y).
0
01

Both A[w, z]/p e A[w, z]/q são isomorphic to à = k[x, y, w]/(w2 − (x + 1), wx − y), the normalisation of
A. Portanto p e q são prime ideals of A[w, z] com p ∩ q ⊃ a ⊃ pq, e portanto p e q correspond to the
,2

minimal prime ideals of B. But como A é reduced e B é étale sobre A, concluı́mos que B é also reduced,
29

e portanto a = p ∩ q. This shows that Spec B has two irredutı́vel components, both isomorphic to the
normalisation of Spec A.
D
ct
,O

4 Morfismos suaves
ET

Lemma 4.1 Seja B be a anel, seja M be a finitamente gerado B-módulo, e N be a free B-módulo of
finite rank. Seja πj : N → B be the j-th projection com respect to a fixed base e1 , . . . , en of N . Seja
φ: M → N be a morphism, e q ∈ Spec B. The seguinte são equivalent:
1. φq : Mq → Nq has a section;
2. φ ⊗ 1: M ⊗B k(q) → N ⊗B k(q) é injective.
127

3. há elements ω1 , . . . , ωm ∈ M whose images generate Mq , e an m × m minor U of the matrix


πj ◦ φ(ωi ) 1≤i≤m com det U ∈ / q.
1≤j≤n

4. existe an element h ∈ B − q tal que φh : Mh → Nh has a section.

Prova Clearly 4 ⇒ 1 ⇒ 2, e 2 ⇒ 3 follows se we choose the ωi de modo que their images form a
minimal set of generators of the Bq -módulo Mq . To see that 3 ⇒ 4, choose h ∈ B − q de modo que the
images of the ωi generate Mh e det U é a unit in Bh . Without loss of generality, we may assume that
U = πj ◦ φ(ωi ) 1≤i≤m . Então we may define ψ: Nh → Mh por
1≤j≤m

   
ψ(e1 /1) ω1
 ..  .
 = U  .. 
−1
 . e ψ(ej /1) = 0 para j = m + 1, . . . , n
ψ(em /1) ωm

Então ψ ◦ φ(ωi ) = ωi para i = 1, . . . , m, e portanto ψ é a section of φh .

Teorema 4.2 (Jacobian criterion of smoothness) Seja A be a anel. Seja B be a finitely presented
A-álgebra, say B = R/b, onde R é the polinômio anel A[x1 , . . . , xn ], e b é a finitamente gerado ideal of
R. Seja Q ∈ Spec R be a prime ideal containing b, seja q be its image in B, e p = Q ∩ A. The seguinte
são equivalent:
1. Bq é formally smooth sobre Ap .
2. The morphism
b
⊗Bq k(q) → ΩRq /Ap ⊗Rq k(q)
b2 q
é injective.
3. há polinômios g1 , . . . , gm ∈ R whose the images generate (b/b2 )q , e an m × m minor U of the
Jacobian matrix  
∂(g1 , . . . , gm ) ∂gi
=
∂(x1 , . . . , xn ) ∂xj 1≤i≤m
1≤j≤n

tal que det U ∈


/ Q.
4. existe an element h ∈ B − q tal que Bh é smooth sobre A.
L
Prova Como ΩR/A = 1≤j≤n Rdxj é free over R, so é N = ΩR/A ⊗R B sobre B; moreover the fact that
FT
b é finitamente gerado sobre R implies that N = b/b2 é also finitamente gerado sobre B. Next observe
that R é formally smooth sobre A, e portanto, para qualquer Q ∈ Spec R e qualquer h ∈ R, RQ e Rh
são formally smooth sobre Ap e A, respectively. Portanto we são in position to apply the differential
)
28

criterion of smoothness. Seja δ: N → M be defined por δ(b mod b2 ) = db ⊗ 1 para todo b ∈ b. Então se
g é a polinômio in b,
7:

X ∂g
(1

δ(g mod b2 ) = dg ⊗ 1 =
RA

dxj
∂xj
1≤j≤n
0
01

Now 1 e 4 can be rephrased in terms of finding sections to δq : Mq → Nq e δh : Mh → Nh , e we can apply


the lemma to our situation to obtain the desired equivalences.
,2
29

Teorema 4.3 Seja k be a perfect corpo, e A be a k-álgebra of finite type. Então A é smooth sobre k se,
D
ct

e só se, A é regular.


,O

Prova We may write A = R/a onde R = k[x1 , . . . , xn ] e a é a finitamente gerado ideal of R. Seja
Q ∈ Spec R be a prime ideal containing a, e seja q be its image in A. It é enough to mostrar que Aq é
ET

formally smooth sobre k se, e só se, Aq é a regular local anel.


Suponha first that Aq é formally smooth sobre k; after renumbering the variables, the Jacobian
∂gi

criterion shows that há polinômios g1 , . . . , gm ∈ R whose images generate aq /a2q e det ∂x j 1≤i,j≤m

/ Q.
We claim that the images of the gi in QRQ ⊗RQ k(Q) = QRQ /(QRQ ) são linearly independent over
2

k(Q), e portanto they são part of a system of parameters of RQ . Então, como RQ é a regular anel,
128 Aspectos Diferenciais de Anéis

RQ /aQ = Aq will also be regular. To prove the claim, suponha that há elements a1 , . . . , am ∈ R tal que
a1 g1 + · · · + am gm ∈ (QRQ )2 . Então, para j = 1, . . . , m,
∂ ∂g1 ∂gm
(a1 g1 + · · · + am gm ) ∈ QRQ ⇒ a1 + · · · + am ∈ QRQ
∂xj ∂xj ∂xj
∂gi

But como det ∂x j
é a unit in RQ , concluı́mos que ai ∈ QRQ para todo i. This finishes the proof of the
claim, e of the first implication of the theorem.
Now suponha that Aq é regular. temos a sequence
aQ λ
- QRQ ⊗R k(Q) - ΩR /k ⊗R k(Q)
µ
⊗RQ k(Q)
a2Q Q Q Q

Como both RQ e Aq = RQ /aQ são regular, a minimal set of generators of aQ must be part of a system
of parameters of RQ , e portanto λ must be injective. por outro lado, como k é perfect, k(Q) é separably
generated sobre k, e portanto k(Q) é formally smooth sobre k. Por the Jacobian criterion, we conclude
então that µ must be injective as well. But então µ ◦ λ é injective, e portanto again por the Jacobian
criterion Aq é smooth sobre k.

Corolário 4.4 Seja k be qualquer corpo, e A be a k-álgebra of finite type. Então A é smooth sobre k se,
e só se, A é geometrically regular.
Prova Seja k alg be the algébrico closure of k. Como k alg é perfect, it é enough to mostrar que A ⊗k k alg
é smooth sobre k alg se, e só se, A é smooth over k. One direction é easy: por base change, se A é smooth
over k, então A ⊗k k alg é smooth sobre k alg .
Now suponha that A ⊗k k alg é smooth sobre k. Write A = R/a onde R = k[x1 , . . . , xn ] e a é a
finitamente gerado ideal of R. Seja Q ∈ Spec R be a prime ideal containing a, e seja q be its image
in A. We denote por a prime the result of tensoring com k alg sobre k; por exemplo A′ = A ⊗k k,
R′ = k alg [x1 , . . . , xn ], e so on. Using the Jacobian criterion e the fact that k alg é faithfully flat over k,
temos que A é smooth sobre k at q se e só se
a   
Q alg
⊗ R k(Q) ⊗ k k → Ω R /k ⊗ R k(Q) ⊗k k alg
a2Q Q Q Q

é injective, i.e., se, e só se,


a′Q
⊗R′Q k(Q)′ → ΩR′Q /kalg ⊗R′Q k(Q)′ (∗)
a′2
Q

é injective. But como A′ = R′ /a′ , again por flatness, e A′ é smooth sobre k alg , por the Jacobian criterion
FT
temos that a′ /a′2 → ΩR′ /kalg ⊗R′ A′ has a section. Localising com respect to the multiplicative set R − Q
e tensoring com k(Q)′ sobre RQ ′
shows that (∗) é injective, as required.
)
28

Teorema 4.5 Seja f : X → Y be a morphism locally of finite presentation. Seja x ∈ X e y = f (x).


7:

Então f é smooth at x se, e só se, there exist afim open neighbourhoods U = Spec B of x e V = Spec A
(1
RA

of y com f (U ) ⊂ V tal que the morphism of anel A → B corresponding to restriction U → V of f factors


0

as
01

A → A[x1 , . . . , xn ] → B
,2

com B étale sobre A[x1 , . . . , xn ].


29

Prova Clearly se there exist afim open neighbourhoods U = Spec B e V = Spec A as above, então f
D

é smooth at x como B é smooth sobre A, being the composition of two smooth morphisms. On the
ct

other hand, suponha that f é smooth at x. Seja (A, p, k) be the local anel OY,y . Como f é of finite
,O

presentation, OX,x é the localisation of algum finitely presented A-álgebra B at a prime ideal q ∈ Spec B
lying sobre p, e it é enough to mostrar que A → Bq factors as above. Write B = R/a para algum
ET

polinômio álgebra R = A[x1 , . . . , xN ] e algum finitely generated ideal a of R, e seja Q ∈ Spec R be a


prime ideal containing a whose image in B é q; temos Q ∩ A = p.
Por the Jacobian criterion,
aQ
δ: ⊗RQ k(q) → ΩRQ /A ⊗RQ k(q)
a2Q
129

∂gi

é injective, e há polinômios g1 , . . . , gm ∈ R whose images generate aQ e det ∂x j 1≤i,j≤m

/ Q. Without
loss of generality we may assume that x1 , . . . , xn são tal que dx1 ⊗ 1, . . . , dxn ⊗ 1 generate the cokernel
of δ. Put S = A[xm+1 , . . . , xN ], Q′ = Q ∩ S, e think of gi as elements of S[x1 , . . . , xm ]. Então

RQ S[x1 , . . . , xm ]Q
Bq = =
aQ (g1 , . . . , gm )
∂gi

e, moreover, det ∂xj 1≤i,j≤m ∈
/ Q. Portanto Bq é standard étale sobre S, as required.

Teorema 4.6 Seja f : X → Y be a morphism locally of finite presentation. Então the seguinte são
equivalent:
1. f é smooth.
2. f é flat e geometrically regular.

Prova Se f é smooth então por the the last theorem f é locally the composition of two flat morphisms,
e por base change f −1 (y) é smooth sobre Spec k(y) para todo y ∈ Y , which é equivalent to f −1 (y) being
geometrically regular. Reciprocamente suponha that 2 holds. Então por the corollary, f −1 (y) é smooth
sobre Spec k(y). Então, como B é flat sobre A,

0 → b ⊗A k(y) → R ⊗A k(y) → B ⊗A k(y) → 0

é exact. Então the seguinte mapa é injective

b ⊗A k(y)
⊗Rq ⊗A k(y) k(q) → ΩRq ⊗A k(y)/k(y) ⊗Rq ⊗A k(y) k(q)
(b ⊗A k(y))2

which é equivalent to
bq
→ ΩRq /Ap ⊗Rq k(q)
qbq
Portanto B é smooth sobre A.

5 Anéis Henselianos
Seja (A, m, k) be a local anel, e B be a finite A-álgebra. Se n é a maximal ideal of B, então B/n é a corpo
which é finite sobre the domain A/(n ∩ A). Portanto this domain é actually a corpo, e n ∩ A = m, i.e.,
the maximal ideals of B são exactly the maximal ideals in the fibre of m. But como B ⊗A k = B/mB é
FT
finite sobre k, B/mB é artinian, e portanto Spec B/mB é a finite set, e todo its elements são maximal
ideals. Portanto B has só finitely many maximal ideals, say n1 , . . . , nn . temos a diagrama comutativo
Y
)
-
28

B⊂ Bni
7:

1≤i≤n
(1
RA

? ?
?
?
0

Y
-∼
01

B/mB Bni /mBni


,2

1≤i≤n
29

Como B/mB é artinian, it é isomorphic to the product of its localisations com respect to its maximal
ideals, e portanto the bottom flecha é an isomorphism. por outro lado, the top flecha é always injective:
D
ct

se b é in the kernel, então ann(b) 6⊂ ni como b/1 = 0 in Bni , so ann(b) é not contained in qualquer
,O

maximal ideal of B, i.e., ann(b) = B ⇐⇒ b = 0.


We will be interested in the case when the top flecha é also an isomorphism, i.e., B é a product
ET

of local anéis. We now claim that this é the case se, e só se, todo idempotent of B/mB lifts to an
idempotent of B. In fact, it é clear that a product of local anéis has the stated idempotent lifting
propriedade. Reciprocamente, suponha that the idempotent

Bn1 Bnn
e1 = (1, 0, 0, . . . , 0) ∈ × ··· ×
mBn1 mBnn
130 Aspectos Diferenciais de Anéis

Q
lifts to an idempotent e1 of B. Então e1 maps to an element (a1 , . . . , an ) ∈ 1≤i≤n Bni , onde ai é an
idempotent of Bni ; but como the latter é a local anel, ai = 0 or ai = 1. Portanto e1 must mapa to
(1, 0, . . . , 0). This argument applies to the other idempotents as well, showing that the top flecha in the
diagram é also surjective in this case, e B é isomorphic to a product of local anéis, as claimed.
Now we consider the special case B = A[x]/f (x) onde f ∈ A[x] é monic. Denote por f the image
of f in k[x] = A[x]/mA[x]. Seja f = f 1 . . . f n be the factorisation of f into powers f i = pei i of distinct
monic irredutı́vel polinômios pi ∈ k[x]; in particular, temos (f i , f j ) = k[x] para i 6= j. Se pi ∈ A[x] é
a lift of pi , we can write the maximal ideals of B as ni = (m, pi ). Now we claim that B é a product of
local anéis se, e só se, the factorisation f = f 1 . . . f n lifts into a factorisation f = f1 . . . fn , onde cada
fi ∈ A[x] é monic e fi lifts f i .
In fact, suponha first that the factorisation of f = f 1 . . . f n lifts into a factorisation of f = f1 . . . fn
itself. Então (fi , fj ) = A[x] para i 6= j: como the fi são monic, M = A[x]/(fi , fj ) é a finite A-módulo
com M ⊗A k = k[x]/(f i , f j ) = 0, e portanto por Nakayama’s lemma M = 0. Por the Chinese remainder
theorem,
A[x] ∼ A[x] A[x]
B= = × ···×
f (x) f1 (x) fn (x)

e cada factor A[x]/fi (x) é finite sobre A; moreover, como A[x]/fi (x) ⊗A k = k[x]/f i (x) = k[x]/pei i has
a single prime ideal pi (x) mod fi (x), concluı́mos que cada factor A[x]/fi (x) é a local anel, e so temos
written B as a product of local anéis.
Reciprocamente, suponha that B = Bn1 × · · · × Bnn com ni = (m, pi ). Seja di be the grau of f i ;
como the images of 1, x, . . . , xdi −1 in Bni /mBni = k[x]/f i form a base sobre k, por Nakayama’s lemma
they generate the finite A-álgebra Bni , e portanto we can find a monic polinômio fi of grau di which lifts
f i e tal que fi (x) = 0 in Bni . Portanto the product f1 . . . fn é 0 in B = A[x]/f , i.e., f divides f1 . . . fn ,
but como both polinômios são monic e of the same grau, we must have f = f1 . . . fn , as required.

Teorema 5.1 Seja (A, m, k) be a local anel. The seguinte condições on A são equivalent:
1. Hensel’s lemma holds para A: se f ∈ A[x] é a monic polinômio whose image f factors as f = gh
com g, h ∈ k[x] monic e relatively prime, então there exist monic polinômios g, h ∈ A[x] tal que
f = gh lifts the factorisation f = gh.
2. Qualquer finite A-módulo B é a product of local anéis.

Prova Por the discussion above, it é clear that 2 ⇒ 1 se we choose B = A[x]/f ; reciprocamente, se
1 holds, então we know that 2 holds para finite A-álgebras of the form B = A[x]/f , onde f é a monic
polinômio.
FT
Seja n1 , . . . , nn be the maximal ideals of B. Para the general case, we must mostrar que qualquer
idempotent of B/mB lifts to an idempotent of B, e it é actually enough to show that the idempotent e
)
of B/mB corresponding to (1, 0, . . . , 0) ∈ Bn1 /mBn1 × · · · × Bnn /mBnn can be lifted to an idempotent
28

e of B. Choose qualquer lift e′ ∈ B of e, e seja B ′ = A[e′ ]. Como B é finite sobre A, existe a monic
7:

polinômio f ∈ A[x] tal que f (e′ ) = 0. Set B ′′ = A[x]/f ; temos a surjective morphism of A-álgebras
φ: B ′′ ։ B ′ sending x mod f to e′ .
(1
RA

Como B ′ e B ′′ são finite sobre A, the maximal ideals of B ′ e B ′′ são the images of n1 , . . . , nn
0

under Spec B → Spec B ′ e Spec B → Spec B ′′ , which we denote por n′1 , . . . , n′n e n′′1 , . . . , n′′n , respectively.
01

These ideals são not necessarily pairwise distinct, however por the choice of e′ temos e′ ∈ ni para
,2

todo i = 2, . . . , n while e′ ∈ / n′1 , e portanto n1 é the só prime of B lying sobre n′1 ; por outro lado,
′ ′′
Spec B → Spec B é a closed immersion, e in particular, é injective. Por the special case above, we
29

know that B ′′ é a product of local anéis; seja e′′ be the idempotent which corresponds to the element
D

com coordinate 1 in Bn′′′′ e 0 in the other components. Então e = φ(e′′ ) é an idempotent of B ′ , e portanto
ct

1
,O

of B, e e ∈ n2 , . . . , nn ondeas e ∈
/ n1 . Portanto e lifts e, e we são done.
ET

Definição 5.2 A local anel A é henselian se it satisfies the condições of the theorem above.

Definição 5.3 Seja (A, m, k) be a local anel. A local A-álgebra B é called a pontoed étale extensão
of A se B é the localisation of an étale A-álgebra C at a prime ideal q ∈ Spec C lying sobre m e tal que
k(q) = k. Notice that B é actually an extensão of A como it é faithfully flat sobre A.
131

Teorema 5.4 Seja (A, m, k) be a local anel. The seguinte condições on A são equivalent:
1. A é henselian;
2. Se f ∈ A[x] é a polinômio whose image f ∈ k[x] has a simple root α ∈ k, então f itself has a
root a ∈ A tal que α = a mod m.
3. Todo pontoed étale extensão of A é trivial.
Prova (1 ⇒ 2) Como f (x) = (x − α) · g(x) para algum g(x) ∈ k[x] com g(α) 6= 0, há monic polinômios
x − a e g(x) in A[x] lifting x − α e g(x), respectively, e such that f (x) = (x − a) · g(x), e 2 follows.
(2 ⇒ 3) Seja B be a pontoed étale extensão of A. Por the local structure theorem, we may assume that
B = A[x]q /f para algum monic polinômio f ∈ A[x], e a prime ideal q ∈ Spec A[x] containing f , tal que
q lies sobre m, has residue corpo k(q) = k, e tal que f ′ é a unit in B. Seja f be the image of f in k[x], e
p ∈ k[x] be the irredutı́vel factor of f corresponding to the prime q in the fibre Spec B ⊗A k = Spec k[x]/f
of m. Como k(q) = k, temos que p(x) = x − α para algum root α ∈ k of f . Como (f, f ′ ) = A[x]q , we

also have (f , f ) = k[x]x−α , so α é a simple root of f . Por hypothesis, f factors as f (x) = (x − a) · g(x),
com (x − a, g) = A[x]q . Portanto B é the localisation of A[x]/f at q = (m, x − a), e portanto B ∼ = A.
(3 ⇒ 1) Seja f (x) = xn + fn−1 xn−1 + · · · + f0 , fi ∈ A, be a monic polinômio whose image f (x) =
xn + f n−1 xn−1 + · · · + f 0 , f i = fi mod m, in k[x] factors as f = gh onde g, h ∈ k[x] são two relatively
prime monic polinômios. Write
g(x) = xr + gr−1 xr−1 + · · · + g 0 , gi ∈ k
s s−1
h(x) = x + hs−1 x + · · · + h0 , hi ∈ k
com n = r + s. Define polinômios F0 , . . . , Fn−1 in n variables y0 , y1 , . . . , yr−1 , z0 , z1 , . . . , zs−1 por
X
Fi (y0 , . . . , yr−1 , z0 , . . . , zs−1 ) = yi zi−j − fi , i = 0, 1, . . . , n − 1
0≤j≤i

Denote por F i the image of Fi in k[y0 , . . . , yr−1 , z0 , . . . , zs−1 ]. Como f = gh, temos que (g 0 , . . . , g r−1 , h0 , . . . , hs−1 ) =
0 é a solution to the system
F i (y0 , . . . , yr−1 , z0 , . . . , zs−1 ) = 0, i = 0, 1, . . . , n − 1 (∗)
e clearly the factorisation f = gh lifts to a factorisation of f se, e só se, the system
Fi (y0 , . . . , yr−1 , z0 , . . . , zs−1 ) = 0, i = 0, 1, . . . , n − 1 (∗∗)
has a solution (g0 , . . . , gr−1 , h0 , . . . , hs−1 ) in A lifting (g 0 , . . . , gr−1 , h0 , . . . , hs−1 ). This can be rephrased
as follows: define the A-álgebra
A[y0 , . . . , yr−1 , z0 , . . . , zs−1 ]
FT
B=
(F0 , . . . , Fn−1 )
)
Notice that the solution (g0 , . . . , g r−1 , h0 , . . . , hs−1 ) to (∗) corresponds to the maximal ideal
28

(y0 − g 0 , . . . , yr−1 − gr−1 , z0 − h0 , . . . , zs−1 − hs−1 )


7:

(F 0 , . . . , F n−1 )
(1
RA

in the fibre
0

k[y0 , . . . , yr−1 , z0 , . . . , zs−1 ]


01

Spec B ⊗A k = Spec
(F 0 , . . . , F n−1 )
,2

of m. Seja q be the corresponding ideal of B. Então an isomorphism Bq ∼ = A will correspond to the


desired solution to (∗∗), so it é enough to mostrar que Bq é a pontoed étale extensão of A. An explicit
29

calculation shows that the Jacobian determinant


D
ct

∂(F0 , . . . , Fn−1 )
,O

∂(y0 , . . . , yr−1 , z0 , . . . , zs−1 )


é the resultant of the two polinômios in the variable x
ET

xr + yr−1 xr−1 + · · · + y0
xs + zr−1 xs−1 + · · · + z0
E this determinant é invertible in Bq se, e só se, its image in Bq /qBq = k é not zero, i.e., se, e só se, the
resultant of g e h é not zero. But g e h são relatively prime por hipótese, e that does it.
132 Aspectos Diferenciais de Anéis

5.1 Henselização e Henselização Estrita


Setup 5.1.1 Seja A be a normal noetherian domain com corpo of fractions K = Frac A. Seja L be a
(possibly infinite) Galois extensão of K com G = Gal(L/K). We denote the integral closure of A in L
por B.
Observe that σ(B) = B para todo σ ∈ G, so B é a G-módulo, e that B G = A, como A é normal
e portanto B G = B ∩ K = A. A useful consequence of this fact é that se L/K é finite então the norm
from L to K defines a multiplicative mapa NL/K : B → A.
The Galois group G also permutes the prime ideals of B, turning Spec B into a G-set. Seja q ∈
Spec B. The stabiliser of q
Dq = {σ ∈ G | σq = q}
é called the decomposition group of q.
Now seja p = q ∩ A ∈ Spec A. Seja k e l the residue corpos of p e q, respectively. Como the inclusão
A ֒→ B induces an injection A/p ֒→ B/q, we may think of k as a subcorpo of l. temos a mapa

Dq → Aut(l/k)

sending σ ∈ Dq to the automorfismo σ ∈ Aut(l/k) dado por

σ(b mod q) = σ(b) mod q para todo b ∈ B

The kernel Iq of Dq → Aut(l/k) é called inertia group of q. Clearly Iq ⊳ Dq .


An important observation é that the above setup e definitions são compatible com localisation: se
S é qualquer multiplicative set of A, então S −1 B é the integral closure of S −1 A in L, e como S ⊂ K,
Spec S −1 B é a G-set e temos a commutative diagram of G-sets

Spec S −1 B ⊂ - Spec B

? ?
Spec S −1 A ⊂ - Spec A

Portanto se q ∈ Spec B does not intersect S então DS −1 q = Dq . Moreover, como q e S −1 q have the same
residue corpos, IS −1 q = Iq . Portanto in most proofs we will be able to localise com respect to S = A − p
FT
e assume that A é local com maximal ideal p, e that q é maximal in B.
Teorema 5.1.2 (Transitive action on primes) Assume the setup above, e seja p ∈ Spec A. The
)
28

group G acts transitively on the set of prime ideals of B lying sobre p.


7:

Corolário 5.1.3 Assume the setup above. Seja q ∈ Spec B, e seja M/K be a Galois subextensão of
(1
RA

L/K. Seja
BM = B ∩ M = integral closure of A in M
0
01

qM = q ∩ M = q ∩ BM ∈ Spec BM
,2

Então DqM é the image of Dq under the projection G = Gal(L/K) ։ Gal(M/K). Portanto
29

DqM
D

D = ←−
lim DqM e B D = −→
lim BM
ct

M M
,O

onde M/K runs sobre todo finite Galois subextensões of L/K.


ET

Prova The projection Gal(L/K) ։ Gal(M/K) maps σ ∈ Gal(L/K) to σ|M ∈ Gal(M/K). Clearly se
σ ∈ Dq então σ|M ∈ DqM , portanto the image of Dq é contained in DqM . Reciprocamente, se φ ∈ DqM ,
seja σ ∈ G be qualquer automorfismo extending φ. Então σ(q) ∩ M = σ(q ∩ M ) = φ(qM ) = qM , portanto
both q e σq lie sobre qM , e por the last theorem, existe τ ∈ Gal(L/M ) tal que τ σ(q) = q. Portanto
τ σ ∈ Dq é a pre-image of φ.
133

Para the next results, we work in the seguinte


Setup 5.1.4 Assume the previous setup. Fix a prime ideal q ∈ Spec B, e seja p = q ∩ A ∈ Spec A.
Denote the decomposition e inertia groups of q por D e I, respectively. Finally set
B ′ = B D = B ∩ LD = integral closure of A in LD
B ′′ = B I = B ∩ LI = integral closure of A in LI
q′ = q ∩ B ′ ∈ Spec B ′
q′′ = q ∩ B ′′ ∈ Spec B ′′

Lemma 5.1.5 Assume the above setup.


1. q é the unique prime of B lying sobre q′ .
2. The inclusão k(p) ֒→ k(q′ ) of residue corpos é an isomorfismo.
3. pBq′ ′ = q′ Bq′ ′ .

Prova 1. Como D = Gal(L/LD ) acts transitively on the primes of B lying sobre q′ e D stabilises q,
the result follows.
Para the next assertions, we may localise todo the anéis e ideals com respect to S = A − p e portanto
assume that (A, p, k) é local e that q e q′ são maximal in B e B ′ , respectively. We now mostrar que
the proof can be reduced to the case onde L/K é finite. Seja BM e qM as in the corollary, e define
′ Dq
BM = BM M , q′M = q ∩ BM ′ ′
e lM to be the residue corpo of q′M . Denote por l′ be the residue corpo of

q . Por the corollary, temos que
[ [ [
B′ = BM′
q′ = q′M l′ = ′
lM
M M M

onde M/K runs sobre todo finite Galois subextensões of L/K. Portanto se we can mostrar que lM =k
′ ′ ′ ′ ′ ′ ′
e that p(BM )q′M = qM (BM )q′M para todo M então we will have that l = k e that pBq′ = q Bq′ .
2. We need to mostrar que para todo b′ ∈ B ′ existe a ∈ A tal que a ≡ b′ (mod q). Seja σ1 , . . . , σg be
a system of representatives of left cosets of D com σ1 = 1. Por the last theorem, the primes of B lying
sobre p, which são the maximal ideals of B, são exactly σ1 q, . . . , σg q. Se i 6= 1 então σi−1 ∈
/ D e portanto
σi−1 q 6= q. Então (1) implies that (σi−1 q) ∩ B ′ 6= q′ , e portanto por the Chinese remainder theorem there
exists x ∈ B ′ tal que
 
x ≡ b′ (mod q′ ) x ≡ b′ (mod q)

x ≡ 1 (mod (σi−1 q) ∩ B ′ ) para i 6= 1 σi (x) ≡ 1 (mod q) para i 6= 1
Então
FT
Y
a = NLD /K (x) = σi (x) ≡ b′ (mod q)
1≤i≤g
)
28

é the required element.


7:

3. Como p ⊂ q′ , we só need to mostrar que q′ ⊂ pBq′ ′ . Seja q′1 = q′ , q′2 , . . . , q′s be the maximal ideals of
B ′ . Notice that the proof of (2) shows that qualquer x ∈ q′1 − (q′2 ∪ · · · ∪ q′s ) belongs to pBq′ ′ . In fact, se
(1
RA

Q
σ∈ / D então σ −1 q′ = q′i para algum i 6= 1; this implies that σ(x) ∈
/ q′ . Portanto 2≤i≤g σi (x) ∈ / q′ onde
0

σi são as in (2). Portanto from


01

Y
,2

x σi (x) = NLD /K (x) ∈ q ∩ A = p


2≤i≤g
29

temos que x ∈ pBq′ .


D
ct

Next, observe that como B ′ é finite sobre A, B ′ /pB ′ é finite sobre k e é portanto an artinian anel.
,O

Portanto B ′ /pB ′ é isomorphic to the product of its localisations com respect to its maximal ideals:

B′ Bq′ ′ Bq′ ′s
ET

= 1
× · · · ×
pB ′ pBq′ ′ pBq′ ′s
1

To prove that pBq′ ′ = q′ Bq′ ′ we must mostrar que the maximal ideal q′1 Bq′ ′ /pBq′ ′ of the first factor é
1 1
0. Seja t ∈ q′1 Bq′ ′ /pBq′ ′ e seja x ∈ B ′ be a lift of the element (t, 1, . . . , 1) in the product above. Então
1 1
x ∈ q′1 − (q′2 ∪ · · · ∪ q′s ) e por the above x ∈ pBq′ . But this implies t = 0, as required.
134 Aspectos Diferenciais de Anéis

Lemma 5.1.6 Assume the setup above.


1. q′′ é the unique prime ideal of B ′′ lying sobre q′ .
2. l/k é a normal corpo extensão, i.e., qualquer irredutı́vel polinômio f (x) ∈ k[x] which has a root
in l splits completely. Moreover the mapa D → Aut(l/k) é surjective.

Prova 1. This follows from (1) of the last lemma e the fact that B é an integral extensão of B ′′ e
portanto Spec B → Spec B ′′ é surjective.
To prove the remaining assertions, observe that por (2) e (3) of the last lemma p e q′ have the same
residue corpos e that pBq′ ′ = q′ Bq′ ′ . Portanto we may replace A por Bq′ ′ e portanto assume that (A, p, k)
é a normal local domain, that D = G, e portanto por (1) of the previous lemma that the integral closure
(B, q, l) of A in L é local. Por (1) above, (B ′′ , q′′ , l′′ ) é also local.
Now we introduce algum notation. Para qualquer b ∈ B, write b ∈ l para b mod q. Seja mb (x) ∈ K[x]
be the minimal polinômio of b; notice that actually mb (x) ∈ A[x] como A é normal e b é integral sobre
A. We also write mb (x) ∈ l[x] para the image of mb (x) under the mapa A[x] → l[x] induced por
A → A/p = k.
2. We first mostrar que l/k é a normal extensão. Seja f (x) ∈ k[x] be an irredutı́vel polinômio e suponha
that it has a root b ∈ l, b ∈ B. Então f (x) é the minimal polinômio of b, e como we also have mb (b) = 0,
concluı́mos que f (x) | mb (x). But como L/K é Galois e mb (x) has a root b ∈ L, mb (x) splits completely
in B[x]. Portanto mb (x) also splits completely in l[x], e portanto so does f (x).
Next we mostrar que D → Aut(l/k) é surjective. Seja ks /k be the maximal separable subextensão
of l/k, e observe that Aut(l/k) = Gal(ks /k), portanto temos to mostrar que D → Gal(ks /k) é surjective.
First we consider the case that ks /k é finite, de modo que por the primitive element theorem we
can write ks = k(b) para algum b ∈ B. Seja f (x) ∈ k[x] be the minimal polinômio of b. Então an

automorfismo φ ∈ Gal(ks /k) é completely determined por the value of φ(b) = b , also a root of f (x),
e como f (x) | mb (x), we may take b′ to be a root of mb (x). But como mb (x) é irredutı́vel sobre K,

existe σ ∈ Gal(L/K) tal que σ(b) = b′ , e portanto σ(b) = b = φ(b), i.e., σ = φ, which shows that
D → Gal(ks /k) é surjective when ks /k é finite.
Para the general case, seja φ ∈ Gal(ks /k), e para cada finite Galois subextensão k ′ /k of ks /k define

T (k ′ ) = {σ ∈ D | σ|k′ = φ|k′ }

In other words, T (k ′ ) é the pre-image under D → Gal(ks /k) of the closed subset in the Krull topology
of Gal(ks /k) consisting of todo automorfismos agreeing com φ on k ′ . But D → Gal(ks /k) é continuous,
portanto T (k ′ ) é closed, e por
T the special case above, T (k ′ ) 6= ∅. Como D é compact, as in the proof
FT
of teorema 5.1.2, temos que k′ T T (k ) 6= ∅, onde k ′ /k runs sobre todo finite Galois subextensão of ks /k.

Então σ = φ para qualquer σ ∈ k′ T (k ′ ), which finishes the proof.


)
28
7:

Corolário 5.1.7 Assume the setup above. Seja q ∈ Spec B, e seja M/K be a Galois subextensão of
L/K. Seja
(1
RA

BM = B ∩ M = integral closure of A in M
0
01

qM = q ∩ M = q ∩ BM ∈ Spec BM
,2

Então IqM é the image of Iq under the projection G = Gal(L/K) ։ Gal(M/K). Portanto
29

Iq
I = ←−
lim IqM e B I = −→
lim BMM
D
ct

M M
,O

onde M/K runs sobre todo finite Galois subextensões of L/K.


ET

Prova Seja lM be the residue corpo of qM . Se σ ∈ I então σ|lM = 1 e portanto the image of I é
contained in IqM . Reciprocamente, dado φ ∈ IqM ⊂ DqM , por corolário 5.1.3 existe σ ∈ D tal que
σ|M = φ. E como σ|lM = 1, σ ∈ Gal(l/lM ) e por (2) of the last lemma existe τ ∈ DqM ⊂ Gal(L/M ) tal
que τ = σ. Então τ −1 σ ∈ I é a pre-image of φ.
135

Lemma 5.1.8 Assume the setup above. Seja l′′ be the residue corpo of q′′ . Então l′′ /k é the maximal
separable subextensão of l/k. Furthermore pBq′′′′ = q′′ Bq′′′′ . Portanto se B ′′ é of finite presentation sobre
A, B ′′ é unramified sobre A at q′′ .
Prova The fact that l/l′′ é purely inseparable follows from (2) of the last lemma applied to B ′′ in place
of A: temos que l/l′′ é a normal extensão, e como D = I in this case, that I ։ Aut(l/l′′ ) é surjective.
But como I é the kernel of D → Aut(l/l′′ ), this implies that Aut(l/l′′ ) é trivial, e portanto that l/l′′ é
purely inseparable.
We são left to mostrar que l′′ /k é separable e that pB ′′ = q′′ . Notice that como I ⊳ D, LI /K
é Galois e portanto in order to prove the two assertions above, we may replace L por LI e (B, q, l)
por (B ′′ , q′′ , l′′ ). In other words, we now assume that I = 1 e G = D, so that D → Aut(l/k) é an
isomorfismo. Furthermore we may reduce the proof to the case onde L/K é finite. Indeed, por the last
Iq Iq
corollary, temos que (B ′′ , q′′ , l′′ ) é the union of the (BMM , qM ∩ BMM , lM
′′
) as M/K runs sobre todo finite
′′ Iq
Galois subextensões of L/K, onde lM denotes the residue corpo of qM ∩ BMM . Portanto se we mostrar
′′ I I
/k é separable e that pBMM = qM ∩ BMM para todo M então we will have that l′′ /k é separable
q q
que lM
e that pB ′′ = q.
Seja b be tal que k(b)/k é the maximal separable subextensão of l/k. We são going to mostrar que
mb (x) ∈ k[x] é the minimal polinômio of b e that the mapa A[x]/mb (x) → B dado por x 7→ b é an
isomorfismo. Então l = k(b) will be separable sobre k e B will be standard étale sobre A, proving that
pB = q.
Seja f (x) ∈ k[x] be the minimal polinômio of b. To prove that mb (x) = f (x), notice that como
f (x) | mb (x) e G = D → Aut(l/k) é an isomorfismo por hipótese, temos que

deg mb (x) = deg mb (x) = [K(b) : K] ≤ [L : K] = |G| = | Aut(l/k)|


= [k(b) : k] = deg f (x) ≤ deg mb (x)

We must portanto have equality everywhere, e portanto mb (x) = f (x) e L = K(b). Portanto A[b] ∼ =
A[x]/mb (x) é standard étale sobre A com Frac A[b] = B. Como A é normal, so é A[b]. However A[b] ⊂ B
e both anéis são normal e have the same corpo of fractions, so we must have A[b] = B, as required.

5.2 Henselisation e Strict Henselisation: existence


See [Nagata], VI.41.2, p. 159, e [Milne], I.3.20, p. 29.
Lemma 5.2.1 Seja (A, p, k) be a local anel, seja (B, q, l) be an étale local A-álgebra e seja (C, r, m) be
a henselian local A-álgebra. temos an isomorfismo
FT
Hom loc A-alg (B, C)

- Hom k-alg (l, m)
)
28

that takes a local morfismo φ: B → C of A-álgebras to the induced morfismo φ: l → m of residue corpos.
7:

Prova First we mostrar que se φ, ψ ∈ Hom loc A-alg (B, C) induce the same morfismos on the residue
(1
RA

corpos então φ = ψ. Para that we will just use the fact that B é unramified sobre A. Set S = Spec A,
X = Spec C, Y = Spec B, e seja f e g be the two S-morfismos from X to Y corresponding to φ e ψ. We
0
01

have to mostrar que f = g. Consider the seguinte cartesian diagram


,2

Z = Y ×(Y ×S Y ) X ⊂ - Spec X
29
D

f ×g
ct
,O

? ∆Y /S ?
Y ⊂ - Y ×S Y
ET

In other words, Z é the largest closed subesquema of X on which f e g coincide. Como Y é unramified
sobre S, ∆Y /S é both an open e a closed immersion, e portanto Z é also an open subesquema of X. But
X é connected, being the spectrum of a local anel, portanto Z = X or Z = ∅. But the fact that φ e ψ
induce the same morfismos on the residue corpos shows that Z(Spec m) 6= ∅, so we must have Z = X,
i.e., f = g.
136 Aspectos Diferenciais de Anéis

Now suponha that we são dado φ ∈ Hom k-alg (l, m). Por the local structure theorem, we can write B
as the localisation of R = A[x]/f (x) at algum maximal ideal Q. But the maximal ideals of A correspond
to the elements of Spec R ⊗A k = k[x]/f (x), i.e., to the prime factors of f (x). But como the image of
f ′ (x) in B = RQ we can write f (x) = f 1 (x) · f 2 (x) onde f 1 (x) é a separable irredutı́vel polinômio in
k[x] e f 2 (x) é relatively prime to f 1 (x). Moreover l ∼
= k[x]/f 1 (x). Now write β = x mod f (x) ∈ B e
β = x mod f 1 (x). Então φ takes β to a root r ∈ m of f 1 (x). But como C é strictly henselian, r lifts to
a root r ∈ C of f (x). Now we define φ(β) = r, e it é clear that φ induces φ on the residue corpos.

Teorema 5.2.2 Seja (A, p, k) be a normal local domain com corpo of fractions K = Frac A. Seja Ks be
the separable closure of K e seja B be the integral closure of A in Ks . Choose a maximal ideal q ∈ Spec B
e seja D e I be its decomposition e inertia groups respectively. Seja

B′ = BD q′ = q ∩ B D B ′′ = B I q′′ = q ∩ B I

Então Bq′ ′ é a henselisation of A onde the local mapa from A to Bq′ ′ é the inclusão A ⊂ Bq′ ′ . Similarly
Bq′′′′ é a strict henselisation of A.
Prova The universal propriedade follows from corolário 5.1.7 e the previous lemma.

Seja us just sketch the general construction of the strict henselisation of an arbitrary local anel
(A, p, k). We say that an A-álgebra B é essentially étale sobre A se B = Cq para algum étale A-
álgebra C e algum q ∈ Spec C lying sobre p. Como the álgebras C são of finite presentation sobre
A, it é easy to see that existe a set E of essentially étale A-álgebras containing one representative per
isomorfismo class. Fix a separable closure ks of k. Now we consider the set N whose elements são pairs
(B, λ) com B ∈ E e λ: B → ks é A-linear com ker λ equal to the maximal ideal of B. We have portanto
a diagrama comutativo

λ
ks  B
6


onde A → ks é the composition of A → k ֒→ ks . Dado two elements (B, λ) e (C, µ) of N , a morfismo


from (B, λ) e (C, µ) é a morfismo φ: B → C such that λ = µ ◦ φ. That makes N into a category, e it é
FT
easy to check that this category é filtred. Portanto we may take the limit
)
lim B
−→
28

(B,λ)
7:

It é not difficult to mostrar que this limit has the required universal propriedade making it into a strict
(1
RA

henselisation of A. Para the henselisation, one restricts the limit to só those étale álgebras that have the
0

same residue corpo as A.


01

6 Exercı́cios
,2
29
D
ct
,O
ET
Chapter 8
esquema
morfismo de r
pré-feixe

Esquemas

Neste capı́tulo, formalizaremos a filosofia de que anéis são objetos geométricos, que estivemos implicita-
mente utilizando até agora. Veremos o importante conceito de esquema, introduzido por Grothendieck,
que nada mais é do que um espaço “localmente isomorfo a um anel”.

1 Espaços Localmente Anulares como Objetos Geométricos Gerais


Basicamente, dar um objeto geométrico consiste em dar não só um “conjunto de pontos” (ou seja, um
espaço topológico) mas também dizer quais os tipos de “funções admissı́veis” estamos considerando sobre
estes conjuntos. Por exemplo, em variedades topológicas, todas as funções contı́nuas são permitidas,
enquanto que em variedades analı́ticas apenas funções holomorfas são consideradas.

1.1 Feixes
Em geral, não estamos só interessados em “funções globalmente defindas”, é preciso também poder dizer
quais as funções admissı́veis em abertos deste espaço. Por exemplo, se estamos trabalhando com var-
iedades analı́ticas compactas conexas, pelo teorema de Liouville as únicas funções holomorfas globalmente
definidas são apenas as funções constantes, o que acaba não dizendo muito sobre a variedade.
O conceito de feixe, que nada mais é do que uma notação para organizar a interação local/global,
surgiu em resposta à situação acima. Antes de dar a definição formal, vejamos um exemplo. Seja X um
espaço topológico qualquer e, para cada aberto U ⊂ X, defina o anel das funções contı́nuas reais em U :

F (U ) = {f : U → R | f é contı́nua}

Estes anéis não são todos independentes entre si: se U ⊃ V é uma inclusão de abertos, temos um
morfismo de restrição
resUV : F (U ) → F (V )
f 7→ f |V
FT
Estes morfismos de restrição
S satisfazem a seguinte propriedade de “cola”: dado um aberto U , uma
cobertura aberta U = i∈I Ui de U e funções fi ∈ F (Ui ) concordando nas intersecções, i.e.,
)
28

fi |Ui ∩Uj = resUi ,Ui ∩Uj (fi ) = resUj ,Ui ∩Uj (fj ) = fj |Ui ∩Uj para todo i, j ∈ I
7:
(1
RA

existe uma única função f ∈ F (U ) tal que f |Ui = resUUi (f ) = fi para cada i ∈ I. De fato, se x ∈ U
0

pertence a Ui , basta definir f (x) = fi (x), o que independe da escolha do aberto Ui que contém x pelo
01

fato de os fi ’s concordarem nas intersecções.


Vamos agora axiomatizar as propriedades essenciais do exemplo anterior. Um espaço topológico X
,2

define uma categoria O(X) dos seus abertos: os objetos desta categoria são todos os abertos de X e as
29

flechas são dadas pelas inclusões:


D


ct

def {U → V } se U ⊂ V
Hom(U, V ) =
,O

∅ caso contrário
ET

Definição 1.1.1 Seja X um espaço topológico. Um pré-feixe F : O(X)◦ → Ab de grupos abelianos em


X é um funtor contravariante de O(X) para a categoria dos grupos abelianos Ab. Explicitamente, temos
1. para cada aberto U de X, um grupo abeliano F (U );
2. para cada inclusão de abertos U ⊃ V , um morfismo de grupos abelianos resUV : F (U ) → F (V ),
chamado de restrição;
138 Esquemas

tais que resUU = id para todo aberto U e resV W ◦ resUV = resUW para todas as inclusões de abertos morfismo de pré-feixe
seções
U ⊃ V ⊃ W.
Se F , G: O(X)◦ → Ab são dois pré-feixes, um morfismo de pré-feixes φ: F → G é um morfismo de
funtores entre F e G. Explicitamente, para cada aberto U temos um morfismo de grupos φU : F (U ) →
G(U ) de modo que se U ⊃ V é uma inclusão de abertos, o diagrama

φU-
F (U ) G(U )

resF
UV resGUV
? φV - ?
F (V ) G(V )

comuta. Denotamos por PSh(X) a categoria de pré-feixes de grupos abelianos sobre X. Pré-feixes de
anéis, conjuntos, etc. são definidos analogamente, substituindo a categoria de grupos abelianos pela
categoria correspondente.

Observação 1.1.2 Alguns comentários acerca da notação e terminologia. Por questões de conveniência,
em deferência ao exemplo inicial denotamos muitas vezes resUV (f ) ∈ F (V ) simplesmente por f |V . Os
elementos de F (V ) são chamados de seções de F sobre V (esta terminologia tem origem nos feixes
de seções de fibrados vetoriais). O grupo F (V ) é às vezes escrito Γ(V, F ) (seções “Γlobais” sobre V ),
especialmente se desejamos enfatizar o caráter funtorial da associação F 7→ Γ(V, F ).

Definição 1.1.3 Um pré-feixo F : O(X)◦ → Ab de grupos abelianos em um espaço topológico X é


chamado de feixe se ele satisfaz o “axioma de cola”: para cada aberto U e qualquer cobertura aberta
{Ui }i∈I de U , dados fi ∈ F (Ui ) elementos que “concordam nas intersecções”, i.e.,

resUi ,Ui ∩Uj (fi ) = resUj ,Ui ∩Uj (fj ) para todo i, j ∈ I

existe um único elemento f ∈ F (U ) tal que resUUi (f ) = fi para cada i ∈ I.


Um morfismo entre feixes é simplesmente um morfismo entre os pré-feixes subjacentes, ou seja, apenas
um morfismo de funtores. Denotamos por Sh(X) a categoria de feixes de grupos abelianos sobre X.
Analogamente, define-se feixes de anéis, conjuntos, etc.

Por conveniência (deste autor, é claro!), daremos todas as definições/resultados para feixes de grupos
abelianos, deixando para o leitor fazer as modificações necessárias para outras categorias.
FT
Observação 1.1.4 Para um feixe de grupos abelianos F : O(X)◦ → Ab, temos que a unicidade do
axioma de cola pode ser expressa dizendo-se que o produto dos mapas de restrição
)
28

Y
F (U ) → F (Ui )
7:

i∈I
(1
RA
0

é injetor, i.e., que seu kernel é trivial. Em particular, se U = ∅, temos que U admite uma cobertura
01

vazia (i.e., com I = ∅) e portanto temos que F (∅) = 0 já que o produto vazio é o grupo trivial. Aqueles
que acharem este tipo de raciocı́nio deveras bizarro estão convidados a incluir a condição F (∅) = 0 na
,2

definição acima.
29

Observação 1.1.5 Simbolicamente, o fato de F satisfazer o axioma de cola é às vezes expresso dizendo-
D
ct

se que a sequência
Y Y
,O

F (U ) → F (Ui ) ⇉ F (Ui ∩ Uj )
i∈I (i,j)∈I×I
ET

é exata. Aqui a primeira flecha é o produto dos mapas de restrição enquanto que as flechas duplas
denotam o produto das restrições com relação à “primeira” e à “segunda” inclusões Ui ∩ Uj ⊂ Ui e
Ui ∩ Uj ⊂ Uj . O fato da sequência acima ser exata deve ser interpretada do seguinte modo: aQ imagem de
F (U ) é precisamente o “kernel dos equalizadores” (as flechas duplas), ou seja, as tuplas em i∈I F (Ui )
que concordam nas intersecções.
139

Exemplo 1.1.6 (Feixe Constante) Seja X um espaço topológico qualquer e A um grupo abeliano, feixe constant
feixe kernel
munido da topologia discreta. Para cada aberto U , defina talo

A(U ) = {f : U → A | f é contı́nua} = Aπ0 (U)

com os mapas de restrição usuais. Aqui π0 (U ) é o conjunto das componentes conexas de U . Temos que
A é um feixe, chamado feixe constante com valores em A. Note que não bastaria definir A(U ) = A
para todo aberto U , pois o axioma de cola não seria satisfeito: por exemplo, se U consiste em duas
componentes conexas disjuntas U0 e U1 , devemos ter F (U ) = F (U0 ) ⊕ F (U1 ) para qualquer feixe F (pois
F (U0 ∩ U1 ) = F (∅) = 0).
Exemplo 1.1.7 O exemplo do feixe de funções contı́nuas reais pode ser facilmente generalizado. Por
exemplo, seja X = C e considere, para cada aberto U ,

def
H(U ) = {f : U → C | f é holomorfo}

com os mapas de restrição usuais (para U = ∅, a definição acima deve ser interpretada como H(∅) = 0;
de agora em diante não mencionaremos mais este detalhe). Temos que H define um feixe de anéis sobre
X = C, o feixe de funções holomorfas.
Exemplo 1.1.8 Tomando os grupos de unidades no exemplo anterior, temos que U 7→ H(U )× (funções
holomorfas que não se anulam em nenhum ponto de U ) define um feixe de grupos abelianos (com os
mapas de restrição usuais) que denotamos por H× .
Exemplo 1.1.9 A função exponencial define um morfismo de feixes de grupos abelianos em X = C:

expU : H(U ) → H× (U )
f 7→ ef

Exemplo 1.1.10 (O Feixe Kernel) Seja φ: F → G um morfismo de feixes de grupos abelianos. Para
cada U , defina
K(U ) = ker(F (U ) - G(U ))
φU

onde os mapas de restrição são induzidos pelos de F , i.e., temos um diagrama comutativo com linhas
exatas

- K(U ) - F (U ) φU-
0 G(U )
FT
resK resF resGUV
)
UV UV
28

? ? φV - ?
- K(U ) - F (V )
7:

0 G(V )
(1
RA

O axioma de cola de K é “herdado” do de F , como é fácil verificar. O feixe K é chamado de feixe kernel
0

de φ e é simplesmente denotado por ker φ. Por exemplo, o kernel da função exponencial exp: H → H×
01

do exemplo acima é o feixe constante 2πiZ, associado ao grupo dos múltiplos inteiros de 2πi.
,2

Em suma, como diz o ditado popular, “kernel de feixe, feixinho é”. Alertamos porém o leitor para o fato
de que U 7→ im φU e U 7→ coker φU (com mapas de restrição induzidos pelos de G) são apenas pré-feixes,
29

mas não feixes em geral. Assim, as definições de feixes imagem e cokernel precisam ser ligeiramente
D
ct

modificadas, como veremos mais abaixo.


,O

Definição 1.1.11 Seja F um pré-feixe de grupos abelianos em um espaço topológico X. Seja x ∈ X.


O talo Fx de F em x é o grupo abeliano de “todas as seções definidas em alguma vizinhaça aberta de
ET

x”, ou seja, o limite direto


lim F (U )
Fx = −→
U∋x

onde U percorre todas as vizinhaças abertas de x, ordenados pela relação U  V ⇐⇒ U ⊇ V e mapas


de transição dados pelos mapas de restrição de F .
140 Esquemas

Explicitamente, os elementos de Fx são classes de equivalência [(U, f )] de pares (U, f ) onde U é um


aberto contendo x e f ∈ F (U ) é uma seção definida em U . A relação de equivalência é dada por

[(U, f )] = [(V, g)] ⇐⇒ existe um aberto W ∋ x tal que W ⊂ U ∩ V e f |W = g|W

Ou seja, dois pares são equivalentes se suas seções concordam em alguma vizinhaça aberta de x. A
soma de duas classes [(U, f )] e [(V, g)] é definida restringindo-se f e g a uma vizinhaça comum de x, por
exemplo W = U ∩ V : [(U, f )] + [(V, g)] = [(W, f |W + g|W )].
Seja φ: F → G é um morfismo de pré-feixes de grupos abelianos em X. Denotamos por φx o morfismo
de grupos abelianos entre talos induzido por φ:

φx : Fx → Gx
[(U, f )] 7→ [(U, φU (f ))]

O seguinte exemplo ilustra bem o fato de que o talo de um feixe corresponde intuitivamente ao
conjunto de todas as “seções definidas em torno de x”:
Exemplo 1.1.12 Se H é o feixe de funções holomorfas em X = C, temos que o talo H0 na origem 0 ∈ X
é isomorfo ao anel C{{z}} das séries de potência a0 + a1 z + · · · convergentes (em alguma vizinhança
aberta de 0). De fato, podemos construir um isomorfismo C{{z}} → H0 da seguinte maneira: cada
f (z) ∈ C{{z}} define uma função holomorfa em alguma vizinhaça U de 0, logo podemos levar f (z)
no elemento [(f (z), U )], que claramente não depende da escolha de U . Para construir o mapa inverso
H0 → C{{z}}, tome um elemento do talo [(f (z), U )] e leve-o na expansão em série f (z) = a0 + a1 z + · · ·
da função holomorfa f (z) em torno do 0.
Uma seção de um feixe é completamente determinada por suas imagens nos talos, como mostra o
seguinte
Exemplo 1.1.13 (Conhecendo Feixes via Talos) Seja F um feixe de grupos abelianos em um espaço
X. Dado um aberto U e um elemento f ∈ F (U ), denotamos por fx = [(U, f )] ∈ Fx a imagem de f no
talo em x ∈ U . Então temos uma injeção de grupos abelianos
Y
F (U ) ֒→ Fx
x∈U
f 7→ (fx )x∈U

que mostra que um feixe é realmente uma coleção de talos (pelo menos em linguagem agricultural!). De
fato, se fx = 0 então existe uma vizinhaça aberta Ux ⊂ U de x para a qual f |Ux = 0. Assim, se f
FT
pertence ao kernel do mapa acima, existe uma cobertura aberta de U na qual f se restringe a 0 em cada
aberto desta cobertura. Pela unicidade no axioma de cola, temos portanto que f = 0, mostrando que o
mapa acima é injetor.
)
28

Em particular, observe que se P é um pré-feixe e temos dois morfismos de pré-feixes φ, ψ: P → F tais


que φx = ψx para todo x ∈ X então φ = ψ. Assim, um morfismo de um pré-feixe para um feixe é
7:

completamente determinado por seus valores nos talos.


(1
RA

Encerramos esta seção com uma importante operação, a feixificação (esta palavra existe?!), que
0
01

nos permitirá definir os feixes quociente, imagem e cokernel.


,2

Teorema 1.1.14 (Feixificação) Seja X um espaço topológico. O funtor de inclusão i: Sh(X) ֒→


Psh(X) possui um adjunto à esquerda s: Psh(X) → Sh(X), chamado funtor de feixificação. Explici-
29

tamente, dado um pré-feixe P ∈ Psh(X), existe um feixe s(P) ∈ Sh(X) e um morfismo de pré-feixes
D

q: P → i ◦ s(P) tal que, para todo feixe F ∈ Sh(X) e todo morfismo de pré-feixes ψ: P → i(F ), existe
ct

um único morfismo de feixes φ: s(P) → F que faz o seguinte diagrama comutar:


,O

ψ-
ET

P i(F )
-
φ)
i(

q
∃!

?
i ◦ s(P)
141

Em outras palavras, temos uma bijeção natural feixe quocient


feixe imagem
HomSh (s(P), F )

- HomPsh (P, i(F )) feixe cokernel

φ 7→ i(φ) ◦ q

Além disso, temos um isomorfismo de talos qx : Px - (s(P))x para todo x ∈ X.


Prova Para todo aberto U de X, uma seção de Γ(U, s(P)) é definida como uma coleção de “seções
compatı́veis” de P:
( )
Y existe uma cobertura aberta S U = U de U e elementos
λ λ
Γ(U, s(P)) = (fx ) ∈ Px
fλ ∈ P(Uλ ) tais que (fλ )x = fx sempre que x ∈ Uλ
x∈U

Aqui, (fλ )x = [(Uλ , fλ )] ∈ Px denota a imagem de fλ no talo em x. Agora, dada uma inclusão de abertos
U ⊃ V , definimos o mapa de restrição resUV ignorando as coordenadas correspondentes a pontos que
não pertencem a V :
resUV : Γ(U, s(P)) → Γ(V, s(P))
(fx )x∈U 7→ (fx )x∈V
É fácil verificar que, com as definições acima, s(P) é de fato um feixe. Definimos ainda q: P → s(P) da
maneira natural:
qU : Γ(U, P) → Γ(U, s(P))
f 7→ (fx )x∈U
Novamente é fácil ver que qx : Px → (s(P))x é um isomorfismo.
Verifiquemos a propriedade universal. Inicialmente, observe que dado um morfismo de pré-feixes
ψ: P → i(F ), existe no máximo um morfismo φ: s(P) → F tal que ψ = i(φ) ◦ q pois isto implica
ψx = φx ◦ qx para todo x ∈ X e, como qx é um isomorfismo, temos que os talos de φ estão completamente
determinados, logo há no máximo um φ pelo exemplo anterior. Agora Q mostremos que φ de fato existe.
Dada uma seção f = (fx )x∈U ∈ Γ(U, s(P)) temos que (ψx (fSx ))x∈U ∈ x∈U Fx define um único elemento
φU (f ) ∈ F (U ). De fato, tomando uma cobertura aberta λ Uλ = U e elementos fλ ∈ P(Uλ ) tais que
(fλ )x = fx , temos que os elementos ψUλ (fλ ) ∈ F (Uλ ) satisfazem o axioma de cola, pois
 
ψUλ (fλ ) x = ψx ((fλ )x ) = ψx (fx ) = ψx ((fµ )x ) = ψUµ (fµ ) x para todo x ∈ Uλ ∩ Uµ
⇒ ψUλ (fλ )|Uλ ∩Uµ = ψUλ (fµ )|Uλ ∩Uµ
pelo exemplo anterior. Assim, estes elementos “colam” e definem um único elemento de F (U ), que
batizamos φU (f ), cujos talos são ψx (fx ). Agora uma verificação rotineira mostra que os morfismos
FT
φU : s(P)(U ) → F (U ) assim definidos são compatı́veis com os mapas de restrição, logo definem um
morfismo de feixes φ: s(P) → F , que satisfaz ψ = i(φ) ◦ q.
Finalmente, se ξ: P → Q é um morfismo de pré-feixes, definimos s(ξ): s(P) → s(Q) como o mapa
)
28

associado pela propriedade universal à composição


7:

P - Q
ξ q
- s(Q)
(1
RA

Uma outra verificação rotineira mostra que, com as definições acima, s define de fato um funtor de
0

Psh(X) em Sh(X).
01
,2

Observação 1.1.15 Como já se era de esperar, se P já é um feixe, então s(P) é um feixe isomorfo a P.
29

A feixificação permite dar a definição “correta” dos feixes quociente, kernel, cokernel, etc.
D

Definição 1.1.16 Seja F um subfeixe de G, i.e., F (U ) ⊂ G(U ) para todo aberto U e os mapas de
ct

restrição de F são induzidos pelos de G. Definimos o feixe quociente F /G como o feixe associado ao
,O

pré-feixe U 7→ G(U )/F (U ) (com mapas de restrição induzidos pelos de G).


ET

Seja φ: F → G um morfismo de feixes de grupos abelianos em um espaço topológico X. Definimos


respectivamente os feixes imagem im φ e cokernel coker φ de φ como sendo os feixes associados aos
pré-feixes
U 7→ im φU e U 7→ cokerU
Dizemos que φ é injetor (resp. sobrejetor) se ker φ = 0 (resp. coker φ = 0).
142 Esquemas

Lemma 1.1.17 Um morfismo de feixes φ: F → G é injetor (resp. sobrejetor) se, e só se, os morfismos
de talos φx : Fx → Gx são injetores (resp. sobrejetores) para todo x.
Prova Como a feixificação preserva talos, temos que (ker φ)x e (coker φ)x são os talos dos pré-feixes
U 7→ ker φU e U 7→ coker φU , respectivamente. Desta forma, temos

(ker φ)x = ker φx e (coker φ)x = coker φx

Por outro lado, pelo exemplo anterior, temos que ker φ = 0 ⇐⇒ (ker φ)x = 0 e coker φ = 0 ⇐⇒
(coker φ)x = 0 para todo x e o resultado segue.

O lema anterior sugere a seguinte


Definição 1.1.18 Uma sequência de morfismos de feixes de grupos abelianos em um espaço X

φi−1 φi φi+1
··· - F i−1 - Fi - F i+1 - ···

é exata se, e só se, as sequências

φi−1 φix φi+1


··· - F i−1 x - Fi - F i+1 x- ···
x x x

de talos são exatas para todo x ∈ X.

Lema 1.1.19 (Funtor seções globais) Seja X um espaço topológico. O funtor “seções globais”

Sh(X) 7→ Ab
F 7→ Γ(X, F )

é exato à esquerda.
Prova Considere uma sequência exata curta de feixes de grupos abelianos sobre X:

0 - F - G
φ
- H
ψ
- 0

Devemos mostrar que a seguinte sequência de grupos abelianos é exata:

0 - Γ(X, F ) φX
- Γ(X, G) - Γ(X, H)
ψX
FT
Já vimos que se ker φx = 0 para todo x então ker φ = 0 e em particular ker φX = 0. Agora seja
)
g ∈ ker ψX . Por hipótese, para cada x ∈ X, existe um único elemento fx ∈ Fx tal que φx (fx ) = gx , logo
28

existe uma vizinhança aberta Ux de x e um elemento f ∈ Γ(Ux , F ) tal que φUx (f ) = g|Ux . Considere o
7:

conjunto
(1
RA

{(U, f ) | U ⊂ X é um conjunto aberto e f ∈ Γ(U, F ) é tal que φU (f ) = g|U }


0

ordenado da maneira usual: (U, f )  (U ′ , f ′ ) ⇐⇒ U ⊂ U ′ e f ′ |U = f . É fácil verificar que toda cadeia


01

é limitada superiormente, logo pelo lema de Zorn existe um elemento maximal (U, f ) neste conjunto.
Afirmamos que U = X e que portanto φX (f ) = g. De fato, caso U 6= X, para x ∈ X \ U haveria uma
,2

vizinhaça aberta V de x e um elemento h ∈ Γ(V, F ) tal que φV (h) = g|V . Pelo que já provamos, φU∩V
29

é injetor, logo
D

φU∩V (f |U∩V ) = g|U∩V = φU∩V (h|U∩V ) ⇒ f |U∩V = h|U∩V


ct
,O

e pelo axioma de cola, existe um elemento t ∈ Γ(U ∪ V, F ) tal que t|U = f e t|V = h, o que implica

ET

 φU∪V (t)|U = g|U


φ (t)|V = g|V ⇒ φU∪V (t) = g|U∪V
 U∪V
φU∪V (t)|U∩V = g|U∩V

novamente pelo axioma de cola (desta vez em G). Mas então (U ∪ V, t) seria um elemento estritamente
maior do que o elemento maximal (U, f ), o que é uma contradição.
143

Exemplo 1.1.20 (A sequência exponencial) Seja X = C e H o feixe de funções holomorfas em X. feixe! injetivo
espaço localm
A sequência imagem diret
0 - Z - H - H× - 0
2πi exp
morfismo de e

é uma sequência exata de grupos abelianos. De fato, a sequência de talos em P ∈ X

0 - ZP -
2πi
HP -
expP
HP× - 0
k k k
Z C{{z − P }} C{{z − P }}×
é exata: para mostrar que exp é sobrejetor, tome h ∈ HP× ; uma pré-imagem de h é log h, onde log
representa qualquer ramo da função logaritmo que esteja definido em h(P ) 6= 0.
Por outro lado, sabe-se que a função logaritmo não pode ser definida “globalmente” e, de fato, tomando
U = C \ {0} temos que expU : H(U ) → H× (U ) não é sobrejetora, já que f (z) = z não pertence à sua
imagem: se g(z) ∈ H(U ) é tal que eg(z) = z para todo z ∈ U então eg(z) · g ′ (z) = 1 ⇐⇒ g ′ (z) = 1/z, o
que leva à contradição Z Z
dz
0= g ′ (z) dz = = 2πi
|z|=1 |z|=1 z

Observação 1.1.21 Um feixe I é injetivo se o funtor HomSh (−, I) é exato. Pode-se demonstrar a
categoria de feixes abelianos possui injetivos “em abundância”, i.e., que para todo feixe F existe um
feixe injetivo I e uma injeção F ֒→ I. Assim, podemos definir os funtores derivados à direita Ri Γ do
funtor seção global Γ(X, −), denotados simplesmente por H i (X, −). No exemplo anterior, demonstra-se
que H 1 (U, Z) = Z e H 1 (U, H) = 0, de modo que temos uma sequência exata longa
1
R h′
h7→ 2πi dz
0 - Z - H(U ) - H (U ) - H 1 (U, Z) = Z - H 1 (U, H) = 0
2πi exp × |z|=1 h

Note que H 1 (U, Z) = Z concorda com o grupo de cohomologia singular usual Hsing
1
(U, Z) (observe que
U é homotópico ao cı́rculo S ). Mais geralmente, demonstra-se que H (X, Z) = Hsing
1 i i
(X, Z) para todo
X paracompacto.

1.2 Espaços Localmente Anulares


Definição 1.2.1 Um espaço localmente anular é um par (X, OX ) formado por um espaço topológico
X e um feixe de anéis OX em X e tal que os talos OX,x são anéis locais para todo x ∈ X.
Definição 1.2.2 Seja φ: X → Y uma função contı́nua entre dois espaços topológicos e seja F um feixe
de grupos abelianos sobre X. A imagem direta φ∗ F de F é o feixe em Y dado em um aberto V ⊂ Y
por
(φ∗ F )(V ) = F (φ−1 (V ))
FT
sendo os mapas de restrição os de F .
)
Definição 1.2.3 Um morfismo de espaço localmente anulares (φ, φ# ): (X, OX ) → (Y, OY ) é um
28

par formado por uma função contı́nua φ: X → Y e por um morfismo φ# : OY → φ∗ OX de feixes em Y


7:

tal que, para todo ponto x ∈ X, o morfismo induzido de talos


(1
RA

φ#
P : OY,φ(x) → OX,x
0

[(V, f )] 7→ [(φ−1 (V ), φ#
V (f ))]
01

é um morfismo local (relembrando: isto significa que a imagem do ideal maximal de OY,φ(x) está contido
,2

no ideal maximal de OX,x ).


29

Morfismos de espaços localmente anulares podem ser compostos: dados


D
ct

(f, f # ): (X, OX ) → (Y, OY ) e (g, g # ): (Y, OY ) → (Z, OZ )


,O

sua composição é dada por


ET

(g ◦ f, (g∗ f # ) ◦ g # ): (X, OX ) → (Z, OZ ),


onde (g∗ f # ) ◦ g # é a composição de mapas de feixes em Z
g# g∗ f #
OZ - g∗ OY - g∗ f∗ OX = (g ◦ f )∗ OX

A categoria de espaços localmente antulares será denotada por LRS.


144 Esquemas

Exemplo 1.2.4 (Variedades Diferenciáveis) Seja X uma variedade diferenciável e seja OX o feixe
de funções reais diferenciáveis sobre X: para um aberto U ⊂ X,
def
OX (U ) = {f : U → R | f é diferenciável}

Temos que o par (X, OX ) é um espaço localmente anular.


Exemplo 1.2.5 (Subespaços Abertos) Seja (X, OX ) um espaço locamente anular e seja U ⊂ X um
conjunto aberto. Então (U, OX |U ) também é um espaço localmente anular; aqui OX |U denota a restrição
do feixe OX aos subconjuntos abertos de U . Temos ainda um morfismo de espaços localmente anulares
(uma “imersão aberta”)
(j, j # ): (U, OX |U ) ֒→ (X, OX )
onde j: U ֒→ X é o mapa de inclusão e j # : OX → j∗ OX |U é induzido pelos mapas de restrição do feixe:
se V ⊂ X é um aberto,
jV# : OX (V ) → Γ(V, j∗ OX |U ) = OX (U ∩ V )
f 7→ f |U∩V
Agora, dado um morfismo qualquer (f, f # ): (X, OX ) → (Y, OY ) de LRS e um aberto U ⊂ X, podemos
definir a restrição (f, f # )|U de (f, f # ) a U como a composição

(f, f # )|U = (f, f # ) ◦ (j, j # )

onde (j, j # ): (U, OX |U ) ֒→ (X, OX ) é o morfismo de inclusão de U definido acima.

Lema 1.2.6 (Colando LRS) Seja {(Xi , OXi )}i∈I uma coleção de espaços localmente anulares. Supo-
nha que para cada par (i, j) ∈ I × I são dados abertos Uij ⊂ Xi e isomorfismos

(φij , φ# - (Uji , OXj |Uji )



ij ): (Uij , OXi |Uij )

tais que φij (Uij ∩ Uik ) = Uji ∩ Ujk de modo que as seguintes condições de cociclo são satisfeitas:
1. (φii , φ# # # −1
ii ) = id e (φij , φij ) = (φji , φji ) ;
2. (φik , φ# # #
ik ) = (φjk , φjk ) ◦ (φij , φij ) em Uij ∩ Uik (tome as restrições como no exemplo anterior).
Então existe um espaço localmente anular (X, OX ) juntamente com morfismos de espaços localmente
S
anulares (ψi , ψi# ): (Xi , OXi ) ֒→ (X, OX ) (“imersões abertas”) tais que X = i∈I ψi (Xi ), ψi (Uij ) =
ψi (Xi ) ∩ ψj (Xj ) e (ψi , ψi# ) = (ψj , ψj# ) ◦ (φij , φ#
ij ) em Uij . O espaço localmente anular (X, OX ) é
FT
unicamente determinado a menos de isomorfismo por estas condições.
F
Prova Considere a seguinte relação ∼ na união disjunta i Xi :
)
28

( xi ∼ xj 
 xi ∈ Uij ,
7:

xi ∈ Xi ⇐⇒ xj ∈ Uji e
(1

 φ (x ) = x
RA

xj ∈ Xj ij i j
0

F
01

As condições de cociclo asseguram que esta relação é de equivalência. Defina X = i Xi / ∼ com a


topologia quociente e sejam ψi : Xi → X os mapas quociente. Vamos agora construir um feixe em X:
,2

para U ⊂ X aberto, uma seção de OX (U ) é uma tupla de seções de ψi∗ OXi (U ) “concordando nas
29

intersecções”:
n Y o
D


ct

def
OX (U ) = (si ) ∈ ψi∗ OXi (U ) si |Uij ∩ψ−1 (U) = φ# −1
ij U ∩ψ (U)
(s j | Uji ∩ψ −1
(U) ) ∀i, j ∈ I
,O

i ji j j
i∈I
ET

Os mapas de restrição são induzidos pelos mapas de restrição dos feixes ψi∗ OXi . Temos ainda um mapa
ψi# : OX → ψi∗ OXi de feixes em X dado pela projeção na i-ésima coordenada. Com estas definições, uma
verificação rotineira mostra que (X, OX ) é um espaço localmente anular e que todas as demais condições
do lema são satisfeitas.

Temos o seguinte “lema companheiro”, cuja prova fica como exercı́cio para o leitor:
145

Lema 1.2.7 S (Colando morfismos de LRS) Sejam (X, OX ) e (Y, OY ) dois espaços localmente an-
ulares. Seja i Ui = X uma cobertura aberta de X e suponha que são dados morfismos de espaços
localmente anulares (fi , fi# ): (Ui , OX |Ui ) → (Y, OY ) concordando nas intersecções, i.e., (fi , fi# )|Ui ∩Uj =
(fj , fj# )|Ui ∩Uj para todo i, j. Então podemos “colar” estes morfismos em um morfismo de espaços local-
mente anulares (f, f # ): (X, OX ) → (Y, OY ) de modo que (fi , fi# ) = (f, f # )|Ui .
Definição 1.2.8 Seja (X, OX ) um espaço localmente anular. Um OX -módulo é um feixe M de grupos
abelianos em X tal que, para todo aberto U , M(U ) é um OX (U )-módulo e tal que os mapas de restrição
são compatı́veis com as estruturas de módulo structures: para toda inclusão U ⊃ V de abertos,
(a · m)|V = a|V · m|V para todo a ∈ OX (U ) e m ∈ M(U )
Ou seja, temos um diagrama comutativo
OX (U ) × M(U ) - M(U )

res res res


? ? ?
OX (V ) × M(V ) - M(V )

Um morfismo de OX -módulos M → N é um morfismo de feixes de grupos abelianos tal que para


cada aberto U o mapa M(U ) → N (U ) é um morfismo de OX (U )-módulos.
Analogamente definimos uma OX -álgebra como sendo um feixe A em X tal que A(U ) é uma OX (U )-
álgebra para cada aberto U , com mapas de restrição compatı́veis com as estruturas de álgebras.

2 Esquemas
O esquema afim associado a um anel comutativo A consiste de duas partes: o espaço topológico Spec(A),
juntamente com um feixe de anéis OA , cujas seções são as “funções” do esquema. Assim como uma
variedade diferenciável é obtida “colando-se” bolas abertas do Rn , um esquema geral será obtido como
“colagem” de esquemas afins.

2.1 Esquemas Afins


Para qualquer anel A, vamos construir um feixe OA de anéis sobre Spec A (munido da topologia de
Zariski) de forma que OA (D(h)) = Ah para todo h ∈ A. Vejamos inicialmente o caso em que A é um
domı́nio com corpo de frações K = Frac A. A ideia é interpretar um elemento f ∈ A como uma “função”
sobre Spec A que “se anula” exatamente nos primos p tais que p ∋ f . O anel OA (U ) ⊂ K por sua vez
deve ser visto como o anel das “funções racionais” que estão “definidas” em todo o aberto U ; aqui um
FT
elemento f ∈ K está “definido” no primo p se podemos escrever f = a/s com a ∈ A e s ∈ A \ p (isto
é, o denominador de f “não se anula” em p). Assim, para um conjunto aberto não vazio U ⊂ Spec A é
)
natural definirmos \
28

def
OA (U ) = Ap ⊂ K
7:

p∈U
(1
RA

Se U ⊃ V então OA (U ) ⊂ OA (V ) e definimos resUV : OA (U ) ֒→ OA (V ) como sendo o mapa de inclusão. É


0

fácil ver que com estas definições OA é de fato de anéis sobre Spec A. Vamos verificar que OA (D(h)) = Ah
01

para todo h ∈ A. Como há uma bijeção


,2

D(h) - Spec Ah

29

p 7→ ph
D
ct

e, além disso, (Ah )ph = Ap , temos que


,O

OAh (Spec Ah ) = OA (D(h))


ET

T
Logo é suficiente considerar o caso h = 1, ou seja, devemos mostrar que p Ap = A, o que segue do
princı́pio local-global (teorema III.1.8). O anel OA (D(h)) = Ah ⊂ K deve ser pensado como o “domı́nio
de definição” da “função” 1/h. Note que o talo de OA em p ∈ Spec A é dado por
OA,p = −→
lim OA (U ) = −→
lim OA (D(h)) = −→
lim Ah = Ap
U∋p D(h)∋p h∈p
/
146 Esquemas

já que os conjuntos D(h) formam uma base da topologia de Zariski. feixe estrutural
Para um anel geral A, não podemos ver as localizações Ap como subanéis do corpo de frações (que
não existe em geral). Entretanto, o mapa natural
Y
A ֒→ Ap
p∈Spec A

ainda é injetivo
Q pelo princı́pio local-global, assim é natural tentar definir OA (U ) como um subanel do
produto p∈Spec A Ap imitando a construção do funtor feixificação (ver teorema 1.1.14).
Para um aberto não vazio U ⊂ Spec A defina
 S 
 Y existe uma cobertura aberta U = i D(hi ) de U por abertos 
def
OA (U ) = (sp )p∈U ∈ Ap básicos (com hi ∈ A) e elementos ai /hni i ∈ Ahi , ai ∈ A, de
 
p∈U modo que sq = ai /hni i ∈ Aq para todo q ∈ D(hi )

Os mapas de restrição são definidos como no teorema 1.1.14. É fácil ver que OA é de fato um feixe de
anéis sobre Spec A, o chamado feixe estrutural de A.
Esta definição pode parecer complicada de inı́cio mas, basicamente, pelo axioma de cola, para
conhecer OA basta saber seus valores nos “abertos básicos” D(h) juntamente com o fato que os mapas
de restrição nada mais são do que os mapas de localização:
Teorema 2.1.1 (Dissecando o feixe estrutural OA ) Seja A um anel e OA seu feixe estrutural.
Para g ∈ A, denote por φg : Ag → OA D(g) o mapa natural que leva a/g n , a ∈ A, na “tupla constante”
(sp ) dada por sp = a/g n ∈ Ap para todo p ∈ D(g). Então φg é um isomorfismo para todo g ∈ A e temos
um diagrama comutativo
φg- 
Ag OA D(g)

ρ res
? φgh ? 
Agh - OA D(gh)


para todo g, h ∈ A, onde ρ: Ag → Agh é o mapa natural de localização. Em particular, OA Spec A ∼=A
e o talo de OA em p ∈ Spec A é dado por

OA,p = −→ lim OA D(g) = −→ lim Ag = Ap
FT
D(g)∋p g∈p
/

 
Prova Como no caso de domı́nios, temos um isomorfismo OA D(g) = OAg D(g) e portanto substi-
)
28

tuindo A por Ag é suficiente mostrar que φ1 : A → OA (Spec A) é um isomorfismo. O princı́pio local-


global (teorema III.1.8) mostra que este mapa é injetor. Para mostrar a sobrejetividade, observamos
7:

inicialmente que dar uma seção de OA (Spec A) é o mesmo que dar uma cobertura aberta básica finita
(1
RA

S
1≤i≤n D(h i ) = Spec A e uma famı́lia de “elementos compatı́veis nas intersecções” ai /hi ∈ Ahi , ou seja,
0

tais que ai /hi = aj /hj em Ahi hj para todo i, j. De fato, como Spec A é quase-compacto (lemma II.4.3)
01

e D(hni i ) = D(hi ), na definição de uma seção de OA (Spec A) podemos assumir que a cobertura básica é
finita e que ni = 1 para todo i. Além disso, da já demonstrada injetividade de φhi hj , se ai /hi = aj /hj
,2

em Ap para todos os primos p ∈ D(hi ) ∩ D(hj ) = D(hi hj ) então ai /hi = aj /hj em Ahi hj também.
S
29

Agora seja 1≤i≤n D(hi ) = Spec A uma cobertura finita e ai /hi ∈ Ahi uma famı́lia de elementos
D

compatı́veis como acima. Nossa missão é encontrar a ∈ A tal que a/1 = ai /hi em Ahi para todo i. Como
ct

ai /hi = aj /hj em Ahi hj , existe N tal que, em A,


,O

N +1
(hi hj )N (ai hj − aj hi ) = 0 ⇐⇒ ai hN
i hj = aj hN +1 N
hj (∗)
ET

Como a cobertura é finita, tomando N grande o suficiente podemos assumir que esta igualdade é válida
para todos os pares i, j. Além disso, temos (ver a demonstração de lemma II.4.3)
[ [ X
D(hN j
+1
)= D(hj ) = Spec A ⇒ hN
j
+1
· tj = 1 (∗∗)
1≤j≤n 1≤j≤n 1≤j≤n
147

P
para certos tj ∈ A, o que fornece uma “partição algébrica da unidade”. Portanto a = 1≤j≤n (ahNj
+1
)·tj esquema afim
N N +1
para qualquer a ∈ A; como estamos procurando a ∈ A tal que a/1 = aj /hj = aj hj /hj em Ahj para
todo j, um bom candidato é
def
X
a = aj hN
j tj
1≤j≤n

Mostremos que isto funciona. Para um i fixado, temos por (∗) e (∗∗)
X X X
ahN
i
+1
= aj hN
i
+1 N
h j tj = ai hN N +1
i hj tj = a i h N
i hN
j
+1
tj = a i h N
i
1≤j≤n 1≤j≤n 1≤j≤n

a ai
⇒ hN
i (ahi − ai ) = 0 ⇒ = em Ahi
1 hi
como desejado. As demais afirmações do teorema são simples consequências do que provamos.

Observação 2.1.2 Se A é um domı́nio, as duas construções de OA dão origem a feixes isomorfos.


Seja η = (0) o “ponto genérico” de Spec A temos OA,η = Aη = K onde K = Frac A. Como η é um
ponto denso, ele pertence a todos os abertos não vazios U de Spec A e portanto temos um mapa natural
OA (U ) → OA,η que, para U = D(h), nada mais é do que a inclusão Ah ֒→ K. Como os D(h) formam
uma base da topologia de Zariski, temos que o axioma de cola implica que OA (U ) ֒→ OA,η = K é injetor
para qualquer aberto U 6= ∅. E da segunda construção de OA (U ) temos que f ∈ K pertence a imagem
de OA (U ) ֒→ K se, e só se, para cada pT∈ U podemos escrever f = a/h com a, h ∈ A e h ∈/ p; em outras
palavras, a imagem de OA (U ) ֒→ K é p∈U Ap . Identificando os anéis OA (U ) com suas imagens em K,
concluı́mos que a nova construção de OA coincide com a anterior.
Exemplo 2.1.3 Vejamos um exemplo concreto da filosofia “diga-me seus valores nos abertos básicos e
te direi quem tu és”. Seja X = Spec C[x, y] e seja U = X \ {(x, y)} (o complemento da “origem” do
“plano” X). Para determinar OC[x,y] (U ), observe inicialmente que U = D(x) ∪ D(y) de modo que temos
uma sequência exata

OC[x,y] (U ) → OC[x,y] (D(x)) ⊕ OC[x,y] (D(y)) ⇉ OC[x,y] (D(x) ∩ D(y))


k k
C[x, y]x ⊕ C[x, y]y C[x, y]xy

Como C[x, y] é um domı́nio, os mapas de restrição/localização C[x, y]x ֒→ C[x, y]xy e C[x, y]y ֒→ C[x, y]xy
nada mais são que os mapas de inclusão e assim
FT
n o

OC[x,y] (U ) = (f, g) ∈ C[x, y, x1 ] × C[x, y, y1 ] f = g em C[x, y, xy
1
]
n o
)

= (f, f ) ∈ C[x, y, x1 ] × C[x, y, y1 ] f ∈ C[x, y]
28
7:

= C[x, y]
(1
RA

Agora podemos associar um objeto geométrico a todo anel.


0
01

Definição 2.1.4 O esquema afim associado ao anel A é o espaço localmente anular (Spec A, OA ). Por
abuso de linguagem, muitas vezes nos referimos ao próprio Spec A como o esquema afim associado a A,
,2

deixando o feixe OA subentendido.


29

Vamos agora mostrar como associar a cada morfismo of anéis φ: A → B um morfismo de espaços
D
ct

localmente anulares
(f, f # ): (Spec B, OB ) → (Spec A, OA )
,O

Definimos f = Spec(φ), que é um mapa contı́nuo (lemma II.4.3). Para definir f # : OA → f∗ OB , observe
ET

inicialmente que, para todo q ∈ Spec B, φ induz um morfismo local

φq : Aφ−1 (q) = Af (q) → Bq


a φ(a) (a ∈ A, s ∈ A \ φ−1 (q))
7→
s φ(s)
148 Esquemas

Agora, para um aberto U ⊂ Spec A, definimos

fU# : OA (U ) → f∗ OB (U ) = OB (f −1 U )
(sp )p∈U 7→ (tq )q∈f −1 U

onde
tq = φq (sf (q) ) ∈ Bq
Vejamos que este mapa está bem definido, ou seja, que a tupla (tq )q∈f −1 U mora de fato em OB (f −1 U ).
Primeiramente, observe que, em termos básicos, f # nada mais é do que a localização de φ: para h ∈ A,
temos um diagrama comutativo
#
 fD(h)  
OA D(h) - OB f −1 (D(h)) == OB D(φ(h))
6 6
≈ ≈

φh - Bφ(h)
Ah

De fato, dado a/hn ∈ Ah (a ∈ A), o elemento correspondente em OA D(h) é a tupla constante
(sp )p∈D(h) com sp = a/hn ∈ Ap para todo p ∈ D(h). Esta tupla é mapeada na tupla constante
(tq )q∈D(φ(h)) onde tq = φ(a)/φ(h)n para todo q ∈ D(φ(h)). Mas este elemento é justamente a imagem
def
de φh (a/hn ) = φ(a)/φ(h)n ∈ Bφ(h) , o que comprova a comutatividade do diagrama.
Para um aberto U ⊂ Spec A arbitrário, temos que um elementoS de OA (U ) pode ser representado
por uma famı́lia de elementos compatı́veis ai /hni i ∈ Ahi onde i D(hi ) = U é uma cobertura de abertos
básicos. Como acima, verifica-se que a imagem deste elemento por fU# pode ser representado S pela
famı́lia compatı́vel de elementos φ(ai )/φ(hi )ni ∈ Bφ(hi ) correspondentes à cobertura i D(φ(hi )) =
S −1
if (D(hi )) = f −1 U de f −1 U , o que mostra que fU# está de fato bem definido.
Para mostrar que (f, f # ) é um morfismo de espaços localmente anulares, precisamos verificar que os
morfimos induzidos nos talos são locais. Mas, das definições, temos para todo q ∈ Spec B um diagrama
comutativo

fq#-
OA,f (q) OB,q
6 6
FT
≈ ≈

φq -
)
Af (q) Bq
28
7:

com flechas verticais que são isomorfismos. Como φq é local, temos que fq# também é local.
(1
RA

Assim, temos um funtor Rings◦ → LRS que associa a um anel A o esquema afim (Spec A, OA ) e
a um morfismo de anéis φ: A → B o morfismo de espaços localmente anulares (f, f # ): (Spec B, OB ) →
0

(Spec A, OA ) como construı́do acima. O próximo teorema mostra que este funtor “geometrização” é
01

fielmente pleno, ou seja, podemos ver a categoria de anéis como uma subcategoria plena da categoria
,2

dos espaços localmente anulares.


29

Teorema 2.1.5 (Anéis como uma subcategoria plena de LRS) O funtor Rings◦ → LRS con-
D

struı́do acima é plenamente fiel: existe uma bijeção natural


ct

 
,O

HomRings (A, B) = HomLRS (Spec B, OB ), (Spec A, OA )


ET

Prova Dado um morfismo (f, f # ): (Spec B, OB ) → (Spec A, OA ) em LRS podemos utilizar os isomor-
#
fismos A = OA (Spec A) e B = OB (Spec B) para definir um morfismo de anéis fSpec A : A → B. Vamos
# # #
agora mostrar que a associação (f, f ) 7→ fSpec A é inversa da associação φ 7→ (f, f ) dada pelo funtor
Rings◦ → LRS.
149

Claramente temos que começando com φ: A → B e aplicando as duas operações acima em sequência
def #
obtemos o morfismo original φ de volta. Por outro lado, começando com (f, f # ), defina φ = fSpec A.
# # #
Seja (g, g ) o morfismo em LRS associado a φ. Vamos mostrar que (f, f ) = (g, g ).
def
Seja q ∈ Spec B e p = f (q) ∈ Spec A. Como f # é um mapa of feixes, obtemos um diagrama
comutativo

#
φ = fSpec
A ====== OA (Spec A) -A
OB (Spec B) ===== B

loc res res loc


? ? fq# ? ?
Ap ======== OA,p - OB,q ======== Bq

onde as flechas verticais são os mapas naturais de restrição/localização. Como fq# é local por hipótese,
temos (fq# )−1 (qBq ) = pAp e assim a pré-imagem de qBq ⊂ Bq em A é p. Por outro lado (literalmente!),
# −1
temos que esta pré-imagem é (fSpec A) (q) = φ−1 (q), assim φ−1 (q) = f (q) e portanto f = g = Spec(φ).
Temos agora que f # e g # são dois mapas entre os feixes OA e f∗ OB = g∗ OB em Spec A. Assim,
para mostrar que f # = g # basta mostrar que estes dois mapas concordam nos abertos básicos D(h),
h ∈ A. Temos o diagrama comutativo

#
φ = fSpec
A ====== OA (Spec A) - OB (Spec B) ====== B
A

loc res res loc


? ? #
fD(h) ? ?
Ah ====== OA (D(h)) - OB (D(φ(h))) ==== Bφ(h)

#
de modo que fD(h) : OA (D(h)) → OB (D(φ(h))) pode ser identificado com a localização φh : Ah → Bφ(h)
# # #
de φ. O mesmo vale para gD(h) : OA (D(h)) → OB (D(φ(h))), de modo que fD(h) = gD(h) , como desejado.

Exemplo 2.1.6 Sejam k e l dois corpos. Temos que ambos os espaços topológicos Spec k e Spec l
FT
consistem apenas de um ponto. Porém, pelo teorema,
)
HomLRS ((Spec l, Ol ); (Spec k, Ok )) = HomRings (k, l)
28
7:

de modo que não há morfismos entre os dois esquemas afins a não ser que k seja isomorfo a um subcorpo
(1
RA

de l e, neste caso, há tantos morfismos quanto imersões de k em l! Isto ilustra uma caracterı́stica muito
importante de esquemas: morfismos requerem certa compatibilidade das “funções” dos dois espaços.
0
01

Exemplo 2.1.7 (“Retas obesas”) Considere os esquemas afins


,2

X = Spec Q[x, y]/(x2 ) e Y = Spec Q[x, y]/(x)


29
D
ct

(aqui começamos a utilizar o abuso de linguagem e omitir os feixes na notação). Note que embora os dois
,O

espaços topológicos sejam homeomorfos, estes dois esquemas não são isomorfos como espaços localmente
anulares: OX contém elementos nilpotentes enquanto que OY não. Geometricamente, imaginamos Y
ET

como a “reta” x = 0 enquanto que X deve ser pensado como uma “reta gorda” obtida a partir do
plano fazendo-se “quase” x = 0 (mas só x2 é realmente 0 em X) e que tem um “tecido adiposo extra”
ou “material infinitesimal” espalhando-se na direção normal a Y . Observe que temos um morfismo de
“inclusão” f : Y ֒→ X da reta magra na gorda correspondente ao morfismo de Q-álgebras Q[x, y]/(x2 ) ։
Q[x, y]/(x), que leva a “função nilpotente” não nula x ∈ OX (X) na função zero em OY (Y ) (ou seja,
x ∈ OX (X) se restringe a 0 em Y ).
150 Esquemas

O último examplo ilustra uma das caracterı́sticas únicas de esquemas dentre os diversos objetos esquema
morfismo de esquema
geométricos encontrados em Matemática: a presença de “funções nilpotentes”. Esta é a razão pela qual
em um morfismo de esquemas o mapa entre feixes não é automaticamente determinado pelo mapa de
espaços topológicos e precisa ser dado explicitamente, ao contrário do caso de variedades diferenciáveis,
por exemplo.
Terminamos esta seção com alguns comentários sobre módulos. Seja A um anel e M um A-módulo.
Imitando a construção de OA , podemos associar a M um OA -módulo M f definido da seguinte forma:
 S 
 Y existe uma cobertura aberta U = i D(hi ) de U por abertos 
f(U ) def n
M = (mp )p∈U ∈ Mp básicos (com hi ∈ A) e elementos mi /hi i ∈ Mhi , mi ∈ M , de
 ni 
p∈U modo que mq = mi /hi ∈ Mq para todo q ∈ D(hi )

Mutatis mutandis, as mesmas provas se aplicam para mostrar que para todo h ∈ A e p ∈ Spec A há
isomorfismos naturais
f(D(g)) ∼
M = Mg e f)p ∼
(M = Mp

e os mapas de restrição são dados por localização. Da mesma forma, um morfismo M → N de A-


módulos induz um morfismo de OA -módulos M f→N e que, para um aberto básico D(h), se resuma ao
mapa localizado Mg → Ng . Assim, temos um funtor da categoria de A-módulos para a de OA -módulos
e como acima temos uma bijeção natural

f, N
HomA-Mod (M, N ) = HomOA -Mod (M e)

Os mesmos comentários também se aplicam para OA -álgebras.

Exemplo 2.1.8 Seja a um ideal de um anel A. Seja i: Spec A/a ֒→ Spec A o morfismo de LRS associado
ao morfismo natural A ։ A/a. Da sequência exata de A-módulos

0 - a - A - A/a - 0

obtemos uma sequência de OA -módulos

0 -e
a - OA - i∗ OA/a - 0

^ = OA/a como uma OA -álgebra via o morfismo i# : OA → i∗ OA/a . Esta é uma


Aqui vemos (A/a)
FT
sequência exata de feixes pois temos uma sequência exata de talos

- e - - - 0
)
0 ap OA,p (i∗ OA/a )p
28

k k k
- - - - 0
7:

0 ap Ap (A/a)p
(1
RA

para todo p ∈ Spec A.


0
01

2.2 Esquemas em geral


,2

Após a introdução de todo este formalismo, estamos finalmente prontos para dar a definição geral de
29

esquemas como objetos geométricos obtidos a partir da “colagem de anéis”:


D
ct

S esquema é um espaço localmente anular (X, OX ) para o qual existe uma cober-
Definição 2.2.1 Um
,O

tura aberta X = α Uα de X com cada (Uα , OX |Uα ) isomorfo na categoria LRS a um esquema afim
(Spec Aα , OAα ). Um morfismo de esquemas é somente um morfismo em LRS. Assim, a categoria de
ET

esquemas Sch é uma subcategoria plena de LRS. Por abuso de linguagem, vamos nos referir ao esquema
(X, OX ) simplesmente por X e ao morfismo de esquemas (f, f # ): (X, OX ) → (Y, OY ) simplesmente por
f : X → Y ; em particular, escreveremos Spec(φ): Spec B → Spec A para o morfismo de esquemas afins
associado ao morfismo de anéis φ: A → B.

O conceito correspondente ao de uma A-álgebra é um S-esquema:


151

Definição 2.2.2 Seja S be a esquema. Um S-esquema ou esquema sobre S é um morfismo de


esquemas p: X → S. Por abuso de linguagem, vamos nos referir a X como o S-esquema se p é claro
pelo contexto. Um morfismo de S-esquemas entre p: X → S e q: Y → S é um morfismo de esquemas
f : X → Y tal que o diagrama

f - Y
X

q
?


S
comuta. A categoria de S-esquemas é denotada por Sch/S. Note que como todo anel é uma Z-álgebra,
todo esquema é um Z-esquema.
Como primeiro exemplo, seja X = Spec A um esquema afim e seja U ⊂ X um conjunto aberto
qualquer. O espaço localmente anular (U, OA |U ) é um esquema (em geral, não afim): isto segue do
seguinte lema e do fato que os abertos principais D(h) formarem uma base para a topologia de Zariski.
Lema 2.2.3 (Subesquema Aberto Principal) Seja A um anel e h ∈ A. Temos um isomorfismo de
esquemas
(Spec Ah , OAh ) ∼
= (D(h), OA |D(h) )
induzido pelo mapa de localização ρ: A → Ah .
Prova Denote por (f, f # ): (Spec Ah , OAh ) → (Spec A, OA ) o morfismo de espaços localmente anulares
correspondente ao morfismo de localização ρ: A → Ah . Temos que f induz um homeomorfismo entre
Spec Ah e D(h) ⊂ Spec A. Portanto restringido f e f # : OA → f∗ OAh a D(h) obtemos um morfismo de
espaços localmente anulares (Spec Ah , OAh ) → (D(h), OA |D(h) ). Falta apenas mostrar que o morfismo de
feixes é um isomorfismo e para isto basta olharmos para os talos: se p ∈ D(h) o mapa fp# : (OA |D(h) )p →
(f∗ OAh )p é um isomorfismo pois Ap ∼= (Ah )ph .

Corolário 2.2.4 Os abertos afins U de um esquema (X, OX ) (i.e. abertos para os quais (U, OX |U ) ∼
=
(Spec A, OA ) para algum anel A) formam uma base de abertos para X.
Note que, como consequência do corolário (ou do lema), temos mais geralmente que para todo
esquema (X, OX ) e todo aberto U ⊂ X o espaço localmente anular (U, OX |U ) também é um esquema, o
subesquema aberto determinado por U .
FT
Observação 2.2.5 Se (X, OX ) é um esquema e Y ⊂ X é um subconjunto fechado, devido a possi-
bilidade de existência de “funções nilpotentes”, existem em geral diversas maneiras de vermos Y como
)
“subesquema fechado” de X, ou seja, existem diversos feixes OY “compatı́veis” com OX . Por exemplo,
28

se X = Spec C[t], temos que Y1 = Spec C[t]/(t) e Y2 = Spec C[t]/(t2 ) são dois esquemas não isomorfos
7:

com o mesmo espaço topológico subjacente {(t)} ⊂ X.


(1
RA

Exemplo 2.2.6 (Reta Projetiva) Podemos utilizar a técnica de colagem de espaços localmente an-
0

ulares para construir esquemas. Para exemplificá-la, vamos construir a versão esquemática da reta
01

projetiva sobre um anel A. Considere os esquemas afins


,2

X0 = Spec A[x] X1 = Spec A[y]


29

∪ ∪
D

U0 = D(x) U1 = D(y)
ct
,O

Seja φ: U0 - U1 o isomorfismo de A-esquemas dado pelo isomorfismo de A-álgebras



ET

A[y]y

- A[x]x
y 7→ 1/x

O A-esquema obtido colando-se X0 e X1 ao longo de φ é chamado reta projetiva sobre A, denotado


por P1A .
152 Esquemas

Por exemplo, se A = C o isomorfismo φ identifica os “pontos genéricos” (0) de X0 = Spec C[x] e


X1 = Spec C[y]; além disso, o ponto fechado (x − a) ∈ X0 , a ∈ C \ {0}, é identificado com ponto fechado
(y − 1/a) ∈ X1 . Portanto P1C possui um único ponto genérico e seus pontos fechados são os mesmos de
X0 acrescido e um ponto extra “no infinito”, que corresponde ao ideal maximal (y) ∈ X1 . Assim como
no caso “clássico” (i.e., visto em classe) da reta projetiva complexa ou esfera de Riemann, que consiste
em uma cópia de C acrescida de um “ponto no infinito”.
No caso clássico, como a esfera de Riemann é compacta, pelo teorema de Liouville não há funções
holomorfas globalmente definidas além das constantes. Vejamos que o mesmo ocorre em “nossa” reta
projetiva. Por construção, temos que

Γ(P1C , OP1C ) = {(s0 , s1 ) ∈ Γ(X0 , OX0 ) × Γ(X1 , OX1 ) | φ#


U1 (s1 |U1 ) = s0 |U0 }

= {(s0 (x), s1 (y)) ∈ C[x] × C[y] | s1 (1/x) = s0 (x) em C[x]x }


= {(a, a) ∈ C[x] × C[y] | a ∈ C}
=C

Em particular, como P1C 6= Spec Γ(P1C , OP1C ) (P1C possui infinitos pontos enquanto Spec C só possui um),
temos que P1C não é afim.
Observação 2.2.7 O espaço projetivo n-dimensional PnA sobre A pode ser definido de maneira análoga:
seja
(i) (i)
X (i) = Spec A[x0 , . . . , x(i)
n ] com a entrada xi omitida
(i) (i) (i) (j)
e Uj = D(xj ) ⊂ X (i) para i, j = 0, 1, . . . , n. Definimos isomorfismos φij : Uj → Ui de A-esquemas
correspondentes aos isomorfismos de A-álgebras
(j) (i)
A[x0 , . . . , x(j) (i)
n ]x(j) → A[x0 , . . . , xn ]x(i)
i j

(i)
(j) xk
xk 7→ (i)
xj

O espaço projetivo PnA é definido pela colagem associada ao pacote acima. Na próxima seção, veremos
uma maneira mais prática de definir PnA , em que tudo já vem “colado de fábrica”.

Lema 2.2.8 (Fatoração Aberta) Seja f : X → Y um morfismo de esquemas. Suponha que a imagem
de f esteja contida em um aberto V ⊂ Y . Então f se fatora unicamente como uma composição

X - V
g ⊂
j
- Y
FT
onde j: V ֒→ Y denota o morfismo de inclusão.
)
Prova Temos que o mapa de espaços topológicos g: X → V é unicamente obtido restringindo-se f a V .
28

Por outro lado, o mapa de feixes f # : OY → f∗ OX em Y se restringe a um mapa de feixes g # : OV → g∗ OX


7:

em V e é claro que f # = (j∗ g # ) ◦ j # .


(1
RA

Teorema 2.2.9 (Morfismos para um esquema afim) Sejam X um esquema e A um anel. Temos
0
01

uma bijeção natural


HomSch (X, Spec A) = HomRings (A, OX (X))
,2

#
(f, f # ) 7→ fSpec A
29

S
D

Prova Seja X = α Uα uma cobertura afim de X. Dado φ: A → OX (X), construı́mos um morfismo


ct

f : X → Spec A da seguinte maneira: seja φα = resXUα ◦φ: A → OX (Uα ) e seja fα : Uα → Spec A o


,O

morfismo correspondente de esquemas afins. Vamos mostrar que os fα ’s concordam nas intersecções e
assim podem ser colados.
ET

S
Seja γ Vγ = Uα ∩ Uβ uma cobertura afim de Uα ∩ Uβ . Temos dois morfismos entre os esquemas
afins Vγ e Spec A: fα |Vγ e fβ |Vγ . Porém ambos correspondem ao mesmo morfismo de anéis resXVγ ◦φ.
Portanto fα |Vγ = fβ |Vγ para todo k e assim fα |Uα ∩Uβ = fβ |Uα ∩Uβ , como desejado.
#
Reciprocamente, dado f : X → Spec(A), definindo φ = fSpec A e repetindo a construção acima,
obtemos um morfismo f ′ : X → Spec(A) tal que f ′ |Uα = f |Uα para todo α, portanto f ′ = f .
153

2.3 Esquemas Projetivos


Anéis graduados são uma grande fonte de esquemas, os chamados esquemas projetivos, que estão
para os conjuntos algébricos projetivos assim como os esquemas afins estão para os conjuntos algébricos
afins.
L
Seja A = d∈Z Ad um anel graduado e S um conjunto multiplicativo cujos elementos são todos
homogêneos. Então S −1 A é também é um anel graduado: definimos deg(a/s) = deg(a) − deg(s) para
a ∈ A e s ∈ S elementos homogêneos. Para um elemento homogêneo h ∈ A, definimos o anel
def
A(h) = {f ∈ Ah | deg(f ) = 0}

Por exemplo, se A = C[x1 , x2 , . . . , xn ] e h = x1 então A(h) = C[ xx21 , . . . , xxn1 ].


Para p ∈ Proj A, seja S o conjunto multiplicativo de todos os elementos homogêneos em A \ p.
Definimos
def
A(p) = {f ∈ S −1 A | deg(f ) = 0}
Agora suponha que A seja N graduado, i.e., Ad = 0 para d < 0. Definimos agora um feixe sobre
Proj A. Para todo aberto U ⊂ Proj A, seja
 existe uma cobertura aberta U = S D (h ) de U por abertos 
 Y i + i 
def n
OProj A (U ) = (sp )p∈U ∈ A(p) básicos (com hi ∈ A homogêneos) e elementos ai /hi i ∈ A(hi ) ,
 a ∈ A, de modo que s = a /hni ∈ A para todo q ∈ D (h ) 
p∈U i q i i (q) + i

Os mapas de restrição são definidos da maneira usual. Agora vamos mostrar que o espaço localmente
anular (Proj A, OProj A ) é um esquema, denominado o esquema projetivo associado ao anel graduado
A.
Lemma 2.3.1 Seja h ∈ A+ um elemento homogêneo. Temos um homeomorfismo

f : D+ (h) - Spec A(h)


p 7→ ph ∩ A(h)

Além disso, se p ∈ D+ (h), temos um isomorfismo A(p) = (A(h) )f (p) .


Prova Observe que se p ∈ D+ (h) então ph ∈ Spec Ah é portantoph ∩ A(h) ∈ Spec A(h) , de modo que f
está bem definido. Além disso, é fácil mostrar que f −1 D(a/hn ) = D+ (h) ∩ D+ (a), de modo que f é
contı́nuo.
Mostremos que f é sobrejetor. Seja ρ: A → Ah o mapa de localização. Seja q ∈ Spec A(h) e considere
√ √
FT
o ideal qAh de Ah ; afirmamos que este ideal é um ideal primo homogêneo de Ah e que p = ρ−1 ( qAh )
está em D+ (h) e que f (p) = q.
)
Primeiro observe que qAh é um ideal homogêneo de Ah , gerado √ por elementos de grau 0. O fato de
28


que qAh é um ideal homogêneo
L segue de um fato geral: o radical
P b√de qualquer ideal homogêneo
P b
7:

em um anel graduado B = d∈Z Bd é homogêneo. De fato, se d bd ∈ b, bd ∈ Bd , então ( d bd )N ∈ b


(1
RA

para algum N ; se d0 é o menor ı́ndice para o qual bd0 6= 0, temos que bN é a componente de grau d0 N
d0 √
P N N
P
em ( d bd ) , logo bd0 ∈ b pois b é homogêneo por hipótese. Assim, bd0 ∈ b. Subtraindo bd0 de d bd
0


01

e repetindo o argumento, eventualmente concluı́remos que cada bd ∈ b.


√ √
Vejamos que qAh ∈ Spec Ah . Este ideal é próprio, pois caso contrário terı́amos 1 ∈ qAh ⇐⇒
,2

1 ∈ qAh e olhando para os termos de grau 0 concluirı́amos que 1 ∈ q = (qAh ) ∩ A(h) , absurdo. Agora se

29

x = a/hn e y = b/hm , com a, b ∈ A homogêneos, são tais que xy ∈ qAh então, multiplicando por uma
D

potência conveniente de h, temos que para algum N


ct
,O

 N
adeg(h) bdeg(h) adeg(h) bdeg(h)
· ∈ qAh ⇒ ∈ q ou ∈q
ET

hdeg(a) hdeg(b) hdeg(a) hdeg(b)

(observe que adeg(h) /hdeg(a) e bdeg(h) /hdeg(b) pertencem a A(h) ). Suponha que tenhamos adeg(h) /hdeg(a) ∈

q; neste caso adeg(h) /1 ∈ qAh ⇒ x ∈ qAh , como desejado.

Agora mostraremos queP p = Spec(ρ)( qAh ) está em√ D+ (h) e que f (p) = q. Primeiro, p é um
√ ideal
primo homogêneo de A: se ad ∈ p, ad ∈ Ad , como qAh é homogêneo, então cada ρ(ad ) ∈ qAh e
154 Esquemas

portanto ad ∈ p para √ todo d. E como p está na imagem de Spec(ρ), temos que p 6∋ h, logo p ∈ D+ (h). funtor de pontos
Finalmente f (p) = qAh ∩ A(h) , que é igual a q: claramente f (p) ⊃ q e se x ∈ f (p) então xN ∈ qAh
para algum N e como deg(x) = 0, temos xN ∈ q = (qAh ) ∩ A(h) ⇐⇒ x ∈ q. Isto completa a prova de
que f é sobrejetor.
Em seguida, vamos mostrar que para ideais homogêneos a, p com p primo temos

f (p) ⊃ f (a) ⇐⇒ p ⊃ a

Aqui estendemos a definição de f para ideias homogêneos arbitrários: f (a) = ah ∩ A(h) . Se provarmos
a equivalência acima, mostraremos que f : D+ (h) → Spec A(h) é injetor e que é um mapa fechado, o que
completará a prova de que f é um homeomorfismo. A implicação ⇐ é trivial. Suponha que f (p) ⊃ f (a)
e seja a ∈ a um elemento homogêneo. Então adeg(h) /hdeg(a) ∈ f (a) ⊂ f (p) e isto implica que a ∈ p, como
querı́amos.
Por fim, dado p ∈ D+ (h), podemos definir morfimos de anéis

A(p) → (A(h) )f (p)


a adeg(h) /hdeg(a) a ∈ A, s ∈ A \ p homogêneos com deg(a) = deg(s)
7→ deg(h) deg(s)
s s /h
e
(A(h) )f (p) → A(p)
a/hm ahn a ∈ A, s ∈ A \ p homogêneos com deg(a) = m deg(h) e deg(s) = n deg(h)
n
7→ m
s/h sh
Agora uma verificação rotineira (lição de casa!) mostra que os mapas acima estão bem definidos (não
dependem dos representantes de classe das frações) e que são inversos um do outro, de modo que temos
um isomorfismo A(p) = (A(h) )f (p) .

Como os abertos D+ (h) formam uma base da topologia de Zariski em Proj A, para provar que
(Proj A, OProj A ) é um esquema basta mostrarmos que há um isomorfismo de espaços localmente anulares
(f, f # ): (D+ (h), OProj A |D (h) )

- (Spec A(h) , OA ). Já sabemos que f como no lema é um home-
+ (h)

omorfismo. Definimos f # : OA(h) → f∗ OProj A |D+ (h) utilizando o isomorfismo φp : (A(h) )f (p)

- A(p) :
para um aberto U ⊂ Spec A(h) ,
fU# : OA(h) (U ) → OProj A (f −1 U )
(sq )q∈U 7→ (tp )p∈f −1 U
FT
onde
tp = φp (sf (p) ) ∈ A(p)
)
Utilizando o lema, como no caso afim é fácil verificar que este mapa está bem definido, i.e., que a tupla
28

(tp )p∈f −1 U é localmente igual a elementos em A(g) para vários g’s. Como por construção f # é um
7:

isomorfismo nos talos, temos que f # é um isomorfismo de feixes e portanto (f, f # ) é um isomorfismo de
(1
RA

espaços localmente anulares.


0

Exemplo 2.3.2 (Reta Projetiva Revisitada) automorfismo Möbius


01

3 Funtor de Pontos e Produto Fibrado


,2

Até o momento, vimos esquemas como uma colagem de anéis. Outro ponto de vista muito importante,
29

talvez o mais natural sob a ótica geométrica, é o chamado funtor de pontos associado a um esquema,
D

que permite vê-lo como um objeto geométrico “estratificado” segundo os “domı́nios de definição” de seus
ct

pontos.
,O

Definição 3.1 Seja S um esquema e seja X um S-esquema. O funtor representado por X, a saber,
ET

def
X(−) = HomSch/S (−, X)

é chamado funtor de pontos de X. Um elemento de X(T ) é chamado de ponto com valores em T


de X. Se T = Spec A, abreviaremos X(Spec A) por X(A) e referiremo-nos aos elementos de X(A) como
pontos com valores em A.
155

Exemplo 3.2 (Conjuntos Algébricos Afins) Seja k um corpo e considere a k-álgebra anel de núme

k[x1 , . . . , xn ]
A= fi ∈ k[x1 , . . . , xn ]
(f1 , . . . , fm )

Considere o k-esquema afim associado X = Spec A. Seja l ⊃ k uma extensão de corpos. Temos que os
pontos com valores em l de X são dados por

X(l) = HomSch/k (Spec l, X) = Homk-alg (A, l)

Mas dar um morfismo de k-álgebras φ: A → l é o mesmo que dar uma n-upla (a1 , . . . , an ) ∈ ln tal
que fi (a1 , . . . , an ) = 0 para todo i = 1, . . . , m, já que neste caso podemos definir φ(x̄i ) = ai . Em
outras palavras, φ corresponde a um ponto com coordenadas em l do conjunto algébrico definido pelos
polinômios fi : temos uma bijeção natural

X(l) = {(a1 , . . . , an ) ∈ ln | fi (a1 , . . . , an ) = 0 para todo i = 1, . . . , m}

Ou seja, um ponto com valores em l de X é realmente um ponto com valores em l! Podemos assim
pensar em um k-esquema X como um “conjunto algébrico definido sobre k”, mesmo quando k não é
algébricamente fechado. Por exemplo, para o R-esquema X = Spec R[x, y]/(x2 + y 2 + 1) temos X(R) = ∅
enquanto que X(C) é o “cı́rculo complexo de raio i”. Supimpa, né?
Exemplo 3.3 (Pontos com Valores sobre o Anel de Números Duais) Sejam k, A e X = Spec A
como no exemplo anterior. Considere a k-álgebra B = k[t]/(t2 ), chamada de anel números duais.
Seguindo a tradição, denotaremos por ǫ a imagem de t em B (notação esta remonta à pré-história; em
algumas pinturas paleolı́ticas, vê-se o desenho de um homem matando um bisão e, ao lado, o anel de
números duais B = k[ǫ]).
Como são os elementos de X(k[ǫ]) = HomSch/k (Spec k[ǫ], A) = Homk−alg (A, k[ǫ])? Dar um morfismo de
k-álgebras φ: A → k[ǫ] agora corresponde a dar uma n-upla (a1 + b1 ǫ, . . . , an + bn ǫ) ∈ k[ǫ]n (aj , bj ∈ k)
tal que fi (a1 + b1 ǫ, . . . , an + bn ǫ) = 0 em k[ǫ] para todo i = 1, . . . , m. Como ǫ2 = 0, aplicando a fórmula
de Taylor, obtemos
X ∂fi
fi (a1 + b1 ǫ, . . . , an + bn ǫ) = fi (a1 , . . . , an ) + (a1 , . . . , an ) · bj ǫ
∂xj
1≤j≤n

que é 0 em k[ǫ] para todo i se, e só se, (a1 , . . . , an ) é um ponto com coordenadas em k do conjunto
algébrico definido por fi (x1 , . . . , xn ) = 0, i = 1, . . . , m, e (b1 , . . . , bn ) é um “vetor tangente” ao conjunto
FT
algébrico neste ponto, também com coordenadas em k.
Por exemplo, para o R-esquema X = R[x, y]/(x2 − y) (uma “parábola”), temos que um R[ǫ]-ponto de X
)
28

é um par consistindo de um ponto real da parábola (a, a2 ), a ∈ R, e um vetor tangente real b · (1, 2a),
b ∈ R. Geometricamente, temos a seguinte “explicação”: Spec R[ǫ] é um “ponto gordo” obtido a partir
7:

da “reta real” Spec R[x] “quase” fazendo-se x = 0, de modo que há certo “material infinitesimal extra”
(1
RA

transbordando para os lados, nas direções normais ao “ponto” Spec R[x]/(x). Assim, para especificar
um morfismo Spec R[ǫ] → X precisamos dizer não só a imagem do “ponto” Spec R[x]/(x) mas também
0
01

a imagem da “parte infinitesimal”, que corresponde ao vetor tangente.


,2

Definição 3.4 Seja X um esquema e x ∈ X. O corpo residual do anel local OX,x é denotado por k(x).
29

Lemma 3.5 Seja X um esquema e l um corpo. Temos uma bijeção natural


D
ct


X(l) = (x, φ) x ∈ X e φ ∈ HomRings (k(x), l)
,O
ET

Prova Seja η = (0) o único elemento em Spec l. A cada elemento (f, f # ): (Spec l, Ol ) → (X, OX ) de
X(l), associamos o ponto x = f (η) ∈ X e o morfismo φ: k(x) → l induzido pelo morfismo de talos
fη# : OX,x → Ol,η = l (note que como este morfismo é local por definição, o ideal maximal mx de OX,x
pertence ao kernel de fη# , logo podemos definir φ(a) = fη# (a) para a ∈ OX,x ). Reciprocamente, dado
um par (x, φ) como acima, podemos definir um morfismo (f, f # ): (Spec l, Ol ) → (X, OX ) decretando
156 Esquemas

f (η) = x, de modo que f∗ Ol é agora “concentrado sobre um único ponto x” (às vezes denominado “feixe produto fibrado
arranha-céu” sobre x): para um aberto U ⊂ X,
n
l se x ∈ U
f∗ Ol (U ) =
0 caso contrário
Assim, para especificar f # : OX → f∗ Ol , basta considerar os abertos U que contém x, para os quais
definimos fU# : OX (U ) → f∗ Ol (U ) = l como a composição

OX (U ) - OX,x -
- k(x) - l
φ

É fácil verificar que, com estas definições, (f, f # ) é um morfismo de esquemas e que as duas associações
acima descritas são inversas uma da outra.
def
Exemplo 3.6 (Reta Projetiva) Seja k um corpo e seja o k-esquema X = P1k = Proj k[x0 , x1 ]. Se
l ⊃ k uma extensão de corpos
Um morfimo de S-esquemas f : X → Y define, por composição, um morfismo entre os funtores de
pontos associados, que ainda denotamos por f : para um S-esquema T ,
f : X(T ) = HomS (T, X) → Y (T ) = HomS (T, Y )
φ 7→ f ◦ φ
Segue imediatamente
S de lema 1.2.7 que o funtor de pontos se comporta como um “feixe na categoria
Sch/S”: se T = α Tα é uma cobertura aberta de T , temos que
n Y o

X(T ) = (tα ) ∈ X(Tα ) tα |Tα ∩Tβ = tβ |Tα ∩Tβ para todo par (α, β)
α

Teorema 3.7 (Princı́pio Yoneda) Sejam X e Y dois S-esquemas. Dar um S-morfismo f : X → Y é


o mesmo que dar um morfismo de seus funtores de pontos fT : X(T ) → Y (T ) para todo S-esquema afim
T.
Exemplo 3.8 möbius em P1A .
Definição 3.9 Seja C uma categoria e seja S ∈ C um objeto desta categoria. Um S-objeto é um
morfismo T → S; um morfismo de S-objetos entre φ: X → S e ψ: Y → S é um morfismo f : X → Y
que é “compatı́vel com os morfismos base”, ou seja, tal que ψ◦f = φ. O conjunto de todos os S-morfismos
entre φ e ψ será denotado por HomS (X, T ). Os S-objetos definem uma subcategoria plena de C que
denotamos por C/S. Dado um S-objeto X → S, definimos o funtor de pontos associado da maneira
FT
usual:
X: C/S → Sets
)
T 7→ HomS (X, T )
28

Dados dois S-objetos φ: X → S e ψ: Y → S, dizemos que um objeto X ×S Y ∈ C é o produto


7:

fibrado de X e Y sobre S se X ×S Y representa o produto cartesiano dos funtores de pontos de X e


(1
RA

Y . Explicitamente: X ×S Y vem equipado de fábrica com dois S-morfismos projeção p: X ×S Y → X e


0

q: X ×S Y → Y de modo que temos uma bijeção natural


01

X ×S Y (T ) = X(T ) × Y (T )
,2

t 7→ (p ◦ t, q ◦ t)
29

Em outras palavras, dados dois S-morfismos f : T → X e g: T → Y , existe um único S-morfismo f ×g: T →


D
ct

X ×S Y para o qual o seguinte diagrama comuta


,O

X ×S Y
6
ET

∃!f × g
 f g -
X - T  Y

φ - ? ψ
S
157

Exemplo 3.10 Na categoria de conjuntos ou espaços topológicos, o produto fibrado de dois S-objetos
φ: X → S e ψ: Y → S sempre existe e é dado por

X ×S Y = {(x, y) ∈ X × Y | φ(x) = ψ(y)}

com os mapas de projeção p: X ×S Y → X e q: X ×S Y → X usuais: p(x, y) = x e q(x, y) = y.


O produto fibrado possui interpretações interessantes para vários casos particulares. Por exemplo, na
categoria de espaços topológicos, se X é um subespaço de S e φ: X ֒→ S é o morfismo de inclusão, então
X ×S Y é homeomorfo a ψ −1 X, a projeção q: X ×S Y ֒→ Y é dada pela inclusão ψ −1 X ֒→ Y enquanto
que a projeção p: X ×S Y → X é dada pela restrição de ψ a ψ −1 X:

q -
ψ −1 X ==== X ×S Y ⊂
Y

ψ|ψ−1 X p ψ
? ? φ - ?
X ======== X ⊂ S
Em geral, podemos pensar na flecha da esquerda como uma “mudança de base” da flecha da direita; a
restrição é apenas um caso particular desta operação.
Queremos mostrar que o produto fibrado sempre existe na categoria de esquemas. Começamos com
um simples
Lemma 3.11 Sejam A e B duas C-álgebras. Então

Spec A ×Spec C Spec B = Spec(A ⊗C B)

onde os morfismos de projeção p: Spec(A ⊗C B) → Spec A e q: Spec(A ⊗C B) → Spec B são induzidos


pelos morfismos naturais de C-álgebras

α: A → A ⊗C B β: B → A ⊗C B
e
a 7→ a ⊗ 1 b 7→ 1 ⊗ b

Prova Temos que mostrar que, para todo (Spec C)-esquema T , o mapa

HomSpec C (T, Spec(A ⊗C B)) → HomSpec C (T, Spec A) × HomSpec C (T, Spec B)
FT
t 7→ (p ◦ t, q ◦ t)

é uma bijeção. Mas pelo teorema 2.2.9, isto equivale a mostrar que
)
28

HomC−alg (A ⊗C B, OT (T )) → HomC−alg (A, OT (T )) × HomC−alg (B, OT (T ))


7:

τ 7→ (τ ◦ α, τ ◦ β)
(1
RA

é uma bijeção, o que é imediato pela propriedade universal do produto tensorial.


0
01
,2

Lemma 3.12 Sejam U ⊂ X e V ⊂ Y dois subesquemas abertos dos S-esquemas X e Y . Se X ×S Y


existe, então
29

U ×S V = p−1 (U ) ∩ q −1 (V ) ⊂ X ×S Y
D
ct

onde p e q são as projeções de X ×S Y para X e Y , respectivamente.


,O

Prova Seja W = p−1 (U ) ∩ q −1 (V ). Pelo lema 2.2.8, temos as seguintes bijeções naturais:
ET

HomS (T, W ) = {τ ∈ HomS (T, X ×S Y ) | τ (T ) ⊂ W }


= {(f, g) ∈ HomS (T, X) × HomS (T, Y ) | f (T ) ⊂ U e g(T ) ⊂ V }
= HomS (T, U ) × HomS (T, V )
158 Esquemas

Agora podemos finalmente mostrar que


Teorema 3.13 Sejam X e Y dois S-esquemas. Então X ×S Y existe na categoria de S-esquemas.
Prova A ideia da demonstração é “colar” os diversos produtos fibrados de subesquemas abertos de X
e Y a fim de obter X ×S Y . A condição de cociclo (ver lema 1.2.6) é automaticamente verificada pela
propriedade universal da definição de produto fibrado.
S
Vamos primeiramente mostrar que se X = α Xα é uma cobertura aberta de X e cada produto
fibrado (Xα ×S Y, pα , qα ) existe então X ×S Y também existe. Defina, para cada par (α, β),

def
Uαβ = p−1
α (Xα ∩ Xβ ) ⊂ Xα ×S Y

Pelo lema anterior, temos que ambos Uαβ e Uβα são produtos fibrados de Xα ∩ Xβ e Y sobre S, portanto
há um único S-isomorfismo φαβ : Uαβ - Uβα compatı́vel com os morfismos de projeção. Em particular,

da unicidade temos φαα = id e φαβ = φ−1


βα . Além disso, para cada tripla (α, β, γ),

def
Uαβγ = Uαβ ∩ Uαγ = p−1
α (Xα ∩ Xβ ∩ Xγ ) ⊂ Xα ×S Y
def
Uγαβ = Uγα ∩ Uγβ = p−1
γ (Xα ∩ Xβ ∩ Xγ ) ⊂ Xγ ×S Y

são todos produtos fibrados de Xα ∩Xβ ∩Xγ e Y sobre S. Como as restrições de φαγ e φβγ ◦φαβ fornecem
isomorfismos entre Uαβγ e Uγαβ compatı́veis com os mapas de projeção, novamente pela unicidade da
propriedade universal temos que φαγ = φβγ ◦ φαβ . Assim, as condições de cociclo se verificam e podemos
colar os esquemas Xα ×S Y de modo a obter um S-esquema X ×S Y , bem como colar os mapas de projeção
pα , qα de modo a obter mapas p: X ×S Y → X e q: X ×S Y → Y . Para mostrar que (X ×S Y, p, q) é
realmente o produto fibrado de X e Y sobre S, identificamos inicialmente Xα ×S Y com os subesquemas
abertos de X ×S Y correspondentes. Agora, dado um S-esquema T e dois pontos f ∈ X(T ) e g ∈ Y (T ),
sendo Tα = f −1 (Xα ×S Y ) e fα = f |Tα , temos que para cada α existe um único ponto tα ∈ Xα ×S Y (Tα )
tal que p ◦ tα = fα e q ◦ tα = g. Novamente pela unicidade da propriedade universal, temos que
tα |Tα ∩Tβ = tβ |Tα ∩Tβ , de modo que temos um único ponto t ∈ X ×S Y (T ) tal que p ◦ t = f e q ◦ t = g,
como desejado.
Agora suponha que S é afim. Se Y é afim, como o produto fibrados de dois S-esquemas afins existe,
pela construção acima X ×S Y existe. Aplicando novamente o argumento acima com X e Y trocados
concluı́mos que o produto fibrado X ×S Y existe para X e Y arbitrários.
Para um S esquema qualquer, seja Sα uma cobertura afim de S. Denote por f : X → S e g: Y → S
os morfismos base e sejam Xα = f −1 (Sα ) e Yα = g −1 (Sα ). Já sabemos que Xα ×Sα Yα existem. Note
que temos um isomorfismo Xα ×Sα Yα = Xα ×S Y . Finalmente, aplicando o argumento inicial mais uma
FT
vez, obtemos que X ×S Y existe no caso geral.
)
28

Exemplo 3.14 Seja B uma A-álgebra. Então P1A ×Spec A Spec B = P1B .
7:

Exemplo 3.15 P1A ×Spec A P1A = Proj A[x, y, z]/()


(1
RA

Definição 3.16 Seja f : X → Y um morfismo de esquemas. Seja y ∈ Y . A fibra de y com relação a f


0

é o k(y)-esquema X ×Y Spec k(y).


01

4 Propriedades Locais
,2
29

Lema 4.1 (Mudança de Carta) Seja X um esquema e sejam U e V dois abertos afins de X com
D

isomorfismos
ct

i: U

- Spec A e j: V

- Spec B
,O

Então para todo x ∈ U ∩ V existe uma vizinhaça aberta W ⊂ U ∩ V de x que é simultaneamente um


ET

aberto principal tanto em Spec A como em Spec B, ou seja, existem h ∈ A, g ∈ B tais que as restrições
de i e j induzem isomorfismos

i|W : W

- D(h) = Spec Ah e j|W : W

- D(g) = Spec Bg
159

Prova Escolha h′ ∈ A tal que i(x) ∈ D(h′ ) e D(h′ ) ⊂ i(U ∩ V); em seguida, escolha g ∈ B tal que
j(x) ∈ D(g) e D(g) ⊂ j i−1 (D(h′ )) . Definamos W = j −1 D(g) ; basta agora mostrarmos que i(W ) é
um aberto principal em Spec A. Temos um morfismo de esquemas afins dado pela composição
i−1
Spec Ah′ = D(h′ ) - U ∩V - Spec B
j


Seja φ: B → Ah′ o morfismo de anéis correspondente. Temos que i(W ) = Spec(φ)−1 D(g) = D(φ(g)) ⊂
n
Spec Ah′ é um aberto principal quando visto como subesquema de Spec Ah′ . Se φ(g) = h/h′ , h ∈ A,
′ ′
temos portanto que i(W ) = D(hh ) = D(h) ∩ D(h ) ⊂ Spec A também é principal quando visto como
subesquema de Spec A.

Como esquemas afins são compactos, obtemos o seguinte


S
Corolário 4.2 Seja X um esquema e Ui = X uma cobertura afim de X com Ui = Spec Ai . Seja
U = Spec B um aberto afim de X. Então U possui uma cobertura finita da forma
U = Vi1 ∪ · · · ∪ Vin com Vik = Spec(Aik )hik = Spec Bgi
com hik ∈ Aik e gi ∈ B.
Definição 4.3 Um esquema X é
1. localmente noetheriano se OX (U ) é um anel noetheriano para todo U afim;
2. reduzido se OX,x é um anel reduzido para todo x;
3. conexo se o espaço topológico X é conexo;
4. irredutı́vel se o espaço topológico X é irredutı́vel;
5. integral se X é reduzido e irredutı́vel;
6. regular se OX,x é um anel regular para todo x ∈ X;
7. Cohen-Macauley se OX,x é um anel Cohen-Macauley para todo x ∈ X.

Teorema 4.4 A propriedade de ser localmente noetheriano


S é local: um esquema X é localmente noethe-
riano se, e só se, X admite uma cobertura X = Ui onde Ui = Spec Ai com Ai anéis noetherianos.
S
Prova Suponha que X admita uma cobertura afim X = Ui onde Ui = Spec Ai com Ai anéis noethe-
rianos. Dado um aberto afim U = Spec B qualquer de X, devemos mostrar que B é noetheriano. Como
a localização de um anel noetheriano é noetheriano,
S pelo corolário 4.2 podemos assumir que U = Spec B
admite uma cobertura aberta finita U = 1≤i≤m D(gi ) com gi ∈ B e tal que cada Bgi é noetheriano.
Agora considere uma cadeia ascendente de ideais b1 ⊂ b2 ⊂ · · · de B. Como os Bgi ’s são noetheri-
anos e há um número finito de tais anéis, existe n0 tal que n ≥ n0 implica (bn )gi = (bn+1 )gi para
FT
todo i = 1, . . . , m. Como os abertos principais D(gi ) cobrem Spec B, isto implica que se n ≥ n0 então
(bn )p = (bn+1 )p para todo p ∈ Spec A. Pelo princı́pio local-global (teorema III.1.8), temos portanto que
)
bn = bn+1 para todo n ≥ n0 .
28
7:

Definição 4.5 Um morfismo f : X → Y de esquemas é


(1
RA

1. localmente de tipo finito se, dado qualquer aberto afim V = Spec A de Y e qualquer aberto
afim U = Spec B de f −1 (V ), temos que B é uma A-álgebra finitamente gerada via o morfismo
0
01

de anéis A → B correspondente ao morfismo de esquemas afins f |U : U → V ;


2. plano se fx# : OY,f (x) → Ox é plano para todo x ∈ X;
,2

3. fielmente plano se f é plano e sobrejetor;


29

4. afim se f −1 (V ) é afim para qualquer aberto afim V de Y ;


D
ct

5. finito se f é afim e se, para todo aberto afim V = Spec A de Y , f −1 (V ) = Spec B com B uma
,O

A-álgebra finita via o morfismo de anéis A → B correspondente ao morfismo de esquemas afins


f |U : U → V ;
ET

6. não ramificado
7. étale se f é plano e não ramificado;
8. suave
9. quase-finito
160 Esquemas

Teorema 4.6 A propriedade de ser localmente finito éSlocal: um morfismo f : X → Y é localmente de tipo
finito se, e só se, Y admite uma cobertura
S afim Y = i Vi com Vi = Spec Ai tal que, para cada i, f −1 Vi
−1
admite uma cobertura afim f Vi = j Uij com Uij = Spec Bij onde Bij é uma Ai -álgebra finitamente
gerada via o morfismo de anéis Ai → Bij correspondente ao morfismo de esquemas Spec Bij → Spec Ai
dado pela restrição de f .
Prova Suponha que Y admita uma cobertura afim como a do enunciado e seja V = Spec A um aberto
afim qualquer de Y e U = Spec B um aberto afim de f −1 (V ); temos que mostrar que B é uma álgebra fini-
tamente gerada sobre A. Como a localização (Bij )hi de Bij em um elemento hi ∈ Ai é uma (Ai )hi -álgebra
finitamente gerada, substituindo Y por V podemos supor que Y = Spec A. Aplicando o corolário 4.2,
podemos ainda assumir que Y é coberto por um número finito de abertos principais D(hi ) ⊂ Spec A
e que Ai = Ahi (hi ∈ A). Observando ainda que os mapas de localização A → Ahi e Bij → (Bij )gij
são álgebras finitamente geradas, mais uma aplicação do corolário 4.2 mostra que podemos até mesmo
assumir que B admite uma cobertura finita por abertos principais D(gj ) onde Bgj é uma A-álgebra
finitamente gerada (gj ∈ B).
n S
Sejam ωjk /gj jk geradores de Bgj sobre A (ωjk ∈ B). Temos que Spec B = j D(gj ) implica que
P
existem bj ∈ B tais que j bj gj = 1. Afirmamos que os elementos bj , gj , ωjk geram a A-álgebra B.
P
Primeiramente, observe que elevando j bj gj = 1 a uma potência suficientemente grande, para qualquer
P
n temos uma expressão da forma j Tj (bj , gj ) · gjn = 1 com Tj (bj , gj ) um polinômio em bj e gj com
coeficientes em A. Agora, dado b ∈ B, como cada Bgj é finitamente gerado sobre A, existe um n tal que
gjn b = Pj (ωjk , gj ) em B para algum polinômio Pj (ωjk , gj ) em ωjk e gj com coeficientes em A. Portanto
X X
b= Tj (bj , gj ) · gjn b = Tj (bj , gj ) · Pj (ωjk , gj )
j j

pertence à A-álgebra gerada por ωjk , bj , gj , como afirmamos.

Virtualmente qualquer propriedade/construção sobre anéis que se comporta bem com relação à
localização é “globalizável”. Temos o seguinte dicionário:
Anéis Esquemas
Ideal Primo Ponto
Localização em um primo Talo
Produto Tensorial Produto Fibrado
Localização em um elemento Subesquema Aberto
Quociente Subesquema Fechado
Anel Noetheriano Esquema Noetheriano
Domı́nio Esquema Integral
FT
Anel Reduzido Esquema Reduzido
Anel Indecomponı́vel Esquema Conexo
)
Álgebra Finita Morfismo Finito
28

Álgebra finitamente gerada Morfismo de tipo finito


7:

Módulo Módulo Quase-coerente


(1
RA

Módulo finitemente presentado Módulo Coerente


0

Observação 4.7 Existem duas propriedades globais (relativos à cola). Um S-esquema é separado se
01

o morfismo diagonal id × id: X → X ×S X é uma imersão fechada. Dizemos que X é próprio sobre S
,2

se X é separado e universalmente fechado: para todo S-esquema T , o morfismo X ×S T → T é fechado.


29

5 Módulos quase-coerentes
D
ct

Definição 5.1 Seja X um esquema. Um OX -módulo M é dito quase-coerente se para todo aberto
afim U = Spec A de X existe um A-módulo M e um isomorfismo de OA -módulos M|U ∼ f. Se, além
,O

=M
disso, M é de presentação finita, dizemos que M é coerente. Analogamente definimos OX -álgebras
ET

quase-coerentes e coerentes.

Exemplo 5.2 Seja X = P1A e considere o A[x0 , x1 ]-módulo graduado N = A[x0 , x1 ] [n]. Definimos
e , um OP1 -módulo coerente.
OP1A (n) = N A

Exemplo 5.3 OP1A (m) ⊗ OP1A (n) = OP1A (m + n)


161

Lemma 5.4 A propriedade de ser quase-coerente (resp. coerente)


S é local: M é quase-coerente (resp.
coerente) se, e só se, existe uma cobertura aberta afim X = i Ui com Ui = Spec Ai e Ai -módulos Mi
tais que M|Ui ∼=M fi para todo i.

Prova Seja U = Spec A um aberto afim qualquer de X.

subesquemas fechados
Teorema 5.5 Seja f : X → Y uma função contı́nua. O funtor imagem direta f∗ : Sh(X) → Sh(Y ) possui
um adjunto à esquerda, o funtor imagem inversa f −1 .
HomSh(Y ) (F , f∗ G) = HomSh(X) (f −1 F , G)

Prova Defina
Γ(V, f −1 F ) = −→
lim F (U )
U⊃f (V )

Exemplo 5.6 talo


Definição 5.7 Seja f : X → Y um morfismo de esquemas e N um OY -módulo. Definimos o pull-back
f ∗ N de N como o OX -módulo
f ∗ N = f −1 N ⊗f −1 OX OX

Lemma 5.8 Seja B uma A-álgebra e f : Spec B → Spec A o morfismo de esquemas afins correspondente.
Seja M um A-módulo. Então
f) = M^
f ∗ (M ⊗A B

Prova Seja g ∈ B.

6 Fibrados Vetoriais e Módulos Localmente Livres


Antes de tratarmos fibrados algébricos vamos rever (ou ver, dependendo do caso) a teoria clássica. Seja
X um espaço topológico. Um fibrado vetorial (real) de posto n sobre X é uma “famı́lia contı́nua”
de espaços vetoriais reais de dimensão n sobre X. Mais precisamente, temos uma função contı́nua
π: E ։ X tal que as fibras φ−1 x ⊂ E de cada ponto x ∈ X possuem a estrutura de um R-espaço
vetorial de dimensão n e π é localmente isomorfo à projeção p1 : U × Rn ։ U , i.e., cada x ∈ X possui
uma vizinhança aberta U ⊂ X juntamente com um homeomorfismo ξU : π −1 U

- U × Rn fazendo o
seguinte diagrama comutar
FT
ξU- −1 - E
U × Rn π U ⊂
)

28
7:

p1 π π
(1
RA

? ? ?
U ========= U ⊂ - X
0
01

e de modo que ξU induz um isomorfismo ξU,x : {x} × Rn



- π −1 x de espaços vetoriais entre as fibras
,2

para cada ponto x ∈ U .


Dizemos que dois fibrados vetoriais π1 : E1 → X e π2 : E2 → X são de posto n são isomorfos se existe
29

um homeomorfismo φ: E1 - E2 fazendo o seguinte diagram comutar



D
ct

φ -
,O

E1 E2

ET

π1 π2
? ?
X ======== X

de modo que o morfismo induzido das fibras φx : π1−1 x - π −1 x é R-linear para cada x ∈ X.

2
162 Esquemas

Exemplo 6.1 Sobre o cı́rculo S 1 , a projeção p1 : S 1 × R → S 1 é um fibrado vetorial de posto 1, o


fibrado trivial. Mas também a projeção π: M → S 1 da “faixa de Möbius infinita” M sobre o cı́rculo
mediano também é um outro fibrado vetorial de posto 1 sobre S 1 : aqui M é o quociente de [0, 1] × R
obtido fazendo a identificação (0, y) ∼ (1, −y) para todo y ∈ R; S 1 é a imagem de [0, 1] × {0} em M e
π: M → S 1 é dado por π(x, y) = x. Embora localmente um produto da forma U × R, este fibrado não é
globalmente um produto S 1 × R (isto é, não é isomorfo ao fibrado trivial) já que M não é orientável, ao
contrário de S 1 × R.

Exemplo 6.2 Sobre a esfera S 2 = {(x, y, z) ∈ R3 | x2 + y 2 + z 2 = 1}, considere

def
T = {(p, v) ∈ S 2 × R3 | v é tangente a S 2 em p}
= {(p, v) ∈ S 2 × R3 | p · v = 0}

e seja π: T ։ S 2 o mapa de projeção. É fácil ver que, localmente, T é da forma U × R2 , de modo que
π é um fibrado vetorial de posto 2 sobre S 2 , o chamado fibrado tangente. Veremos a seguir que este
fibrado não é trivial. Temos também
def
N = {(p, v) ∈ S 2 × R3 | v é normal a S 2 em p}
= {(p, λp) ∈ S 2 × R3 | λ ∈ R}

Se τ : N → S 2 é o mapa de projeção, temos que τ é um fibrado vetorial de posto 1, o chamado fibrado


normal de S 2 ⊂ R3 . Este último fibrado é isomorfo ao trivial via

- S2 × R

FT
ξ: N
(p, λp) 7→ (p, λ)
)
28

Por definição, para qualquer fibrado vetorial π: E → X existe uma cobertura aberta U = {Ui } de X
que trivializa π, isto é, π restrito a cada Ui é isomorfo a um fibrado trivial Ui × Rn ։ Ui . Esta simples
7:

observação é a base da receita da construção de todos os fibrados trivializados por U.


(1
RA

Seja
0

ξi : Ui × Rn - π −1 Ui

01

uma trivialização do fibrado π: E → X com relação à cobertura aberta U = {Ui }. Então, para cada
,2

par (i, j), temos um automorfismo de fibrado vetoriais φij : (Ui ∩ Uj ) × Rn



- (Ui ∩ Uj ) × Rn sobre
29

Ui ∩ Uj dado pela composição


D
ct

ξj−1
- π −1 (Ui ∩ Uj )
ξi
- (Ui ∩ Uj ) × Rn
,O

φij : (Ui ∩ Uj ) × Rn
ET

onde, para não sobrecarregar a notação, ainda denotamos por ξi e ξj−1 as suas restrições a (Ui ∩ Uj ) × Rn
e π −1 (Ui ∩ Uj ) respectivamente. Estes automorfismos clearmente satisfazem as seguintes relações de
compatibilidade:
1. φii = id para todo i;
−1
2. φij = φji para todo par i, j;
163

3. (condição de cociclo) para toda tripla i, j, k,

φik = φjk ◦ φij

sobre Ui ∩ Uj ∩ Uk .
Uma coleção de automorfismos {φij } satisfazendo as condições acima é chamada de 1-cociclo de Čech.
Associamos um 1-cociclo de Čech a cada trivialização de um fibrado vetorial. Reciprocamente, dado um
1-cociclo de Čech {φij }, podemos construir um fibrado π: E → X “colando” os fibrados triviais Ui × Rn
com ajuda dos automorfismos φij : tome E como o quociente
F
i (Ui × Rn )
E=

da união disjunta dos Ui × Rn pela relação de equivalência

v ∼ w para v ∈ Ui × Rn e w ∈ Uj × Rn ⇐⇒ φij (v) = w sobre Ui ∩ Uj

As condições 1–3 acima asseguram que ∼ é de fato uma relação de equivalência, de modo que tudo
está bem definido. Agora os mapas de projeção Ui × Rn → Ui induzem um mapa global de projeção
π: E → X, que é o fibrado vetorial prometido.
Exemplo 6.3 Seja ǫ > 0 pequeno e considere a seguinte cobertura aberta de S 2 (formada pelos hem-
isférios norte e sul):

2 def def
S+ = {(x, y, z) ∈ R3 | x2 + y 2 + z 2 = 1 e z ≥ −ǫ} 2
S− = {(x, y, z) ∈ R3 | x2 + y 2 + z 2 = 1 e z ≤ +ǫ}

trivialização do fibrado tangente π: T → S 2 de

Observação 6.4 Seja λ: E



- F um isomorfismo de fibrados e sejam ξi : Ui × Rn ∼- π −1 Ui e
E
χi : Ui × Rn - πF−1 Ui trivializações de modo que φij = ξj−1 ◦ ξi e ψij = χ−1

j ◦ χ i sobre U i ∩ U j . Então,
sobre Ui , temos um automorfismo de fibrados µi : Ui × Rn → Ui × Rn dado pela composição

χ−1
µi : U i × R n
ξi
- π −1 Ui λ
- π −1 Ui i- U i × Rn
E F

Agora um cálculo simples mostra que, para todo i, j,


FT
ψij = µj ◦ φij ◦ µ−1
i sobre Ui ∩ Uj

Caso haja automorfismos µi of Ui × Rn tais que a relação acima se verifica, dizemos que os 1-cociclos
)
28

{ψij } e {φij } são cohomólogos. É fácil verificar que a relação acima é de equivalência e que temos uma
7:

bijeção ( )
classes de isomorfismo de fibra- n o
(1
RA

dos vetoriais de posto n trivial- ↔ classes de cohomologia


de U-Čech 1-cociclos
0

izados por U
01

7 Topologias de Grothendieck e Descenso Fielmente Plano


,2

F
Agora sejam X e U como na seção anterior e seja Y = i Ui a união disjunta dos Ui . Considere o mapa
29

h: Y → X induzido pelas inclusões Ui ֒→ X. Então temos homeomorfismos


D
ct

G G
,O

Y ×X Y = Ui ∩ Uj e Y ×X Y ×X Y = Ui ∩ Uj ∩ Uk
i,j i,j,k
ET

e mapas
12 p
p1
Y ×X Y ×X Y −→
h
−→
−→ Y ×X Y −→
−→
p2
Y −→ X
p23

Aqui p1 e p2 são os mapas de projeção e p12 (y1 , y2 , y3 ) = (y1 , y2 ), p13 (y1 , y2 , y3 ) = (y1 , y3 ) (a “flecha
do meio” na flecha tripla, não indicada por falta de espaço), p23 (y1 , y2 , y3 ) = (y2 , y3 ). Agora dar uma
164 Esquemas

colerção {φij } de automorfismos de fibrados (Ui ∩ Uj ) × Rn é o mesmo que dar um único automorfismo
de fibrados sobre Y ×X Y :
φ: Y ×X Y × Rn - Y ×X Y × Rn

A condição de 1-cociclo é agora resumida por uma única relação

p∗13 φ = p∗23 φ ◦ p∗12 φ

onde p∗ij φ denotam os pull-backs de φ com relação a pij , isto é, o morfismo em Y ×X Y ×X Y × Rn
induzido por φ na componente (i, j) e pela identidade na componente remanescente: por exemplo,
p∗13 φ(y1 , y2 , y3 , v) = (y1 , y2 , y3 , w) onde φ(y1 , y3 , v) = (y1 , y3 , w). Agora dois automorfismos φ e ψ são
cohomólogos se, e só se, existe um automorfismo µ: Y × Rn - Y × Rn tal que ψ = p∗ µ ◦ φ ◦ p∗ µ−1 .

2 1
Algebricamente, Y = Spec B terá o papel de união disjunta da cobertura de X = Spec A. Como
trabalhamos com anéis em vez de esquemas, as flechas serão invertidas e os pull-backs serão dados pelo
produto tensorial. Escrevemos

p1 : B → B ⊗ A B p2 : B → B ⊗ A B
b 7→ b ⊗ 1 b 7→ 1 ⊗ b

Se N é um B-módulo, sejam

p∗1 N = N ⊗B (B ⊗A B) = N ⊗A B e p∗2 N = (B ⊗A B) ⊗B N = B ⊗A N

os pull-backs de N , ou seja, os B ⊗A B-módulos obtidos tensorizando N com respeito às primeira e


segunda entradas respectivamente. Então p∗i N fará o papel do fibrado trivial Y × Rn . Seja

φ: p∗1 N

- p∗2 N

be um isomorfismo de B ⊗A B-módulos (φ terá o papel de um automorfismo de Y ×X Y × Rn ). Considere


os “mapas de projeção duais”

p12 : B ⊗A B → B ⊗A B ⊗A B p13 : B ⊗A B → B ⊗A B ⊗A B
b1 ⊗ b2 7→ b1 ⊗ b2 ⊗ 1 b1 ⊗ b2 7→ b1 ⊗ 1 ⊗ b2
FT
p23 : B ⊗A B → B ⊗A B ⊗A B
)
28

b1 ⊗ b2 7→ 1 ⊗ b1 ⊗ b2
7:

e os pull-backs de N para B ⊗A B ⊗A B, obtidos tensorizando o último anel com N sobre B com respeito
(1
RA

às primeira, segunda e terceira entradas de B ⊗A B ⊗A B, respectivamente:


0
01

N ⊗A B ⊗A B e B ⊗A N ⊗A B e B ⊗A B ⊗A N
,2

Tensorizando φ com id sobre B com respeito a terceira, segunda e primeira entradas de B ⊗A B ⊗A B,


29

obtemos morfismos de B ⊗A B ⊗A B-módulos


D
ct

P
,O

p∗12 φ: N ⊗A B ⊗A B → B ⊗A N ⊗A B p∗12 φ (n ⊗ 1 ⊗ 1) = i bi ⊗ ni ⊗ 1
P
ET

p∗13 φ: N ⊗A B ⊗A B → B ⊗A B ⊗A N p∗13 φ (n ⊗ 1 ⊗ 1) = i bi ⊗ 1 ⊗ ni
P
p∗23 φ: B ⊗A N ⊗A B → B ⊗A B ⊗A N p∗23 φ (1 ⊗ n ⊗ 1) = i 1 ⊗ b i ⊗ ni
P
para φ(n ⊗ 1) = i bi ⊗ ni . Estes mapas farão o papel de pull-backs do automorfismo de Y ×X Y ×X Rn
para Y ×X Y ×X Y × Rn . Podemos agora enunciar e provar
165

Teorema 7.1 (Grothendieck’s Faithfully Flat Descent) Seja B ⊃ A uma extensão fielmente plana
de anéis.
1. Para qualquer A-módulo M , a sequência

d0 d1 d2
0 - M ǫ
- M ⊗A B - M ⊗A B ⊗A B - M ⊗A B ⊗A B ⊗A B - ···

é exata. Aqui ǫ(m) = m ⊗ 1 e


X
dr (m ⊗ b0 ⊗ · · · ⊗ br ) = (−1)i · m ⊗ b0 ⊗ · · · ⊗ bi−1 ⊗ 1 ⊗ bi ⊗ · · · ⊗ br
0≤i≤r

2. Seja N um B-módulo (ou uma B-álgebra). Então existe uma bijeção entre as classes de iso-
morfismo de A-módulos (ou A-álgebras) M tais que M ⊗A B ∼
= N e as classes equivalência de
B ⊗A B-isomorfismos
φ: N ⊗A B - B ⊗A N

satisfazendo a condição de 1-cociclo p∗13 φ = p∗23 φ ◦ p∗12 φ, de modo que temos um diagrama
comutativo

p∗12-
φ
N ⊗A B ⊗A B B ⊗A N ⊗A B
p1∗

p∗23 φ
- ?
B ⊗A B ⊗A N

Dois B ⊗A B-isomorfismos φ e ψ são equivalentes se, e só se, existe um B-automorfismo


µ: N

- N tal que
ψ = p∗2 µ ◦ φ ◦ p∗1 µ−1

onde p∗1 µ = µ⊗ id: N ⊗A B - N ⊗A B e p∗2 µ = id ⊗µ: B ⊗A N - B ⊗A N são os pull-backs


∼ ∼

de φ com relação a p1 e p2 .

Prova 1. Primeiro vamos mostrar que o complexo dado é homotópico a 0 se o mapa ǫ: M → M ⊗A B


FT
admite uma seção s: M ⊗A B → M (isto é, s ◦ ǫ = id). Defina
)
k r : M ⊗A B ⊗(r+1) → M ⊗A B ⊗r
28

m ⊗ b0 ⊗ b1 ⊗ · · · ⊗ br 7→ s(m ⊗ b0 ) ⊗ b1 ⊗ · · · ⊗ br
7:
(1
RA

Agora verifique que id = dr−1 ◦ k r + k r+1 ◦ dr para todo r. Isto mostra que o complexo é exato. Para
0

o caso geral, como B é fielmente plano sobre A, basta mostrar que a sequência é exata após a mudança
01

de base − ⊗A B. Portanto é suficiente mostrar que o mapa ǫB : M ⊗A B → M ⊗A B ⊗A B admite uma


seção. Defina s: M ⊗A B ⊗A B → M ⊗A B por s(m ⊗ b1 ⊗ b2 ) = m ⊗ b1 b2 e verifique que s ◦ ǫ = id.
,2

2. Suponha que M é um A-módulo tal que M ⊗A B = N . Então podemos definir um isomorfismo


29

φ: N ⊗A B - B ⊗A N de B ⊗A B-módulos por

D
ct
,O

φ: (M ⊗A B) ⊗A B

- B ⊗A (M ⊗A B)
ET

m ⊗ b1 ⊗ b2 7→ b1 ⊗ m ⊗ b2

que, em nossa discussão geométrica, corresponde ao automorfismo de Y ×X Y × Rn que identifica as


restrições de Ui × Rn e Uj × Rn sobre Ui ∩ Uj . Agora um cálculo direto mostra que φ satisfaz a
condição de 1-cociclo. Observe que por (1), podemos identificar M com o A-submódulo de N dado por
{m ⊗ 1 | m ∈ M }.
166 Esquemas

Agora sejam M1 e M2 dois A-módulos tais que M1 ⊗A B = M2 ⊗A B = N e sejam φ1 e φ2 os iso-


morfismos correspondentes N ⊗A B → B ⊗A N . Se existe um isomorfismo de A-módulos ν: M1 - M2 ,

∗ ∗ −1
então ele induz um B-automorfismo µ = ν ⊗ id de N e temos que φ2 = p2 µ ◦ φ1 ◦ p1 µ pois

p∗2 µ ◦ φ1 ◦ p∗1 µ−1 (m2 ⊗ b1 ⊗ b2 ) = (id ⊗ν ⊗ id) ◦ φ1 ◦ (ν −1 ⊗ id ⊗ id)(m2 ⊗ b1 ⊗ b2 )


= (id ⊗ν ⊗ id) ◦ φ1 (ν −1 m2 ⊗ b1 ⊗ b2 )
= (id ⊗ν ⊗ id)(b1 ⊗ ν −1 m2 ⊗ b2 )
= b1 ⊗ m2 ⊗ b2 = φ2 (m2 ⊗ b1 ⊗ b2 )

para todo m2 ∈ M2 e b1 , b2 ∈ B. Reciprocamente, se φ2 = p∗2 µ ◦ φ1 ◦ p∗1 µ−1 ⇐⇒ φ2 ◦ p∗1 µ = p∗2 µ ◦ φ1


então µ: N - N se restringe a um isomorfismo ν: M1 - M2 já que
∼ ∼

φ2 (µ(m1 ) ⊗ 1) = φ2 ◦ p∗1 µ(m1 ⊗ 1) = p∗2 µ ◦ φ1 (m1 ⊗ 1) = p∗2 µ(1 ⊗ m1 ) = 1 ⊗ µ(m1 ),

i.e., µ(m1 ) ∈ M2 para todo m1 ∈ M1 .


Agora temos que mostrar que dado um isomorfismo φ: N ⊗A B - B ⊗A N satisfazendo a condição

de 1-cociclo, existe um A-módulo M tal que M ⊗A B = N . “Secretamente”, sabemos que M existe e


que φ é dado pela “fórmula de troca” acima, portanto é natural definir

def
M = {m ∈ N | φ(m ⊗ 1) = 1 ⊗ m}
F
que, em termos geométricos, corresponde ao subconjunto de Y × Rn = i Ui × Rn dos elementos que
“concordam nas intersecções” Ui ∩ Uj .
O subconjunto M ⊂ N é claramente um A-módulo. Vamos mostrar que o mapa natural

λ: M ⊗A B → N
m ⊗ b 7→ bm

é um isomorfismo de B-módulos. Para isto, considere o seguinte diagrama

α ⊗-id
M ⊗A B - N ⊗A B - B ⊗A N ⊗A B
β ⊗ id

λ ≈ φ ≈ ≈ p∗23 φ
FT
? ǫ ? e0 - ?
N - B ⊗A N - B ⊗A B ⊗ N
)
e1
28
7:

onde os mapas horizontais de linha inferior são os de (1), a saber, e1 (b⊗n) = 1⊗b⊗n e e0 (b⊗n) = b⊗1⊗n
(1
RA

para b ∈ B, n ∈ N , e os mapas da linha superior são obtidos pela mudança de base fielmente plana
− ⊗A B da sequência exata
0

α
-
01

M - N - B ⊗A N
β
,2

que define o A-módulo M , onde α(n) = φ(n ⊗ 1) e β(n) = 1 ⊗ n. Em particular, temos que M ⊗A B
29

é o equalizador de α ⊗ id e β ⊗ id, enquanto que N é o equalizador de e0 e e1 . Portanto o isomorfismo


D

φ induzirá o isomorfismo pedido λ uma vez que mostrarmos que o diagrama acima comuta. Mas é fácil
ct

checar que o quadrado da direita formado pelas flechas inferiores β ⊗ id e e1 comuta e o mesmo vale para
,O

o quadrado da esquerda pela definição de M . O quadrado da direita formado pelas flechas superiores
α⊗id e e0 também comuta pois p∗23 φ◦α⊗id(n⊗b) = p∗23 φ◦p∗12 φ(n⊗1⊗b) = p∗13 φ(n⊗1⊗b) = e0 ◦φ(n⊗b).
ET

Se N é uma B-álgebra, temos também que mostrar que o mapa de multiplicação m: N ⊗B N → N


também “desce,” o que pode ser feito por cálculos similares aos anteriores e que deixamos como exercı́cio
para o leitor.

Exemplo 7.2 (Variedade de Brauer-Severi)


167

8 Exercı́cios
01. Mostre que Proj A[x, y, z, w]/(yw − xz) = P1A ×Spec A P1A .
02. Seja S um esquema noetheriano. Sejam X e Y dois S-esquemas de tipo finito. Seja x ∈ X e
y ∈ Y dois pontos com mesma imagem s ∈ S. Sejam f : X → Y e g: X → Y dois morfismos tais que
f (x) = g(x) = y e os morfismos OS,s -álgebras fx# : OY,y → OX,x e gx# : OY,y → OX,x são iguais. Prove
que existe uma vizinhança aberta U de x tal que f |U = g|U .
Teorema 8.1 Para qualquer esquema Y , existe a reduced esquema Yred e a morfismo Yred → Y com
the seguinte propriedade: se X é a reduced esquema, a morfismo f : X → Y factors through Yred → Y ,
i.e., f determines a unique g: X → Yred tal que f é the composition of g e the mapa Yred → Y .

FT
)
28
7:
(1
RA
0
01
,2
29
D
ct
,O
ET
D
ET
,O
ct
29
RA
,2
01
0
(1
FT
7:
28
)
Chapter 9
base

Apêndice

Para a conveniência do leitor, recordamos neste apêndice alguns conceitos básicos que são frequentemente
utilizados ao longo de todo o livro.

1 Topologia Geral
Um espaço topológico é um conjunto X juntamente com uma coleção O de subconjuntos de X,
chamados de abertos, satisfazendo os seguintes axiomas:
1. ∅ ∈ O e X ∈ O (vazio e todo o espaço são abertos);
2. U, W ∈ O ⇒ U ∩ W ∈ O (intersecção finita de abertos é aberto);
S
3. se {Uλ }λ∈Λ é uma famı́lia qualquer com Uλ ∈ O então λ∈Λ Uλ ∈ O (união arbitrária de
abertos é aberto).
Um subconjunto F ⊂ X é chamado de fechado se ele é o complementar de algum aberto em X. Os
conjuntos fechados satisfazem portanto os seguintes axiomas:
1. ∅ e X são fechados;
2. se F e G são fechados então F ∪ G também é fechado;
T
3. se {Fλ }λ∈Λ é uma famı́lia qualquer de fechados então λ∈Λ Fλ também é fechado.
Se a coleção O estiver clara pelo contexto, por abuso de linguagem diremos que o conjunto X é ele
próprio um espaço topológico. Todo conjunto X admite ao menos a chamada topologia discreta, em
que todo subconjunto de X é aberto (e portanto fechado).
Dados dois espaços topológicos X e Y , dizemos que uma função f : X → Y é contı́nua se, para todo
aberto V de Y , a sua pré-imagem f −1 (V ) é um aberto de X. Equivalentemente, f é contı́nua se a pré-
imagem de um conjunto fechado de Y é fechado em X. Se f é uma bijeção contı́nua e sua inversa também
é contı́nua, dizemos que f é um homeomorfismo e que X e Y são espaços homeomorfos. Uma função
contı́nua f : X → Y é dita aberta (resp. fechada) se a imagem de todo aberto (resp. fechado) de X é
FT
um aberto (resp. fechado) de Y . Assim, uma bijeção contı́nua f : X → Y é um homeomorfismo se, e só
se, f é um mapa aberto (ou fechado).
)
Em vez de prescrever todos os conjuntos abertos (ou fechados) de X, uma outra maneira de descrever
28

a topologia de X é por meio de uma base B, isto é, uma coleção de subconjuntos abertos de X tal que
7:

qualquer aberto de X pode ser escrito como uma união de elementos de B. Por exemplo, a coleção
(1

de todas bolas abertas formam uma base para a topologia usual do Rn . Uma base B de X satisfaz as
RA

seguintes duas propriedades:


0

S
01

1. U∈B U = X
2. se U, U ′ ∈ B então U ∩ U ′ pode ser escrito como união de elementos em B.
,2

Por outro lado, dada uma famı́lia B de subconjuntos de um conjunto X satisfazendo os dois axiomas
29

acima, podemos definir uma topologia em X, bastando para isto declarar um subconjunto U de X aberto
D

se ele se escreve como união de elementos de B. Trabalhar com bases em geral simplifica algumas tarefas.
ct

Por exemplo, para verificar que um mapa f : X → Y é contı́nuo, é suficiente verificar que as pré-imagens
,O

f −1 (V ) de abertos V de uma base de Y são abertos em X.


ET

Seja X um espaço topológico e x ∈ X um ponto deste espaço. Uma vizinhança aberta ou


simplesmente vizinhança é qualquer conjunto aberto contendo x. Para qualquer subconjunto S ⊂ X,
definimos o seu fecho S como sendo o menor conjunto fechado que contém S, i.e.,
\ n o

F = x ∈ X toda vizinhança de x tem intersecção não
def
S =
F ⊃S
vazia com S
F fechado
170 Apêndice

Em outras palavras, S é o conjunto de pontos x que estão “arbitrariamente próximos” de S. Se S = X, topologia induzida
topologia quociente
dizemos que S é denso em X. topologia produto
Dado um subconjunto arbitrário Y ⊂ X de um espaço topológico X, Y também é um espaço Hausdorff
quasi-compacto
topológico com a chamada topologia induzida: os abertos de Y são aqueles da forma U ∩ Y onde U compacto
é um conjunto aberto em X. A topologia induzida é a topologia mais grossa para a qual o mapa da desconexo
conexo
inclusão Y ֒→ X é contı́nuo. redutı́vel
lema de Zorn
Lemma 1.1 Seja Uα be an open cobertura of X. A subset F ⊂ X é closed in X se, e só se, F ∩ Uα é limitante superior
closed in Uα (com the subspace topology) para todo α. elemento maximal
limitante inferior
T elemento minimal
Prova Se Fα são closed sets in X tal que F ∩ Uα = Fα ∩ Uα , então F = (Fα ∪ Uαc ), which é a closed universo de Grothend
set in X.

Se ∼ é um relação de equivalência em X, podemos definir uma topologia no espaço quociente X/ ∼,


chamada de topologia quociente, da seguinte forma: um conjunto de classes de equivalência U em
X/ ∼ é aberto se, e somente se, a sua união é um aberto em X. A topologia quociente é a mais fina
para a qual a projeção X → X/ ∼ é contı́nua.
Finalmente,
Q se {Xi }i∈I é uma coleção de espaços topológicos, definimos a topologia produto em
X = i∈I Xi como a mais grossa das topologias para a qual todos os mapas de projeção pi : X → Xi são
T
contı́nuos. Uma base de X é formada pelos conjuntos dos forma i∈I0 p−1 i (Ui ) onde I0 é um subconjunto
finito de I e cada Ui ⊂ Xi é aberto no respectivo espaço.
Dizemos ainda que um espaço topológico X é:
1. Hausdorff se para cada par de pontos distintos x, x′ ∈ X existem dois conjuntos abertos
disjuntos U e U ′ tais que x ∈ U e x′ ∈ U ′ ;
S
2. quasi-compacto se toda cobertura aberta X = λ Uλ de X admite uma subcobertura finita
X = Uλ1 ∪ . . . ∪ Uλn ; seguindo a tradição francesa, dizemos que X é compacto se ele é quasi-
compacto e Hausdorff;
3. desconexo se ele é a união de dois fechados disjuntos não vazios (que, portanto, também são
abertos). Caso contrário, X é dito conexo;
4. redutı́vel se ele é a união de dois conjunotos fechados próprios. Caso contrário, X é chamado
(adivinhe!) de irredutı́vel. Equivalentemente, X é irredutı́vel se, e somente se, quaisquer dois
subconjuntos abertos não vazios têm uma intersecção não vazia. Em particular, os espaços
irredutı́veis são conexos. Este conceito só é interessante para os espaços não Hausdorff.
Se Y é um subconjunto arbitrário de X, dizemos que Y é quasi-compacto, irredutı́vel e assim por diante,
se Y tem a propriedade correspondente com relação à topologia induzida de X.
O seguinte resultado é equivalente ao axioma da escolha (e não somos um daqueles hereges que não
FT
acreditam no axioma da escolha!).
Teorema 1.2 (Tychonoff ) O produtos de espaços quasi-compactos é quasi-compacto.
)
28

Já que mencionamos o axioma da escolha, convém também relembrar uma de suas formas equiv-
7:

alentes mais úteis, o chamado lema de Zorn. Seja (P, ≤) um conjunto parcialmente ordenado. Uma
cadeia C é um subconjunto de P no qual quaisquer dois elementos x, y são comparáveis, isto é, se x, y ∈ C
(1
RA

então ou x ≤ y ou y ≤ x. Um limitante superior de C é um elemento u ∈ P tal que x ≤ u para


0

todo x ∈ C (observe que u não pertence necessariamente a C). Um elemento maximal m de P é um


01

elemento para o qual m ≤ x implica x = m. Analogamente definimos limitante inferior e elemento


,2

minimal.
29

Teorema 1.3 (Zorn’s lemma) Seja (P, ≤) um conjunto parcialmente ordenado em que toda cadeia
possui um limitante superior. Então (P, ≤) possui um elemento maximal.
D
ct

Temos um resultado análogo trocando limitante superior por inferior e elemento maximal por mini-
,O

mal. Na maioria das aplicações, P é uma famı́lia de subconjuntos de um conjunto fixado e ≤ é a inclusão
de conjuntos ⊂.
ET

2 Categorias e Funtores
Um universo de Grothendieck U é um conjunto muito grande no qual podemos realizar as operações
usuais da teoria dos conjuntos em seus elementos sem jamais sair de U. Precisamente, um conjunto não
vazio U é universo de Grothendieck se satisfaz os seguintes axiomas:
171

1. se x ∈ U e y ∈ x então y ∈ U; axioma de un
categoria
2. se x, y ∈ U então {x, y} ∈ U; flechas
morfismo iden
3. se x ∈ U então 2x ∈ U (aqui 2x denota o conjunto das partes de x); dual
S oposta
4. se I ∈ U e (xi )i∈I é uma famı́lia de elementos de U então i∈I xi ∈ U. isomorfismo
funtor covaria
O axioma de universo garante que, dado qualquer conjunto x, existe um universo U tal que funtor
x ∈ U. Adotamos esta axioma, que é independente dos outros axiomas da teoria dos conjuntos. Com
isto, evitaremos certas dificuldades lógicas do tipo “conjuntos de todos os conjuntos” no que segue.
Uma categoria C consiste em
1. um conjunto Obj(C), cujos elementos são chamados de objetos de C;
2. para cada par de objetos X, Y ∈ Obj(C), um conjunto HomC (X, Y ), cujos elementos são
chamados de morfismos ou flechas de X em Y ;
3. para cada terna de objetos X, Y, Z ∈ Obj(C), uma lei de composição de morfismos

HomC (X, Y ) × HomC (Y, Z) → HomC (X, Z)


(f, g) 7→ g ◦ f

satisfazendo os seguintes axiomas:


i. (identidade) para cada objeto X ∈ Obj(C), existe um morfismo idX ∈ HomC (X, X),
chamado de morfismo identidade de X, tal que

f ◦ idX = f e idX ◦g = g

para todo f ∈ HomC (X, Y ) e g ∈ HomC (Z, X).


ii. (associatividade) para todo f ∈ HomC (W, X), g ∈ HomC (X, Y ) e h ∈ HomC (Y, Z),

h ◦ (g ◦ f ) = (h ◦ g) ◦ f

Por exemplo, fixado um universo de Grothendieck U, podemos considerar a categoria dos conjuntos
Set, cujos objetos são todos os conjuntos que pertencem a U e HomSet (X, Y ) é simplesmente o conjunto
de todas as funções f : X → Y ; a lei de composição de morfismos nada mais é do que a composição usual
de funções. Outro exemplo é a categoria dos grupos abelianos Ab, cujos objetos são todos os grupos
abelianos que pertencem a U e HomAb (X, Y ) é simplesmente o conjunto de todos os morfismos de grupos
f : X → Y ; a lei de composição é a usual. Temos ainda a categoria Top de todos os espaços topológicos
em U e cujos morfismos são as aplicações contı́nuas. No que segue, frequentemente omitiremos referência
FT
explı́cita ao universo U fixado, referindo-nos às categorias de “todos” os conjuntos, grupos abelianos,
espaços topológicos, etc.
)
28

Uma maneira prática de se pensar em um categoria é em termos de seu grafo subjacente (na verdade,
um multigrafo dirigido), cujo vértices são os objetos da categoria e cujas arestas são os morfismos da
7:

categoria. Cada vértice tem um laço distinto, a flecha identidade, e temos uma regra de composição de
(1
RA

arestas satisfazendo os axiomas usuais.


Se C é uma categoria denotamos por C◦ a chamada dual ou oposta, obtida invertendo-se o sentido
0
01

de todas as flechas de C (ou seja, HomC◦ (X, Y ) = HomC (Y, X)). Por abuso de linguagem que costu-
mamos escrever X ∈ C se X é um objeto de C. Dizemos que h ∈ HomC (X, Y ) é um isomorfismo se
,2

existe g ∈ HomC (Y, X) tal que g ◦ f = idX e f ◦ g = idY . Nós escrevemos X ∼ = Y se há um isomorfismo
29

entre X e Y .
Sejam C e D duas categorias. Um funtor covariante ou simplesmente funtor F : C → D de C em
D
ct

D é uma regra que associa


,O

1. um objeto F (X) ∈ D para cada objeto X ∈ C;


ET

2. um morfismo F (φ) ∈ HomD (F (X), F (Y )) para cada morfismo φ ∈ HomC (X, Y ), respeitando
morfismos identidade bem como as leis de composição:

F (idX ) = idF (X) e F (φ ◦ ψ) = F (φ) ◦ F (ψ)

para todo X ∈ C e todos os morfismos φ e ψ de C que podem ser compostos.


172 Apêndice

Em termos dos grafos subjacentes, um funtor nada mais é do que um morfismo de grafos que re- funtor contravariante
morfismo de funtores
speitam os laços identidade e as leis de composição das arestas. Um funtor F : C◦ → D é às vezes transformação natura
chamado de funtor contravariante de C para D, uma vez que ele inverte os sentidos das flechas. fiel
plenamente fiel
Exemplos clássicos de funtores são: o funtor inclusão F : Ab ֒→ Set, também chamado funtor esqueci- subcategoria
mento (basta “esquecer” as estruturas de grupo abeliano, tanto para objetos como para morfismos); o plena
essencialmente sobrej
funtor π1 : PTop → Group dado pelo grupo fundamental π1 (X, x0 ) de um espaço topológico pontuado lema de Yoneda
(X, x0 ) (aqui os objetos de PTop são pares (X, x0 ) onde X é um espaço topológico e x0 ∈ X é um “ponto
base” e um morfismo f : (X, x0 ) → (Y, y0 ) é um mapa contı́nuo f : X → Y preservando pontos base, i.e.,
f (x0 ) = y0 ).
Dados dois funtores F, G: C → D, um morfismo de funtores ou transformação natural φ: F →
G entre F e G é uma coleção de morfismos φX ∈ HomD (F (X), G(X)), um para cada objeto X ∈ C, tal
que

φX-
F (X) G(X)

F (f ) G(f )
? φY- ?
F (Y ) G(Y )

comuta para todos X, Y ∈ C e todos f ∈ HomC (X, Y ). O conjunto de todos os morfismos (em algum
universo) entre F e G será denotado Hom(F, G).
Um funtor F : C → D dá origem a um mapa

FX,Y : HomC (X, Y ) → HomD (F (X), F (Y ))

para cada par de objetos X, Y ∈ C. Dizemos que é F pleno (respectivamente fiel, plenamente fiel)
se FX,Y é sobrejetora (respectivamente injetora, bijetora) para todo X, Y ∈ C. Um funtor fiel F não
precisa ser injetor em objetos ou morfismos: podemos ter f : X → Y e f ′ : X ′ → Y ′ com F (X) = F (X ′ ),
F (Y ) = F (Y ′ ) e F (f ) = F (f ′ ); a injetividade só é garantida apenas para morfismos entre um par fixo
de objetos. Da mesma forma um funtor pleno não precisa ser sobrejetor em objetos ou morfismos.
Uma subcategoria C′ de C é uma categoria cujos objetos e morfismos são subconjuntos dos de C
e cuja regra de composição de flechas é a mesma de C; um subcategoria C′ de C é dita plena se o funtor
inclusão C′ ֒→ C é plenamente fiel. Em termos dos grafos subjacentes, uma subcategoria corresponde
a um subgrafo e uma subcategoria completa, a um subgrafo induzido, ou seja, um subgrafo obtido
escolhendo vértices de um grafo e incluindo todas as arestas entre eles.
FT
Dizemos que duas categorias C e D são equivalentes se houver funtores F : C → D e G: D → C e
isomorfismos G ◦ F ∼ = idC e F ◦ G ∼ = idD , onde id denota o funtor identidade. Um funtor F : C → D dá
)
uma equivalência das categorias se, e somente se, ele é plenamente fiel e essencialmente sobrejetor:
28

cada objeto de D é isomorfo a F (X) para algum X ∈ D. Em particular, uma equivalência de categorias
7:

não precisa ser uma bijeção. Por exemplo, as categorias dadas pelos seguintes grafos são equivalentes:
(1
RA

• e • ⇄ •
0

  
01

Um funtor plenamente fiel F : C → D estabelece uma equivalência entre C e sua imagem, uma subcategoria
,2

plena de D.
29

Para cada objeto X ∈ C, temos um funtor X: C → Sets dado por X(−) = HomC (X, −). Dizemos
que um funtor F : C → Sets é representável por um objeto X ∈ C se existe um isomorfismo F ∼
D

= X.
ct

Uma consequência direta, mas útil, das definições acima é o chamado lema de Yoneda, que afirma que
,O

para qualquer funtor F : C → Sets e para cada X ∈ C existe uma bijeção natural
ET

Hom(X, F ) - F (X)

φ 7→ φX (idX )

que é funtorial em X, ou seja, quando fazemos X percorrer os objetos de C as bijeções acima produzem
um isomorfismo entre os funtores X 7→ Hom(X, F ) e F . Em particular, temos um isomorfismo natural
173

Hom(X, Y ) = HomC (X, Y ) para cada X, Y ∈ C, mostrando que um objeto que representa um funtor é adjunto à esq
adjunto à dir
único a menos de isomorfismo. direcionado
Sejam F : C → D e G: D → C funtores. Dizemos que F é um adjunto à esquerda de G e que filtrante
(surpresa!) G é um adjunto à direita de F se, para cada X ∈ C e Y ∈ D, existe uma bijeção natural

HomD (F (X), Y ) - HomC (X, G(Y ))


funtorial em ambas as entradas X e Y : para X fixo, conforme Y percorre os objetos de D, as bijeções


acima dão origem a um isomorfismo entre os funtores HomD (F (X), −) e HomC (X, G(−)), e analogamente
para Y fixo.

3 Limites

3.1 Conjuntos Direcionados e Categorias Filtrantes


Começamos definindo o nosso “conjunto de ı́ndices”.
Definição 3.1.1 Uma relação  em um conjunto I é uma pré-ordem se satisfaz os seguintes dois
axiomas:
1. (Reflexividade) i  i para todo i ∈ I.
2. (Transitividade) se i  j e j  k então i  k.
Um conjunto pré-ordenado (I, ) é chamado de direcionado se, para quaisquer i, j ∈ I, existe k ∈ I
tal que i  k e j  k (ou seja, existe k “majorando” dois elementos i e j quaisquer).
Exemplo 3.1.2 Conjuntos direcionados que aparecem na Natureza são, por exemplo,
1. o conjunto 2X das partes de um conjunto X (i.e., o conjunto de todos os subconjuntos de X),
pré-ordenados pela inclusão ⊆. Dados dois subconjuntos A e B de X, temos que A ∪ B majora
A e B.
2. os conjuntos de submódulos finitamente gerados de um módulo M , pré-ordenados pela inclusão.
Dados dois submódulos N e P , temos que N + P é finitamente gerado e majora N e P .
3. os elementos de um conjunto multiplicativo S de um anel, pré-ordenados pela relação de divisi-
bilidade | em S (i.e., s  t ⇐⇒ s | t ⇐⇒ existe u ∈ S tal que t = su). Dados dois elementos
t, s ∈ S, temos que o produto st majora s e t.
Note que nos dois primeiros exemplos, a relação de pré-ordem é uma relação de ordem (i.e., vale a
propriedade anti-simétrica), mas não no último exemplo: se S é o conjunto multiplicativo de Z formado
por todos os elementos não nulos, temos que 2 | −2 e −2 | 2 mas 2 6= −2.
FT
Um conjunto direcionado (I, ) pode ser visto como uma categoria da seguinte forma: os objetos
desta categoria são os elementos de I e as flechas são dadas por

)
Hom(i, j) = {i → j} se i  j
28

def
∅ caso contrário
7:

para quaisquer i, j ∈ I. Esta categoria é o exemplo mais importante de uma categoria filtrante, como na
(1
RA

seguinte
0
01

Definição 3.1.3 Uma categoria I é dita filtrante se satisfaz os seguintes axiomas:


1. Dadas duas flechas
,2

j
29

ր
i
D
ct

ց
,O

k
elas “eventualmente convergem”: existem j → l e k → l tais que o seguinte diagrama comuta:
ET

j
ր ց
i l
ց ր
k
174 Apêndice

2. Dadas duas flechas conexa


cofiltrante
i⇉j colimite
limite direto
elas podem ser “equalizadas”: existe j → k tal que as composições
i⇉j→k
de i para k são iguais.
3. I é conexa vista como um grafo não direcionado, i.e., quaisquer dois objetos i e j de I podem
ser conectados por um caminho de flechas, ignorando as suas orientações:
i ← x1 → x2 ← x3 → · · · ← xn−1 → xn ← j
Dizemos que a categoria I é cofiltrante se a categoria oposta I◦ é filtrante.

3.2 Colimite ou Limite Direto


Seja I uma categoria filtrante e C uma categoria qualquer. Seja F : I → C um funtor (covariante). Pense
em I com um “conjunto de ı́ndices” para objetos F (i) ∈ C. Queremos definir agora uma espécie de
“união” dos F (i), o chamado colimite ou limite direto de F . Formalmente, o limite direto é um
lim F (i) ∈ C, juntamente com “mapas de inclusão” em C
objeto −→
i∈I

φi : F (i) - lim F (i) i∈I


−→
i∈I

que são “compatı́veis entre si”, isto é, para qualquer flecha i → j em I, temos um diagrama comutativo
φ-
j
F (j) lim F (i)
−→
6 i∈I
-
i
φ

F (i)
lim F (i) é o “menor objeto” com a propriedade acima, i.e., satisfaz a seguinte propriedade universal:
e −→
i∈I
dado um “objeto de teste” T ∈ I e uma coleção de morfismos fi : F (i) → T , i ∈ I, compatı́veis entre si,
ou seja, para qualquer flecha i → j em I, temos um diagrama comutativo em C
φj -
F (j) T
-

6
FT
i
φ

)
28

F (i)
7:

lim F (i) → T tal que fi = f ◦ φi para todo i ∈ I.


existe um único morfismo f : −→
(1
RA

i∈I

Exemplo 3.2.1 (União como limite direto) Seja X um conjunto qualquer e seja I a categoria
0
01

filtrante associada ao conjunto das partes de X, com elementos ordenados por inclusão. Considere o
funtor de “inclusão”
,2

F : I → Set
29

S 7→ S
D

Temos que −→lim F = X com os mapas de inclusão usuais S ֒→ X. De fato, basta verificar que a
ct

propriedade universal é satisfeita, o que é claro: dar um morfismo (vulgo função) f : X → T em Set é o
,O

mesmo que dar uma famı́lia de funções fS : S → T compatı́veis no sentido que fS = fS ′ |S sempre que
ET

S ⊂ S ′.
Da mesma forma, seja A um anel e M um A-módulo qualquer. Se I é a categoria filtrante associada ao
conjunto de submódulos de M finitamente gerados e F : I → A−Mod é o funtor de inclusão, temos que
lim F = M : definir um morfismo de A-módulos f : M → T é o mesmo que definir uma famı́lia compatı́vel
−→
de morfismos fN : N → T , N ∈ I, já que todo elemento m ∈ M pertence a um submódulo finitamente
gerado (por exemplo, N = Am).
175

Podemos expressar a propriedade universal de uma maneira mais sucinta da seguinte forma. Para funtor consta
limite inverso
um objeto X ∈ C, seja cX : I → C o funtor constante que leva todo i ∈ I em X e toda flecha em idX . limite projeti
Note que dar uma famı́lia compatı́vel de morfismos fi : F (i) → X, i ∈ I, é o mesmo que dar um morfismo limite
de funtores F → cX . Assim, o limite direto de F pode ser definido agora como qualquer objeto (caso
exista) representando o funtor X 7→ Hom(F, cX ), ou seja, temos uma bijeção natural
HomC (−→
lim F, X) = Hom(F, cX )
f 7→ {f ◦ φi }i∈I
Pelo lema de Yoneda ou pela propriedade universal, temos que quaisquer dois limites diretos são isomorfos
entre si.
Exemplo 3.2.2 (Limite Direto de Grupos Abelianos) Seja I uma categoria filtrante e F : I → Ab
um funtor. Vamos mostrar que o limite direto de F sempre existe; o mesmo método de construção se
aplica a outras categorias além de Ab. Defina
L
def i∈I F (i)
lim F =
−→ H
onde H é o seguinte subgrupo: identificando um elemento g ∈ F (i) com o vetor na soma direta cuja
i-ésima componente é g e cujas demais componentes são nulas, temos que H é gerado pelas diferenças
xi − xj , xi ∈ F (i), xj ∈ F (j)
onde existem flechas f : i → k, g: j → k em I com
F (f )(xi ) = F (g)(xj )
Em outras palavras, identificamos dois elementos na “união” dos F (i)’s desde que eles “eventualmente
concordem”.

Lemma 3.2.3 Se as sequências


0 - M′ - Mi - M ′′ - 0
i i
são exatas, então
0 - lim M ′ - lim Mi - lim M ′′ - 0
−→ i −→ −→ i
i∈I i∈I i∈I
também é exato.

3.3 Limite ou Limite Inverso


O conceito “dual” de limite direto é o de limite inverso. Novamente seja I uma categoria filtrante, C uma
categoria qualquer e F : I → C um funtor. O limite inverso ou limite projetivo ou ainda simplesmente
limite de F , em sı́mbolos,
FT
lim F (i),
←−
i∈I
)
28

é um objeto em C que representa o funtor X 7→ HomC (cX , F ), ou seja, temos a “fórmula”


HomC (X, ←− lim F ) ∼
= Hom(cX , F )
7:
(1
RA

Explicitamente, isto significa que temos uma famı́lia de “mapas de projeção”


lim F (i) - F (i)
ψi : ←− i∈I
0
01

i∈I
compatı́veis entre si, ou seja, tais que, para cada flecha i → j, os diagramas
,2

ψi -
29

lim F (i)
←− F (i)
i∈I
D
ct
ψj

,O

?
-
ET

F (j)
comutam, e ←−
lim F (i) é o “menor objeto” com esta propriedade: dado um “objeto de teste” T juntamente
i∈I
com mapas de projeção pi : T → F (i), compatı́veis entre si como acima, existe um único morfismo
p: T → ←−
lim F (i) tal que pi = ψi ◦ p para todo i ∈ I.
i∈I
176 Apêndice

Exemplo 3.3.1 Seja k um corpo e ks seu fecho separável. Seja I a categoria filtrante cujos objetos são separável
separável
os subcorpos l de ks que são extensões Galois finitas de k; temos uma flecha l → l′ se, e só se, se l ⊃ l′ . puramente inseparáve
Considere ainda o funtor fecho separável
normal
F : I → Groups Galois
l 7→ Gal(l/k)

e que associa, para cada flecha l → l′ , isto é, para cada inclusão l ⊃ l′ , a projeção natural dos grupos de
Galois
Gal(l/k) ։ Gal(l′ /k)
l ⊃ l′
φ 7→ φ|l′

Então afirmamos que ←− lim F = Gal(ks /k), com os mapas de projeção Gal(ks /k) ։ Gal(l/k) usuais. Isto
segue do seguinte fato: dar um elemento σ ∈ Gal(ks /k) é dar uma coleção de elementos σl ∈ Gal(l/k)
compatı́veis entre si, ou seja, tais que σl |l′ = σl′ sempre que l ⊃ l′ . De fato, a partir de σ ∈ Gal(ks /k)
podemos definir σl = σ|l e, reciprocamente, dada uma famı́lia de elementos σl ∈ Gal(l/k) compatı́veis,
como todo elemento a ∈ ks pertence a alguma extensão Galois finita l ⊃ k, podemos definir σ(a) = σl (a),
definição esta que claramente independe da escolha de l. Portanto, dar um morfismo de grupos p: T →
Gal(ks /k) é o mesmo que dar uma famı́lia de morfismos pl : T → Gal(l/k) compatı́veis, logo Gal(ks /k)
possui a propriedade universal do limite inverso.

4 Alguns Resultados Algébricos

4.1 Extensões Algébricas


Seja L ⊃ K uma extensão finita de corpos de grau n = [L : K]. Dizemos que L é separável sobre K se
existem exatamente n K-imersões σ1 , . . . , σn : L ֒→ K alg de L no fecho algébrico de K.

Teorema 4.1.1 (Elemento Primitivo) Seja L ⊃ K uma extensão separável finita. Então L = K(θ)
para algum θ ∈ L.

Teorema 4.1.2 As seguintes condições são equivalentes:


1. L ⊃ K é separável.
2. Todo elemento b ∈ L é separável, i.e., o polinômio minimal de b sobre K não possui raı́zes
múltiplas.
FT
3. L ⊗K K alg é reduzida (i.e., não possui elementos nilpotentes além de 0).

Um polinômio f (x) ∈ K[x] possui raı́zes múltiplas se, e só se, gcd(f (x), f ′ (x)) 6= 1. Se f (x) é
)
28

irredutı́vel, isto ocorre se, e só se, gcd(f (x), f ′ (x)) = f (x), isto é, se, e só se, f ′ (x) = 0 (pois deg f ′ (x) <
deg f (x)). Isto significa que f (x) = g(xp ) para algum g(x) ∈ K[x] e p = char K > 0. Em particular,
7:

extensões de corpos de caracterı́stica 0 são sempre separáveis.


(1
RA

Definição 4.1.3 Seja L ⊃ K uma extensão de corpos de caracterı́stica p > 0. Dizemos que L ⊃ K é
0
01

n
puramente inseparável se para todo b ∈ L existe um n (que depende de b) tal que bp ∈ K.
,2

Note que toda extensão de corpos L ⊃ K pode ser quebrada em uma extensão separável M ⊃ K
29

seguida de uma puramente inseparável L ⊃ M . Basta tomar M como o conjunto de todos os elementos
separáveis de L. O corpo M é chamado de fecho separável de K em L.
D
ct

Uma extensão de corpos L ⊃ K é dita normal se, para todo b ∈ L, todos os seus conjugados
,O

pertencem a L. Uma extensão de corpos é Galois se é normal e separável.


ET

Teorema 4.1.4 Seja L ⊃ K uma extensão normal e seja G = AutK (L). Seja M o corpo fixo por G:

def
M = K G = {b ∈ L | σ(b) = b para todo σ ∈ G}

Então L ⊃ M é Galois e M ⊃ K é puramente inseparável.


177

4.2 Grau de Transcendência algebricamen


base de trans
Seja L ⊃ K uma extensão de corpos. Um subconjunto Ω = {ωi } de L é algebricamente indepen- grau de trans
dente sobre K se não existe relação polinomial não trivial entre os ωi , ou seja, dado um polinômio traço
norma
f (x1 , . . . , xn ) ∈ K[x1 , . . . , xn ],

f (ωi1 , . . . , ωin ) = 0 ⇒ f (x1 , . . . , xn ) = 0

para todo subconjunto finito {ωi1 , . . . , ωin } de Ω.


Um subconjunto maximal (com relação à inclusão) Ω ⊂ L algebricamente independente sobre K
é chamado de base de transcendência de L sobre K. Note que Ω é uma base de transcendência de
L sobre K se, e só se, a K-subálgebra K[Ω] de L é isomorfa a um anel de polinômios nas “variáveis”
ω ∈ Ω e L é algébrico sobre K(Ω) = Frac K[Ω]. Por exemplo, {x1 , . . . , xn } e {x21 − x2 , x2 , . . . , xn } são
duas bases de transcendência de K(x1 , . . . , xn ) = Frac K[x1 , . . . , xn ] sobre K. Quaisquer duas bases de
transcendência possuem mesma cardinalidade, que é chamada de grau de transcendência de L sobre
K e é denotada por tr. degK L. Assim, no exemplo tr. degK K(x1 , . . . , xn ) = n.
No caso finito, como para espaços vetoriais, para mostrar que quaisquer duas bases possuem mesma
cardinalidade, basta provar o
Lema 4.2.1 (Axioma de troca de Steinitz) Sejam Ω e Ω′ duas bases de transcendência de L sobre
K. Dado qualquer ω ∈ Ω, existe ω ′ ∈ Ω′ tal que (Ω \ {ω}) ∪ {ω ′ } é uma base de transcendência.
Prova Sejam Ω = {ω1 , . . . , ωr } e Ω′ = {ω1′ , . . . , ωs′ } com ω = ω1 . Para cada i = 1, . . . , s, considere uma
relação polinomial não trivial pi (ωi′ , ω1 , . . . , ωr ) = 0 entre ωi′ , ω1 , . . . , ωr , que existe devido à maximalidade
de Ω. Então pelo menos um dos pi ’s deve envolver ω1 não trivialmente, caso contrário todos os elementos
de Ω′ , e portanto de L, seriam algébricos sobre K(ω2 , . . . , ωr ). Mas então ω1 seria algébrico sobre
K(ω2 , . . . , ωr ), o que contradiz a independência algébrica de Ω.
Sem perda de generalidade, seja p1 (ω1′ , ω1 , . . . , ωr ) = 0 tal relação envolvendo ω = ω1 . Afirmamos
que (Ω \ {ω}) ∪ {ω1′ } = {ω1′ , ω2 , . . . , ωr } é uma base de transcendência de L sobre K. De fato, da maxi-
malidade de Ω, temos que L é algébrico sobre K(ω1′ , ω1 , . . . , ωr ) e a relação polinomial acima mostra que
ω1 é algébrico sobre K(ω1′ , ω2 , . . . , ωr ), de modo que L também é algébrico sobre este último corpo, o que
mostra a maximalidade de (Ω \ {ω}) ∪ {ω1′ }. Por outro lado, se este último conjunto não fosse algebrica-
mente independente, como {ω2 , . . . , ωr } é algebricamente independente, terı́amos que ω1′ seria algébrico
sobre K(ω2 , . . . , ωr ). Mas como acabamos de ver, temos que L é algébrico sobre K(ω1′ , ω2 , . . . , ωr ) e
portanto L seria algébrico sobre K(ω2 , . . . , ωr ), o que contraria a maximalidade de Ω. Isto completa a
prova.

Agora, aplicando Steinitz um número finito de vezes, podemos transformar Ω em uma base contida
em Ω′ e vice-versa, mostrando que |Ω| = |Ω′ |.
FT
4.3 Traço e Norma
)
28

Seja B uma A-álgebra que é livre como A-módulo de posto finito n. Todo elemento b ∈ B define, por
multiplicação, uma transformação A-linear
7:
(1
RA

Mb : B → B
0

x 7→ bx
01

Definimos o traço TrB/A (b) ∈ A de b ∈ B como o traço de Mb ; da mesma forma, a norma NB/A (b) ∈ A
,2

de b ∈ B é definida como o determinante de Mb . Explicitamente, se {ω1 , . . . , ωn } é uma base de B sobre


29

A, escrevendo
b · ω1 = a11 ω1 + a12 ω2 + · · · + a1n ωn
D
ct

b · ω2 = a21 ω1 + a22 ω2 + · · · + a2n ωn


,O

..
.
ET

b · ωn = an1 ω1 + a2n ω2 + · · · + ann ωn


com aij ∈ A, temos
X
TrB/A (b) = Tr(aij ) = aii e NB/A (b) = det(aij )
1≤i≤n
178 Apêndice

e, como em álgebra linear, mostra-se que estes dois valores independem da escolha da base de B sobre
A. Diretamente das definições, temos que o traço é aditivo e a norma é multiplicativa: para b1 , b2 ∈ B,

TrB/A (b1 + b2 ) = TrB/A (b1 ) + TrB/A (b2 ) e NB/A (b1 · b2 ) = NB/A (b1 ) · NB/A (b2 )

Observe ainda que se a ∈ A, então TrB/A (a) = na e NB/A (a) = an .


Se C é uma B-álgebra que é livre de posto finito como B-módulo, temos ainda que

TrC/A = TrB/A ◦ TrC/B e NC/A = NB/A ◦ NC/B

L do cálculo explı́cito com matrizes, utilizando bases B = Aω1 ⊕ · · · ⊕ Aωn , C = Bτ1 ⊕ · · · ⊕ Bτm
Isto segue
eC= 1≤i≤n Aωi τj .
1≤j≤m

Agora, vamos especializar a discussão acima para corpos: seja L ⊃ K uma extensão algébrica finita
de grau n = [L : K]. Pelo teorema do elemento primitivo, podemos escrever L = K(θ) = K[θ] para
algum θ ∈ L com polinômio minimal p(x) ∈ K[x], que possui n raı́zes distintas θ = θ1 , θ2 , . . . , θn em
K alg . Neste caso, podemos descrever explicitamente as imersões σi como as composições de K-álgebras

≈ K[x] ≈
σi : L = K[θ]   - K[θi ] ֒→ K alg
p(x)

θ1 7→ x 7→ θi

Assim, se p(x) = xn + an−1 xn−1 + · · · + a0 , utilizando a base 1, θ, . . . , θn−1 de L sobre K, temos que para
calcular a norma e o traço de θ, precisamos calcular o determinante e o traço da “matriz companheira”
de p(x):
 
0 0 ··· 0 −a0
1 0 ··· 0 −a1 
 
0 1 ··· 0 −a2 
 .. 
 . 
0 0 ··· 1 −an−1
Portanto X X
TrL/K (θ) = −an−1 = θi = σi (θ)
1≤i≤n 1≤i≤n
Y Y
NL/K (θ) = (−1)n a0 = θi = σi (θ)
1≤i≤n 1≤i≤n
FT
e, em geral, temos
)
Lemma 4.3.1 Seja L ⊃ K uma extensão algébrica finita separável de corpos de grau n = [L : K] e
28

sejam σ1 , . . . , σn : L ֒→ K alg as n K-imersões de L no fecho algébrico de K. Então, para todo b ∈ L,


7:

X Y
(1
RA

TrL/K (b) = σi (b) e NL/K (b) = σi (b)


1≤i≤n 1≤i≤n
0
01

Prova Seja m = [K(b) : K]. Temos, pelo cálculo acima, que


,2

X
29

TrL/K (b) = TrL/K(b) ◦ TrK(b)/K (b) = m · τ (b)


D

τ
ct
,O

onde τ percorre as m K-imersões de K(b) em K alg . Como L ⊃ K(b) é separável, temos que cada
K-imersão τ : K(b) ֒→ K alg se estende para exatamente n/m = [L : K(b)] K-imersões de L em K alg .
ET

Portanto X X
TrL/K (b) = m · τ (b) = σi (b)
τ 1≤i≤n

como desejado. A prova para a norma é análoga.


179

Teorema 4.3.2 (Basic properties of norms e traces) Seja K/k be a finite extensão of corpos of
grau n.
1. The trace é k-linear e the norm é multiplicative: para todo α, β ∈ K e a, b ∈ k,

TrK/k (aα + bβ) = a TrK/k (α) + b TrK/k (β)


NK/k (αβ) = NK/k (α)NK/k (β)
TrK/k (a) = na e NK/k (a) = an

2. Seja ns e ni denote respectively the graus of separability e inseparability of K/k . há exactly ns
k-embeddings (i.e., embeddings that restrict to the identity on k) of K into an algébrico closure
k alg of k; denote them por σ1 , σ2 , . . . , σns . Então, para qualquer α ∈ K,
X  Y ni
TrK/k (α) = ni σj (α) e NK/k (α) = σj (α)
1≤j≤ns 1≤j≤ns

In particular, the trace é zero se the extensão é inseparable.


3. (Transitivity) Seja L/K be another finite extensão of corpos e seja α ∈ L. Então

TrL/k (α) = TrK/k ◦ TrL/K (α)


NL/k (α) = NK/k ◦ NL/K (α)

Prova 1. Follows easily from the definitions.


2. Seja p be the characteristic of k. Se p(X) denotes the minimal polinômio of α sobre k, we may write
e
p(X) = f (X p ) para algum f (X) = X d + cd−1 X d−1 + · · · + c0 com distinct roots r1 , r2 , . . . , rd in k alg .
e
We may assume that αp = r1 . Como ni /pe é an integer e ni ns = n, temos
 Y ni  Y ni /pe  Y ni /pe  Y n/(dpe )
n /d
σj (α) = σj (r1 ) = rj s = rj
1≤j≤ns 1≤j≤ns 1≤j≤d 1≤j≤d

But por the last example we also have


 Y n/(dpe )  Y n/(dpe )
n/(dpe )
NK/k (α) = (−1)n c0 = (−1)n (−1)d rj = rj
1≤j≤d 1≤j≤d

The argument works similarly para the trace. Moreover, se the extensão é inseparable, ni é a power of
FT
p, so the trace vanishes.
3. Seja ms e ns be respectively the graus of separability of L/K e K/k , e mi e ni be respectively
)
28

the graus of inseparability of L/K e K/k . Seja τ1 , . . . , τms be the K-embeddings of L into k alg , e seja
σ1 , . . . , σns denote extensões of the k-embeddings of K to algum normal extensão of k containing L.
7:

Então σi τj são the ms ns k-embeddings of L. Como the grau of inseparability of L/k é mi ni , temos
(1
RA

 Y Y mi ni  Y ni
0

NL/k (α) = σi τj (α) = σi (NL/K (α))


01

1≤i≤ns 1≤j≤ms 1≤i≤ns


,2

= NK/k ◦ NL/K (α)


29

e similarly para the trace.


D
ct

4.4 Discriminante
,O

Sejam ω1 , . . . , ωn e τ1 , . . . , τn bases de K sobre Q e seja C = (cij ) a matriz de mudança de base:


ET

ωi = ci1 τ1 + · · · + cin τn i = 1, . . . , n

Sejam ∆(ω1 , . . . , ωn ) = (TrK/Q (ωi ωj )) e ∆(τ1 , . . . , τn ) = (TrK/Q (τi τj )) os discriminantes das duas bases.
Então
∆(ω1 , . . . , ωn ) = ∆(τ1 , . . . , τn ) · (det C)2
180 Apêndice

e ambos os discriminantes são não nulos. torsion free módulo


Sejam σi : K ֒→ C as imersões de K em C e considere a matriz δ(ω1 , . . . , ωn ) = (σj (ωi )). Multipli-
cando pela transposta, temos
 X   
δ(ω1 , . . . , ωn ) · δ(ω1 , . . . , ωn )T = σk (ωi )σk (ωj ) = TrK/Q (ωi ωj ) .
1≤k≤n

Por outro lado,  X 


δ(ω1 , . . . , ωn ) = cik σj (τk ) = C · δ(τ1 , . . . , τn ).
1≤k≤n

Assim,
∆(ω1 , . . . , ωn ) = (det δ(ω1 , . . . , ωn ))2 = (det C)2 · (det δ(τ1 , . . . , τn ))2
= (det C)2 · ∆(τ1 , . . . , τn )
Como det C 6= 0, para mostrar que estes discriminantes são não nulos, basta mostrar isto para uma base
especı́fica. Escrevendo K = Q(θ) (teorema do elemento primitivo ), temos que 1, θ, . . . , θn−1 é uma base
de K sobre Q. Sendo θi = σi (θ) os conjugados de θ, temos o determinante de Vandermonde
Y
det δ(1, θ, θ2 , . . . , θn−1 ) = det(θji−1 ) = (θi − θj ) 6= 0.
1≤i<j≤n

Este determinante, e portanto ∆(1, θ, . . . , θn−1 ) = det δ(1, θ, . . . , θn−1 )2 , são não nulos pois os conjugados
θi são dois a dois distintos

4.5

Teorema 4.5.1 Todo PID é a UFD.


Prova Seja A be a PID; we first mostrar que todo element r ∈ R − (R× ∪ {0}) can be written as a
product of irredutı́vels. Se r é not itself an irredutı́vel, we can write it as r = ab com a, b ∈ / R× . Se either
a or b é not irredutı́vel, say the first, we can repeat the process e write a = cd, c, d ∈ / R . The question
×

então é whether this will eventually stop. The answer é yes: se not, there would be a sequence of elements
Sr1 = a, r2 = c, . . . of A e a strictly increasing chain of ideals (r0 ) ( (r1 ) ( (r2 ) ( · · ·; however,
r0 = r, S the
ideal i≥0 (ri ) = (r0 , r1 , r2 , . . .) é principal. Seja s ∈ R be a generator. In particular, como s ∈ i≥0 (ri ),
S
existe n tal que s = xrn para algum x ∈ R. But we also have rn+1 ∈ i≥0 (ri ) = (s) = (xrn ) ⊂ (rn ),
FT
portanto (rn+1 ) = (rn ), which é a contradiction.
Second, now that we know that factorisation into irredutı́vels é possible, temos to prove that todo
irredutı́vel p ∈ R é prime. So suponha that p | xy but p ∤ x; we need to mostrar que p | y. The ideal
)
28

(p, x) é principal; como p é irredutı́vel e p ∤ x, it must be equal to (1). In other words, há a, b ∈ R tal
que 1 = ap + bx. Multiplying por y, temos y = ay · p + b · xy, which shows that p | y.
7:
(1
RA

The last topic of our review é an easy special case of the structure theorem of finitamente gerado
0

módulos sobre a PID, which will be sufficient para our applications. Recall that an A-módulo M é said
01

to be torsion free se rm = 0 para algum r ∈ R e m ∈ M implies r = 0 or m = 0.


,2

Teorema 4.5.2 (Módulos sobre a PID) Seja A be a PID.


29

1. Seja M be a free A-módulo of finite rank. Então qualquer submódulo N of M é also free of
D

finite rank.
ct
,O

2. Todo finitamente gerado torsion free A-módulo é free.


ET

Prova 1. Seja e1 , . . . , en be a base of M e seja πi : M → R, i = 1, . . . , n, be the corresponding


coordinate functions, i.e., the A-módulo morfismos defined por πi (ei ) = 1 e πi (ej ) = 0 para j 6= i.
Consider the ideal π1 (N ); como A é a PID, we can write π1 (N ) = (r1 ) para algum r1 ∈ R. Seja v1′ ∈ N
be tal que π1 (v1′ ) = r1 . Next, we look at the submódulo N1 = {v ∈ N | π1 (v) = 0} of N e repeat the
procedure: write π2 (N1 ) = (r2 ) e choose algum v2′ ∈ N1 tal que π2 (v2′ ) = r2 . In general, we recursively
′ ′
define Ni = {v ∈ Ni−1 | πi (v) = 0} e pick vi+1 ∈ Ni tal que πi+1 (vi+1 ) generates πi+1 (Ni ). Deleting the
181

zeros amongst the v1′ , . . . , vn′ , obtemos elements v1 , . . . , vm ∈ N which we claim é a base para N . In fact, diagonal
por construction, the first i − 1 coordinates of vi′ são 0:

v1′ = r1 e1 + · · ·
v2′ = r2 e2 + · · ·
v3′ = r3 e3 + · · ·

e so on. This implies that the vi são linearly independent. Furthermore, se v ∈ N has the form
v = a1 e1 + · · · + an en , ai ∈ R, como a1 ∈ (r1 ), existe s1 ∈ R tal que v − s1 v1′ ∈ N1 ; applying the same
reasoning com v − s1 v1′ e N1 in place of v e N , we can find s2 ∈ R tal que v − s1 v1′ − s2 v2′ ∈ N2 e so on.
Portanto v = s1 v1′ + · · · + sn vn′ para algum si ∈ R, which shows that the vi generate N .
2. Seja M be a finitamente gerado torsion free A-módulo e seja e1 , . . . , em be a set of generators; we may
assume that e1 , . . . , en , n ≤ m, são linearly independent, e that they são maximal com this propriedade
among todo the subsets of {ei }. Então há nonzero Lai ∈ R tal que ai en+i = ri1 e1 + · · · + rin en para algum
rij ∈ R. Seja a = a1 . . . am−n 6= 0. Então aM ⊂ 1≤i≤n Rei , portanto aM é free por (1). But como M
é torsion free, multiplication por a é injetivo, portanto M é also free.

5 Exercı́cios
01. (Furstenberg) Para a, b ∈ Z, seja Ua,b = {a + nb | n ∈ Z}.
(a) mostrar que the sets Ua,b form a base para a topology on Z. Observe that in this topology qualquer
non-empty open set é infinite e Ua,b é both open e closed.
S
(b) mostrar que Z \ {−1, 1} = p U0,p , onde p runs sobre todo prime numbers. Use this fact to mostrar
que há infinitamente many primes.
02. mostrar que X é Hausdorff se, e só se, the diagonal ∆(X) = {(X, x) ∈ X × X | X ∈ X} é a closed
set in X × X.
03. (Irredutı́vels in Z[i])
(a) Seja p ≡ 1 (mod 4) be a prime number. mostrar que the equation X 2 + 1 ≡ 0 (mod p) has a
solution. Conclude that p é not an irredutı́vel in Z[i] e portanto é the sum of two squares in Z.
(b) mostrar que the irredutı́vels of Z[i] são 1 + i, prime numbers p ≡ 3 (mod 4), e elements of the form
a + bi, onde a e b são integers tal que a2 + b2 = p para a prime number p ≡ 1 (mod 4).
04. Seja A be a UFD.
(a) (Gauss’ lemma) Seja S be the set of polinômios p(X) = cn xn + · · · + c0 in R[X] whose coefficients
são relatively prime, i.e., u | ci para todo i implies u ∈ R× . mostrar que se p(X), q(X) ∈ S então
FT
p(X)q(X) ∈ S. Conclude that a polinômio in R[X] é irredutı́vel in R[X] se, e só se, it é irredutı́vel
in k[X], onde k é the quociente corpo of A.
)
(b) Prove that R[X] é a UFD. Portanto k[X1 , . . . , Xn ], k a corpo, é a UFD. Caution: Se A é a UFD,
28

então R[[X]] (see definition below) é not necessarily a UFD.


7:

05. Seja K be a number corpo. mostrar que α ∈ OK é a unit se, e só se, NK/Q (α) = ±1. Find the group
(1
RA

of units of OQ(√d) , onde d < 0 é a square-free integer.


0
01

06. Seja p be a prime number e ζ be a primitive p-th root of 1. mostrar que


(a) TrQ(ζ)/Q (ζ k ) = p − 1 se k = 0 e TrQ(ζ)/Q (ζ k ) = 0 se k = 1, . . . , p − 1.
,2

p−1
(b) the discriminant ∆(1, ζ, . . . , ζ p−2 ) = (−1) 2 pp−2 .
29
D
ct
,O
ET
D
ET
,O
ct
29
RA
,2
01
0
(1
FT
7:
28
)
1.
Bibliografia

D
ET
,O
ct
29
RA
,2
01
0
(1
FT
7:
28
)
Chapter 10
D
ET
,O
ct
29
RA
,2
01
0
(1
FT
7:
28
)

Vous aimerez peut-être aussi